XII JEE Maths Theory
XII JEE Maths Theory
INDEX
Topic Page No.
1. Graphs............................................................................. 01
4. Method of Differentiation................................................ 61
5. Application of Derivatives............................................... 75
then graph of
(a) y = – f(x) is
y
(b) y = f(–x) is
1
–4 –3 3
x
–2 –1 1 2
–1
(c) y = f(|x|) is
–4 –3 3 4
x
–2 –1 1 2
–1
RESONANCE JEE (Main + Advacnced) Graph Related to Modulus & Graphical Transformation - 1
(d) y = f(–|x|) is y
–3 3
x
–2 –1 1 2
–1
(e) y = |f(x)| is
y
x
–3 –2 –1 1 2 3 4
RESONANCE JEE (Main + Advacnced) Graph Related to Modulus & Graphical Transformation - 2
Graphical Trasformation : y
(1, 0)
x
(–2, 0) –1
(a) y – k = f(x – h) is
y
(h, 1 + k)
(–2 + h, k) (1 + h, k)
(h, k)
x
(h, –2+k)
(b) y = kf(x) is
(k > 0)
RESONANCE JEE (Main + Advacnced) Graph Related to Modulus & Graphical Transformation - 3
(c) y = f(kx) is
(k > 0)
x
Example # 5 : y = sin Solution period is 4
2
1
Example # 6 : y = sin x Solution Graphical Transformation
2
RESONANCE JEE (Main + Advacnced) Graph Related to Modulus & Graphical Transformation - 4
Functions & Inverse Trigonometric Functions
Definition :
Function is a rule (or correspondence), from a non empty set A to a non empty set B, that associates
each member of A to a unique member of B. Symbolically, we write f: A B. We read it as "f is a
function from A to B".
For example, let A {–1, 0, 1} and B {0, 1, 2}.
Then A × B {(–1, 0), (–1, 1), (–1, 2), (0, 0), (0, 1), (0, 2), (1, 0), (1, 1), (1, 2)}
Now, " f : A B defined by f(x) = x 2 " is the function such that
f {(–1, 1), (0, 0), (1, 1)}
f can also be shown diagramatically by following mapping.
A B
Note : Every function say y = f(x) : A B. Here x is independent variable which takes its values from A while
'y' takes its value from B. A relation will be a function if and only if
(i) x must be able to take each and every value of A and
and (ii) one value of x must be related to one and only one value of y in set B.
Graphically : If any vertical line cuts the graph at more than one point, then the graph does not
represent a function.
Example # 1 : (i) Which of the following correspondences can be called a function ?
(A) f(x) = x3 ; {–1, 0, 1} {0, 1, 2, 3}
(B) f(x) = ± x ; {0, 1, 4} {–2, –1, 0, 1, 2}
(C) f(x) = x ; {0, 1, 4} {–2, –1, 0, 1, 2}
(A) (B)
(C) (D)
Solution :
(i) f(x) in (C) and (D) are functions as definition of function is satisfied. while in case of (A) the
given relation is not a function, as f(–1) 2nd set. Hence definition of function is not satisfied.
While in case of (B), the given relation is not a function, as f(1) = ± 1 and f(4) = ± 2 i.e. element
1 as well as 4 in 1st set is related with two elements of 2nd set.Hence definition of function is not
satisfied.
(ii) B and D. In (A) one element of domain has no image, while in (C) one element of 1st set has
two images in 2nd set
FNT - 5
Functions & Inverse Trigonometric Functions
Self practice problem :
(1) Let g(x) be a function defined on [1, 1]. If the area of the equilateral triangle with two of its
vertices at (0,0) and (x,g(x)) is 3 / 4 sq. unit, then the function g(x) may be.
2
(A) g(x)= (1 x ) (B) g(x) = (1 x 2 ) (C) g(x) = (1 x 2 ) (D) g(x) = (1 x 2 )
(2) Represent all possible functions defined from {} to {1, 2}.
Answers : (1) B, C
Let y = f(x) : A B, then the set A is known as the domain of f and the set B is known as co-domain
of f.
x 2 5 is real iff x – 5 0
Solution : (i) f(x) = 2
|x| 5 x – 5 or x 5
the domain of f is (–, – 5 ] [ 5 , )
(ii) x3 – x R
domain is x R
Algebraic Operations on Functions :
If f and g are real valued functions of x with domain set A and B respectively, then both f and g are
defined in A B. Now we define f + g, f g, (f . g) and (f /g) as follows:
f f( x )
(x) =
(iii)
g g( x ) domain is {x x A B such that g(x) 0}.
Note : For domain of (x) = {f(x)}g(x) , conventionally, the conditions are f(x) > 0 and g(x) must be real.
For domain of (x) = f(x)Cg(x) or (x) = f(x)Pg(x) conventional conditions of domain are f(x) g(x)
and f(x) N and g(x) W.
FNT - 6
Functions & Inverse Trigonometric Functions
Example # 3 : Find the domain of following functions :
3
(i) f(x) = sin x 16 x 2 (ii) f(x) = 2
log(x3 x)
4x
Solution : (i) sin x is real iff sin x 0 x[2n, 2n + ], nI.
16 x 2 is real iff 16 x 0 4 x 4.
2
Thus the domain of the given function is {x : x[2n, 2n + ], nI }[4, 4]=[4, ] [0, ].
x2 x 1
Example # 4 : Find the range of f(x) =
x2 x 1
x2 x 1
Solution : f(x) = {x 2 + x + 1 and x2 + x – 1 have no common factor}
x2 x 1
x2 x 1
y=
x2 x 1
yx 2 + yx – y = x2 + x + 1
(y – 1) x 2 + (y – 1) x – y – 1 = 0
If y = 1, then the above equation reduces to –2 = 0. Which is not true.
Further if y 1, then (y – 1) x 2 + (y – 1) x – y – 1 = 0 is a quadratic and has real roots if
(y – 1)2 – 4 (y – 1) (–y – 1) 0
i.e. if y –3/5 or y 1 but y 1
Thus the range is (–, –3/5] (1, )
x2 4
Example # 5 : Find the range of f(x) =
x2
x2 4
Solution : f(x) = = x + 2; x 2
x2
graph of f(x) would be
Thus the range of f(x) is R – {4}
FNT - 7
Functions & Inverse Trigonometric Functions
Further if f(x) happens to be continuous in its domain
then range of f(x) is [min f(x), max. f(x)]. However for
sectionally continuous functions, range will be union
of [min f(x), max. f(x)] over all those intervals where
f(x) is continuous, as shown by following example.
Then range of above sectionally continuous function is [y2, y3] [y7, y6) (y4, y5]
(iii) Using monotonocity : Many of the functions are monotonic increasing or monotonic decreasing. In
case of monotonic continuous functions the minimum and maximum values lie at end points of domain.
Some of the common function which are increasing or decreasing in the interval where they are
continuous is as under.
FNT - 8
Functions & Inverse Trigonometric Functions
Classification of Functions :
Functions can be classified as "One One Function (Injective Mapping)" and "Many One Function":
OR
OR
OR
Into function :
If f : A B is such that there exists atleast one element in codomain which is not the image of any
element in domain, then f(x) is into.
FNT - 9
Functions & Inverse Trigonometric Functions
Diagrammatically into function can be shown as
OR
Note : (i) If f is both injective and surjective, then it is called a bijective mapping. The bijective functions
are also named as invertible, non singular or biuniform functions.
(ii) If a set A contains 'n' distinct elements, then the number of different functions defined from
A A is nn and out of which n! are one one.
(iii) If f and g both are onto, then gof or fog may or may not be onto.
(iv) The composite of two bijections is a bijection iff f and g are two bijections such that gof is
defined, then gof is also a bijection only when co-domain of f is equal to the domain of g.
ve ; 0 x 1
f(x) =
ve ; 1 x 3
(5) For each of the following functions find whether it is one-one or many-one and also into or
onto
1
(i) f(x) = 2 tan x; (/2, 3/2) R (ii) f(x) = ; (–, 0) R
1 x2
(iii) f(x) = x 2 + n x
Answers : (i) one-one onto (ii) one-one into (iii) one-one onto
FNT - 10
Functions & Inverse Trigonometric Functions
Equal or dentical Functions :
Two functions f and g are said to be identical (or equal) iff :
(i) The domain of f the domain of g.
(ii) f(x) = g(x), for every x belonging to their common
domain.
1 x
e.g. f(x) = and g(x) = 2 are identical functions.
x x
Clearly the graphs of f(x) and g(x) are exactly same
x2
But f(x) = x and g(x) = are not identical functions.
x
Clearly the graphs of f(x) and g(x) are different at x = 0.
x2 1
(i) f(x) = and g(x) = x + 1
x 1
(ii) f(x) = sin2x + cos2x and g(x) = sec 2x – tan2x
x
x0
(i) f(x) = sgn (x) and g(x) = | x |
0 x0
(ii) f(x) = cosec 2x – cot2x and g(x) = 1
Answers : (i) Yes (ii) No
Composite Function :
Let f: XY1 and g: Y2 Z be two functions and D is the set of values of x such that if x X, then f(x) Y2.
If D , then the function h defined on D by h(x) = g{f(x)} is called composite function of g and f and is
denoted by gof. It is also called function of a function.
Note : Domain of gof is D which is a subset of X (the domain of f ). Range of gof is a subset of the range of
g. If D = X, then f(X) Y2.
Example # 10 :Describe fog and gof wherever is possible for the following functions
(i) f(x) =x 3 , g(x) = 1 + x2 (ii) f(x) = x , g(x) = x 1.
2
FNT - 11
Functions & Inverse Trigonometric Functions
For gof(x)
Since range of f is a subset of domain of g,
domain of gof is [3, ) {equal to the domain of f }
gof (x) = g{f(x)} = g ( x 3 ) = 1 + (x+3) = x + 4. Range of gof is [1, ).
For fog(x)
since range of g is a subset of domain of f,
domain of fog is R {equal to the domain of g}
fog (x) = f{g(x)}= f(1+ x 2 ) = x 2 4 Range of fog is [2, ).
(ii) f(x) = x , g(x) = x2 1.
Domain of f is [0, ), range of f is [0, ).
Domain of g is R, range of g is [1, ).
For gof(x)
Since range of f is a subset of the domain of g,
domain of gof is [0, ) and g{f(x)}= g(x) = x 1. Range of gof is [1, )
For fog(x)
Since range of g is not a subset of the domain of f
i.e. [1, ) [0, )
fog is not defined on whole of the domain of g.
Domain of fog is {xR, the domain of g : g(x) [0, ), the domain of f}.
Thus the domain of fog is D = {xR: 0 g(x) < }
i.e. D = { xR: 0 x 2 1}= { xR: x 1 or x 1 }= (, 1] [1, )
fog (x) = f{g(x)} = f(x 21) = x 2 1 Its range is [0, ).
Example # 11 : Let f(x) = ex ; R+ R and g(x) = sin–1 x; [–1, 1] , . Find domain and range of fog(x)
2 2
Solution : Domain of f(x) : (0, ) Range of g(x) : ,
2 2
values in range of g(x) which are accepted by f(x) are 0,
2
0 < g(x) 0 < sin–1x 0<x1
2 2
Hence domain of fog(x) is x (0, 1]
1 x 0 x 1
| x 1 | 0 x 3
= = x 1 1 x 3
| x 5 | 3 x 4 5 x 3 x 4
g(x) = 4 – |2 – x| 1 x 3
4 ( 2 x ) 1 x 2 2 x 1 x 2
= =
4 ( x 2 ) 2 x 3 6 x 2 x 3
FNT - 12
Functions & Inverse Trigonometric Functions
1 (2 x ) 0 2 x 1 and 1 x 2
1 g( x ) 0 g( x ) 1 2 x 1 1 2 x 3 and 1 x 2
5 (2 x ) 3 2 x 4 and 1 x 2
g( x ) 1 1 g( x ) 3
fog (x) = =
5 g( x ) 3 g( x ) 4 1 6 x 0 6 x 1 and 2 x 3
6 x 1 1 6 x 3 and 2x3
5 6 x 3 6 x 4 and 2x3
1 x 2 x 1 and 1 x 2 1 x 2 x 1 and 1 x 2
1 x 1 x 1 and 1 x 2 1 x 1 x 1 and 1 x 2
3 x 1 x 2 and 1 x 2 3 x 1 x 2 and 1 x 2
= =
x 5 6 x 5 and 2x3 x5 5x6 and 2x3
5 x 5 x 3 and 2x3 5x 3x5 and 2x3
x 1 3 x 2 and 2x3 x 1 2x3 and 2x3
1 x 1 x 1
= 3 x 1 x 2
x 1 2 x 3
Alternate method for finding fog
2 x 1 x 2
g(x) =
6 x 2 x 3
graph of g(x) is
1 g( x ) 0 g( x ) 1
g( x ) 1 1 g( x ) 3
fog(x) =
5 g( x ) 3 g( x ) 4
1 g( x ) for no value 2 x 1 1 x 1 x 1 1 x 1
g( x ) 1 1 x 1
= = 5 (2 x ) 1 x 2 = 3 x 1 x 2
5 g( x ) 1 x 3 5 (6 x ) 2 x 3 x 1 2 x 3
Self practice problems
(7) Define fog(x) and gof(x). Also find their domain and range.
(i) f(x) = [x], g(x) = sin x (ii) f(x) = tan x, x (–/2, /2); g(x) = 1 x 2
(8) Let f(x) = ex : R + R and g(x) = x 2 – x : R R. Find domain and range of fog (x) and gof (x)
Answers :
(7) (i) gof = sin [x] domain : R range { sin a : a }
fog = [ sin x] domain : R range : {–1, 0, 1}
(ii) gof 1 tan 2 x , domain : , range : [0, 1]
4 4
fog tan 1 x 2 domain : [–1, 1] range [0, tan 1]
FNT - 13
Functions & Inverse Trigonometric Functions
(8) fog (x) gof (x)
Domain : (–, 0) (1, ) Domain : (0, )
Range : (1, ) Range : (0, )
(b) All functions (whose domain is symmetrical about origin) can be expressed as the sum of an
even and an odd function, as follows
f(x) =
(c) The only function which is defined on the entire number line and is even and odd at the same
time is f(x) = 0.
(d) If f and g both are even or both are odd, then the function f.g will be even but if any one of them
is odd and the other even then f.g will be odd.
(e) If f(x) is even then f(x) is odd while derivative of odd function is even. Note that same cannot be
said for integral of functions.
2 2
x 1 x x 1 x 1
= log = log = – log x x 2 1 = –f(x)
x2 1 x 2
x 1 x
or f(x) + f(–x) = 0
Hence f(x) is an odd function.
FNT - 14
Functions & Inverse Trigonometric Functions
Example # 14 : Show that ax +a–x is an even function.
Solution : Let f(x) = ax + a–x
Then f(–x) = a–x + a–(–x) = a–x +ax = f(x). Hence f(x) is an even function
Self practice problems
(9) Determine whether the following functions are even or odd?
e x e x 2
(i) (ii) log x 1 x
x
e e x
2
(iii) x log x x 1
Answers (i) Odd (ii) Odd (iii) Even
Periodic Functions :
A function f(x) is called periodic with a period T if there exists a real number T > 0 such that for each x
in the domain of f the numbers x – T and x + T are also in the domain of f and f(x) = f(x + T) for all x in
the domain of f(x). Graph of a periodic function with period T is repeated after every interval of 'T'.
e.g. The function sin x and cos x both are periodic over 2 and tan x is periodic over
The least positive period is called the principal or fundamental period of f(x) or simply the
period of the function.
Note : Inverse of a periodic function does not exist.
Properties of Periodic Functions :
1
(a) If f(x) has a period T, then and f( x ) also have a period T.
f( x )
T
(b) If f(x) has a period T, then f (ax + b) has a period | a | .
(c) Every constant function defined for all real x, is always periodic, with no fundamental period.
(d) If f (x) has a period T 1 and g (x) also has a period T 2 then period of f(x) ± g(x) or f(x) . g(x) or
f (x)
is L.C.M. of T 1 and T 2 provided their L.C.M. exists. However that L.C.M. (if exists) need
g( x )
f (x)
not to be fundamental period. If L.C.M. does not exists then f(x) ± g(x) or f(x) . g(x) or is
g( x )
nonperiodic.
L.C.M. of =
e.g. |sinx| has the period , | cosx | also has the period
|sinx| + |cosx| also has a period . But the fundamental period of |sinx| + |cosx| is .
2
(e) If g is a function such that gof is defined on the domain of f and f is periodic with T, then gof is
also periodic with T as one of its periods.
Example # 15 :Find period of the following functions
x x
(i) f(x) = sin + cos (ii) f(x) = {x} + sin x, where {.}denotes fractional
2 3
part function
3x x 2x
(iii) f(x) = cos x . cos 3x (iv) f(x) = sin – cos – tan
2 3 3
x x x x
Solution : (i) Period of sin is 4 while period of cos is 6 . Hence period of sin + cos
2 3 2 3
is 12
{L.C.M. of 4 and 6 is 12}
(ii) Period of sin x = 2
Period of {x} = 1
but L.C.M. of 2 and 1 is not possible as their ratio is irrational number
it is aperiodic
FNT - 15
Functions & Inverse Trigonometric Functions
(iii) f(x) = cos x . cos 3x
2
period of f(x) is L.C.M. of 2, = 2
3
2
but 2 may or may not be fundamental periodic, but fundamental period = , where
n
n N. Hence cross-checking for n = 1, 2, 3, ....we find to be fundamental period
f( + x) = (– cos x) (– cos 3x) = f(x)
2 2 4 3
(iv) Period of f(x) is L.C.M. of , , = L.C.M. of , 6 , = 12
3 / 2 1/ 3 2 / 3 3 2
Self practice problems :
(10) Find the period of following function.
x
(i) f(x) = sin x + | sin x | (ii) f(x) = 3 cos x – sin 3
2x 3x
(iii) sin – cos (iv) f(x) = sin2x + cos4x
5 7
Answers : (i) 2 (ii) 6 (iii) 70 (iv)
Inverse of a Function :
Let y = f(x) : A B be a one-one and onto function. i.e. bijection, then there will always exist bijective
function x = g(y) : B A such that if (p, q) is an element of f, (q, p) will be an element of g and the
functions f(x) and g(x) are said to be inverse of each other. g(x) is also denoted by f 1(x) and f(x) is
–1
denoted by g (x)
Note : (i) The inverse of a bijection is unique.
(ii) Inverse of an even function is not defined.
Properties of Inverse Function :
(a) The graphs of f and g are the mirror images of each other in the line y = x. For example f(x) =
ax and g(x) = log a x are inverse of each other, and their graphs are mirror images of each other
on the line y = x as shown below.
(b) Normally points of intersection of f and f–1 lie on the straight line y = x. However it must be noted
–1
that f(x) and f (x) may intersect otherwise also. e.g f(x) = 1/x
(c) In general fog(x) and gof(x) are not equal. But if f and g are inverse of each other, then gof = fog.
fog(x) and gof(x) can be equal even if f and g are not inverse of each other. e.g. f(x) = x + 1, g(x)
–1
= x + 2. However if fog(x) = gof(x) = x, then g(x) = f (x)
(d) If f and g are two bijections f : A B, g : B C, then the inverse of gof exists and
(gof) 1 = f 1 o g1.
1
(e) If f(x) and g(x) are inverse function of each other, then f(g(x)) = g( x )
FNT - 16
Functions & Inverse Trigonometric Functions
2x 3
Example # 16 :(i) Determine whether f(x) = for f : R R, is invertible or not? If so find it.
4
(ii) Let f(x) = x 2 + 2x; x –1. Draw graph of f –1(x) also find the number of solutions of the
equation, f(x) = f –1(x)
(iii) If y = f(x) = x 2 – 3x + 1, x 2. Find the value of g(1) where g is inverse of f
Solution : (i) Given function is one-one and onto, therefore it is invertible.
2x 3 4y 3 –1 4x 3
y= x= f (x) =
4 2 2
(ii)
FNT - 17
Functions & Inverse Trigonometric Functions
Inverse Trigonometry Functions
Introduction : The student may be familiar about trigonometric functions viz sin x, cos x, tan x, cosec x, sec x,
cot x with respective domains R, R, R – {(2n + 1) /2}, R – {n}, R – {(2n + 1) /2}, R – {n} and respective
ranges [–1, 1], [–1, 1], R, R – (–1, 1), R – (–1, 1), R.
Correspondingly, six inverse trigonometric functions (also called inverse circular functions) are defined.
sin–1x : The symbol sin–1x or arcsinx denotes the angle so that sin = x. As a direct meaning, sin–1x is not a
function, as it does not satisfy the requirements for a rule to become a function. But by a suitable choice
[–1, 1] as its domain and standardized set [–/2, /2] as its range, then rule sin–1 x is a single valued function.
Thus sin–1x is considered as a function with domain [–1, 1] and range [–/2, /2].
The graph of y = sin–1x is as shown below, which is obtained by taking the mirror image, of the portion of the
graph of y = sin x, from x = –/2 to x = /2, on the line y = x.
cos –1 x : By following the discussions, similar to above, we have cos–1 x or arccos x as a function with
domain [–1, 1] and range [0, ].
The graph of y = cos–1x is similarly obtained as the mirror image of the portion of the graph of y = cos x from
x = 0 to x =
tan –1 x : We get tan–1 x or arctanx as a function with domain R and range (–/2, /2).
Graph of y = tan–1x
y
/2
o x
/2
FNT - 18
Functions & Inverse Trigonometric Functions
cosec –1 x : cosec–1x or arccosec x is a function with domain R – (–1, 1) and range [–/2, /2] – {0}.
Graph of y = cosec–1x
sec –1 x : sec–1x or arcsec x is a function with domain R – (–1, 1) and range [0,] – {/2}.
Graph of y = sec–1x
1 1 1 1
Example # 17 : Find the value of tan cos tan .
2 3
1 1 1 1
Solution : tan cos tan = tan
= tan = .
2 3 3 6 6
FNT - 19
Functions & Inverse Trigonometric Functions
x2
(15) Find the domain of y = cos –1
1 x2
(16) Find the domain of y = tan–1 ( x 2 1)
Answers : (12) 1 (13) –1
(14) (– , – 3] [ – 2, – 1] [0, ) (15) R (16) (– , –1] [1, )
Property 1 : “–x”
The graphs of sin–1x, tan–1 x, cosec –1x are symmetric about origin.
Hence we get sin–1 (–x) = – sin–1x
tan–1 (–x) = – tan–1x
cosec –1 (–x) = – cosec –1x.
Also the graphs of cos –1x, sec –1x, cot–1x are symmetric about the point (0, /2). From this, we get
cos –1 (–x) = – cos –1x
sec –1 (–x) = – sec –1x
cot–1 (–x) = – cot–1x.
Property 2 : T(T –1 )
FNT - 20
Functions & Inverse Trigonometric Functions
The graph of y = cosec (cosec –1x) sec (sec –1x)
Property 3 : T –1 (T)
2n x, x [2n / 2, 2n / 2]
(i) sin–1 (sin x) =
( 2n 1) x, x [( 2n 1) / 2, ( 2n 1) / 2], n Z
Proof : If x [2n – /2, 2n + /2], then –2n + x [–/2, /2] and sin (–2n + x) = sin x.
Hence sin–1 (sin x) = –2n + x for x [2n – /2, 2n + /2].
Proof of 2nd part is left for the students.
FNT - 21
Functions & Inverse Trigonometric Functions
(iii) tan–1 (tan x) = – n + x, n – /2 < x < n + /2, n Z
Graph of y = tan–1 (tan x)
FNT - 22
Functions & Inverse Trigonometric Functions
(vii) cot–1 (cot x) = –n + x, x (n , (n + 1) ), n Z
Graph of y = cot –1 (cot x)
Remark : sin (sin–1x), cos (cos –1x), .... cot (cot–1x) are aperiodic (non periodic) functions where as
sin–1 (sin x), ..., cot–1(cot x) are periodic functions.
Property 4 : “1/x”
Property 5 : “/2”
(i) , 1 x 1
sin1 x + cos 1 x =
2
Proof : Let A = sin–1x and B = cos –1x sin A = x and cos B = x
sin A = cos B sin A = sin (/2 – B)
A = /2 – B, because A and /2 – B [–/2, /2]
A + B = /2.
Similarly, we can prove
(ii) tan1 x + cot1 x = , xR (iii) cosec1 x + sec1 x = , x 1
2 2
1 3
Example # 19 : Find the value of cosec cot cot .
4
Solution :
cot (cot–1 x) = x, x R
1 3 3
cot cot =
4 4
1 3 3
cosec cot cot 4 = cosec = 2.
4
FNT - 23
Functions & Inverse Trigonometric Functions
3
Example # 20 Find the value of tan–1 tan .
4
Solution :
tan–1 (tan x) = x if x ,
2 2
3 3 3
As , tan–1 tan
4 2 2 4 4
3 3
,
4 2 2
graph of y = tan–1 (tan x) is as :
3
from the graph we can see that if < x< ,
2 2
then tan–1 (tan x) = x –
3 3
tan–1 tan = – =–
4 4 4
Example # 21 : Find the value of sin–1 (sin7) and sin–1 (sin (–5)).
5
From the graph we can see that if 2 x , then
2
y = sin–1(sin x ) can be written as :
y = x – 2
sin–1 (sin 7) = 7 – 2
FNT - 24
Functions & Inverse Trigonometric Functions
Similarly if we have to find sin–1 (sin(–5)) then
3
– 2 < – 5 < –
2
from the graph of sin–1 (sin x), we can say that sin–1 (sin(–5)) = 2 + (–5) = 2 – 5
Note that : – 2 < 5 < – graph of cos–1 (cos x ) is as :
2
From the graph we can see that if – 2 x – then cos–1 (cosx) = x + 2
5
from the graph cos–1 cos 5 = 5 + 2 = 5
2 2 2
from (i), we get
5 3
y=– 5 y=5– .
2 2
2
Example # 23 : Find the value of tan cot 1
3
2
Solution : Let y = tan cot 1 ........(i)
3
cot–1 (–x) = – cot–1x, x R
(i) can be written as
2
y = tan cot 1
3
2
y = – tan cot 1
3
3 3
y = – tan tan 1 y=–
2 2
FNT - 25
Functions & Inverse Trigonometric Functions
3
Example # 24 : Find the value of sin tan 1 .
4
3 3 3
Solution : sin tan 1 = sin sin 1 =
4 5 5
1 1 5
Example # 25 : Find the value of tan cos
2 3
1 1 5
Solution : Let y = tan cos ..........(i)
2 3
5 5
Let cos–1 = 0, and cos =
3 2 3
(i) becomes y = tan ..........(ii)
2
5
1
1 cos 3 3 5 (3 5 ) 2
tan2 = = = =
2 1 cos 5 3 5 4
1
3
3 5
tan =± .........(iii)
2 2
0, tan >0
2 4 2
3 5
from (iii), we get y = tan = 2
2
1
Example # 26 : Find the value of cos (2cos–1x + sin–1x) when x =
5
1 1 1 1 1
Solution : cos 2 cos 1 sin 1 = cos cos 1 sin 1 cos 1
5 5 5 5 5
1 1 1
= cos cos = – sin cos 1 .........(i)
2 5 5
2
1 2 6
=– 1 =– .
5 5
1 1
Aliter : Let cos 1 = cos = and 0,
5 5 2
24
sin =
5
24
sin–1 (sin ) = sin–1 5 ..........(ii)
0, 2 sin–1 (sin ) =
FNT - 26
Functions & Inverse Trigonometric Functions
equation (ii) can be written as
24 1
= sin–1
= cos–1
5 5
1 24
cos–1 = sin–1
5 5
1 24
Now equation (i) can be written as y = – sin sin
........(iii)
5
24 1 24 24
[–1, 1] sin sin =
5 5 5
24
from equation (iii), we get y = –
5
Example # 27 : Solve sin–1 (x2 – 2x + 1) + cos–1(x2 – x) =
2
Solution : sin–1(f(x)) + cos–1(g(x)) = f(x) = g(x) and –1 f(x), g(x) 1
2
x2 – 2x + 1 = x2 – x x = 1, accepted as a solution
7
(21) Find the value of cos–1 (– cos 4) (22) Find the value of tan–1 tan
8
1 2
(23) Find the value of tan–1 cot (24) Find the value of sec cos 1
4 3
1
(25) Find the value of cosec sin1
3
1
(26) Find the value of sin (2cos–1x + sin–1x) when x =
5
(27) Solve the following equations (i) 5 tan–1x + 3 cot–1x = 2 (ii) 4 sin–1x = – cos–1x
1 41 16
(28) Evaluate tan cos ec (29) Evaluate sec cot 1
4 63
1 1 3 1
(30) Evaluate sin cot (31) Evaluate tan 2 tan 1
2 4 5 4
(32) Solve sin–1(x2 – 2x + 3) + cos–1(x2 – x) =
2
FNT - 27
Functions & Inverse Trigonometric Functions
3
Answers : (17) (18) not defined
2
(19) 13 – 4 (20) 3 – , – 2, – 3, – 2
1
(21) 4– (22) (23)
8 4 2
3 1
(24) (25) – 3 (26)
2 5
1
(27). (i) x=1 (ii) x=
2
4 65 2 5 7
(28) (29) (30) (31)
5 16 5 17
(32) No solution
sin (A + B) = x 1 y 2 + y 1 x 2
1 2 2
cos xy 1 x 1 y ; 0 x y 1
(iv) cos–1x – cos–1y =
cos 1 xy 1 x 2 1 y 2 ; 0 y x 1
/2 if x, y 0 & xy 1
/2 if x, y 0 & xy 1
1 x y
tan if x, y 0 & xy 1
1 xy
(v) tan x + tan y =
–1 –1
1 x y
tan if x, y 0 & xy 1
1 xy
xy
(vi) tan–1x – tan–1y = tan–1 1 xy , x 0, y 0
FNT - 28
Functions & Inverse Trigonometric Functions
Notes :(i) x2 + y2 1 & x, y 0 0 sin1 x + sin1 y
2
and x 2 + y2 1 & x, y 0 sin1 x + sin1 y
2
(ii) xy < 1 and x, y 0 0 tan1 x + tan1 y < ; xy > 1 and x, y 0 < tan1 x + tan1 y <
2 2
(iii) For x < 0 or y < 0 these identities can be used with the help of property “– x”
i.e. change x or y to x or y which are positive .
3 15 84
Example # 28 : Show that sin–1 + sin–1 = – sin–1
5 17 85
2 2
3 15 3 15 8226
Solution : > 0, > 0 and + = >1
5 17 5 17 7225
3 15 3 225 15 9
1 1
sin–1 + sin–1 = – sin–1 5 289 17 25
5 17
3 8 15 4 84
= – sin–1 . . = – sin–1
5 17 17 5 85
12 4 63
Example # 29 : Evaluate cos–1 + sin–1 – tan–1
13 5 16
12 4 63
Solution : Let z = cos–1 + sin–1 – tan–1
13 5 16
4 4
sin–1 = – cos–1
5 2 5
12 1 4 63
z = cos–1 + cos – tan–1 .
13 2 5 16
1 4 12 63
z= – cos cos 1 – tan–1 .........(i)
2 5 13 16
4 12 4 12
> 0, > 0 and <
5 13 5 13
4 12 4 12 16 144 63
cos–1 – cos–1 = cos–1 5 13 1 25 1 –1
169 = cos 65
5 13
equation (i) can be written as
63 63
z= – cos–1 – tan–1
2 65 16
63 63
z = sin–1 – tan–1 .........(ii)
65 16
63 63
sin–1 = tan–1
65 16
from equation (ii), we get
63 63
z = tan–1 – tan–1 z=0
16 16
FNT - 29
Functions & Inverse Trigonometric Functions
5
Example # 30 : Evaluate tan–1 9 + tan–1
4
5 5
Solution : 9 > 0, > 0 and 9 > 1
4 4
5
5 9
4
tan–1 9 + tan–1 = + tan–1 = + tan–1 (– 1) = – = .
4 5 4
1 9.
4
Example # 31 : Define y = cos–1 (4x3 – 3x) in terms of cos–1 x and also draw its graph.
Solution : Part - 1: Let y = cos–1 (4x3 – 3x)
Domain : [–1, 1] and range : [0, ]
Let cos–1 x = [0, ] and x = cos
y = cos–1 (4 cos3 – 3 cos )
y = cos–1 (cos 3) ...........(i)
FNT - 30
Functions & Inverse Trigonometric Functions
1
y = – 2 + 3 cos–1 x if – 1 x –
2
from (i), (ii) & (iii), we get
1 1
3 cos x ; x 1
2
1 1
y = cos–1 (4x3 – 3x) = 2 3 cos 1 x ; x
2 2
2 3 cos 1 x ; 1 x 1
2
dy 1 1
<0 if x , 1 decreasing if x , 1
dx 2 2
again if we differentiate equation (i) w.r.t. ‘x’, we get
d2 y 3x d2 y 1 1
=– <0 if x , 1 concavity downwards if x , 1
dx 2 (1 x 2 )3 / 2 dx 2 2 2
1 1
(ii) if – x < , y = 2 – 3cos–1 x.
2 2
dy 3 dy
if x ,
1 1
= >0
dx 1 x 2 dx 2 2
1 1 d2 y 3x
increasing if x , and =
2 2 dx 2 (1 x 2 )3 / 2
1 d2 y 1
(a) if x , 0 then <0 concavity downwards if x , 0
2 dx 2 2
1 d2 y
(b) if x 0, then >0 concavity upwards if x 0, 1
2 dx 2 2
1 dy d2 y
(iii) Similarly if – 1 x < – then < 0 and > 0.
2 dx dx 2
the graph of y = cos–1 (4x3 – 3x) is as
FNT - 31
Functions & Inverse Trigonometric Functions
Self practice problems:
4 5 16
(33) Evaluate sin–1 + sin–1 + sin–1
5 13 65
(34) If tan 4 + tan 5 = cot , then find ‘’
–1 –1 –1
3 16 1 7
(35) Prove that 2 cos–1 + cot–1 + cos–1 =
13 63 2 25
(36) Solve the equation tan–1 (2x) + tan–1 (3x) =
4
2
(37) Solve the equation sin–1x + sin–1 2x =
3
(38) Define y = sin–1 (3x – 4x3) in terms of sin–1x and also draw its graph.
3x x 3
(39) Define y = tan–1 in terms of tan–1 x and also draw its graph.
1 3x 2
19 1 1
Answers. (33) (34) =– (36) x= (37) x=
2 9 6 2
1 1 1
3 sin x ; x
2 2
1
1
(38) y = sin–1 (3x – 4x3) = 3 sin x ; x 1
2
3 sin 1 x ; 1
1 x
2
1 1 1
3 tan x ; x
3 3
3x x 3
1 1
(39) y = tan 1 3 x 2 = 3 tan x ; x
–1
3
1 1
3 tan x ; x
3
3x x 3
Fig.: Graph of y = tan –1
1 3x 2
FNT - 32
Functions & Inverse Trigonometric Functions
Property 7 : Miscellaneous identities
2 sin x
1
if |x| 1
2
(i)
sin 2 x 1 x 2
1
1
= 2 sin x if 1 x 1
2
2 sin 1 x
if 1 x 1
2
graph of y = sin1 2 x 1 x 2
2 cos 1 x if 0 x 1
(ii) cos1 (2 x 2 1) = 1
2 2 cos x if 1 x 0
graph of y = cos 1 (2 x 2 1)
2 tan 1x if | x | 1
2x 1
(iii) tan 1
= 2 tan x if x 1
1 x2
2 tan 1x
if x 1
2x
graph of y = tan1
1 x2
2 tan 1 x if | x | 1
2x
(iv) sin1
= 2 tan 1 x if x 1
1 x2
2 tan 1 x if x 1
2x
graph of y = sin1
1 x2
FNT - 33
Functions & Inverse Trigonometric Functions
1 x2 2 tan 1x if x 0
(v) cos1 = 1
1 x 2 2 tan x if x 0
1 x2
graph of y = cos 1
1 x 2
(vi) If tan1 x + tan1 y + tan1 z = , then x + y + z = xyz
(vii) If tan1 x + tan1 y + tan1 z = , then xy + yz + zx = 1
2
1 1
(ix) tan1 1 + tan1 + tan1 =
2 3 2
FNT - 34
Limits, Continuity & Derivability
Calculus required continuity, and continuity was supposed to require the infinitely little; But nobody could discover what the infinitely little
might be............Russell, Bertrand
(E) im f (g(x)) = f im g ( x ) = f (m); provided f is continuous at g (x) = m.
x a
x a
Solution : (i) im (x + 2) = 2 + 2 = 4
x + 2 being a polynomial in x, its limit as x 2 is given by x 2
Indeterminate forms :
0
If on putting x = a in f(x), any one of, , 0 , º, 0º,1form is obtained, then the limit
0
has an indeterminate form. All the above forms are interchangeable, i.e. we can change one form to
other by suitable substitutions etc.
im f(x) may exist.
In such cases x a
LCD - 35
Limits, Continuity & Derivability
x2 4 im x 2 – 4 = 0 and im x – 2 = 0
Consider f(x) = . Here x x 2
x2 2
NOTE :
(i) + =
(ii) x =
a
(iii) = 0, if a is finite.
a
(iv) is not defined for any a R.
0
2
(v) im x is an indeterminate form whereas im [ x ] is not an indeterminate form (where [.]
x 0 x x 0
x2
represents greatest integer function)
Students may remember these forms alongwith the prefix ‘tending to’
To evaluate a limit, we must always put the value where ' x ' is approaching to in the function. If we get
a determinate form, then that value becomes the limit otherwise if an indeterminate form comes, we
have to remove the indeterminancy, once the indeterminancy is removed the limit can be evaluated by
putting the value of x, where it is approaching.
Factorisation method :-
We can cancel out the factors which are leading to indeterminancy and find the limit of the remaining
expression.
x 2 2x 3 1 2 (2 x 3)
Example # 2 : (i) im (ii) im 3
x 3 2
x 4x 3 x 2 x 2 x 3 x 2 2x
x 2 2x 3 ( x 3)( x 1)
Solution : (i) im im
= x =2
x 3 2
x 4x 3 3 ( x 3)( x 1)
LCD - 36
Limits, Continuity & Derivability
1 2 (2 x 3) 1 2 (2 x 3 )
(ii) im 3 im
= x
x 2 x 2 x 3 x 2 2x 2 x 2 x ( x 1)( x 2)
x( x 1) 2( 2 x 3 )
im
= x
2 x( x 1)( x 2)
x 2 5x 6
im
= x 2 x( x 1)( x 2)
( x 2)( x 3) x3 1
im
= x im
= x =–
2
x( x 1)( x 2) 2
x( x 1) 2
Rationalisation method :-
We can rationalise the irrational expression in numerator or denominator or in both to remove the
indeterminancy.
Example # 3 : Evaluate :
4 15x 1 1 x 1 x
(i) im (ii) im
x 1 2 3x 1 x 0
x
(15 15x) 2 3x 1 5
= xim × =
1 (3 3x) 4 15x 1 2
0
(ii) The form of the given limit is when x 0. Rationalising the numerator, we get
0
1 x 1 x 1 x 1 x
im 1 x 1 x im
= x x
x 0
x 0
1 x 1 x
(1 x ) (1 x ) 2x
im
= x = im
0 x ( 1 x 1 x ) x 0
x ( 1 x 1 x )
2 2
im
= x = =1
0
1 x 1 x 2
Self practice problems
Evaluate the following limits : -
(3)
im (2x 3) x 1 (4)
im 1 (sin x )1/ 3
x
x 1 2x 2 x 3 2 1 (sin x )2 / 3
LCD - 37
Limits, Continuity & Derivability
Standard limits :
(a) (i) im sinx = im tanx = 1 [ Where x is measured in radians ]
x 0 x 0
x x
1 1
(ii) im tan x = im sin x = 1
x 0 x 0
x x
1
x 0 0 (1 ax ) x e a
x
im 1 1 = e ;
x
im
tan f ( x )
im ef(x) 1
(iii) =1 (iv) =1
x a f ( x) x a f(x)
f(x)
1 n(1 f ( x ))
(v) im b = n b, (b > 0) (vi) im =1
x a f (x) x a f ( x)
1
(vii) im (1 f ( x )) f ( x ) e
x a
(c) im f(x) = A > 0 and im (x) = B(a finite quantity), then im [f(x)](x) = AB.
x a x a x a
n
im (1 x ) 1
Example # 4 : Evaluate : x 0
x
n
im (1 x ) 1 = im (1 x ) 1 = n
n
Solution : x 0 x 0 (1 x ) 1
x
3x
im e 1
Example # 5 : Evaluate : x 0
x/2
3x 3x
Solution : im e 1 = im 2 × 3 e 1 = 6.
x 0 x 0 3x
x/2
im tan x sin x
Example # 6 : Evaluate : x 0 x3
x
tan x . 2 sin2
Solution : im tan x sin x = im tan x(1 cos x ) = im 2
x 0 x3 x 0 x3 x 0
x3
2
x
sin
1 tan x 2 1
im
= x . . x = .
0 2 x 2
2
LCD - 38
Limits, Continuity & Derivability
im sin 2x
Example # 7 : Evaluate : x 0 sin 3 x
sin 2x 2x 3x sin 2x 2 3x
Solution : im sin 2x im
= x . . = 2xim . . im
x 0 sin 3 x 0 2x 3 x sin 3 x 0 2 x 3 3 x 0 sin 3 x
2 sin 3 x 2 2
=1. × im = ×1=
3 3 x 0 3 x 3 3
x
im 1 2
Example # 8 : Evaluate : x x
x2
Solution : im 1 2 = xim x .x = e2.
x e
x
x 3 x
Example # 9 : Evaluate : (i) im e e (ii) im x(e 1)
x 3 x3 x 0 1 cos x
im e3 . e y e3 ey 1
3 im
= y 0 = e y 0 = e3 . 1 = e3
y y
x ex 1 x2
im x(e 1)
x
(ii) im x(e 1) = 1 . im
= x . = 2.
x 0 1 cos x 0 x 2 x 0 x x
2 sin 2 sin 2
2 2
im 8 x2 x2 x2 x2
(9) 1 – cos – cos cos cos
x 0 2 4 2 4
x8
im 1 sin 2 x
(10) x
4 4x
x x
(11) im 5 9
x 0
x
1 5
Answers : (8) 2 (9) (10) does not exist (11) n
32 9
(12) b
LCD - 39
Limits, Continuity & Derivability
Use of substitution in solving limit problems
Sometimes in solving limit problem we convert xim
a
f(x) into him
0
f(a + h) or him
0 f(a – h) according as
im 1 tan x
Example # 10 : Evaluate x
1 2 sin x
4
Solution : Put x= +h
4
x h0
4
2 tan h
1 tan h 1 tan h
4 1 1 tan h
im im 1 tan h im
h0 1 2 sin = h0 1 sin h cos h
= h0 h h h
h 2 sin2 2 sin cos
4 2 2 2
tanh
2
2 tan h 1 h 1 2
im
= h im
= h = = 2.
0 h h h (1 tanh) 0 h (1 tanh) 1
2 sin sin cos sin
2 2 2 2 sin h cos h
h 2 2
2
Limits using expansion
x n a x 2 n 2 a x 3 n 3 a
(a) ax 1 ........., a 0
1! 2! 3!
x x2 x3
(b) e x 1 ......
1! 2! 3!
x2 x3 x4
(c) n (1+x) = x ........., for 1 x 1
2 3 4
x3 x5 x7
(d) sin x x .....
3! 5! 7!
x2 x 4 x6
(e) cosx 1 .....
2! 4! 6!
x 3 2x 5
(f) tan x = x ......
3 15
x3 x5 x7
(g) tan-1x = x ....
3 5 7
x 2 5 x 4 61x 6
(i) sec -1x = 1 ......
2! 4! 6!
n(n 1) n(n 1)(n 2)
(j) for |x| < 1, n R; (1 + x) n = 1 + nx + 1. 2 x 2
+ 1. 2 . 3 x3 + ............
1 x 11 2
(k) (1 x ) x = e 1 2 24 x .......... ...
LCD - 40
Limits, Continuity & Derivability
x
im e 1 x
Example # 11 : Evaluate x 0
x2
x2
x
1 x ...... 1 x
Solution : im e 1 x = im 2! =
1
x0 2 x0
x x 2
2
im tan x sin x
Example # 12 : Evaluate x 0
x3
x3 x3
x ....... x ......
Solution : im tan x sin x = im 3 3! = 1 + 1 = 1.
x0 x0
x3 x 3 3 6 2
7 x ) 3 2
Example # 13 : Evaluate xim
1
x 1
Solution : Put x = 1 + h
1
im (8 h) 2
3
h0
h
1 1 h
2
1
1 h 3 3 8
1 2 1 . ....... 1
h 3
3 8 1 . 2
2 . 1 2
im 8 im im 2 × 1 = 1
= h 0 = h 0 = h 0
h h 24 12
x2
n(1 x ) sin x
im
Example # 14 : Evaluate x 2
0
x tan x sin x
2 3 3 5 2
x2 x x x ..... x x x ..... x
n(1 x ) sin x 2 3 3! 5! 2 1 1
im 2 = im 1
Solution : x0 x0 = + =
x tan x sin x tan x sin x 3 6 2
x3 . .
x x
1
im e (1 x )
x
Example # 15 : Evaluate x 0
tan x
x
1
e e 1 ......
Solution : im e (1 x ) = im
x
2 = im e × x = e
x0 x0 x0 tan x
tan x tan x 2 2
ae x b cos x ce x
Example # 16 : Find the values of a,b and c so that im =2
x 0 x sin x
ae x b cos x ce x
Solution : im =2 .....(1)
x 0 x sin x
at x 0 numerator must be equal to zero
a–b+c=0 b=a+c .....(2)
LCD - 41
Limits, Continuity & Derivability
ae x (a c ) cos x ce x
From (1) & (2), im =2
x 0 x sin x
x x2 x3 x2 x4 2 3
a 1 ...... – (a c ) 1 ...... c 1 x x x ......
1! 2 ! 3 !
im 1 ! 2 ! 3 !
x 2! 4! =2
0
x 3
x 5
x x .....
3 ! 5!
(a c ) x
(a c ) (a c ) .....
im x 3 !
x 0
=2
2 4
1 x x ......
3! 5!
Since R.H.S is finite,
0 2a 0 ....
a–c=0 a = c, then =2
1
a = 1 then c = 1
From (2), b=a+c=1+1=2
Limit when x
In these types of problems we usually cancel out the greatest power of x common in numerator and
1
denominator both. Also sometime when x , we use to substitute y = and in this case y 0+.
x
im x sin 1
Example # 17 : Evaluate x
x
1
sin
im 1 im x
Solution : x x sin = x =1
x 1
x
im x2
Example # 18 : Evaluate x 2x 3
2
1
im x 2 im x 1
Solution : x 2x 3 x 3
= .
2 2
x
im x 2 4x 5
Example # 19 : Evaluate x
3x 2 x 3 2
1 4 5
2 3
im x 2 4x 5 im x x x
Solution : x = x 3 2 =0
3x 2 x 3 2 1 3
x x
LCD - 42
Limits, Continuity & Derivability
im 3x 2 2
Example # 20 : Evaluate x
x2
im 3x 2 2 1
Solution : x (Put x = – , x – t 0 +)
x2 t
1
3 2t 2 .
t2 3 2t 2 t 3
= tim
0 ( 1 2 t ) = tim
0 = =– 3.
(1 2t ) | t | 1
t
Some important notes :
n
x
(i) im nx = 0 (ii) im =0 (iii) im x = 0
x x x x e x
x e
im x1000
Example # 21 : Evaluate x
ex
1000
Solution : im x =0
x ex
Limits of form 1 , 0 0 , 0
0
(A) All these forms can be converted into form in the following ways
0
(a) If x 1, y , then z = (x) y is of 1 form
n z = y n x
nx 0
n z = form
1 0
y
1
As y y 0 and x 1 nx 0
LCD - 43
Limits, Continuity & Derivability
(B) (1) type of problems can be solved by the following method
1
(a) im (1 x ) x = e
x 0
( f ( x )1) g( x )
1 1
{ f ( x ) 1} . g( x )
im [ f ( x ) 1] g( x )
im 1 f ( x ) 1
= x
f ( x ) 1
im
= x a [1 ( f ( x ) 1)] )1
f ( x = e xa
a
4 x 2
2x 2 1
im
Example # 22 : Evaluate x
2x 2 3
Solution : Since it is in the form of 1
4 x 2
2x 2 1 2x 2 1 2x 2 3
im = exp im ( 4 x 2 2) = e–8
x 2x 2 3 x
2x 2 3
4
tan / 4
2
1(1 tan / 4 ) 1
= e = e–1 =
e
x
tan
im 2 a
Example # 24 : Evaluate x
2a
.
a x
x
tan
im 2 a
2a
Solution : x a
put x=a+h
x
h h h h
tan cot im cot . 1 1
im 1 h
h
2 2a im 1 h
h
2a
= e
h0 2a a h
0 0
( a h) ah
h 2a
im 2a .
h0 h ah 2
tan –
e 2a = e
im x x
Example # 25 : Evaluate x0
Solution : im xx
Let y = x0
1
n 1
n y im x n x = im –
= x0 x = 0, as
x 0 1 x
x
y=1
LCD - 44
Limits, Continuity & Derivability
Sandwitch theorem or squeeze play theorem:
Suppose that f(x) g(x) h(x) for all x in some open interval containing a, h
except possibly at x = a itself. Suppose also that g
im f(x) = = im h(x), f
x a x a
im g(x) = .
Then x a
x im [ x ] [2x ] .... [nx ] x
< n
2 n2 2
[r x]
r 1
= [x] + [2x] + .... + [nx]
= (x + 2x + 3x + ... + nx) – ({x} + {2x} + .... + {nx})
xn(n 1)
= – ({x} + {2x} + .. + {nx})
2
n
n
1
2
[r x] =
r 1
x 1 1
2 n
–
{ x } {2x } .... {nx }
n2
n
im
{rx}
n
r 1 =0
n2
n n
{rx}
n
[rx]
[rx]
im r 1 im x 1 1 – im
= n
r 1
im r 1 =
x
n n n
n 2 2 n n2 2
n2
LCD - 45
Limits, Continuity & Derivability
Continuity & Derivability :
A function f(x) is said to be continuous at x = c, if Limit f(x) = f(c)
x c
i.e. f is continuous at x = c
if Limit f(c
h 0
h) = Limit
h 0
f(c+h) = f(c).
If a function f (x) is continuous at x = c, the graph of f (x) at the corresponding point ( c, f (c)) will not be
broken. But if f (x) is discontinuous at x = c, the graph will be broken when x = c
(i) Limit f(x) does not exist i.e. Limit f(x) Limit f (x) [figure (i)]
x c x c x c
x
sin , x 1
Example # 27 :If f(x) = 2 , then find whether f(x) is continuous or not at x = 1, where
[ x] , x 1
[ . ] is greatest integer function.
x
Solution : f(x) = sin 2 , x 1
[ x] , x 1
For continuity at x = 1, we determine f(1), xlim
1–
f(x) and xlim
1
f(x).
Now, f(1) = [1] = 1
so f(1) = xlim
1–
f(x) = xlim
1
f(x) = 1 f(x) is continuous at x = 1
Self practice problems :
(13) If possible find value of for which f(x) is continuous at x =
2
1 sin x
, x
1 cos 2 x 2
f(x) = , x
2
2x
, x
4 2x 2 2
LCD - 46
Limits, Continuity & Derivability
(14) Find the values of a and b such that the function
x a 2 sin x ; 0x
4
f ( x ) 2x cot x b ; x is continuous at x = and x =
4 2 4 2
a cos 2x b sin x ;
x
2
(1 ax )1/ x ; x0
b ; x 0 , then find the values of a, b, c, for which f(x) is continuous at x = 0
(15) If f ( x )
1/ 3
(x c) 1
; x0
x
Answers : (13) discontinuous (14) a= ,b=
6 12
1
(15) a = –n 3, b = ,c=1
3
Theorems on continuity :
(i) If f & g are two functions which are continuous at x = c, then the functions defined by:
F1(x) = f(x) g(x) ; F2(x) = K f(x), K is any real number ; F3(x) = f(x).g(x) are also continuous at
f (x)
x = c. Further, if g (c) is not zero, then F4(x) = g( x ) is also continuous at x = c.
(ii) If f(x) is continuous & g(x) is discontinuous at x = a, then the product function
(x) = f(x). g(x) may or may not be continuous but sum or difference function (x) = f(x) g(x)
will necessarily be discontinuous at x = a.
sin x 0
x
e.g. f (x) = x & g(x) =
0 x0
(iii) If f (x) and g(x) both are discontinuous at x = a, then the product function (x) = f(x). g(x) is not
necessarily be discontinuous at x = a.
1 , x0
e.g. f (x) = g(x) =
1 , x 0
and atmost one out of f(x) + g(x) and f(x) – g(x) is continuous at x = a.
Example # 28 : If f(x) = [sin(x–1)] – {sin(x–1)}. Comment on continuity of f(x) at x = 1
2
(where [ . ] denotes G.I.F. and { . } denotes fractional part function).
LCD - 47
Limits, Continuity & Derivability
Continuity of composite functions :
1
1 x 3 , x 0 ( x 1) 3 , x0
(16) f ( x) 2 and g( x ) 1
x 1 , x 0
( x 1) 2 , x0
Then define fog (x) and comment on the continuity of gof(x) at x = 1
Answer : [fog(x) = x, x R and gof(x) is discontinous at x = 1]
Continuity in an Interval :
(a) A function f is said to be continuous in (a, b) if f is continuous at each & every point (a, b).
(b) A function f is said to be continuous in a closed interval [ a, b ] if:
(i) f is continuous in the open interval (a, b),
(c) All Polynomial functions, Trigonometrical functions, Exponential and Logarithmic functions are
continuous at every point of their respective domains.
On the basis of above facts continuity of a function should be checked at the following
points
(i) Continuity of a function should be checked at the points where definition of a function
changes.
(ii) Continuity of {f(x)} and [f(x)] should be checked at all points where f(x) becomes integer.
(iii) Continuity of sgn (f(x)) should be checked at the points where f(x) = 0 (if f(x) = 0 in any
open interval containing a, then x = a is not a point of discontinuity)
(iv) In case of composite function f(g(x)) continuity should be checked at all possible points
of discontinuity of g(x) and at the points where g(x) = c, where x = c is a possible point
of discontinuity of f(x).
[sin x ] ; 0 x 1
Example # 29: If f(x) = x 2 · sgn x 5 ; 1 x 2 , where { . } represents fractional part function and
3
4
[ . ] is greatest integer function, then comment on the continuity of function in the interval [0, 2].
Solution : (i) Continuity should be checked at the end-points of intervals of each definition i.e. x = 0,
1, 2
(ii) For [sin x], continuity should be checked at all values of x at which sin x
1
i.e. x = 0,
2
2 5 5
(iii) For x · sgn x , continuity should be checked when x – =0
3 4 4
LCD - 48
Limits, Continuity & Derivability
5 2
(as sgn (x) is discontinuous at x = 0) i.e. x = and when x –
4 3
5
i.e. x= (as {x} is discontinuous when x )
3
1 5 5
overall discontinuity should be checked at x = 0, , 1, , and 2
2 4 3
check the discontinuity your self.
1 5 5
discontinuous at x = ,1 ,
2 4 3
x 1 1
Example # 30 : If f(x) = and g(x) = , then discuss the continuity of f(x), g(x) and fog (x).
x 1 x2
x 1
Solution : f(x) =
x 1
f(x) is a rational function it must be continuous in its domain
and f is not defined at x = 1
f is discontinuous at x = 1
1
g(x) =
x2
g(x) is also a rational function. It must be continuous in its domain and g is not defined at
x=2
g is discontinuous at x = 2
Now fog (x) will be discontinuous at
(i) x=2 (point of discontinuity of g(x))
(ii) g(x) = 1 (when g(x) = point of discontinuity of f(x))
1
if g(x) = 1 =1 x=3
x2
discontinuity of fog(x) should be checked at x = 2 and x = 3
at x = 2
1
1
x2
fog (x) = 1
1
x2
fog (2) is not defined
1
1
lim fog (x) = lim x2 1 x 2
x 2 x 2 1 = xlim
2 1 x 2
=1
1
x2
fog (x) is discontinuous at x = 2 and it is removable discontinuity at x = 2
fog (3) = not defined
1
1
lim fog (x) = lim x2
x 3 x 3 1 =
1
x2
1
1
lim fog (x) = lim x2
x 3 x 3 1 = –
1
x2
fog (x) i s discontinuous at x = 3 and it is non removable discontinuity of nd kind.
LCD - 49
Limits, Continuity & Derivability
Self practice problem :
1
[n x ] . sgn x ; 1 x 3
(17) If f(x) = 2 . Find the pointswhere the continuity of f(x),
{ x 2 } ; 3 x 3.5
should be checked, where [ . ] is greatest integer function and {.} fractional part function.
3 5
Answer : { 1, , , e, 3 , 10 , 11 , 12 , 3.5 }
2 2
Intermediate value theorem :
A function f which is continuous in a , b possesses the following properties:
(i) If f(a) & f(b) possess opposite signs, then there exists at least one solution of the equation
f(x) = 0 in the open interval (a, b).
(ii) If K is any real number between f(a) & f(b), then there exists at least one solution of the
equation f(x) = K in the open interval (a, b).
Example # 31 : Given that a > b > c > d, then prove that the equation (x – a) (x – c) + 2(x – b) (x – d) = 0
will have real and distinct roots.
Solution : (x – a) (x – c) + 2 (x – b) (x – d) = 0
f(x) = (x – a) (x – c) + 2 (x – b) (x – d)
f(a) = (a – a) (a – c) + 2 (a – b) (a – d) = + ve
f(b) = (b – a) (b – c) + 0 = –ve
f(c) = 0 + 2 (c – b) (c – d) = –ve
f(d) = (d – a) (d – c) + 0 = +ve
hence (x – a) (x – c) + 2(x – b) (x – d) = 0
have real and distinct roots
1
Example # 32:Let f(x) = Lim , then find f and also comment on the continuity at x = 0
2
n 1 n sin x 4
1
Solution : Let f(x) = nlim
1 n sin2 x
1 1
f = nlim = nlim =0
4 2
1
1 n . sin 1 n
Now 4 2
1 1
f(0) = nlim 2 = =1
n . sin (0) 1 1 0
1
lim f(x) = lim lim =0
x 0 x 0 n 1 n sin 2 x
{here sin2x is very small quantity but not zero and very small quantity when multiplied with
becomes }
f(x) is not continuous at x = 0
LCD - 50
Limits, Continuity & Derivability
Example # 33 : f(x) = maximum (sin t, 0 t x), 0 x 2discuss the continuity of this function at x=
2
Solution : f(x) = maximum (sin t, 0 t x), 0 x 2
if x 0, , sin t is increasing function
2
Hence if t [0, x], sin t will attain its maximum value at t = x.
f(x) = sin x if x 0,
2
if x , 2 and t [0, x]
2
then sin t will attain its maximum value when t =
2
sin x , if x 0,
2
f(x) = sin = 1 if x , 2 f(x) =
2 2
1 , if x , 2
2
Now f =1
2
lim lim
x
f(x) = x sin x = 1
2 2
lim lim
x
f(x) = x
1=1
2 2
as f(/2) = L.H.S. = R.H.S. f(x) is continuous at x =
2
Differentiability of a function at a point :
(i) The right hand derivative of f (x) at x = a denoted by f (a+) is defined by:
f(a h)f(a)
R.H.D. = f (a+) = Limit
h 0 , provided the limit exists.
h
(ii) The left hand derivative of f(x) at x = a denoted by
f (a–) is defined by:
f(a h)f(a )
L.H.D. = f (a– ) = Limit
h 0 , provided the limit exists.
h
A function f(x) is said to be differentiable at x = a if f (a+) = f (a–) = finite
f(a h)f(a)
By definition f (a) = Limit
h 0
h
x , x 1
Example # 34 : Comment on the differentiability of f(x) = 2 at x = 1.
x , x 1
f (1 h) f (1)
Solution : R.H.D. = f (1+) = Limit
h 0
h
(1 h)2 1 1 h2 2h 1
= Limit
h 0 = Limit
h 0 = Limit
h 0 (h + 2) = 2
h h
f (1 h) f (1) 1 h 1
L.H.D. = f(1–)= Limit
h 0 = Limit
h 0 =1
h h
As L.H.D. R.H.D. Hence f(x) is not differentiable at x = 1.
LCD - 51
Limits, Continuity & Derivability
A Bx 2 , x 1
Example # 35 : If f(x) = , then find A and B so that f(x) become differentiable at x = 1.
3 Ax – b 2 , x 1
f (1 h) f (1) 3 A (1 h) B 2 3 A B 2 3 Ah
Solution : f(1+) = hlim
0
= hlim
0 = hlim
0 = 3A
h h h
2
f (1 h) f (1) lim A B(1 h) 3A B 2
f(1–) = hlim =
0 h h0 h
( 2A 2B 2) Bh 2 2Bh
= hlim
0 h
hence for this limit to be defined
– 2A + 2B – 2 = 0
B=A+1
f(1–) = hlim
0 – (Bh – 2B) = 2B
[cos x ], x 1
Example # 36 : If f(x) = , then comment on the derivability at x = 1,
2{ x } 1, x 1
where [ . ] is greatest integer function and {.} is fractional part function.
f (1 h) f (1) [cos( h)] 1 1 1
Solution : f(1–) = h lim
0 = h lim
0 = h lim
0 =0
h h h
f (1 h) f (1) 2{1 h} 1 1 2h
f(1+) = h lim
0 = h lim
0 = h lim
0 =2
h h h
f(1+) f(1–)
f(x) is not differentiable at x = 1.
x tan 1 1/ x, x 0
(21) If f(x) = , then comment on the derivability of f(x) at x = 0.
0 , x0
Answers : (20) Discontinuous and non-differentiable at x = 1
(21) non-differentiable at x = 0
f (a h ) f ( a )
slope of the line joining (a,f(a)) and (a + h, f(a + h)) =
h
f (a h ) f (a )
Slope of tangent at P = f(a) = Lim
h 0 h
LCD - 52
Limits, Continuity & Derivability
The tangent to the graph of a continuous function f at the point P(a, f(a)) is
(i) the line through P with slope f(a) if f(a) exists ;
(ii) the line x = a if L.H.D. and R.H.D. both are either or – .
If neither (i) nor (ii) holds then the graph of f does not have a tangent at the point P.
In case (i) the equation of tangent is y – f(a) = f(a) (x – a).
In case (ii) it is x = a
Note : (i) tangent is also defined as the line joining two infinitesimally close points on a curve.
(ii) A function is said to be derivable at x = a if there exist a tangent of finite slope at that point.
f(a+) = f(a–) = finite value
(iii) y = x 3 has x-axis as tangent at origin.
(iv) y = |x| does not have tangent at x = 0 as L.H.D. R.H.D.
Example # 37 : Find the equation of tangent to y = (x) 1/3 at x = 1 and x = 0.
f (1 h) f (1) (1 h)1/ 3 1 1
R.H.D. = f(1+) = h lim
0 = h lim
0 =
h h 3
1
As R.H.D. = L.H.D. =
3
1 1
slope of tangent = y – f(1) = (x – 1)
3 3
1
y–1= (x – 1)
3
3y – x = 2 is tangent to y = x1/3at (1, 1)
At x = 0
(0 h)1/ 3 0
L.H.D. = f(0–) = h lim
0 =+
h
(0 h)1/ 3 0
R.H.D. = f(0+) = hlim
0 =+
h
As L.H.D. and R.H.D are infinite.y = f(x) will have a vertical tangent at origin.
x = 0 is the tangent to y = x 1/3 at origin.
Self Practice Problems :
(22) If possible find the equation of tangent to the following curves at the given points.
(i) y = x 3 + 3x 2 + 28x +1 at x = 0.
(ii) y = (x – 8) 2/3 at x = 8.
Answers : (i) y = 28x + 1 (ii) x=8
Note : The converse of the above result is not true i.e. "If 'f' is continuous at x = a, then 'f' is differentiable at
x = a is not true.
e.g. the functions f(x) = x 2 is continuous at x = 2 but not differentiable at x = 2.
LCD - 53
Limits, Continuity & Derivability
Case -
If = m = some finite value, then the function f(x) is differentiable as well as continuous.
Case -
if m = but both have some finite value, then the function f(x) is non differentiable but it is continuous.
Case -
If at least one of the or m is infinite, then the function is non differentiable but we can not say about
continuity of f(x).
lim f (a + h) = f(a).
h0
Similarly hlim
0
[f(a – h)– f(a)] = – h lim f(a – h) = f(a)
h0
x 2 sgn[ x] { x } , 0 x 2
Example # 39 : If f(x) = , comment on the continuity and differentiability of f(x),
sin x | x 3 | , 2 x 4
where [ . ] is greatest integer function and {.} is fractional part function, at x = 1, 2.
Solution : Continuity at x = 1
lim f(x) = lim (x 2 sgn[x] + {x}) = 1 + 0 = 1
x 1 x 1
=1 sgn (0) + 1 = 1
f(1) = 1
L.H.L = R.H.L = f(1). Hence f(x) is continuous at x = 1.
Now for differentiability,
f (1 h) f (1)
R.H.D. = f(1+) = hlim
0 h
(1 h)2 sgn[1 h] {1 h} 1
= hlim
0
h
(1 h)2 h 1 2 2
lim 1 h 2h h 1 = lim h 3h = 3
= hlim
0 = h0 h0
h h h
LCD - 54
Limits, Continuity & Derivability
e [ x ] | x | 1
x0
(23) If f(x) = [ x ] {2x } , comment on the continuity at x = 0 and differentiability at
1 / 2 x 0
x = 0, where [ . ] is greatest integer function and {.} is fractional part function.
Answer : discontinuous hence non-differentiable at x = 0
(ii) If f(x) is not differentiable at x = a & g(x) is differentiable at x = a, then the product function
F(x) = f(x) . g(x) can still be differentiable at x = a
e.g. f(x) = x and g(x) = x 2.
(iii) If f(x) & g(x) both are not differentiable at x = a, then the product function
F(x) = f(x) . g(x) can still be differentiable at x = a e.g. f(x) = x & g(x) = x.
(iv) If f(x) & g(x) both are non-differentiable at x = a, then the sum function F(x) = f(x) + g(x) may be
a differentiable function. e.g. f(x) = x & g(x) = x.
f (a g(h)) f (a p(h))
(v) If f is differentiable at x = a, then hlim
0 g(h) p(h) = f(a), where
Solution :
Non-differentiable at x = 0.
LCD - 55
Limits, Continuity & Derivability
Example # 41 : Discuss the differentiability of f(x) = x|x|
x 2 , x 0
Solution : f(x) = 2
x , x 0
Differentiable at x = 0
Example # 42 : If f(x) is differentiable and g(x) is differentiable, then prove that f(x) . g(x) will be differentiable.
Solution : Given, f(x) is differentiable
lim f (a h ) f ( a )
i.e. h 0 = f(a)
h
g(x) is differentiable
g(a h) g(a ) f ( a h) f ( a )
= h lim
0
f (a h). g(a). = f(a) . g(a) + g(a) f(a) = p(a)
h h
Hence p(x) is differentiable.
x 3 , x0
Example # 43 : If f(x) = 2 and g(x) = f(|x|) + |f(x)|, then comment on the continuity and
x 3x 2 , x 0
differentiability of g(x) by drawing the graph of f(|x|) and, |f(x)|.
Solution :
LCD - 56
Limits, Continuity & Derivability
Graph of f(|x|) and |f(x)|
If f(|x|) and |f(x)| are continous, then g(x) is continuous. At x = 0 f(|x|) is continuous, and |f(x)| is
discontinuous therefore g(x) is discontineous at x = 0.
g(x) is non differentiable at x = 0, 1, 2, (find the reason yourself).
1
x [sin x] , 0 x 1
3
Example # 44 : If f(x) = , find those points at which continuity and differentiability
[2x ] sgn x 4 , 1 x 2
3
should be checked, where [ . ] is greatest integer function and {.} is fractional part function.
Also check the continuity and differentaibility of f(x) at x = 1.
1
x [sin x] , 0 x 1
3
Solution : f(x) = The points, where we should check the continuity and
[2x ] sgn x 4 , 1 x 2
3
LCD - 57
Limits, Continuity & Derivability
1 2 4 3
differentiability are x = 0, , , 1, , ,2
2 3 3 2
At x = 1
1
L.H.L. = xlim f(x) = xlim x [sin x] = 0
1 1 3
4
R.H.L. = xlim
1
f(x) = xlim
1
[2x] sgn x 3 = 2 (– 1) = – 2
L.H.L R.H.L. hence f(x) is discontinuous at x = 1 and hence it is non diffferentiable at x = 1.
Self Practice Problems:
(24) If f(x) = [x] + [1 – x] , – 1 x 3, then draw its graph and comment on the continuity and
differentiability of f(x), where [ . ] is greatest integer function.
| 1 4 x 2 | , 0 x 1
(25) If f(x) = 2 , then draw the graph of f(x) and comment on the
[ x 2x] , 1 x 2
Example # 45 : If f(x) is a function satisfies the relation for all x, y R, f(x + y) = f(x) + f(y) and if f(0) = 2 and
function is differentiable every where, then find f(x).
f ( x h) f ( x ) f ( x ) f (h ) f ( x ) f ( 0 )
Solution : f(x) = hlim
0
= hlim
0
( f(0) = 0)
h h
f (h) f (0)
= hlim
0
= f(0)
h
f(x) = 2 f (x) dx 2 dx
f(x) = 2x + c
f(0) = 2.0 + c as f(0) = 0
c=0 f(x) = 2x
Second Method :
Since f(x + y) = f(x) + f(y) is true for all values of x and y is independent of differentiating both sides
w.r.t x (here y is constant with respect to x).
f(x + y) = f(x)
put x = 0
Example # 46 : f(x + y) = f(x) . f(y) x, y R and f(x) is a differentiable function and f(0) = 1, f(x) 0 for
any x. Find f(x)
Solution : f(x) is a differentiable function
f ( x h) f ( x )
f(x) = hlim
0 h
LCD - 58
Limits, Continuity & Derivability
f ( x ).f (h) f ( x ).f (0)
= hlim
0
( f(0) = 1)
h
f ( x ).( f (h) f (0))
= hlim
0
= f(x) . f(0) = f(x)
h
f(x) = f(x)
f ( x )
f ( x) dx = 1 dx
n f(x) = x + c n 1 = 0 + c
c=0 n f(x) = x
f(x) = ex
Important formula :
f (a g(h)) f (a p(h))
For finding limit hlim = f(a),
0 g(h) p(h)
if hlim
0
p(h) = hlim
0
g(h) = 0 and f(x) is differentiable at x = a
f (a 2h) f (a 3h)
Example # 48 : Evaluate hlim
0
, if f(a) = 3
h
(27) If f(x) and g(x) are differentiable, then prove that f(x) ± g(x) will be differentiable.
f (2 h) f (2 sinh)
(28) If f(2) = 4, then find the value of hlim
0 h. sinh . tanh .
LCD - 59
Limits, Continuity & Derivability
General tips :
If x, y are independent variables, then:
(i) f (xy) = f (x) + f (y) f (x) = k ln x or f (x) = 0.
(ii) f (xy) = f (x). f (y) f (x) = xn, n R
(iii) f (x + y) = f (x). f (y) f (x) = akx.
(iv) f (x + y) = f (x) + f (y) f(x) = kx, where k is a constant.
1 1
Example # 49 : If f(x) is a polynomial function satisfying f(x) . f = f(x) + f x R – {0} and f(2) =9,
x x
then find f (3)
Solution : f(x) = 1 ± x n
As f(2) = 9 f(x) = 1 + x3
3
Hence f(3) = 1 + 3 = 28
Self practice problems
1 1
(29) If f(x) is a polynomial function satisfying f(x) . f = f(x) + f x R – {0} and
x
x
f(3) = – 8, then find f(4)
f (x)
(30) If f(x + y) = f(x) . f(y) for all real x, y and f(0) 0, then prove that the function, g(x) =
1 f 2 (x)
is an even function.
Answer : (29) – 15
LCD - 60
Method of Differentiation
Mathematics is very much like poetry ... what makes a good poem — a great poem — is that there is a large amount of thought expressed in
i
1 0 e
x2
very few words. In this sense formulas like e or dx are poems......
1. The derivative of a given function f at a point x = a in its domain represent the slope of the tangent at
that point, and it is defined as:
f ( x )f (a)
i.e. f (a) = Limit
x a , provided the limit exists.
xa
f ( x h)f ( x )
i.e., f (x) = Limit
h0
h
This method of differentiation is also called ab-initio method or first principle method.
( x h) 2 x 2 2xh h2
Solution : (i) f(x) = hlim
0 = hlim
0 = 2x.
h h
sin( x h) sin x
tan( x h) tan x
(ii) f(x) lim
= h0 lim
= h 0 cos( x h) cos x
h h
sin( x h – x )
= hlim = sec 2x.
0 h cos x. cos( x h)
xh x xhx
2 cos sin
lim sin( x h) sin x = esin x lim 2 2
= esin x h0 h0
h h
h sin(h / 2)
= esin x lim cos x sin x cos x
h0
2 h/2 = e
MOD - 61
Method of Differentiation
Derivative of some elementary functions :
f(x) f(x)
1. xn nx n – 1 , (x R, n R)
2. ax ax n a , a>0
1
3. n |x|
x
1
4. logax x n a
5. sin x cos x
6. cos x – sin x
9. tan x sec 2 x
Basic theorems :
Sum of two differentiable functions is always differentiable.
Sum of two non-differentiable functions may be differentiable.
d d
2. (k f(x)) = k f(x)
dx dx
d
3. (f(x) . g(x)) = f(x) g(x) + g(x) f(x)
dx
d f ( x) g( x ) f ( x ) f ( x ) g( x )
4. =
dx g( x ) g2 ( x )
d
5. (f(g(x))) = f(g(x)) g(x)
dx
This rule is also called the chain rule of differentiation and can be written as
dy dy dz
= .
dx dz dx
Note that an important inference obtained from the chain rule is that
dy dy dx
=1= .
dy dx dy
dy 1
=
dx dx / dy
MOD - 62
Method of Differentiation
another way of expressing the same concept is by considering y = f(x) and x = g(y) as inverse functions
of each other.
dy dx
= f(x) and dy = g(y)
dx
1
g(y) =
f ( x )
d 1 d
f(x) = ( sin ( 2 x 3 ) ) = . (sin (2x + 3)) (chain rule)
dx 2 sin (2x 3) dx
cos(2x 3)
=
sin (2x 3)
x (1 x 2 ) x(2x )
(ii) f(x) = f(x) = (Quotiant rule)
1 x2 (1 x 2 )2
1 x2
=
(1 x 2 )2
(iii) f(x) = x sin x
f(x) = x. cos x + sin x (Product rule)
f (5 t ) f (5 t )
(2) If f(5) = 7, then find the value of tlim
0 2t
1 x
(iii) 1 x 2 (iv)
1 x
(v) cos 3 x sin x (vi) x ex sin x
sin x
(vii) (viii) n (sin x – cos x)
1 cos x
MOD - 63
Method of Differentiation
Answers : (1) (i) x cosx + sinx (ii) 2sin x cos x
(2) 7
x 2 2x 1
(3) (i) 6x (5x 3 + x – 1) (ii)
( x 2) 2 ( x 3 ) 2
x 1
(iii) (iv)
1 x 2 (1 x )1 / 2 (1 x )3 / 2
1 x cos x sin x
(vii) sec 2 (viii)
2 2 sin x cos x
1. y = sin–1 x – y and x = sin y
2 2
dx dy 1 1
= cos y = = , siny ± 1
dy dx cos y 1 sin 2 y
dy 1
= , – 1 < x < 1.
dx 1 x 2
In general |cos y| = 1 sin 2 y . But here since – y |cos y| = cosy
2 2
2. y = tan–1x x = tan y and – y
2 2
dx
= sec 2y = 1 + tan2 y
dy
dx dy 1
dy = 1 + x
2 = (x R)
dx 1 x2
3. y = sec –1x y [0, ] – and x = secy
2
dx dy 1
dy = sec y tan y = sec y. tan y , tan y 0
dx
1
= | sec y. tan y | , y 0, ,
2 2
1 1 dy 1
= | sec y || tan y | = = x (– , – 1) (1, )
| x | x2 1 dx | x | x2 1
MOD - 64
Method of Differentiation
Results for the derivative of inverse trigonometric functions can be summarized as :
f(x) f(x)
1
sin–1x ; |x| < 1
1 x 2
1
cos –1x ; |x| < 1
1 x 2
1
tan–1x ; xR
1 x2
1
cot–1x ; xR
1 x2
1
sec –1 x ; |x| > 1
| x | x2 1
1
cosec -1 x ; |x| > 1
| x | x2 1
1 1 2x
Solution : f(x) = . . 2x = 1
1
(sin x )2
1 ( x 2 )2 (sin x 2 ) 1 x 4
2x 1
Solution : f(x) = 2 sec –1(x) + 2
+
| x | x 1 | x | x2 1
2 1 4 3
Hence, f(–2) = 2.sec –1(– 2) – + f(–2) = –
3 2 3 3 2
Logarithmic differentiation :
The process of taking logarithm of the function first and then differentiate is called Logarithmic
differentiation. It is often useful in situations when
dy
Example # 6 : If y = (sin x) n x, find
dx
Solution : n y = n x . n (sin x)
1 dy 1 cos x
y dx = n (sin x) + n x.
x sin x
dy n sin x
= (sin x) n x cot x n x
dx x
MOD - 65
Method of Differentiation
x1/ 2 (1 2x )2 / 3 dy
Example # 7 : If y = 3/4 4/5 , then find .
(2 3x ) (3 4 x ) dx
x1/ 2 (1 2x )2 / 3
Solution : y=
( 2 3 x )3 / 4 (3 4 x ) 4 / 5
taking loge on both side
1 2 3 4
n y = n x + n (1 – 2x) – n (2 – 3x) – n (3 – 4x)
2 3 4 5
1 dy 1 4 9 16
= – + 4 (2 3 x ) + 5 (3 4 x )
y dx 2x 3(1 2x )
dy 1 4 9 16
= y 2x 3 (1 2 x ) 4(2 3 x ) 5 (3 4 x )
dx
Implicit differentiation :
If f(x, y) = 0, is an implicit function i.e. y can't be expressed explicitly as a function of x then in order to
find dy/dx, we differentiate each term w.r.t. x regarding y as a function of x and then collect terms in
dy/dx.
dy
Example # 8 : If x3 + y3 = 3xy, then find .
dx
Solution : Differentiating both sides w.r.t.x, we get
dy dy
3x 2 + 3y2 = 3x + 3y
dx dx
dy y x2
= 2
dx y x
Note that above result holds only for points where y2 – x 0
dy
Example # 9 : If x y + yx = 2 then find
dx
Solution : u+v=2
du dv
+ =0 ........(i)
dx dx
where u = x y & v = yx
n u = y n x & n v = x n y
1 du y dy 1 dv x dy
= + n x & = n y + y
u dx x dx v dx dx
du y dy dv x dy
= x y n x & = yx n y y dx
dx x dx dx
Now, equation (i) becomes
y dy x dy
x y n x + y x n y
x dx y dx = 0.
x y y
y n y x .
dy x
=–
dx y x
x n x y x .
y
MOD - 66
Method of Differentiation
Self Practice Problems
dy
(5) Find if
dx
(i) y = cos (x + y) (ii) x 2/3 + y2/3 = a2/3
(iii) x = y n (x – y)
dy n x
(6) If xy = ex – y, then prove that = .
dx (1 n x )2
x a dy x
(7) If = log , then prove that =2– .
xy xy dx y
2 1
Answers : (4) (i) (ii)
4
x x 1 1 x2
x
1 1 1 x
(iii) 1 n 1 (iv) xx. e x (nx + 1)
x x 1 x
1 sin x
(v) n (nx ) nx (n x)x + x sinx cos x nx
x
1/ 3
sin( x y ) y y( x 2 y )
(5) (i) 1 sin( x y ) (ii) – (iii)
x x( x y ) y 2
(i) x 2 a2 x = a tan , where << or x = a cot , where 0 < <
2 2
(ii) a2 x 2 x = a sin , where or x = a cos , where 0
2 2
(iii) x 2 a2 x = a sec , where [0 –
2
or x = a cosec , where , – {0}
2 2
xa
(iv) x = a cos , where 0
ax
MOD - 67
Method of Differentiation
1 x 2 1
Example # 10 : Differentiate y = tan–1 with respect to x.
x
Solution : Let x = tan , where , – {0}
2 2
| sec | 1
y = tan–1 | sec | sec ,
tan 2 2
1 cos
y = tan–1
sin
y = tan–1 tan
2
1
y= tan (tan x ) x for x ,
2 2 2
1 dy 1
y= tan–1 x =
2 dx 2(1 x 2 )
dy 1 x 1 x
Example # 11 : Find , where y = tan–1 .
dx 1 x 1 x
Solution : Let x = cos , where [0, ]
1 cos 1 cos
y = tan–1
1 cos 1 cos
2 cos 2 sin
2 2
y = tan–1
2 cos 2 sin
2 2
1 cos 2 cos but for 0, , 2 cos 2 cos
2 2 2 2 2
1 tan
2
y = tan–1 y= – as –
1 tan 4 2 4 4 2 4
2
1 dy 1
y= – cos –1x =
4 2 dx 2 1 x2
MOD - 68
Method of Differentiation
2x
Example # 12 : If f(x) = sin–1 , then find
1 x2
1
(i) f(2) (ii) f (iii) f(1)
2
Solution : x = tan , where – <<
2 2
y = sin–1 (sin 2)
2 ,
2
2 2 tan 1 x x 1
1
2 tan x 1 x 1
y = 2 , 2 f(x) =
2 2 ( 2 tan 1 x ) x 1
( 2) , 2
2
2
2
x 1
1 x
2
2
1 x 1
f(x) = 1 x
2
x 1
1 x 2
2 1 8
(i) f(2) = – (ii) f =
5 2
5
(iii) f(1+) = – 1 and f(1–) = + 1
f(1) does not exist.
Aliter
Above problem can also be solved without any substution also, but in a little tedious way.
2x
f(x) = sin–1
1 x2
1 2{(1 x 2 ) 2x 2 }
f(x) = .
4x 2 (1 x 2 )2
1
(1 x 2 )2
(1 x 2 ) 2(1 x 2 )
= .
(1 x 2 )2 (1 x 2 )2
2 (1 x 2 )
f(x) = 2 .
(1 x ) | 1 x2 |
2
2
, |x|1
thus 1
f(x) = 2 x
, | x|1
1 x 2
MOD - 69
Method of Differentiation
y = sin where – .
2 2
cos + cos = a (sin – sin)
cot =a
2
– = 2 cot–1 (a) or – + 2 cot–1a
sin–1 x – sin–1 y = 2 cot–1a or – + 2 cot–1a
differentiating w.r.t to x.
1 1 dy
– =0
1 x 2
1 y 2 dx
dy 1 y2
=
dx 1 x2
Aliter
Using implicit differentiation
x y dy dy
– = a 1
1 x 2 1 y2 dx dx
x
a
y dy x dy 1 x2
a =a+ =
1 y2 dx 1 x 2 dx
a
y
1 y2
1 x 2 1 y 2 x
dy xy 1 x 2
=
dx 1 x2 1 y2 y
xy 1 y2
dy (1 x 2 ) (1 x 2 )(1 y 2 ) x 2 xy 1 y2 1 (1 x 2 )(1 y 2 ) xy 1 y2
= . = .
dx (1 x 2 )(1 y 2 ) (1 y 2 ) xy y 2 1 x2 1 (1 x 2 )(1 y 2 ) xy 1 x2
dy 1 y2
= Hence proved
dx 1 x2
MOD - 70
Method of Differentiation
Parametric differentiation :
dy dy / d
If y = f( ) & x = g( ) where is a parameter, then dx dx / d .
dy
Example # 14 : If x= a cos 3t and y = a sin3t, then find the value of .
dx
dy dy / dt 3a sin 2 t cos t
Solution : = = = – tan t
dx dx / dt 3a cos 2 t sin t
dy
Example # 15 : If y = a cos t and x = a (t – sint), then find the value of at t = .
dx 2
dy a sin t dy
Solution : = dx
= – 1.
dx a(1 cos t ) t
2
dy dy / dx 1
Solution : = =
dz dz / dx xex
dy
(8) Find when
dx
(i) x = a (cos t + t sin t) and y = a (sin t – t cos t)
1 t2 2t
(ii) x=a 2 and y=b
1 t 1 t 2
x2 2 xa 2
dy
(9) If y = sin–1 4 4 , then prove that dx = x 4 a 4 .
x a
2x dy 2
(10) If y = tan–1 , then prove that = (| x | 1)
1 x2 dx 1 x2
du 1 u2
(11) If u = sin (m cos –1x) and v = cos (m sin–1 x), then prove that = .
dv 1 v 2
( t 2 1)b
Answer : (8) (i) tan t (ii)
2at
MOD - 71
Method of Differentiation
Higher order derivatives :
Let a function y = f(x) be defined on an open interval (a, b). It’s derivative, if it exists in (a, b) is a certain
function f (x) [or (dy/dx) or y ] & is called the first derivative of y w. r. t. x.
If it happens that the first derivative has a derivative in (a, b) then this derivative is called the second
derivative of y w. r. t. x & is denoted by f (x) or (d2y/dx 2) or y . While the first derivative denotes slope
of the graph, the second derivative denotes it's concavity.
d3 y d d2 y
Similarly, the 3rd order derivative of y w. r. t. x, if it exists, is defined by
dx 3 dx dx 2 , it is also
denoted by f (x) or y .
It must be carefully noted that in case of parametric functions
dy dy / dt d2 y d2 y / dt 2 d2 y d dy / dt
although = but rather =
dx dx / dt dx 2 2
dx / dt 2
dx 2 dx dx / dt
which on applying chain rule can be resolved as
dx d2 y dy d2 x
. .
dt dt 2 dt dt 2
d2 y d dy / dt dt d2 y dt
= . = 2 .
dx 2 dt dx / dt dx dx 2
dx dx
dt
dx d2 y dy d2 x
. 2 .
d2 y dt dt dt dt 2
= 3
dx 2 dx
dt
Example # 17 : If y = x3 n x, then find yand y
1
Solution : y = 3x 2 n x + x 3 = 3x 2 n x + x 2
x
1
y = 6x n x + 3x2 . + 2x = 6x n x + 5x
x
y = 6 n x + 11
x
1
Example # 18 : If y = , then find y(1)
x
Solution : Now taking loge both sides, we get
n y = – x n x when x = 1, then y = 1
n y = – x n x
y
y = – (1 + n x) y = – y (1 + n x) ......(i)
1 y
y = – y(1 + n x) – y . y = y (1 + n x)2 – (using (i))
x x
y(1) = 0
MOD - 72
Method of Differentiation
d2 y
Example # 19 : If x = t + 1 and y = t2 + t3, then find .
dx 2
dy dx
Solution : = 2t + 3t2 ; =1
dt dt
dy d2 y d dt
= 2t + 3t2 2 = (2t + 3t2) .
dx dx dt dx
d2 y
= 2 + 6t.
dx 2
dy dy / dx cos x
Solution : = =
dz dz / dx ex
d2 y d cos x d2 y d cos x dx
= = .
dz 2 dz e x dx 2 dx e x dz
e x sin x cos x e x 1
= .
x 2
(e ) ex
d2 y (sin x cos x )
=–
dz 2
e2x
Example # 21 : y = f(x) and x =g(y) are inverse functions of each other, then express g(y) and g(y) in terms
of derivative of f(x).
dy dx
Solution : = f(x) and dy = g(y)
dx
1
g(y) = ...........(i)
f ( x )
again differentiating w.r.t. to y
d 1
g(y) = dy
f ( x )
d 1 dx
= .
dx f ( x ) dy
f ( x )
=– . g(y)
f ( x )2
f ( x )
g(y) = – .........(ii)
f ( x )3
which can also be remembered as
d2 y
2
d x dx 2
2 = – 3 .
dy dy
dx
MOD - 73
Method of Differentiation
Example # 22 : y = sin (sinx) then prove that y + (tanx) y + y cos2x = 0
n x d2 y
(12) If y = , then find the value of .
x dx 2
d2 y
(13) Prove that y = x + tan x satisfies the differential equation cos 2 x – 2y + 2x = 0.
dx 2
d2 y
(14) If x = a (cos + sin ) and y = a(sin – cos), then find the value of .
dx 2
(15) Find the second order derivative of nx with respect to sin x.
d2 y dy
(16) If y = e– x (A cos x + B sin x), then prove that 2 +2. + 2y = 0.
dx dx
Derivative of a determinant :
f ( x ) g( x ) h( x )
l( x ) m( x ) n( x )
If F(x) = , where f, g, h, l, m, n, u, v, w are differentiable functions of x, then
u( x ) v( x ) w( x)
L’ hospital’s rule :
If f(x) & g(x) are functions of x such that:
(i) Limit f(x) = = Limit g(x)
x a x a
OR
(ii) Limit f(x) = 0 = Limit g(x), both f(x) and g(x) are continuous at x = a, both f(x) and
x a x a
g(x) are differentiable at x = a and both f (x) and g (x) are continuous at x = a,
MOD - 74
Application of Derivatives
APPLICATION OF DERIVATIVES
An economist may want to study how the investment changes with respect to variations in interest
rates.
A physician may want to know, how small changes in dosage can affect the body's response to a drug.
A physicist may want to know the rate of charge of distance with respect to time.
All questions of the above type can be interpreted and represented using derivatives.
Definition : The average rate of change of a function f(x) with respect to x over an interval [a, a + h] is
f (a h ) f ( a )
defined as .
h
f (a h ) f ( a )
Definition : The instantaneous rate of change of f(x) with respect to x is defined as f(x) = hlim
0
,
h
provided the limit exists.
Note : To use the word 'instantaneous', x may not be representing time. We usually use the word 'rate of
change' to mean 'instantaneous rate of change'.
Example # 1 How fast the area of a circle increases when its radius is 5cm;
(i) with respect to radius (ii) with respect to diameter
dA
Solution : (i) A = r2 , = 2r
dr
dA
dr r 5 = 10 cm /cm.
2
2 dA
(ii) A= D , = D
4 dD 2
dA
dD D 10 = 2 . 10 = 5 cm /cm.
2
Example # 2 If area of circle increases at a rate of 2cm 2 /sec, then find the rate at which area of the inscribed
square increases.
Solution : Area of circle, A1 = r2. Area of square, A2 = 2r 2 (see figure)
dA 1 dr dA 2 dr
= 2r , = 4r .
dt dt dt dt
dr dr 1
2 = 2r . r =
dt dt
dA 2 1 4
=4. = cm 2/sec
dt
4
Area of square increases at the rate cm 2/sec.
AOD - 75
Application of Derivatives
Example # 3 The volume of a cube is increasing at a rate of 7 cm 3/sec. How fast is the surface area increasing
when the length of an edge is 4 cm?
Solution. Let at some time t, the length of edge is x cm.
dv dx dv
v = x3 = 3x 2 (but = 7)
dt dt dt
dx 7
= cm/sec.
dt 3x 2
Now S = 6x2
dS dx dS 7 28
= 12x = 12x. 2 =
dt dt dt 3x x
dS
when x = 4 cm, = 7 cm 2/sec.
dt
Example # 4 Sand is pouring from pipe at the rate of 12 cm 3/s. The falling sand forms a cone on the ground
in such a way that the height of the cone is always one - sixth of radius of base. How fast is the
height of the sand cone increasing when height is 4 cm?
1
Solution. V= r 2 h
3
r
but h=
6
1
V= (6h)2 h
3
V = 12 h3
dV dh
= 36 h2.
dt dt
dV
when, = 12 cm 3/s and h = 4 cm
dt
dh 12 1
= = cm/sec.
dt 36.( 4)2 48
(1) Radius of a circle is increasing at rate of 3 cm/sec. Find the rate at which the area of circle is
increasing at the instant when radius is 10 cm.
(2) A ladder of length 5 m is leaning against a wall. The bottom of ladder is being pulled along the
ground away from wall at rate of 2cm/sec. How fast is the top part of ladder sliding on the wall
when foot of ladder is 4 m away from wall.
(3) Water is dripping out of a conical funnel of semi-vertical angle 45° at rate of 2cm 3/s. Find the
rate at which slant height of water is decreasing when the height of water is 2 cm.
(4) A hot air balloon rising straight up from a level field is tracked by a range finder 500 ft from the
lift-off point. At the moment the range finder's elevation angle is /4, the angle is increasing at
the rate of 0.14 rad/min. How fast is the balloon rising at that moment.
8 1
Answers : (1) 60 cm 2/sec (2) cm/sec (3) cm/sec. (4) 140 ft/min.
3 2
AOD - 76
Application of Derivatives
Error and Approximation :
Let y = f(x) be a function. If these is an error x in x then corresponding error in y is y = f(x + x) – f(x).
f ( x x ) f ( x ) dy
We have lim = = f(x)
x 0 x dx
We define the differential of y, at point x, corresponding to the increment x as f(x) x and denote it by dy.
i.e. dy = f(x) x.
Let P(x, f(x)), Q((x + x), f(x + x)) (as shown in figure)
y = QS,
x = PS,
dy = RS
In many practical situations, it is easier to evaluate dy but not y.
Example # 5. Find the approximate value of 251/3.
Sol. Let y = x1/3
Let x = 27 and x = –2
Now y = (x + x)1/3 – x1/3 = (25)1/3 – 3
dy
x = 251/3 – 3
dx
At x = 27, 251/3 = 3 – 0.074 = 2.926
Monotonicity of a function :
Let f be a real valued function having domain D(D R) and S be a subset of D. f is said to be monotonically
increasing (non decreasing) (increasing) in S if for every x 1, x 2 S, x1 < x2 f(x1) f(x2). f is said to be
monotonically decreasing (non increasing) (decreasing) in S if for every x 1, x 2 S, x1 < x2 f(x 1) f(x 2)
f is said to be strictly increasing in S if for x1, x2 S, x 1 < x2 f(x 1) < f(x2). Similarly, f is said to be
strictly decreasing in S if for x 1, x 2 S, x 1 < x 2 f(x1) > f(x 2) .
Notes : (i) f is strictly increasing f is monotonically increasing (non decreasing). But converse need
not be true.
(ii) f is strictly decreasing f is monotonically decreasing (non increasing). Again, converse need
not be true.
(iii) If f(x) = constant in S, then f is increasing as well as decreasing in S
(iv) A function f is said to be an increasing function if it is increasing in the domain. Similarly, if f is
decreasing in the domain, we say that f is monotonically decreasing
(v) f is said to be a monotonic function if either it is monotonically increasing or monotonically
decreasing
(vi) If f is increasing in a subset of S and decreasing in another subset of S, then f is non monotonic
in S.
AOD - 77
Application of Derivatives
Here also f(x) 0 for all x (a, b). But, note that in this case, f(x) = 0 holds for all x (c, d) and (e,b).
Thus the given function is increasing (monotonically increasing) in (a, b), but not strictly increasing.
AOD - 78
Application of Derivatives
for increasing f(x) 0 cosx sinx
5
f is increasing in 0 , and , 2
4 4
5
f is decreasing in ,
4 4
Note : If a function f(x) is increasing in (a, b) and f(x) is continuous in [a, b], then f(x) is increasing on [a, b]
Example # 10: f(x) = [x] is a step up function. Is it a strictly increasing function for x R.
Solution : No, f(x) = [x] is increasing (monotonically increasing) (non-decreasing), but not strictly increasing
function as illustrated by its graph.
Example # 11 : If f(x) = sin4x + cos 4x + bx + c, then find possible values of b and c such that f(x) is monotonic
for all x R
Solution : f(x) = sin4x + cos4x + bx + c
f(x) = 4 sin3x cosx – 4cos 3x sinx + b = – sin4x + b.
Case - (i) : for M.I. f(x) 0 for all x R
b sin4x for all x R b1
Case - (ii) : for M.D. f(x) 0 for all x R
b sin4x for all x R b–1
Hence for f(x) to be monotonic b (– , – 1] [1, ) and c R.
Example # 12:Find possible values of 'a' such that f(x) = e2x – (a + 1) ex + 2x is monotonically increasing for
xR
Solution : f(x) = e2x – (a + 1) ex + 2x
f(x) = 2e2x – (a + 1) ex + 2
Now, 2e2x – (a + 1) ex + 2 0 for all x R
x 1
2 e x – (a + 1) 0 for all x R
e
x 1
(a + 1) < 2 e x for all x R
e
1
a+14 e x x has min imum value 2
e
a3
Aliter (Using graph)
2e2x – (a + 1) ex + 2 0 for all x R
putting ex = t ; t (0, )
2t2 – (a + 1) t + 2 0 for all t (0, )
Case - (i) : D0
(a + 1) 2 – 4 0
(a + 5) (a – 3) 0
a [– 5, 3]
or
Case - (ii) : both roots are non positive
b
D0 & – <0 & f(0) 0
2a
AOD - 79
Application of Derivatives
a 1
a (– , – 5] [3, ) & <0 & 20
4
a (– , – 5] [3, ) & a<–1 & aR
a (– , – 5]
Taking union of (i) and (ii), we get a (– , 3].
(6) Let f(x) = x – tan–1x. Prove that f(x) is monotonically increasing for x R.
(7) If f(x) = 2ex – ae–x + (2a + 1) x – 3 monotonically increases for x R, then find range of values
of a
(8) Let f(x) = e2x – aex + 1. Prove that f(x) cannot be monotonically decreasing for x R for any
value of 'a'.
(9) The values of 'a' for which function f(x) = (a + 2) x3 – ax2 + 9ax – 1 monotonically decreasing for
x R.
Answers : (5) (i) in [1, 3] ; D in (– , 1] (3, )
(ii) in (– , – 2] [0, ) ; D in [– 2, – 1) (–1, 0]
1 1
(iii) in , 1 ; D in , [1, )
2 2
(iv) I for x R
(7) a0 (9) – < a – 3
2. A function f(x) is called a strictly decreasing function about a point x = a, if it is strictly decreasing in
an open interval containing a (as shown in figure).
AOD - 80
Application of Derivatives
Note : If x = a is a boundary point then use the appropriate one sided inequality to test monotonicity of f(x).
e.g. : Which of the following functions (as shown in figure) is increasing, decreasing or neither increasing nor
decreasing at x = a.
(i) (ii)
(iii) (iv)
AOD - 81
Application of Derivatives
(i) f(0) = – 3 decreasing at x = 0
(ii) f(1) = 0
also, f(x) is positive on left neighbourhood and f(x) is negative in right neighbourhood.
neither increasing nor decreasing at x = 1.
(iii) f(2) = 9 increasing at x = 2
Note : Above method is applicable only for functions those are continuous at x = a.
Self practice problems :
(10) For each of the following graph comment on monotonicity of f(x) at x = a.
(11) Let f(x) = x 3 – 3x2 + 3x + 4, comment on the monotonic behaviour of f(x) at (i) x = 0 (ii) x = 1.
x 0 x 1
(12) Draw the graph of function f(x) = . Graphically comment on the monotonic
[ x ] 1 x 2
behaviour of f(x) at x = 0, 1, 2. Is f(x) M.. for x [0, 2] ?
Answers : (10) (i) neither M.. nor M.D. (ii) M.D. (iii) M.D (iv) M..
(11) M.. both at x = 0 and x = 1.
(12) M.. at x = 0, 2 ; neither M.. nor M.D. at x = 1. No, f(x) is not M.. for x [0, 2].
Global Maximum :
A function f(x) is said to have global maximum on a set E if there exists at least one c E such that f(x) f(c)
for all x E.
We say global maximum occurs at x = c and global maximum (or global maximum value) is f(c).
Local Maxima :
A function f(x) is said to have a local maximum at x = c if f(c) is the greatest value of the function in a small
neighbourhood (c – h, c + h), h > 0 of c.
i.e. for all x (c – h, c + h), x c, we have f(x) f(c).
i.e. f(c – ) f(c) f(c + ), 0 < h
Note : If x = c is a boundary point then consider (c – h, c) or (c, c + h) (h > 0) appropriately.
Global Minimum :
A function f(x) is said to have a global minimum on a set E if there exists at least one c E such that f(x)
f(c) for all x E.
Local Minima :
A function f(x) is said to have a local minimum at x = c if f(c) is the least value of the function in a small
neighbourhood (c – h, c + h), h > 0 of c.
i.e. for all x (c –h, c + h), x c, we have f(x) f(c).
i.e. f(c – ) f(c) f(c + ), 0 < h
Extrema :
A maxima or a minima is called an extrema.
Explanation : Consider graph of y = f(x), x [a, b]
AOD - 82
Application of Derivatives
x = a, x = c2, x = c4 are points of local maxima, with maximum values f(a), f(c2), f(c4) respectively.
x = c1, x = c3, x = b are points of local minima, with minimum values f(c1), f(c3), f(b) respectively
x = c2 is a point of global maximum
x = c3 is a point of global minimum
Consider the graph of y = h(x), x [a, b)
h(c4)
h(c1)
h(c3)
h(c2)
h(a)
a c1 c2 c3 c4 b
x = c1, x = c4 are points of local maxima, with maximum values h(c1), h(c4) respectively.
x = a, x = c2 are points of local minima, with minimum values h(a), h(c2) respectively.
x = c3 is neither a point of maxima nor a minima.
Global maximum is h(c4)
Global minimum is h(a)
| x | 0 | x | 2
Example # 14: Let f(x) = . Examine the behaviour of f(x) at x = 0.
1 x0
Solution : f(x) has local maxima at x = 0 (see figure).
(b 3 b 2 b 1)
– x3 0 x 1
Example # 15: Let f(x) = (b 2 3b 2)
2x 3 1 x 3
Find all possible values of b such that f(x) has the smallest value at x = 1.
Solution. Such problems can easily be solved by graphical approach (as in figure).
Hence the limiting value of f(x) from left of x = 1 should be either greater or equal to the value of function
at x = 1.
AOD - 83
Application of Derivatives
lim f(x) f(1)
x 1
(b 3 b 2 b 1)
–1+ –1
(b 2 3b 2)
(b 2 1)(b 1)
0
(b 1) (b 2)
b (– 2, –1) [1, + )
(14) Examine the graph of following functions in each case identify the points of global maximum/
minimum and local maximum / minimum.
Consider f(x) = x3
f(0) = 0
but f(0) is not an extremum (see figure).
AOD - 84
Application of Derivatives
Sufficient condition for an extrema :
x = c is a point of local minima (see figure), f(x) changes sign from negative to positive.
Stationary points :
The points on graph of function f(x) where f(x) = 0 are called stationary points.
Rate of change of f(x) is zero at a stationary point.
Example # 16: Find stationary points of the function f(x) = 4x3 – 6x2 – 24x + 9.
Solution : f(x) = 12x2 – 12x – 24
f(x) = 0 x = – 1, 2
f(– 1) = 23, f(2) = – 31
(– 1, 23), (2, – 31) are stationary points
Example # 17: If f(x) = x3 + ax2 + bx + c has extreme values at x = – 1 and x = 3. Find a, b, c.
Solution. Extreme values basically mean maximum or minimum values, since f(x) is differentiable function so
f(– 1) = 0 = f(3)
f(x) = 3x 2 + 2ax + b
f(3) = 27 + 6a + b = 0
f(– 1) = 3 – 2a + b = 0
a = – 3, b = – 9, c R
AOD - 85
Application of Derivatives
examining the sign change of f(x)
Note : In case of continuous functions points of maxima and minima are alternate.
Example # 19: Find the points of maxima, minima of f(x) = x 3 – 12x. Also draw the graph of this functions.
For tracing the graph let us find maximum and minimum values of f(x).
x f(x)
2 16
2 16
Example # 20 : Show that f(x) = (x3 – 6x 2 + 12x – 8) does not have any point of local maxima or minima. Hence
draw graph
Solution. f(x) = x3 – 6x 2 + 12x – 8
f(x) = 3(x 2 – 4x + 4)
f(x) = 3(x – 2) 2
f(x) = 0 x=2
but clearly f(x) does not change sign about x = 2. f(2+) > 0 and f(2–) > 0. So f(x) has no point
of maxima or minima. In fact f(x) is a monotonically increasing function for x R.
Example # 21 : Let f(x) = x3 + 3(a – 7)x 2 + 3(a2 – 9) x – 1. If f(x) has positive point of maxima, then find possible
values of 'a'.
Solution. f(x) = 3 [x2 + 2(a – 7)x + (a2 – 9)]
Let , be roots of f(x) = 0 and let be the smaller root. Examining sign change of f(x).
Maxima occurs at smaller root which has to be positive. This basically implies that both
roots of f(x) = 0 must be positive and distinct.
29
(i) D>0 a<
7
AOD - 86
Application of Derivatives
b
(ii) – >0 a<7
2a
(iii) f(0) > 0 a (– , – 3) (3, )
29
from (i), (ii) and (iii) a (– , – 3) 3,
7
Self practice problems :
(15) Find the points of local maxima or minima of following functions
(i) f(x) = (x – 1)3 (x + 2)2
(ii) f(x) = x 3 + x2 + x + 1.
4
Answer : (15) (i) Maxima at x = – 2, Minima at x = –
5
(ii) No point of local maxima or minima.
Critical points :
The points where f(x) = 0 or f(x) is not differentiable are called critical points.
Example # 22 : Find critical points of f(x) = max (sinx, cosx) , x (0, 2).
Solution :
5
From the figure it is clear that f(x) has three critical points x = , , .
4 2 4
Important Note :
For f(x) defined on a subset of R, points of extrema (if exists) occur at critical points
x 2 2x x 2
2
Solution. f(x) = 2x x 0x2
x 2x x 0
2
2( x 1) x 2
f(x) = 2(1 x ) 0 x 2
2( x 1) x 0
f(x) = 0 at x = 1 and f(x) does not exist at x = 0, 2. Thus these are critical points.
AOD - 87
Application of Derivatives
x 3 x 2 10 x x0
Example # 24 : Let f(x) = . Examine the behaviour of f(x) at x = 0.
3 sin x x0
Solution. f(x) is continuous at x = 0.
3 x 2 2x 10 x0
f(x) =
3 cos x x0
f(0+) = 3 and f(0–) = – 10 thus f(x) is non-differentiable at x = 0 x = 0 is a critical point.
Also derivative changes sign from negative to positive, so x = 0 is a point of local minima.
Example # 25 : Find the critical points of the function f(x) = 4x 3 – 6x 2 – 24x + 9 if (i) x [0, 3] (ii) x [–3, 3]
(iii) x [– 1, 2].
Solution. f(x) = 12(x2 – x – 2)
= 12(x – 2) (x + 1)
f(x) = 0 x = – 1 or 2
(i) if x [0, 3] , x = 2 is critical point.
(ii) if x [– 3, 3], then we have two critical points x = – 1, 2.
(iii) If x [– 1, 2], then no critical point as both x = 1 and x = 2 become boundary points.
Note : Critical points are always interior points of an interval.
Global extrema for continuous functions :
(i) Function defined on closed interval
AOD - 88
Application of Derivatives
Self Practice Problems :
(16) Let f(x) = 2x3 – 9x 2 + 12x + 6
(i) Find the possible points of Maxima/Minima of f(x) for x R.
(ii) Find the number of critical points of f(x) for x [0, 2].
(iii) Discuss absolute (global) maxima/minima value of f(x) for x [0, 2]
(iv) Prove that for x (1, 3), the function does not has a Global maximum.
Answers :
(16) (i) x = 1, 2 (ii) one
(iii) f(0) = 6 is the global minimum, f(1) = 11 is global maximum
x2 x ; 1 x 0
; x0
Example # 27 : Let f(x) =
1 3
log1 / 2 x ; 0 x
2 2
Discuss global maxima, minima for = 0 and = 1. For what values of does f(x) has global
maxima
–1/2 3/2
–1
–1
–1/2 3/2
–1
–1
AOD - 89
Application of Derivatives
f(–3) = – 75, f(1) = 53 are local minima
f(0) = 60 is local maxima
Lim f(x) = , Lim f(x) =
x x
Hence global maxima does not exists : Global minima is – 75
40
g(x) = f(x) g(x) has same critical points as that of f(x).
( f ( x ))2
A rough sketch of y = f(x) is
60
53
–3
1
– 75
Let zeros of f(x) be ,
g(), g() are undefined,
Lim g(x) = , Lim g(x) = – , Lim g(x) = – , Lim g(x) =
x x x x
x = , x = are asymptotes of y = g(x).
Lim g(x) = 0, Lim g(x) = 0
x x
y = 0 is also an asymptote. x = –3, x = 1 are local minima of
y = f(x) x = – 3, x = 1 are local maxima of y = g(x)
similarly, x = 0 is local minima of y = g(x)
Global extrema of g(x) does not exists.
A rough sketch of y = g(x) is
1
40/53
2/3
–3
1
–(8/15)
AOD - 90
Application of Derivatives
(i) If f(c) = 0 then further investigation is required
(ii) If f(c) > 0 then x = c is a point of minima.
(iii) If f(c) < 0 then x = c is a point of maxima.
For maxima f(x) changes from positive to negative (as shown in figure).
f(x) is decreasing hence f(c) < 0
Example # 29 : Find the points of local maxima or minima for f(x) = sin2x – x, x (0, ).
Solution. f(x) = sin2x – x
f(x) = 2cos2x – 1
1 5
f(x) = 0 cos 2x = x= ,
2 6 6
f(x) = – 4 sin 2x
f < 0 Maxima at x =
6 6
5 5
f >0 Minima at x =
6 6
Self practice problems :
(19) Find the points of local maxima or minima of f(x) = sin 2x – x
(20) Let f(x) = sinx (1 + cosx) ; x (0, 2). Find the number of critical points of f(x). Also identify
which of these critical points are points of Maxima/Minima.
Answer : (19) Maxima at x = n + ; Minima at x = n –
6 6
(20) Three
x= is point of maxima.
3
x = is not a point of extrema.
5
x= is point of minima.
3
n th Derivative test :
Let f(x) have derivatives up to nth order
If f(c) = f(c) = ..........= fn–1(c) = 0 and
f (c) 0 then we have following possibilities
n
AOD - 91
Application of Derivatives
so, f(x) = 30 (2x2 – 4x + 1)
f(0) = 30
Neither maxima nor minima at x = 0.
Note : It was very convenient to check maxima/minima at first step by examining the sign
change of f(x) no sign change of f(x) at x = 0
f(x) = 5x 2 (x – 1) (x – 3)
3. Volume of cube = a3
1 2
5. Volume of a cone = r h.
3
4 3
9. Volume of a sphere = r .
3
1 2
11. Area of a circular sector = r , when is in radians.
2
12. Volume of a prism = (area of the base) × (height).
14. Total surface area of a prism = (lateral surface area) + 2 (area of the base)
(Note that lateral surfaces of a prism are all rectangle).
1
15. Volume of a pyramid = (area of the base) × (height).
3
1
16. Curved surface area of a pyramid = (perimeter of the base) × (slant height).
2
(Note that slant surfaces of a pyramid are triangles).
Example # 31: If the equation x3 + px + q = 0 has three real roots, then show that 4p3 + 27q2 < 0.
Solution: f(x) = x3 + px + q, f(x) = 3x2 + p
f(x) must have one maximum > 0 and one minimum < 0. f(x) = 0
AOD - 92
Application of Derivatives
p
x=± ,p0
3
p p
f is maximum at x = – and minimum at x =
3 3
p p
f – f 0
3 3
q 2p p q 2p p 0
3
3 3 3
4p3
q2 + < 0, 4p3 + 27q2 < 0.
27
Example # 32 : Find two positive numbers x and y such that x + y = 60 and xy3 is maximum.
Solution. x + y = 60
x = 60 – y xy3 = (60 – y)y3
Let f(y) = (60 – y) y3 ; y (0, 60)
for maximizing f(y) let us find critical points
f(y) = 3y2 (60 – y) – y3 = 0
f(y) = y2 (180 – 4y) = 0
y = 45
f(45+) < 0 and f(45–) > 0. Hence local maxima at y = 45.
So x = 15 and y = 45.
Example # 33 : Rectangles are inscribed inside a semicircle of radius r. Find the rectangle with maximum
area.
Solution. Let sides of rectangle be x and y (as shown in figure).
A = xy.
Here x and y are not independent variables and are related by Pythogorus theorem with r.
x2 x2
+ y2 = r 2 y= r2
4 4
x2
A(x) = x r2
4
x4
A(x) = x 2r 2
4
x4
Let f(x) = r 2x 2 – ; x (0, r)
4
A(x) is maximum when f(x) is maximum
Hence f(x) = x(2r2 – x 2) = 0 x=r 2
r
confirming at f(x) is maximum when x = r 2 & y = .
2
Aliter Let us choose coordinate system with origin as centre of circle (as shown in figure).
A = xy
AOD - 93
Application of Derivatives
A = 2 (rcos) (rsin) A = r2 sin2 0,
2
r
Clearly A is maximum when = x=r 2 and y= .
4 2
Example # 34. Show that the least perimeter of an isosceles triangle circumscribed about a circle of radius ‘r’ is
6 3r.
Sol. AQ = r cot = AP
AO = r cosec A
(
x
= tan
AO ON
x = (r cosec + r) tan Q P
x = r(sec + tan)
Perimeter = p = 4x + 2AQ r r
p = 4r(sec + tan) + 2rcot x O x
p = r(4sec + 4tan + 2cot)
dp B x C
= r[4sec tan + 4sec2 – 2cosec2] x N
d
dp
for max or min =0 2sin3 + 3sin2 – 1 = 0 (sin + 1) (2sin2 + sin – 1) = 0
d
(sin + 1)2 (2sin – 1) = 0 sin = 1/2 = 30° = /6
4.2
pleast = r
4 8 4 6
2 3 = r =r
6 3 3 =6 3 r
3 3 3 3
Example # 35 : If a right circular cylinder is inscribed in a given cone. Find the dimensions of the cylinder
such that its volume is maximum.
Solution. Let x be the radius of cylinder and y be its height v = x 2y
x, y can be related by using similar triangles (as shown in figure).
y h h
= y= (r – x)
rx r r
h
v(x) = x 2 (r – x) x (0, r)
r
h
v(x) = (rx2 – x 3)
r
h
v(x) = x (2r – 3x)
r
2r 2r
v = 0 and v < 0
3 3
2r h
Thus volume is maximum at x = and y = .
3 3
Note : Following formulae of volume, surface area of important solids are very useful in problems of maxima
& minima.
AOD - 94
Application of Derivatives
Example # 36 : Among all regular square pyramids of volume 36 2 cm 3. Find dimensions of the pyramid
having least lateral surface area.
Solution. Let the length of a side of base be x cm and y be the perpendicular height of the pyramid
(see figure).
1
V= × area of base x height
3
1 2 108 2
V= x y = 36 2 y=
3 x2
1
and S= × perimeter of base x slant height
2
1
= (4x).
2
x2 x2
but = y2 S = 2x y2 = x 4 4x 2 y 2
4 4
2
108 2
S=
4
x 4x 2
x2
8.(108 )2
S(x) = x4
x2
8.(108 )2
Let f(x) = x 4 + for minimizing f(x)
x2
16(108 )2 (x 6 66 )
f(x) = 4x 3 – 3 =0 f(x) = 4 =0
x x3
x = 6, which a point of minima
Hence x = 6 cm and y = 3 2 .
Example # 37 : Let A(1, 2) and B(– 2, – 4) be two fixed points. A variable point P is chosen on the straight line
y = x such that perimeter of PAB is minimum. Find coordinates of P.
Solution. Since distance AB is fixed so for minimizing the perimeter of PAB, we basically have to
minimize (PA + PB)
Let A be the mirror image of A in the line y = x (see figure).
F(P) = PA + PB
F(P) = PA + PB
But for PAB
PA + PB AB and equality hold when P, A and B becomes collinear. Thus for minimum path
length point P is that special point for which PA and PB become incident and reflected rays
with respect to the mirror y = x.
Equation of line joining A and B is y = 2x intersection of this line with y = x is the point P.
Hence P (0, 0).
AOD - 95
Application of Derivatives
Note : Above concept is very useful because such problems become very lengthy by making perimeter as a
function of position of P and then minimizing it.
Self Practice Problems :
(21) Find the two positive numbers x and y whose sum is 35 and the product x 2 y5 maximum.
(22) A square piece of tin of side 18 cm is to be made into a box without top by cutting a square
from each corner and folding up the slops to form a box. What should be the side of the square
to be cut off such that volume of the box is maximum possible.
(23) Prove that a right circular cylinder of given surface area and maximum volume is such that the
height is equal to the diameter of the base.
x2 y2
(24) A normal is drawn to the ellipse + = 1. Find the maximum distance of this normal from
25 16
the centre.
(25) A line is drawn passing through point P(1, 2) to cut positive coordinate axes at A and B. Find
minimum area of PAB.
(26) Two towns A and B are situated on the same side of a straight road at distances a and b
respectively perpendiculars drawn from A and B meet the road at point C and D respectively.
The distance between C and D is c. A hospital is to be built at a point P on the road such that
the distance APB is minimum. Find position of P.
Answers : (21) x = 25, y = 10. (22) 3 cm (24) 1 unit
ac
(25) 4 units (26) P is at distance of from C.
ab
3. If f is continuous at x = c and f(x) has opposite signs on either sides of c, then the
point (c, f(c)) is a point of inflection of the curve
4. If f(c) = 0 and f(c) 0, then the point (c, f(c)) is a point of inflection
AOD - 96
Application of Derivatives
2 x1 x 2
x1 x2
Example # 38 : Prove that for any two numbers x 1 & x2 , 2e e > e 3
3
Solution : Assume f(x) = ex and let x 1 & x2 be two points on the curve y = ex.
Let R be another point which divides PQ in ratio 1 : 2.
2 x1 x 2
x1 x2
y coordinate of point R is 2e e and y coordinate of point S is e 3 . Since f(x) = ex is
3
concave up, the point R will always be above the point S.
2x x
2e x1 e x 2 1 2
> e 3
3
Alternate : Above inequality could also be easily proved using AM and GM.
x1 x 2 x 3 sin x 1 sin x 2 sin x 3
Example # 39 : If 0 < x 1 < x2 < x3 < then prove that sin > . Hence
3 3
prove that : if A, B, C are angles of a triangle then maximum value of
3 3
sinA + sinB + sinC is .
2
Solution :
AOD - 97
Application of Derivatives
If A + B + C = , then
A B C sin A sin B sin C
sin
3 3
Example # 40 : Find the points of inflection of the function f(x) = sin 2x x [0, 2]
Solution : f(x) = sin2x
f(x) = sin2x
f(x) = 2 cos2x
f(0) = 0
3
x= ,
4 4
both these points are inflection points as sign of f(x) change on either sides of these points.
Example # 41 : Find the inflection point of f(x) = 3x 4 – 4x3. Also draw the graph of f(x) giving due importance to
concavity and point of inflection.
Solution : f(x) = 3x 4 – 4x3
f(x) = 12x 3 – 12x 2
f(x) = 12x 2 (x – 1)
f(x) = 12(3x 2 – 2x)
f(x) = 12x(3x – 2)
2
f(x) = 0 x = 0, .
3
Again examining sign of f(x)
2
thus x = 0, are the inflection points
3
Hence the graph of f(x) is
f 1( x1 ) f 1( x 2 ) x1 x 2
> f –1 .
2 2
1 e2
Answer : (27)
e
AOD - 98
Application of Derivatives
Use of monotonicity for proving inequalities
Comparison of two functions f(x) and g(x) can be done by analysing the monotonic behaviour of
h(x) = f(x) – g(x)
Example # 42 : For x 0, prove that sin x < x < tan x
2
Solution : Let f(x) = x – sin x f(x) = 1 – cos x
f(x) > 0 for x 0,
2
f(x) is M.I. f(x) > f(0)
x – sin x > 0 x > sin x
Similarly consider another function g(x) = x – tan x g(x) = 1 – sec 2x
g(x) < 0 for x 0, g(x) is M.D.
2
Hence g(x) < g(0)
x – tan x < 0 x < tan x
sin x < x < tan x Hence proved
x3 x3 tan 1 x
Example # 43 : For x (0, 1) prove that x – < tan x < x –
–1 lim
hence or otherwise find x 0 x
3 6
x3
Solution : Let f(x) = x – – tan–1x
3
1
f(x) = 1 – x2 –
1 x2
x4
f(x) = –
1 x2
f(x) < 0 for x (0, 1) f(x) is M.D.
x3
f(x) < f(0) x– – tan–1x < 0
3
x3
x– < tan–1x ...........(i)
3
x3
Similarly g(x) = x – – tan–1x
6
x2 1
g(x) = 1 – –
2 1 x2
x 2 (1 x 2 )
g(x) =
2(1 x 2 )
g(x) > 0 for x (0, 1) g(x) is M.I.
g(x) > g(0)
x3
x– – tan–1x > 0
6
x3
x– > tan–1x ........(ii)
6
from (i) and (ii), we get
x3 x3
x– < tan–1x < x – Hence Proved
3 6
AOD - 99
Application of Derivatives
x2 tan 1 x x2
Also, 1– < <1– , for x > 0
3 x 6
tan 1 x
Hence by sandwich theorem we can prove that xlim
0
= 1 but it must also be noted that
x
tan 1 x tan 1 x
as x 0, value of 1 from left hand side i.e. <1
x x
tan 1 x
lim =0
x 0
x
NOTE : In proving inequalities, we must always check when does the equality takes place because the point of
equality is very important in this method. Normally point of equality occur at end point of the interval or
will be easily predicted by hit and trial.
x3
Example # 44 : For x 0, , prove that sin x > x –
2 6
3
x
Solution : Let f(x) = sin x – x +
6
x2
f(x) = cos x – 1 +
2
we cannot decide at this point whether f(x) is positive or negative, hence let us check for
monotonic nature of f(x)
f(x) = x – sinx
Since f(x) > 0 f(x) is M.I. for x 0,
2
f(x) > f(0) f(x) > 0
f(x) is M.. f(x) > f(0)
x3 x3
sin x – x + >0 sin x > x – . Hence proved
6 6
sin x tan x
Example # 45 : Examine which is greater : sin x tan x or x 2. Hence evaluate xlim0 , where
x2
x 0,
2
Solution : Let f(x) = sinx tanx – x2
f(x) = cos x . tan x + sin x . sec2x – 2x
f(x) = sin x + sin x sec 2x – 2x
f(x) = cos x + cos x sec 2x + 2sec2x sin x tan x – 2
f(x) = (cos x + sec x – 2) + 2 sec 2x sin x tan x
Now 2
cos x + sec x – 2 = cos x sec x and 2 sec 2x tan x . sin x > 0 because x 0,
2
f(x) > 0 f(x) is M.I.
Hence f(x) > f(0)
f(x) > 0 f(x) is M.I.
f(x) > 0 sin x tan x – x 2 > 0
Hence sin x tan x > x 2
x
1
Example # 46 : Prove that f(x) = 1 is monotonically increasing in its domain. Hence or otherwise draw
x
graph of f(x) and find its range
AOD - 100
Application of Derivatives
x
1 1
Solution : f(x) = 1 , for Domain of f(x), 1 + >0
x x
x 1
>0 (–, –1) (0, )
x
x 1 x 1
1 n1
Consider f(x) = 1 x 1 2
x 1 x
x
x
1 1 1
f(x) = 1 n1
x x x 1
x
1 1 1
Now 1 is always positive, hence the sign of f(x) depends on sign of n 1 –
x x 1 x
1 1
i.e. we have to compare n 1 and
x 1 x
1 1
So lets assume g(x) = n 1 –
x x 1
1 1 1 1
g(x) = + g(x) =
1 x 2 ( x 1)2 x( x 1)2
1
x
(i) for x (0, ), g(x) < 0 g(x) is M.D. for x (0, )
g(x) > xlim
g(x)
g(x) > 0. and since g(x) > 0 f(x) > 0
(ii) for x (– , – 1), g(x) > 0 g(x) is M.I. for x (– , –1)
g(x) > xlim
g(x) g(x) > 0 f(x) > 0
Hence from (i) and (ii) we get f(x) > 0 for all x (– , –1) (0, )
f(x) is M.I. in its Domain
For drawing the graph of f(x), its important to find the value of f(x) at boundary points
i.e. ± , 0, –1
x
lim 1
1 = e
x x
x x
lim 1 1 = 1 and lim 1
1 =
x 0 x x 1 x
so the graph of f(x) is
AOD - 101
Application of Derivatives
Self practice problems :
(30) Prove the following inequalities
(i) x < – n(1 – x) for x (0, 1)
(ii) x > tan–1(x) for x (0, )
(iii) ex > x + 1 for x (0, )
x
(iv) n (1 + x) x for x (0, )
1 x
2 sin x
(v) < <1 for x 0,
x 2
Rolle’s Theorem :
AOD - 102
Application of Derivatives
a b
f(0) = 0 and f(1) =
+ + c = 2a + 3b + 6c = 0
3 2
If f(0) = f(1) then f(x) = 0 for some value of x (0, 1)
ax 2 + bx + c = 0 for at least one x (0, 1)
Self Practice Problems :
(31) If f(x) satisfies condition in Rolle’s theorem then show that between two consecutive zeros of f(x)
there lies at most one zero of f(x).
(32) Show that for any real numbers , the polynomial P(x) = x7 + x3 + , has exactly one real root.
Lagrange’s Mean Value Theorem (LMVT) :
If a function f defined on [a, b] is
(i) continuous on [a, b] and
(ii) derivable on (a, b)
f (b ) f (a )
then there exists at least one real numbers between a and b (a < c < b) such that = f(c)
ba
AOD - 103
Application of Derivatives
Example # 51 : Using Lagrange’s mean value theorem, prove that if b > a > 0,
ba ba
then 2 < tan–1 b – tan–1 a <
1 b 1 a2
Solution : Let f(x) = tan–1 x ; x [a, b] applying LMVT
tan 1 b tan 1 a
f(c) = for a < c < b and f(x) = ,
ba 1 x 2
Now f(x) is a monotonically decreasing function
Hence if a < c < b f(b) < f(c) < f(a)
1 tan 1 b tan 1 a 1
2 < < Hence proved
1 b ba 1 a2
Example # 52 : Let f : R R be a twice differentiable function such that f(2) = 8, f(4) > 64, f(7) = 343 then show that
there exists a c (2, 7) such that f(c) < 6c.
Solution: Consider g(x) = f(x) – x3
By LMVT
g ( 4 ) g ( 2) g (7 ) g ( 4 )
= g(c1) , 2 < c1 < 4 and = g(c2), 4 < c2 < 7
42 74
g(c1) > 0 , g(c2) < 0
By LMVT
g (c 2 ) g (c 1 )
c 2 c1 = g(c), c1 < c < c2 g(c) < 0 f(c) – 6c < 0
(33) If a function f(x) satisfies the conditions of LMVT and f(x) = 0 for all x (a, b), then f(x) is constant
on [a, b].
(34) Using LMVT, prove that if two functions have equal derivatives at all points of (a, b), then they differ
by a constant
(35) If a function f is
(i) continuous on [a, b],
(ii) derivable on (a, b) and (iii) f(x) > 0, x (a, b), then show that f(x) is strictly increasing on [a, b].
AOD - 104
Tangent Normal & Its Applications in Conics
Geometrical Meaning of
Also, since normal is a line perpendicular to tangent at (x 1 , y1) so its equation is given by
1
(y – y1) = – (x – x 1), when f(x 1) is nonzero real.
f ( x1 )
If f(x 1) = 0, then tangent is the line y = y1 and normal is the line x = x 1.
f ( x 1 h) f ( x 1 )
If hlim
0
= or – , then x = x 1 is tangent (VERTICAL TANGENT) and y = y1 is normal.
h
Solution At x = 0 y = e0 = 1
dy dy
= ex dx =1
dx x 0
TN - 105
Tangent Normal & Its Applications in Conics
1
Example # 2 Find the equation of all straight lines which are tangent to curve y = and which are
x 1
parallel to the line x + y = 0.
Solution : Suppose the tangent is at (x1, y1) and it has slope – 1.
dy
= – 1.
dx ( x1 , y1 )
1
– = – 1.
( x1 1) 2
x1 = 0 or 2
y1 = – 1 or 1
Hence tangent at (0, – 1) and (2, 1) are the required lines (see figure) with equations
– 1(x – 0) = (y + 1) and – 1 (x – 2) = (y – 1)
x+y+1=0 and y+x=3
1
So the slope of normal at (– 2, 2) is .
3
Hence equation of normal is
1
(x + 2) = y – 2 3y = x + 8
3
Example # 4 Prove that sum of intercepts of the tangent at any point to the curve x + y = a on the
coordinate axis is constant.
Solution : Let P(x 1, y1) be a variable point on the curve x + y = a , as shown in figure.
equation of tangent at point P is
y1
– (x – x 1) = (y – y1)
x1
x y
– + x1 = – y1
x1 y1
x y x y
+ = x1 + y 1 + = a ( x1 y 1 a )
x1 y1 x1 y1
= a ( x1 + y1 ) = a. a =a (which is constant)
Example # 5 Find value of c such that line joining points (0, 3) and (5, – 2) becomes tangent to curve
c
y= .
x 1
Solution : Equation of line joining A & B is x + y = 3
TN - 106
Tangent Normal & Its Applications in Conics
c
3–x= x 2 – 2x + (c – 3) = 0 ......(i)
x 1
For tangency, roots of this equation must be equal.
Hence discriminant of quadratic equation = 0
4 = 4 (c – 3) c=4
Putting c = 4, equation (i) becomes
x 2 – 2x + 1 = 0 x=1
Hence point of contact becomes (1, 2).
Note : If a line touches a curve then on solving the equation of line and curve we get at least two repeated roots
corresponding to point of contact.
(2) Find the equation of the tangent and normal to the given curves at the given points.
(i) y = x 4 – 6x3 + 13x 2 – 10x + 5 at (1, 3)
x3
(ii) y2 = at (2, – 2).
4x
(3) Prove that area of the triangle formed by any tangent to the curve xy = c 2 and coordinate axes
is constant.
(4) A curve is given by the equations x = at2 & y = at3 . A variable pair of perpendicular lines through
the origin 'O' meet the curve at P & Q . Show that the locus of the point of intersection of the
tangents at P & Q is 4y2 = 3ax - a2 .
a
Answers : (1) –
2b
(2) (i) Tangent : y = 2x + 1, Normal :x + 2y = 7
(ii) Tangent : 2x + y = 2, Normal :x – 2y = 6
Given a point P(a, b) which does not lie on the curve y = f(x), then the equation of possible tangents to
the curve y = f(x), passing through (a, b) can be found by solving for the point of contact Q.
f (h ) b
f(h) =
ha
f (h ) b
And equation of tangent is y – b = (x – a)
ha
TN - 107
Tangent Normal & Its Applications in Conics
But, since line is tangent at P so x = 2 will be a repeated root of equation x 3 – 12x + 16 = 0 and
another root will be x = h. Using theory of equations :
sum of roots 2+2+h=0 h=–4
Hence coordinates of Q are (– 4, – 64)
(5) How many tangents are possible from origin to the curve y = (x + 1) 3. Also find the equation of
these tangents.
x9
(6) Find the equation of tangent to the hyperbola y = which passes through (0, 0) origin
x5
Let P (h, k) be any point on curve y = f(x). Let tangent drawn at point P meets x-axis at T & normal at
point P meets x-axis at N. Then the length PT is called the length of tangent and PN is called length of
normal. (as shown in figure)
Projection of segment PT on x-axis, TM, is called the subtangent and similarly projection of line segment
dy
PN on x axis, MN is called subnormal. Let m = = slope of tangent.
dx (h, k )
Hence equation of tangent is m (x – h) = (y – k).
k
Putting y = 0, we get x - intercept of tangent is x = h –
m
Similarly, the x-intercept of normal is x = h + km
Now, length PT, PN,TM, MN can be easily evaluated using distance formula
1
(i) PT = | k | 1 = Length of Tangent (ii) PN = | k | 1 m2 = Length of Normal
m2
k
(iii) TM = = Length of subtangent (iv) MN = |km| = Length of subnormal.
m
TN - 108
Tangent Normal & Its Applications in Conics
Example # 8 Prove that for the curve y = bex/a, the length of subtangent at any point is always constant.
Solution : y = bex/a
dy b.e x1 / a y1
Let the point be (x 1, y1) m= =
dx x x1 a a
y1 y1
Now, length of subtangent = = y1 / a = | a | ; which is always constant.
m
Example # 9 For the curve y = a n (x2 – a2) show that sum of lengths of tangent & subtangent at any point
is proportional to coordinates of point of tangency.
Solution : Let point of tangency be (x 1, y1)
dy 2ax1
m= = 2
dx x x1 x 1 a2
1 y1
Length of tangent + subtangent = |y1| 1 2 +
m m
( x 21 a 2 ) 2 2
y 1( x 1 a 2 ) 4
x1 a 4 2a 2 x1
2 2
y 1( x 1 a 2 )
= |y1| 1 2 + = |y1| +
4a 2 x 1 2ax 1 2 | ax 1 | 2ax 1
2 2
y 1( x 1 a 2 ) y 1( x 1 a 2 ) 2
| y 1 | ( 2 x1 ) x 1 y1
= 2ax 1 + 2ax 1 = =
2 | ax 1 | a
a a a 2 x 2
(8) Prove that the segment of the tangent to the curve y = n – a 2 x 2 contained
2 a a 2 x 2
between the y-axis & the point of tangency has a constant length .
(9) Find the length of the subnormal to the curve y2 = x 3 at the point (4, 8).
Answer : (9) 24
CHORD :
1. The equation of a chord joining t1 & t2 on parabola y2 = 4ax is 2x (t1 + t2) y + 2 at1 t2 = 0.
2. If t1 & t2 are the ends of a focal chord of the parabola y² = 4ax then t1t2 = 1. Hence the
a 2a
coordinates at the extremities of a focal chord can be taken as (at², 2at) & 2 ,
t t
Focal chord
A
S (focus)
B
3. Length of the focal chord making an angle with the x axis is 4acosec²
4. Semi latus rectum of the parabola y² = 4ax, is the harmonic mean between segments of any
focal chord of the parabola.
P
S 2(PS )(SQ )
= 2a
PS SQ
Q
TN - 109
Tangent Normal & Its Applications in Conics
Position of line w.r.t. conic :
(a) The line y = mx + c meets the parabola y² = 4ax in two points real, coincident or imaginary according
as a cm condition of tangency is, c = a/m.
Tangent
Secant
A
x2 y2
(b) The line y = mx + c meets the ellipse + = 1 in two points real, coincident or imaginary
a2 b2
according as c² is < = or > a²m² + b².
2 y2
Hence y = mx + c is tangent to the ellipse x 2 2 = 1 if c² = a²m² + b².
a b
x2 y2
(c) The straight line y = mx + c is a secant, a tangent or passes outside the hyperbola 1
a2 b2
according as : c 2 > or = or < a2 m 2 b2, respectively.
Example # 11 : Find the set of value(s) of '' for which the line 3x – 4y + = 0 intersect the ellipse
x2 y2
+ = 1 at two distinct points.
16 9
( 4 y )2 y2
Solution : Solving given line with ellipse, we get + =1
9 16 9
2y 2 y 2
– + –1 = 0
9 18 144
Since, line intersect the parabola at two distinct points,
roots of above equation are real & distinct
2 8 2
D0 – . 144 1 > 0 –12 2 < < 12 2
(18)2 9
x2 y2
Example # 12 : Prove that the straight line x + my + n = 0 touches the hyperbola – =1
a2 b2
if a22 – b2 m 2 = n2.
n
Solution : The given line is x + my + n = 0 or y=– x–
m m
Comparing this line with y = Mx + c
n
M=– and c = – ..........(1)
m m
x2 y2
This line (1) will touch the hyperbola – =1 if c 2 = a2M2 – b2
a2 b2
n2 a22
= – b2 or a 2 2 – b 2m 2 = n 2
m2 m2
TN - 110
Tangent Normal & Its Applications in Conics
Self practice problems :
(10) If the line y = 3x + intersect the parabola y2 = 4x at two distinct point's then set of value's of
'' is
x2 y2
(11) Find the value of '' for which 2x – y + = 0 touches the ellipse + =1
25 9
x2 y2
(12) Show that the line x cos + y sin = p touches the hyperbola – =1
a2 b2
if a2 cos2 – b2 sin2 = p2.
(13) For what value of does the line y = 2x + touches the hyperbola 16x 2 – 9y2 = 144 ?
Tangents to a conic :
(a) Tangents to the parabola y² = 4ax :
Equation of tangent at a point on the parabola can be obtained by replacement method or using derivatives.
In replacement method, following changes are made to the second degree equation to obtain T.
x2 x x1, y2 y y1, 2xy xy1 + x1y, 2x x + x1, 2y y + y1
So, it follows that the targents are :
a a 2a
(m 0) at 2 ,
(ii) y = mx +
m m m
(iii) t y = x + a t² at (at², 2at).
(iv) Point of intersection of the tangents at the point t1 & t2 is { at1 t2 , a(t1 + t2) }.
xcos ysin x2 y2
(iii) Parametric form: 1 is tangent to the ellipse 2 + 2 = 1 at the point
a b a b
(a cos , b sin ).
Note : (i) There are two tangents to the ellipse having the same m, i.e. there are two tangents parallel
to any given direction.These tangents touches the ellipse at extremities of a diameter.
cos sin 2
(ii) Point of intersection of the tangents at the point & is, a 2
, b
cos cos 2
2
(iii) The eccentric angles of the points of contact of two parallel tangents differ by .
x2 y2
1 , having slope 'm'.
a2 b2
TN - 111
Tangent Normal & Its Applications in Conics
x2 y2
(ii) Point form : Equation of tangent to the hyperbola 1 at the point (x 1, y1) is
a2 b2
.
x2 y2
(iii) Parametric form : Equation of the tangent to the hyperbola 1 at the point.
a2 b2
y nθ
(a sec , b tan ) is .
Note : (i) Point of intersection of the tangents at P(1) & Q(2) is
(ii) If |1 + 2| = , then tangents at these points (1 & 2) are parallel.
(iii) There are two parallel tangents having the same slope m. These tangents touches the hyperbola
at the extremities of a diameter.
a
Example # 13 : Prove that the straight line y = mx + c touches the parabola y2 = 4a (x + a) if c = ma +
m
a 1
y = m(x + a) + y = mx + a m
m m
but the given tangent is y = mx +c
a
c = am +
m
Example # 14 : A tangent to the parabola y2 = 8x makes an angle of 45° with the straight line y = 3x + 5. Find
its equation and its point of contact.
3 1
Solution : Slope of required tangent’s are m =
1 3
1
m 1 = – 2, m2 =
2
a
Equation of tangent of slope m to the parabola y2 = 4ax is y = mx + .
m
1 1
tangent’s y = – 2x – 1 at , 2 y= x + 4 at (8, 8)
2 2
Example # 15 : Find the equation to the tangents to the parabola y2 = 9x which goes through the point (4, 10).
9
Solution : Equation of tangent to parabola y2 = 9x is y = mx +
4m
Since it passes through (4, 10)
9
10 = 4m + 16 m 2 – 40 m + 9 = 0
4m
1 9
m= ,
4 4
x 9
equation of tangent’s are y= +9 & y= x + 1.
4 4
TN - 112
Tangent Normal & Its Applications in Conics
Example # 16 : Find the equations to the common tangents of the parabolas y2 = 4ax and x 2 = 4by.
Solution : Equation of tangent to y2 = 4ax is
a
y = mx + ........(i)
m
Equation of tangent to x = 4by is
2
b 1 b
x = m 1y + y= x– ........(ii)
m1 m1 (m1 ) 2
for common tangent, (i) & (ii) must represent same line.
1/ 3
1 a b a a
m1 = m & = – 2 = – bm 2 m =
m m1 m b
1/ 3 1/ 3
a b
equation of common tangent is y = x + a .
b a
Example # 17 : Find the equations of the tangents to the ellipse 3x 2 + 4y2 = 12 which are perpendicular to the
line y + 2x = 4.
1 x2 y2
Solution : Slope of tangent = m = . Given ellipse is + =1
2 4 3
Equation of tangent whose slope is 'm' is y = mx ± 4m 2 3
1 1
m= y= x± 1 3 2y = x ± 4
2 2
x2 y2
Example # 18 : A tangent to the ellipse 2 +
= 1 touches at the point P on it in the first quadrant and
a b2
meets the co-ordinate axes in A and B respectively. If P divides AB in the ratio 3 : 1, find the
equation of the tangent.
Solution : Let P ( a cos, b sin)
equation of tangent is
x y
cos + sin = 1
a b
A (a sec, 0)
B (0, b cosec)
P divide AB internally in the ratio 3 : 1
a sec 1 1
a cos = cos 2 = cos =
4 4 2
3b cos ec 3
and b sin = sin =
4 2
x 3y
tangent is + =1 bx + 3 ay = 2ab
2a 2b
Example # 19 : Prove that the locus of the point of intersection of tangents to an ellipse at two points whose
eccentric angle differ by a constant is an ellipse.
Solution : Let P (h, k) be the point of intersection of tangents at A() and B() to the ellipse.
a cos b sin 2 2
2 2 h k
h= &k= + = sec 2
a b 2
cos cos
2 2
but given that – =
TN - 113
Tangent Normal & Its Applications in Conics
x2 y2
locus is + =1
a 2 sec 2 b 2 sec 2
2 2
Example # 20 : Find the locus of foot of perpendicular drawn from centre to any tangent to the ellipse
x2 y2
+ = 1.
a2 b2
Solution : Let P(h, k) be the foot of perpendicular to a tangent y = mx + a 2m 2 b 2 .......(i)
from centre
k h
.m=–1 m=– .......(ii)
h k
P(h, k) lies on tangent
k = mh + a 2m 2 b 2 .......(iii)
from equation (ii) & (iii), we get
2
2 2 2
k h = a h + b2
k
k2
locus is (x 2 + y2)2 = a2x 2 + b2y2
Example # 21 : Find the equation of the tangent to the hyperbola x 2 – 4y2 = 36 which is perpendicular to the
line x – y + 4 = 0.
Solution : Let m be the slope of the tangent. Since the tangent is perpendicular to the line x – y = 0
m × 1 = –1 m=–1
Since x 2 – 4y2 = 36
x2 y2
or – =1
36 9
x2 y2
Comparing this with – =1
a2 b2
a2 = 36 and b2 = 9
y = –x + 27 x+y+3 3 =0
x2 y2
Example # 22 : Find the equation and the length of the common tangents to hyperbola – = 1 and
a2 b2
y2 x2
– = 1.
a2 b2
Solution : Tangent at (a sec , b tan ) on the 1st hyperbola is
x y
sec – tan = 1 .....(1)
a b
Similarly tangent at any point (b tan , a sec ) on 2nd hyperbola is
y x
sec – tan = 1 .....(2)
a b
TN - 114
Tangent Normal & Its Applications in Conics
If (1) and (2) are common tangents then they should be identical. Comparing the co–effecients of x
and y
sec tan
= – .....(3)
a b
tan sec
and – =
b a
a
or sec = – tan ......(4)
b
sec 2 – tan2 = 1
a2 b2
tan2 – sec2= 1 {from (3) and (4)}
b2 a2
a2 b2 a2 b2 b2
or tan2 – (1+ tan 2) = 1 or b2 a2 tan 2
= 1 +
b2 a2 a2
b2 a2
tan2 = and sec 2 = 1 + tan2 =
a2 b2 a2 b2
Hence the point of contact are
a2 b2 b2 a2
, and , {from (3) and (4)}
2 2 2 2 2 2 2 2
(a b ) (a b ) (a b ) (a b )
(a 2 b 2 )
Length of common tangent i.e., the distance between the above points is 2 and
(a 2 b 2 )
equation of common tangent on putting the values of sec and tan in (1) is
x y
+ 2 2 =1 or x y=+ (a 2 b 2 )
(a b ) (a b 2 )
2
Example # 23 :Find the locus of the point P from which tangents are drawn to parabola y2 = 4ax having slopes m 1,
m2 such that
(i) m1 + m2 = m0 (const) (ii) 1 + 2 = 0 (const)
Solution : Equation of tangent to y2 = 4ax, is
a
y = mx +
m
Let it passes through P(h, k)
m2h – mk + a = 0
k
(i) m1 + m2 = m0 = y = m 0x
h
m1 m 2 k /h
(ii) tan0 = 1 m m = y = (x – a) tan0
1 2 1 a/h
(15) Find the area of parallelogram formed by tangents at the extremities of latera recta of the
x2 y2
ellipse 1.
a2 b2
TN - 115
Tangent Normal & Its Applications in Conics
(16) If y1 is ordinate of a point P on the ellipse then show that the angle between its focal radius
b2
and tangent at it, is tan –1
aey .
1
x2 y2
(17) Find the eccentric angle of the point P on the ellipse + = 1 tangent at which, is
a2 b2
equally inclined to the axes.
(18) Find the equation of the tangent to the hyperbola x2 – y2 = 1 which is parallel to the line
4y = 5x + 7.
(19) If two tangents to the parabola y2 = 4ax from a point P make angles 1 and 2 with the axis of the
parabola, then find the locus of P in each of the following cases.
(i) tan21 + tan22 = (a constant) (ii) cos 1 cos 2 = (a constant)
x 2a 3
Answers : (14) y 2 (15)
2 a2 b2
b b b
(16) = ± tan–1 , – tan–1 , – + tan–1 (18) 4y = 5x ± 3
a a a
(19) (i) y2 – 2ax = x2 , (ii) x2 = 2 {(x – a)2 + y2}
Normal of conic :
(a) Normals to the parabola y² = 4ax :
2a P
Slope of tangent at (x1 , y1) = y
1 Normal
y1
Slope of normal = –
2a
y1
(i) y y1 = – (x x 1) at (x1, y1) ;
2a
(ii) y = mx 2am am 3 at (am 2, 2am)
(iii) y + tx = 2at + at3 at (at2, 2at).
NOTE :
2
t2, then t2 = – t1 .
t1
Q(t2)
(iii) If the normals to the parabola y² = 4ax at the points t 1 & t2 intersect again on the parabola at
the point 't3' then t1 t2 = 2; t3 = (t1 + t2) and the line joining t1 & t2 passes through a fixed point
(2a, 0)
TN - 116
Tangent Normal & Its Applications in Conics
(b) Normals to Ellip se:
x2 y2 a2 x b2 y
(i) Equation of the normal at (x1, y1) to the ellipse + = 1 is = a² b².
a2 b2 x1 y1
x2 y2
(ii) Equation of the normal at the point (acos , bsin ) to the ellipse + = 1 is;
a2 b2
ax. sec by. cosec = (a² b²).
x2 y2
(ii) The equation of the normal at the point P (a sec , b tan ) on the hyperbola 1 is
a2 b2
= a 2 + b 2 = a 2 e 2.
( )
(iii) Equation of normals in terms of its slope 'm' are y = mx .
2
Example # 24 : If the normal at point ‘t1’ intersects the parabola again at ‘t2’ then show that t2 = –t1 –
t1
2
Solution : Slope of normal at P = – t1 and slope of chord PQ =
t1 t 2
2
– t1 =
t1 t 2
2 2
t1 + t 2 = – t2 = – t 1 – .
t1 t1
Example # 25 : If the normals at points t1, t2 meet at the point t3 on the parabola then prove that
(i) t1 t2 = 2 (ii) t1 + t2 + t3 = 0
Solution : Since normal at t1 & t2 meet the curve at t3
2
t3 = – t 1 – .....(i)
t1
2
t3 = – t 2 – .....(ii)
t2
(t12 + 2) t2 = t1 (t22 + 2)
t1t2 (t1 – t2) + 2 (t2 – t1) = 0
t1 t2 , t1t2 = 2 ......(iii)
Hence (i) t1 t2 = 2
from equation (i) & (iii), we get t3 = – t1 – t2
Hence (ii) t1 + t2 + t3 = 0
TN - 117
Tangent Normal & Its Applications in Conics
x2 y2
Example # 26 : P and Q are corresponding points on the ellipse + = 1 and the auxiliary circles
a2 b2
respectively. The normal at P to the ellipse meets CQ in R, where C is the centre of the
ellipse. Prove that CR = a + b
Solution : Let P (acos , b sin)
Q (a cos, a sin)
Equation of normal at P is
(a sec) x – (b cosec ) y = a2 – b2 ..........(i)
equation of CQ is y = tan . x .........(ii)
Solving equation (i) & (ii), we get (a – b) x = (a2 – b2) cos
x = (a + b) cos, & y = (a + b) sin
R ((a + b) cos, (a + b) sin)
CR = a + b
Example # 27 : Prove that, in an ellipse, the distance between the centre and any normal does not exceed
the difference between the semi-axes of the ellipse.
x2 y2
Solution : Let the equation of ellipse is 1
a2 b2
Equation of normal at P () is (a sec)x – (bcosec )y – a2 + b2 = 0
distance of normal from centre
| a2 b2 |
= OR =
a 2 b 2 (a tan )2 (b cot )2
| a2 b2 |
=
(a b)2 (a tan b cot )2
| a2 b2 |
(a + b)2 + (a tan – b cot)2 (a + b) 2 or
(a b ) 2
|OR| (a – b)
x2 y2
Example #28 : A normal to the hyperbola – = 1 meets the axes in M and N and lines MP and NP are
a2 b2
drawn perpendicular to the axes meeting at P. Prove that the locus of P is the hyperbola
a2x 2 – b2y2 = (a2 + b2)2.
x2 y2
Solution : The equation of normal at the point Q (a sec , b tan ) to the hyperbola – = 1 is
a2 b2
ax cos + by cot = a2 + b2 ........(1)
The normal (1) meets the x–axis in
a2 b2 a2 b2
M sec , 0 and y–axis in N 0, tan
b
a
a2 b2 ax
x= sec or sec = (a 2 b 2 ) ........(2)
a
TN - 118
Tangent Normal & Its Applications in Conics
and the equation of NP, the line through N and perpendicular to the y–axis is
a2 b2 by
y= b tan or tan = (a 2 b 2 ) .........(3)
The locus of the point of intersection of MP and NP will be obtained by eliminating from (2)
and (3), we have sec 2 – tan2 = 1
a2 x 2 b2 y 2
– =1 or a2x 2 – b2y2 = (a2 + b2)2
(a 2 b 2 ) 2 (a 2 b 2 ) 2
is the required locus of P
(21) If the normal at point P(1, 2) on the parabola y2 = 4x cuts it again at point Q then Q = ?
(22) Find the length of normal chord at point ‘t’ to the parabola y2 = 4ax.
(23) If normal chord at a point 't' on the parabola y2 = 4ax subtends a right angle at the vertex then
prove that t2 = 2
(24) Prove that the chord of the parabola y2 = 4ax, whose equation is y – x 2 + 4a 2 = 0, is a
normal to the curve and that its length is 6 3a .
x2 y2
(25) Find the value(s) of 'k' for which the line x + y = k is a normal to the ellipse 1
a2 b2
x2 y2
(26) If the normal at the point P() to the ellipse = 1 intersects it again at the point Q(2)
14 5
then cos =
2 2 6 6
(A) – (B) (C) – (D)
3 3 7 7
x2 y2
(27) Prove that the line lx + my – n = 0 will be a normal to the hyperbola 2 – =1
a b2
a2 b2 (a 2 b 2 ) 2
if – = .
2 m2 n2
(28) Find the locus of the foot of perpendicular from the centre upon any normal to the hyperbola
x2 y2
– = 1.
a2 b2
3
a 2a a 2a 4a( t 2
1) 2
Answers : (20) , , , (21) (9, – 6) (22)
3
3 3 3 t2
(25) k=
a 2
b2
2
(26) A
a2 b2
(28) (x 2 + y2)2 (a2y2 – b2x 2) = x2y2 (a2 + b2)2
Co-normal points
If normal are drawn from a point P(h, k) to the parabola y2 = 4ax then
k = mh 2am am 3 i.e. am 3 + m(2a h) + k = 0.
2a h k
m1 + m2 + m3 = 0 ; m 1m 2 + m 2m 3 + m 3m 1 = ; m 1 m2 m3 = .
a a
TN - 119
Tangent Normal & Its Applications in Conics
Where m 1, m 2, & m 3 are the slopes of the three concurrent normals. Note that
P(h, k)
A, B, C Conormal points
B
C
Example # 29 Find the equation of all possible normal/s to the parabola x 2 = 4y drawn from point (1, 2).
h2
Solution : Let point Q be h, on parabola x 2 = 4y as shown in figure
4
Note : The equation gives only one real value of h, hence there is only one point of contact implying that only
one real normal is possible from point (1, 2).
Example # 30 : Find the locus of the point N from which 3 normals are drawn to the parabola y2 = 4ax are such that
(i) Two of them are equally inclined to x-axis
(ii) Two of them are perpendicular to each other
Solution : Equation of normal to y2 = 4ax is
y = mx – 2am – am 3
Let the normal passes through N(h, k)
k = mh – 2am – am3 am3 + (2a – h) m + k = 0
For given value’s of (h, k) it is cubic in ‘m’.
Let m1, m2 & m3 are root’s of above equation
m1 + m2 + m3 = 0 ......(i)
2a h
m 1m 2 + m 2m 3 + m 3m 1 = ......(ii)
a
k
m 1m 2m 3 = – ......(iii)
a
(i) If two normal are equally inclined to x-axis, then m 1 + m2 = 0
m3 = 0 y=0
TN - 120
Tangent Normal & Its Applications in Conics
(ii) If two normal’s are perpendicular
m1 m2 = – 1
k
from (3) m3 = .....(iv)
a
k 2a h
from (2) –1+ (m1 + m2) = .....(v)
a a
k
from (1) m1 + m2 = – .....(vi)
a
from (5) & (6), we get
k2 h
–1– =2–
a a
y2 = a(x – 3a)
Pair of tangents:
The equation to the pair of tangents which can be drawn from any point (x 1, y1) to the parabola y² = 4ax
is given by: SS1 = T² where :
S y² 4ax ; S1 = y1² 4ax1 ; T y y1 2a(x + x1).
A
(x1, y1 ) P
B
The equation to the pair of tangents which can be drawn from any point (x1, y1) to the ellipse
x2 y2
= 1 is given by: SS1 = T² where :
a2 b2
2 2
x2 y2 x1 y1 xx1 yy1
S –1 ; S1 = 2
2 –1; T + – 1.
2
a 2
b 2 a b a b2
The equation to the pair of tangents which can be drawn from any point (x 1, y1) to the hyperbola
x2 y
2
Example # 31 : Write the equation of pair of tangents to the parabola y2 = 4x drawn from a point P(–1, 2)
Solution : We know the equation of pair of tangents are given by SS 1 = T²
(y2 – 4x) (4 + 4) = (2y – 2 (x – 1))2
8y2 – 32x = 4y2 + 4x2 + 4 – 8xy + 8y – 8x
y2 – x2 + 2xy – 6x – 2y = 1
TN - 121
Tangent Normal & Its Applications in Conics
x2 y2
Example # 32 : How many real tangents can be drawn from the point (4, 3) to the ellipse + = 1. Find
16 9
the equation of these tangents & angle between them.
Solution : Given point P (4, 3)
x2 y2
ellipse S + –1=0
16 9
16 9
S1 + –1=1>0
16 9
Point P (4, 3) lies outside the ellipse.
Two tangents can be drawn from the point P(4, 3).
Equation of pair of tangents is SS1 = T 2
x2 y2 2
1 . 1 =
4x 3y
1
16 9
16 9
x2 y2 x2 y2 xy x 2y
+ –1= + +1+ – –
16 9 16 9 6 2 3
– xy + 3x + 4y – 12 = 0 (4 – x) (y – 3) = 0
x=4&y=3
and angle between them =
2
Alternative
By direct observation
x = 4, y = 3 are tangents.
x2 y2
Example # 33 : How many real tangents can be drawn from the point (4, 3) to the hyperbola – =1.
16 9
Find the equation of these tangents & angle between them.
Solution : Given point P (4, 3)
x2 y2
Hyperbola S – –1=0
16 9
16 9
S1 – –1=–1<0
16 9
Point P (4, 3) lies outside the hyperbola.
Two tangents can be drawn from the point P(4, 3).
Equation of pair of tangents is SS1 = T 2
x2 y2 2
1 . (– 1) = 4 x 3 y 1
16 9
16 9
x2 y2 x2 y2 xy x 2y
– + +1= + +1– – +
16 9 16 9 6 2 3
3x 2 – 4xy – 12x + 16y = 0
4
= tan–1
3
TN - 122
Tangent Normal & Its Applications in Conics
Self Practice Problem
(30) Find the locus of point of intersection of the tangents drawn at the extremities of a focal
x2 y2
chord of the ellipse + = 1.
a2 b2
a
Answer : (27) x = ±
e
Director circle:
Locus of the point of intersection of the perpendicular tangents to a curve is called the Director Circle.
For parabola y2 = 4ax it’s equation is x + a = 0 which is parabola’s own directrix.
x2 y2
For ellipse = 1, t. The equation to this locus is x² + y² = a² + b² i.e. a circle whose centre
a2 b2
is the centre of the ellipse & whose radius is the length of the line joining the ends of the major & minor
axes.
x2 y2
For hyperbola = 1, t the equation to the director circle is : x 2 + y2 = a2 b2.
a2 b2
If b2 < a2 , then the director circle is real.
If b2 = a2 (i.e. rectangular hyperbola), then the radius of the director circle is zero and it reduces to a
point circle at the origin. In this case centre is the only point from which two perpendicular tangents
can be drawn on the curve.
If b2 > a2, then the radius of the director circle is imaginary, so that there is no such circle and
so no pair of tangents at right angle can be drawn to the curve.
x2 y2
Example # 34 : Find the locus of point of intersection of perpendicular tangents to the ellipse =1
a2 b2
Solution : Let P(h, k) be the point of intersection of two perpendicular tangents
equation of pair of tangents is SS1 = T 2
x2 y2 h2 k 2 hx ky
2
1 1
a2 b2 a2 b2 = 2 2 1
a b
x 2 k 1 h2
2
y2 1 + ........ = 0
2 + a2 .........(i)
a2 b b2
Since equation (i) represents two perpendicular lines
1 k2 1 h2
1 + 1 = 0
b 2 a 2
a2 b2
k 2 – b2 + h2 – a2 = 0 locus is x 2 + y2 = a2 + b2
Example # 35 : An ellipse slides between two perpendicular lines. Show that the locus of its centre is a circle.
Solution : Let length of semi-major and semi-minor axis are 'a' and 'b' and centre is C (h, k)
Since ellipse slides between two perpendicular lines, therefore point of intersection of two
perpendicular tangents lies on director circle.
Let us consider two perpendicular lines as x & y axes
point of intersection is origin O (0, 0)
OC = radius of director circle
2
h2 k 2 = a b
2
locus of C (h, k) is
x 2 + y2 = a2 + b2 which is a circle
TN - 123
Tangent Normal & Its Applications in Conics
x2 y2
Example # 36 : Find the locus of point of intersection of perpendicular tangents to the hyperbola =1
a2 b2
Solution : Let P(h, k) be the point of intersection of two perpendicular tangents. Equation of pair of
tangents is SS1 = T 2
x2 y2 h2 k 2 hx ky
2
1 1 1
a2 b2 a2 b2 =
a2 b2
x2 k2 2 h2
1 – y 1 + ........ = 0 .........(i)
b2 a2
a2 b2
Since equation (i) represents two perpendicular lines
1 k2 1 h2
1 – 1 = 0
b2 a2
a2 b2
– k 2 – b2 – h2 + a2 = 0 locus is x 2 + y2 = a2 – b2
(31) A tangent to the ellipse x 2 + 4y2 = 4 meets the ellipse x2 + 2y2 = 6 at P and Q. Prove that the
tangents at P and Q of the ellipse x 2 + 2y2 = 6 are at right angles.
Chord of contact:
Equation to the chord of contact of tangents drawn from a point P(x 1, y1) to parabola y2 = 4ax is
yy1 = 2a (x + x1). A
P
(x1, y1 ) Chord of contact
x2 y2
Equation to the chord of contact of tangents drawn from a point P(x1, y1) to the ellipse 2 + = 1 is
a b2
xx1 yy1
T = 0, where T= 2 + –1
a b2
Equation to the chord of contact of tangents drawn from a point P(x 1, y 1 ) to the hyperbola
x2 y2 xx1 yy1
– = 1 is , T = 0, where T = 2 – –1
a 2
b 2
a b2
NOTE : The area of the triangle formed by the tangents to parabola y2 = 4ax from the point (x 1, y1) &
1
the chord of contact is (y1² 4ax 1)3/2
2a
Chord with a given middle point:
Equation of the chord of the parabola y² = 4ax whose middle point is
2a M(x,y)
(x 1, y1) is y y1 = (x x 1) T = S1
y1
2 y2
Equation of the chord of the ellipse x 2 2 = 1 whose middle point is (x 1, y1) is T = S1,
a b
TN - 124
Tangent Normal & Its Applications in Conics
2 2
x1 y1 xx1 yy1
where S1 = 2
2 –1; T + – 1.
2
a b a b2
x 2 y2
Equation of the chord of the hyperbola a = 1 whose middle point is (x 1, y1) is T = S1,
2
b2
2 2
x1 y1 xx1 yy1
where S1 = –1; T – – 1.
a2 b2 a 2
b2
Example # 37 : If t1, t2 are end points of a focal chord then show that t1 t2 = –1.
Solution : Let parabola is y2 = 4ax 2
P (at1 , 2at1)
since P, S & Q are collinear
S(a, 0)
2 2t 1
mPQ = mPS t1 t 2 = t12 1
2
Q (at2 , 2at2)
t1 2 – 1 = t 1 2 + t 1 t2 t1 t2 = – 1
Example # 38 : If the endpoint t1, t2 of a chord satisfy the relation t1 t2 = k (const.) then prove that the chord always
passes through a fixed point. Find the point?
Solution : Equation of chord joining (at12, 2at1) and (at22, 2at2) is
2
y – 2at1 = t t (x – at12) (t1 + t2) y – 2at12 – 2at1t2 = 2x – 2at12
1 2
2
y = t t (x + ak) ( t1t2 = k)
1 2
This line passes through a fixed point (– ak, 0).
Example # 39 : Find the length of chord of contact of the tangents drawn from point (x 1, y1) to the parabola y2 = 4ax.
Solution : Let tangent at P(t1) & Q(t2) meet at (x1, y1)
at1t2 = x1 & a(t1 + t2) = y1
PQ = (at 12 at 22 )2 (2a( t1 t 2 ))2
( y12 4ax 1 )( y12 4a 2 )
=a (( t1 t 2 )2 4t 1t 2 )((t1 t 2 )2 4) =
a2
Example # 40 : If the line x – y – 1 = 0 intersect the parabola y2 = 8x at P & Q, then find the point of intersection of
tangents at P & Q.
Solution : Let (h, k) be point of intersection of tangents then chord of contact is
yk = 4(x + h)
4x – yk + 4h = 0 .....(i)
But given is x – y – 1 = 0
4 k 4h
= = h = – 1, k = 4
1 1 1
point (–1, 4)
TN - 125
Tangent Normal & Its Applications in Conics
Example # 41 : Find the locus of point whose chord of contact w.r.t to the parabola y2 = 4bx is the tangents of the
parabola y2 = 4ax.
a
Solution : Equation of tangent to y2 = 4ax is y = mx + ......(i)
m
Let it is chord of contact for parabola y2 = 4bx w.r.t. the point P(h, k)
Equation of chord of contact is yk = 2b(x + h)
2b 2bh
y= x+ .....(ii)
k k
From (i) & (ii)
2b a 2bh 4b 2h
m= , = a=
k m k k2
4b 2
locus of P is y2 = x
a
Example # 42 : Find the locus of middle point of the chord of the parabola y2 = 4ax which pass through a given
point (p, q).
Solution : Let P(h, k) be the mid point of chord of parabola y2 = 4ax,
so equation of chord is yk – 2a(x + h) = k 2 – 4ah.
Since it passes through (p, q)
qk – 2a (p + h) = k2 – 4ah
Required locus is y2 – 2ax – qy + 2ap = 0.
Example # 43 : Find the locus of middle point of the chord of the parabola y2 = 4ax whose slope is ‘m’.
Solution : Let P(h, k) be the mid point of chord of parabola y2 = 4ax,
so equation of chord is yk – 2a(x + h) = k 2 – 4ah.
2a 2a
but slope = =m locus is y =
k m
x2 y2
Example # 44 : If tangents to the parabola y2 = 4ax intersect the ellipse 2 + = 1 at A and B, then find the
a b2
locus of point of intersection of tangents at A and B.
Solution : Let P (h, k) be the point of intersection of tangents at A & B
xh yk
equation of chord of contact AB is 2 + =1 .............(i)
a b2
which touches the parabola equation of tangent to parabola y2 = 4ax
a
y = mx +
m
a
mx – y = – .............(ii)
m
equation (i) & (ii) as must be same
a
1
m m
= =
h k 1
2 2
a
b
2
h b ak
m=– 2 & m =
k a b2
hb 2 ak b4
– = 2 locus of P is y2 = – .x
ka 2 b a3
TN - 126
Tangent Normal & Its Applications in Conics
x2 y2
Example # 45 : Find the locus of the mid - point of focal chords of the ellipse + = 1.
a2 b2
Solution : Let P (h, k) be the mid-point
xh yk h2 k2
equation of chord whose mid-point is given 2 + 2 –1= + –1
a b a2 b2
since it is a focal chord,
it passes through focus, either (ae, 0) or (–ae, 0)
If it passes through (ae, 0)
ex x2 y2
locus is = 2 + 2
a a b
If it passes through (–ae, 0)
ex x2 y2
locus is – = 2 + 2
a a b
x2 y2
Example # 46 : Find the condition on 'a' and 'b' for which two distinct chords of the ellipse + =1
2a 2 2b 2
passing through (a, – b) are bisected by the line x + y = b.
x y(b ) 2 (b ) 2
+ = +
2a 2 2b 2 2a 2 2b 2
Since it passes through (a, –b)
(b ) 2 (b )2
– = 2 +
2a 2b 2a 2b 2
1 1 1 1 2
– 1 = 2 2 2 – +1
a b a b b
1 1 3 1
2 2 2 – + 2 = 0
a b b a
since line bisect two chord
above quadratic equation in must have two distinct real roots
2
3 1 1 1
–4 2 2 .2>0
b a a b
9 1 6 8 8 1 7 6
2 + 2 + – 2 – 2 >0 2 – + >0
b a ab a b b a2 ab
a2 – 7b2 + 6ab > 0
a2 > 7b2 – 6ab which is the required condition.
x2 y2
Example # 47 : If tangents to the parabola y2 = 4ax intersect the hyperbola – = 1 at A and B, then find
a2 b2
the locus of point of intersection of tangents at A and B.
Solution : Let P (h, k) be the point of intersection of tangents at A & B
xh yk
equation of chord of contact AB is – =1 .............(i)
a 2 b2
TN - 127
Tangent Normal & Its Applications in Conics
which touches the parabola
equation of tangent to parabola y2 = 4ax
a a
y = mx + mx – y = – .............(ii)
m m
equation (i) & (ii) as must be same
a
1 2
ak
m m h b
= = m= 2 & m = –
h k 1 k a b2
2 2
a b
hb 2 ak b4
=– locus of P is y2 = – .x
ka 2 b2 a3
x2 y2
Example # 48 : Find the locus of the mid - point of focal chords of the hyperbola – = 1.
a2 b2
Solution : Let P (h, k) be the mid-point
xh yk h2 k2
equation of chord whose mid-point is given is – –1= – –1
a2 b2 a2 b2
since it is a focal chord,
it passes through focus, either (ae, 0) or (–ae, 0)
If it passes through (ae, 0)
ex x2 y2
locus is = 2 – 2
a a b
If it passes through (–ae, 0)
ex x2 y2
locus is – = 2 – 2
a a b
x2 y2
Example # 49 : Find the condition on 'a' and 'b' for which two distinct chords of the hyperbola 2 – =1
2a 2b 2
passing through (a, b) are bisected by the line x + y = b.
Solution : Let the line x + y = b bisect the chord at P(, b – )
equation of chord whose mid-point is P(, b – )
x y(b ) 2 (b ) 2
– = –
2a 2 2b 2 2a 2 2b 2
Since it passes through (a, b)
(b ) 2 (b )2
– = –
2a 2b 2a 2 2b 2
1 1 1 1
2 2 2 + = 0
a b b a
1
= 0, = a±b
1 1
a b
x2 y2
Example # 50 : Find the locus of the mid point of the chords of the hyperbola – = 1 which subtend a
a2 b2
right angle at the origin.
Solution : let (h,k) be the mid–point of the chord of the hyperbola. Then its equation is
hx ky h2 k2 hx ky h2 k2
– –1= – –1 or – = – ........(1)
a2 b2 b2 b2 a2 b2 a2 b2
TN - 128
Tangent Normal & Its Applications in Conics
The equation of the lines joining the origin to the points of intersection of the hyperbola and the chord
(1) is obtained by making homogeneous hyperbola with the help of (1)
2
hx ky
2 2
x2 y2 a b
– = 2
a2 b2 h2 k 2
a2 b2
2
1 h k
2 2 2
h2 k 2 h2 2 k2
2 2
2 x2 – 1 y2 =
2hk
x + 4 y2 – 2 2 xy .......(2)
a a b b2 a2 b2 a 4
b a b
The lines represented by (2) will be at right angle if coefficient of x + coefficient of y2 = 0
2
2 2
1 h2 k 2 2 h2 k 2 2
– h – 1 – k =0
a2 a2 b2 a4 b2 a2 b2 b4
2
h2 k 2 1 1 h2 k2
2 2 = +
a2 b2 a b a4 b4
2
x2 y2 1 1 x2 y2
hence, the locus of (h,k) is 2 2 2 2 = +
a b a b a4 b4
Self Practice Problems
(33) If one end of focal chord of parabola y2 = 16x is (16, 16) then coordinate of other end is.
(34) If PSQ is focal chord of parabola y2 = 4ax (a > 0), where S is focus then prove that
1 1 1
+ = .
PS SQ a
(35) Find the length of focal chord whose one end point is ‘t’.
(36) Prove that locus of a point whose chord of contact w.r.t. parabola passes through focus is directrix
(37) If from a variable point ‘P’ on the line x – 2y + 1 = 0 pair of tangent’s are drawn to the parabola
y2 = 8x then prove that chord of contact passes through a fixed point, also find that point.
(38) Find the equation of chord of parabola y2 = 4x whose mid point is (4, 2).
(39) Find the locus of mid - point of chord of parabola y2 = 4ax which touches the parabola x2 = 4by.
(40) Find the locus of point of intersection of tangents at the extremities of normal chords of the
x2 y2
ellipse + = 1.
a2 b2
(41) Find the locus of point of intersection of tangents at the extremities of the chords of the
x2 y2
ellipse 2 + = 1 subtending a right angle at its centre.
a b2
x2 y2
(42) Find the equation of the chord + = 1 which is bisected at (2, 1).
36 9
TN - 129
Tangent Normal & Its Applications in Conics
x2 y2
(43) Find the locus of the mid-points of normal chords of the ellipse + = 1.
a2 b2
x2 y2 1 2
(44) Find the length of the chord of the ellipse + = 1 whose middle point is ,
25 16 2 5
x2 y2
(45) Find the equation of the chord – = 1 which is bisected at (2, 1).
36 9
(46) Find the point 'P' from which pair of tangents PA & PB are drawn to the hyperbola
x2 y2
– = 1 in such a way that (5, 2) bisect AB
25 16
(47) From the points on the circle x 2 + y2 = a2, tangent are drawn to the hyperbola x 2 – y2 = a2,
prove that the locus of the middle points of the chords of contact is the curve
(x 2 – y2)2 = a2 (x 2 + y2).
a6 b6
(39) y (2ax – y ) = 4a b
2 2
(40) + = (a2 – b2)2
x2 y2
x2 y2 1 1
(41) + = 2 + (42) x + 2y = 4
a 4
b 4
a b2
2
x2 y2 a6 b6 7
(43) a2 b2 x 2 y 2 = (a – b )
2 2 2
(44) 41
5
20 8
(45) x = 2y (46) ,
3 3
Important Highlights:
(i) If the tangent & normal at any point ‘P’ of the parabola intersect the axis at T & G then
ST = SG = SP where ‘S’ is the focus. In other words the tangent and the normal at a point P on
the parabola are the bisectors of the angle between the focal radius SP & the perpendicular
from P on the directrix. From this we conclude that all rays emanating from S will become
parallel to the axis of theparabola after reflection.
TN - 130
Tangent Normal & Its Applications in Conics
x2 y2
(ii) Refering to the ellipse =1
a2 b2
The tangent & normal at a point P on the ellipse bisect the external & internal angles between
the focal distances of P. This refers to the well known reflection property of the ellipse which
states that rays from one focus are reflected through other focus & viceversa. Hence we can
deduce that the straight lines joining each focus to the foot of the perpendicular from the other
focus upon the tangent at any point P meet on the normal PG and bisects it where G is the
point where normal at P meets the major axis.
(iii) The tangent & normal at any point of a hyperbola bisect the angle between the focal radii.
This explains the reflection property of the hyperbola as "An incoming light ray " aimed
towards one focus is reflected from the outer surface of the hyperbola towards the other
focus. It follows that if an ellipse and a hyperbola have the same foci, they cut at right angles
at any of their common point.
2 2
x2 y2 x y
Note that the ellipse 2 1 & the hyperbola 2 2
2 = 1 (a > k > b > 0) are
a 2
b a k k b2
confocal and therefore orthogonal.
(iv) The portion of a tangent to a parabola cut off between the directrix & the curve subtends a right
angle at the focus.
(v) The portion of the tangent to an ellipse between the point of contact & the directrix subtends
a right angle at the corresponding focus.
(vi) The portion of the tangent to a hyperbola between the point of contact & the directrix subtends
a right angle at the corresponding focus.
S
T
TN - 131
Tangent Normal & Its Applications in Conics
(vii) Any tangent to a parabola & the perpendicular on it from the focus meet on the tangent at the
vertex.
x2 y2
(viii) Locus of the feet of the perpendicular drawn from focus of the hyperbola 1 upon
a2 b2
any tangent is its auxiliary circle i.e. x2 + y2 = a2 & the product of these perpendiculars is b2.
P
T
S S
T
(ix) The area of the triangle formed by three points on a parabola is twice the area of the triangle
formed by the tangents at these points.
(xi) If the normal at any point P on the ellipse with centre C meet the major & minor axes in G &
g respectively, & if CF be perpendicular upon this normal, then
(i) PF. PG = b²
(ii) PF. Pg = a²
(iii) PG. Pg = SP. S P
(iv) CG. CT = CS2
(v) locus of the mid point of Gg is another ellipse having
the same eccentricity as that of the original ellipse.
[where S and S are the focii of the ellipse and T is the
point where tangent at P meet the major axis]
(xii) The circle on any focal distance as diameter touches the auxiliary circle. Perpendiculars from
the centre upon all chords which join the ends of any perpendicular diameters of the ellipse
are of constant length.
(xiii) If the tangent at the point P of a standard ellipse meets the axis in
T and t and CY is the perpendicular on it from the centre then,
(i) T t. PY = a2 b2
and
(ii) least value of T t is a + b
TN - 132
Tangent Normal & Its Applications in Conics
Example # 51 : A ray originating from the point (5, 0) is incident on the hyperbola 9x 2 – 16y2 = 144 at the point
P with abscissa 8. Find the equation of the reflected ray after first reflection and point P lying
in first quadrant.
Solution : Given hyperbola is 9x2 – 16y2 = 144. This equation can be
x2 y2
rewritten as – =1 ......(1)
16 9
Since x co–ordinate of P is 8. Let y co–ordinate of P is .
(8,) lies on (1)
64 2
– =1 2 = 27
16 9
=3 3 ( P lies in first quadrant)
03 3
Its equation is y – 3 3 = (x – 8)
58
or 13y – 39 3 = 3 3 x – 24 3 or 3 3 x – 13y + 15 3 = 0
(52) Show that the locus of the foot of the perpendicular on a varying tangent to an ellipse from
either of its foci is a concentric circle.
(53) Prove that the portion of the tangent to an ellipse intercepted between the ellipse and the
directrix subtends a right angle at the corresponding focus.
Angle between the curves
Angle between two intersecting curves is defined as the acute angle between their tangents (or normals) at
the point of intersection of two curves (as shown in figure).
m1 m 2
tan =
1 m1m 2
where m1 & m2 are the slopes of tangents at the intersection point (x1, y1).
Notes : (i) The angle is defined between two curves if the curves are intersecting. This can be ensured by
finding their point of intersection or graphically.
(ii) If the curves intersect at more than one point then angle between curves is found out with respect to
the point of intersection.
(iii) Two curves are said to be orthogonal if angle between them at each point of intersection is right
angle. i.e. m1 m2 = – 1.
Example # 52 Find angle between y2 = 4x and x2 = 4y. Are these two curves orthogonal?
Solution : y2 = 4x and x2 = 4y intersect at point (0, 0) and (4, 4) (see figure).
C1 : y2 = 4x C2 : x2 = 4y
TN - 133
Tangent Normal & Its Applications in Conics
dy 2 dy x
= y =
dx dx 2
dy dy
dx dx =0
( 0, 0 ) ( 0, 0 )
1
2
2 3
tan = 1 =
1 2. 4
2
Two curves are not orthogonal because angle between them at (4, 4) is not 90°.
dy 1 x / 2 y1
and for y = e–x/2 dx =– e 1 =–
( x1, y1 ) 2 2
m1 m2 = – 1 Hence = 90°
Note that we have not actually found the intersection point but geometrically we can see that the curves intersect.
Example # 54 Find possible values of p such that the equation px 2 = nx has exactly one solution.
Solution : Two curves must intersect at only one point.
(i) (ii)
1 1
also y1 = px12 y1 = p y1 = .........(ii)
2p 2
1
and y1 = nx1 = nx1 x1 = e1/2 .........(iii)
2
1 1 1
x 12 = e= p=
2p 2p 2e
1
Hence possible values of p are (– , 0]
2e
TN - 134
Tangent Normal & Its Applications in Conics
Self practice problem :
(54) Find the angle of intersection of the following curves:
x2 y2
(i) y = x2 & 6y = 7 x3 at (1, 1) (ii) x 2 – y2 = 5 & = 1.
18 8
Answers : (54) (i) /2 (ii) /2
Example # 55 Find the shortest distance between the line y = x – 2 and the parabola y = x 2 + 3x + 2.
Solution : Let P(x1, y1) be a point closest to the line y = x – 2
dy
then dx = slope of line
( x1 , y1 )
2x1 + 3 = 1
x1 = – 1
y1 = 0
Hence point (– 1, 0) is the closest and its perpendicular distance from the line y = x – 2 will give the
shortest distance
3
p= .
2
x2 y2
Example # 56 : Find the shortest distance between the line x + y = 10 and the ellipse =1
16 9
Solution : Shortest distance occurs between two non-intersecting curve always along common normal.
Let 'P' be a point on ellipse and Q is a point on given line for which PQ is common normal.
Tangent at 'P' is parallel to given line
Equation of tangent parallel to given line is (y = mx ± a 2m 2 b 2 )
y=–x±5
x + y + 5 = 0 or x+y–5=0
minimum distance = distance between
x + y – 10 = 0 & x + y – 5 = 0
| 10 5 | 5
shortest distance = =
1 1 2
(55) Find the minimum & maximum values of (x + 2)2 + (y – 1)2, if (x – 2)2 + (y + 1) 2 4.
Answer : (55) 2 5 2, 2 5 2
TN - 135
Tangent Normal & Its Applications in Conics
Section (A) : Equation of Tangent and Normal, length of tangent, normal, subtangent &
subnormal
A-1. Find the equation of normal to the curve x3 + y3 = 8xy at point where it is meet by the curve y2 = 4x,
other than origin.
A-2. If the tangent to the curve xy + ax + by = 0 at (1, 1) is inclined at an angle tan–1 2 with positive x-axis
in anticlockwise, then find a and b ?
A-3. The normal to the curve 5x5 – 10x3 + x + 2y + 6 = 0 at the point P(0, –3) is tangent to the curve at the point(s).
Find those point(s)?
A-4. Prove that the length of segment of all tangents to curve x 2/3 + y2/3 = a2/3 intercepted between coordinate
axes is same.
A-5. If the tangent at (1, 1) on y2 = x(2 – x)2 meets the curve again at P, then find coordinates of P
A-6. Find equations of tangents drawn to the curve y2 – 2x 2 – 4y+ 8 = 0 from the point (1, 2).
A-7_. Find equation of all possible normals to the parabola x2 = 4y drawn from point (1, 2).
A-8_. Find equations of tangents to curve y = x 4 drawn from point (2, 0).
A-9. Find the equation of tangents to the parabola y2 = 9x, which pass through the point (4, 10).
A-10_. If ax + by = 1 is a normal to the parabola y2 = 4Px, then prove that Pa3 + 2aPb2 = b2.
A-11. Find length of subnormal to x = 2 cos t, y = – 3sin t at t = 4 .
A-12. Show that subnormal at any point on the curve x 2y2 = a2(x 2 – a2) varies inversely as the cube of its
abscissa.
B-1_. A line y = x + 5 intersect the parabola (y – 3)2 = 8(x + 2) at A & B. Find the length of chord AB.
16
B-2_. Chord joining two distinct points P(2, k1) and Q k 2 , on the parabola y2 = 16x always passes through
a fixed point. Find the co-ordinate of fixed point.
B-3_. Find the set of possible value of for which point P(, 3) lies on the smaller region of the ellipse
9x2 + 16y2 = 144 divided by the line 3x + 4y = 12.
x2 y2
B-5_. Find the length of chord x – 3y – 3 = 0 of hyperbola = 1.
9 4
B-6. Prove that the straight line x + my + n = 0 touches the parabola y2 = 4ax if n = am 2.
TN - 136
Tangent Normal & Its Applications in Conics
B-7. Find the range of c for which the line y = mx + c touches the parabola y2 = 8 (x + 2).
B-8. Find the equation of that tangent to the parabola y2 = 7x which is parallel to the straight line
4y – x + 3 = 0. Find also its point of contact.
B-9. If two tangents to the parabola y2 = 4ax from a point P make angles 1 and 2 with the axis of the parabola,
then find the locus of P in each of the following cases.
(i) 1 + 2 = (a constant) (ii) 1 + 2 =
2
(iii) tan 1 + tan2 = (is constant)
x2 y2
B-10. Check whether the line 4x + 5y = 40 touches the ellipse + = 1 or not. If yes, then also find its
50 32
point of contact.
B-11. The equation of a tangent to the parabola y2 = 8x is y = x + 2. Find the point on this line from which the other
tangents to the parabola is perpendicular to the given tangent.
x2 y2
B-12. Find the equation of tangents to the ellipse + = 1 which passes through a point (15, – 4).
50 32
B-13. If tangent at P and Q to the parabola y2 = 4ax intersect at R then prove that mid point of R and M lies
on the parabola, where M is the mid point of P and Q.
B-14. For what value of , does the line y = 3x + touch the hyperbola 9x 2 – 5y2 = 45 ?
x2 y2
B-15. If the straight line 2x + 2 y + n = 0 touches the hyperbola – = 1, then find the value of n.
9 16
B-16. An ellipse passes through the point (4, 1) and touches the line x + 4 y 10 = 0. Find its equation if
its axes coincide with coordinate axes.
x2 y2
B-17. Find the equation of the tangents at the ends of the latus rectum of the ellipse + = 1 and also
a2 b2
show that they pass through the points of intersection of the major axis and directrices.
B-18. Any tangent to an ellipse is cut by the tangents at the ends of major axis in the points T and T . Prove
that the circle, whose diameter is T T will pass through the focii of the ellipse.
B-19. Find the equation of the tangent to the hyperbola x² 4y² = 36 which is perpendicular to the line
xy+4=0.
B-20. The tangent & normal at a point on x 2/a2 y2/b2 = 1 cut the y axis respectively at A & B. Prove that the
circle on AB as diameter passes through the focii of the hyperbola .
B-21. If m 1 & m 2 are the slopes of the tangents to the hyperbola x 2/25 – y2/16 = 1 which passes through the
point (4, 2), find the value of (i) m 1 + m 2 & (ii) m 1 m 2 .
Section (C) : Equation of normal, co-normal points, Properties of tangent and normal in
conics
TN - 137
Tangent Normal & Its Applications in Conics
C-2. The normal at the point P(ap2, 2ap) meets the parabola y2 = 4ax again at Q(aq2 , 2aq) such that the lines
joining the origin to P and Q are at right angle. Then prove that p2 = 2.
x2 y2
C-3. If the normal at an end of a latus-rectum of an ellipse + = 1 passes through one extremity of
a2 b2
the minor axis, show that the eccentricity of the ellipse is given by e4 + e2 – 1 = 0
C-4. The normal at P to a hyperbola of eccentricity e, intersects its transverse and conjugate axes at L and
e
M respectively. Show that the locus of the middle point of LM is a hyperbola of eccentricity .
2
( e 1)
C-5. If a line x + y = 1 cut the parabola y2 = 4ax in points A and B and normals drawn at A and B meet at C (C does
not lies on parabola). The normal to the parabola from C other, than above two meet the parabola in D, then
find D
x2 y2 25
C-6. If 'P' be a moving point on the ellipse + = 1 in such a way that tangent at 'P' intersect x =
25 16 3
at Q then circle on PQ as diameter passes through a fixed point. Find that fixed point.
C-7. A ray emanating from the point ( 4, 0) is incident on the ellipse 9x² + 25y² = 225 at the point P with
abscissa 3. Find the equation of the reflected ray after first reflection.
Section (D) : Pair of tangents, Director Circle, Chord of contact, Chord in terms of midpoint
D-1. If normal of circle x2 + y2 + 6x + 8y + 9 = 0 intersect the parabola y2 = 4x at P and Q then find the locus of
point of intersection of tangent’s at P and Q.
x2 y2
D-2. If 3x + 4y = 12 intersect the ellipse + = 1 at P and Q, then find the point of intersection of
25 16
tangents at P and Q.
x2 y2
D-3. Find the equation of chord of ellipse + = 1 whose mid point is (3, 1).
25 16
D-4. Two perpendicular chords are drawn from the origin ‘O’ to the parabola y = x 2, which meet the parabola at P
and Q Rectangle POQR is completed. Find the locus of vertex R.
D-5. Find the equations of the tangents to the hyperbola x 2 9 y2 = 9 that are drawn from (3, 2) . Find the
area of the triangle that these tangents form with their chord of contact.
D-6. Find the locus of the mid-points of the chords of the parabola y2 = 4ax which subtend a right angle at the
vertex of the parabola.
x2 y2
D-7. Find the locus of the middle points of chords of an ellipse + = 1 which are drawn through the
a2 b2
positive end of the minor axis.
D-8. Find the locus of the mid points of the chords of the circle x 2 + y 2 = 16, which are tangent to the
hyperbola 9 x 2 16 y2 = 144.
TN - 138
Tangent Normal & Its Applications in Conics
D-9. Chords of the hyperbola, x2 y2 = a2 touch the parabola, y2 = 4 a x. Prove that the locus of their middle
points is the curve, y2 (x a) = x 3.
x2 y2
D-10_. Prove that the locus of the middle points of the chords of the hyperbola = 1 which pass through a
a2 b2
fixed point (, ) is a hyperbola whose centre is , .
2 2
E-1. Find angle of intersection of the curves y = 2 sin2x and y = cos 2x.
E-2. Let f(x) and g(x) be two functions which cut each other orthogonally at their common point of intersection
(x 1). Both f(x) and g(x) are equal to 0 at x = x1 . Also |f(x 1)| = |g(x 1)|, then find im [f(x) . g(x)], where
x x1
E-3. Find the angle of intersection of the curves y2 = 4ax and x 2 = 32ay.
E-4. Find the condition that curves ax2 + by2 = 1 and a x 2 + b y2 = 1 may cut each other orthogonally
E-5. Find the co-ordinates of the point on the curve x2 = 4y, which is at least distance from the line
y = x 4.
E-6. Find the point on hyperbola 3x2 – 4y2 = 72 which is nearest to the straight line 3x + 2y + 1 = 0
E-7. Find the shortest distance between the curves f(x) = – 6x 6 – 3x4 – 4x2 – 6 and g(x) = ex + e–x + 2
A-1. Equation of the normal to the curve y = x + 2 at the point of its intersection with the curve
y = tan (tan 1 x) is
(A) 2x y 1 = 0 (B) 2x y + 1 = 0 (C) 2x + y 3 = 0 (D) none of these
A-3. The abscissa of point on curve ay2 = x3, normal at which cuts off equal intercepts from the coordinate
axes is
2a 4a 4a 2a
(A) (B) (C) (D)
9 9 9 9
A-4. If the tangent to the curve x = a ( + sin ), y = a (1 + cos ) at = makes an angle (0 < ) with
3
x-axis, then =
2 5
(A) (B) (C) (D)
3 3 6 6
TN - 139
Tangent Normal & Its Applications in Conics
A-5. Number of tangents drawn from the point (–1/2, 0) to the curve y = e{x}. (Here { } denotes fractional part
function).
(A) 2 (B) 1 (C) 3 (D) 4
3 2
A-6. The lines y = – x and y = – x intersect the curve 3x2 + 4xy + 5y2 4 = 0 at the points P and Q
2 5
respectively. The tangents drawn to the curve at P and Q:
(A) intersect each other at angle of 45º (B) are parallel to each other
(C) are perpendicular to each other (D) none of these
A-8. The number of tangents drawn to the curve xy = 4 from point (0, 1) is
(A) 0 (B) 1 (C) 2 (D) Infinite
A-9_. The line xcos + ysin = P is the normal to the curve (x + a)y = c 2, if
3
(A) 2n , 2n 1 2n , 2n 2
2 2
3
(B) 2n , 2n 1 2n , 2n 2
3 4
3
(C) 2n , 2n 1 2n , 2n 2
6 6
3
(D) 2n , 2n 1 2n , 2n 2
4 2
A-11. The subtangent, ordinate and subnormal to the parabola y2 = 4ax at a point (different from the origin)
are in
(A) AP (B) GP (C) HP (D) none of these
B-1_. The locus of point of trisections of the focal chords of the parabola, y2 = 4x is:
(A) y2 = x 1 (B) 9y2 = 4.(3x – 4) (C) y2 = 2 (1 x) (D) None of these
B-3_. A chord is drawn to the hyperbola xy = 4 from a point A(2, 2) which cuts it again at point B. The locus of
point P such that AP : PB = 2 :1 (Internally)
(A) (3x – 2) (3y – 2) = 16 (B) (2x – 3)(2y – 3) = 16
(C) xy = 2 (D) (3x – 2)(2y – 3) = 16
B-4. A variable chord PQ of the parabola, y2 = 4x is drawn parallel to the line y = x. If the parameters of the
points P & Q on the parabola be p & q respectively, then (p + q) equal to.
(A) 1 (B) 1/2 (C) 2 (D) 4
TN - 140
Tangent Normal & Its Applications in Conics
B-5. The length of the chord of the parabola, y2 = 12x passing through the vertex & making an angle of 60º
with the axis of x is:
(A) 8 (B) 4 (C) 16/3 (D) none of these
1
B-6. If y = 2 x 3 is a tangent to the parabola y2 = 4a x , then ' a ' is equal to, where a 0 :
3
14 14
(A) 1 (B) 1 (C) (D)
3 3
x2 y2
B-7. If the line y = 2x + c be a tangent to the ellipse + = 1, then c is equal to
8 4
(A) ± 4 (B) ± 6 (C) ± 1 (D) ± 8
B-8. The latus rectum of a parabola whose focal chord is PSQ such that SP = 3 and SQ = 2 is given by:
(A) 24/5 (B)) 12/5 (C) 6/5 (D) none of these
B-9. The distance of the point of contact from the origin of the line y = x – 7 with the ellipse 3x2 + 4y2 = 12, is
5
(A) 3 (B) 2 (C) (D) none of these
7
x y x2 y2
B-11. If + = 2 touches the ellipse
se + = 1 at a point P, then eccentric angle of P is
a b a2 b2
(A) 0 (B) 45° (C) 60° (D) 90°
x2 y2
B-12. The point of intersection of the tangents at the point P on the ellipse = 1 and its corresponding
a2 b2
point Q on the auxiliary circle, lies on the line :
(A) x = a/e (B) x = 0 (C) y = 0 (D) none of these
x2 y2
B-13. The equation of the locus of the middle point of the portion of the tangent to the ellipse + =1
16 9
included between the co-ordinate axes is the curve:
(A) 9x 2 + 16y2 = 4 x 2y2 (B) 16x 2 + 9y2 = 4 x 2y2
(C) 3x 2 + 4y2 = 4 x 2y2 (D) 9x 2 + 16y2 = x 2y2
B-14. The number of possible tangents which can be drawn to the curve 4x 2 9y2 = 36, which are perpendicular
to the straight line 5x + 2y 10 = 0 is :
(A) zero (B) 1 (C) 2 (D) 4
B-15. The angle between the tangents drawn from a point ( – a, 2a) to y2 = 4 ax is
(A) (B) (C) (D)
4 2 3 6
Section (C) : Equation of normal, co-normal points, Properties of tangent and normal in
conics
C-1. Equation of the normal to the parabola, y2 = 4ax at its point (am 2, 2 am) is:
(A) y = mx + 2am + am 3 (B) y = mx 2am am 3
(C) y = mx + 2am + am 3
(D) none
TN - 141
Tangent Normal & Its Applications in Conics
C-2. At what point on the parabola y2 = 4x the normal makes equal angles with the axes?
(A) (4, 4) (B) (9, 6) (C) (4, – 1) (D) (1, 2)
C-4. The normal chord of a parabola y2 = 4ax at the point P(x 1, x1) subtends a right angle at the
(A) focus (B) vertex
(C) at both ends of the latus rectum (D) none of these
C-5. The equation of the other normal to the parabola y2 = 4ax which passes through the intersection of
those at (4a, 4a) & (9a, 6a) is:
(A) 5x y + 115 a = 0 (B) 5x + y 135 a = 0
(C) 5x y 115 a = 0 (D) 5x + y + 115 = 0
x2 y2
C-6. If the line x cos + y sin = p be normal to the ellipse 2 += 1, then
a b2
(A) p2 (a2 cos 2 + b2 sin2 ) = a2 – b2 (B) p2 (a2 cos2 + b2 sin2 ) = (a2 – b2)2
(C) p2 (a2 sec2 + b2 cosec 2 ) = a2 – b2 (D) p2 (a2 sec2 + b2 cosec2 ) = (a2 – b2)2
c
C-7. If the normal at ct, on the curve xy = c 2 meets the curve again at t, then
t
1 1 1 1
(A) t = – (B) t = (C) t = 2 (D) t 2 = –
t 3 t t t2
x2 y2
C-8. If F 1 & F2 are the feet of the perpendiculars from the focii S1 & S2 of an ellipse = 1 on the
5 3
tangent at any point P on the ellipse, then (S1F1). (S2F2) is equal to :
(A) 2 (B) 3 (C) 4 (D) 5
C-9. The feet of the perpendicular drawn from focus upon any tangent to the parabola, y = x 2 2x 3 lies on
(A) y + 4 = 0 (B) y = 0 (C) y = – 2 (D) y + 1 = 0
C-10. T is a point on the tangent to a parabola y2 = 4ax at its point P. TL and TN are the perpendiculars on the
focal radius SP and the directrix of the parabola respectively. Then:
(A) SL = 2 (TN) (B) 3 (SL) = 2 (TN) (C) SL = TN (D) 2 (SL) = 3 (TN)
C-12. A circle described on any focal chord of the parabola, y2 = 4ax as its diameter will touch
(A) the axis of the parabola
(B) the directrix of the parabola
(C) the tangent drawn at the vertex of the parabola
(D) latus rectum
x2 y2
C-13. P & Q are corresponding points on the ellipse + = 1, and the auxiliary circle respectively. The
16 9
normal at P to the ellipse meets CQ in R where C is centre of the ellipse. Then (CR) is
(A) 5 units (B) 6 units (C) 7 units (D) 8 units
TN - 142
Tangent Normal & Its Applications in Conics
Section (D) : Pair of tangents, Director Circle, Chord of contact, Chord in terms of mid
point
D-1. The line 4x 7y + 10 = 0 intersects the parabola, y2 = 4x at the points A & B. The co-ordinates of the
point of intersection of the tangents drawn at the points A & B are:
7 5 5 7 5 7 7 5
(A) , (B) , (C) , (D) ,
2 2 2 2 2 2 2 2
D-2. The locus of the middle points of the focal chords of the parabola, y2 = 4x is:
(A) y2 = x 1 (B) y2 = 2 (x 1) (C) y2 = 2 (1 x) (D) none
D-3. The equation of the chord of the ellipse 2x 2 + 5y2 = 20 which is bisected at the point (2, 1) is
(A) 4x + 5y + 13 = 0 (B) 4x + 5y = 13 (C) 5x + 4y + 13 = 0 (D) 5x + 4y = 13
D-4. The equation to the chord of the hyperbola x 2 – y2 = 9 which is bisected at (5, – 3) is
(A) 5x + 3y = 9 (B) 5x – 3y = 16 (C) 5x + 3y = 16 (D) 5x – 3y = 9
D-5. The locus of the middle points of chords of hyperbola 3x 2 – 2y2 + 4x – 6y = 0 parallel to y = 2x is
(A) 3x – 4y = 4 (B) 3y – 4x + 4 = 0 (C) 4x – 4y = 3 (D) 3x – 4y = 2
x2 y2
D-6. The chords passing through L(2, 1) intersects the hyperbola – = 1 at P and Q. If the tangents
16 9
at P and Q intersects at R then Locus of R is
(A) x – y = 1 (B) 9x – 8y = 72 (C) x + y = 3 (D) None of these
D-7. Locus of the middle points of the parallel chords with gradient m of the rectangular hyperbola xy = c 2 is:
(A) y + mx = 0 (B) y mx = 0 (C) my x = 0 (D) my + x = 0
D-8. The tangents from (1, 2 2 ) to the hyperbola 16x2 – 25y2 = 400 include between them an angle equal to:
(A) (B) (C) (D)
6 4 3 2
D-9. The locus of the mid points of the chords passing through a fixed point (, ) of the hyperbola
x2 y 2
= 1 is :
a 2 b2
(A) a circle with centre , (B) an ellipse with centre ,
2 2 2 2
(C) a hyperbola with centre , (D) straight line passing through ,
2 2 2 2
D-10. The locus of the middle points of normal chords of the parabola y2 = 4a x is
(A) y4 2 a (x 2 a). y2 + 8 a4 = 0 (B) y4 2 a (x 2 a). y2 + 8 a4 = 0
(C) y4 2 a (x 2 a). y2 + 8 a4 = 0 (D) y4 2 a (x 2 a). y2 – 8 a4 = 0
D-11. The tangent at any point P(x1, y1) on the hyperbola xy = c2 meets the co-ordinate axes at points Q & R. The
circumcentre of OQR has co-ordinates.
x1 y1 2x1 2y1
(A) (0, 0) (B) (x1, y1) (C) , (D) ,
2 2 3 3
TN - 143
Tangent Normal & Its Applications in Conics
Section (E) : Angle between curves, Shortest distance
E-2. The coordinates of the point of the parabola y2 = 8x , which is at minimum distance from the circle
x 2 + (y + 6) 2 = 1 are
(A) (2 , 4) (B) (18 , 12) (C) (2 , 4) (D) none of these
2
2 2
(C) Maximum value of – 3 4 x – x 4 ( x – 5) (r) 5/4
(where 1 x 3) is
(D) If normal at point (6, 2) to ellipse passes through its nearest (s) 0
focus (5, 2) having centre at (4, 2) then its eccentricity is
2. AB is a chord of the parabola y2 = 4ax joining A(at12, 2at1) and B (at22, 2at2). Match the following
Column – I Column – II
1
(C) AB subtends 90º at (0, 0) (r) t2 = – t
1
2
(D) AB is inclined at 45º to the axis of parabola (s) t 2 = – t1 – t
1
4 x2 y2
3. A tangent having slope – touches the ellipse + = 1 at point P and intersects the major and
3 18 32
minor axes at A & B respectively, O is the centre of the ellipse
Column - I Column - II
4
(A) Distance between the parallel tangents having slopes – , is (p) 24
3
TN - 144
Tangent Normal & Its Applications in Conics
4. Column - I Column - II
(A) Number of positive integral values of b for which tangent (p) 16
parallel to line y = x + 1 can be drawn to hyperbola
x2 y2
= 1 is
5 b2
(B) The equation of the hyperbola with vertices (3, 0) and (q) 2
(–3, 0) and semi-latusrectum 4, is given by is
4x 2 – 3y2 = 4k, then k =
2. If tangents are drawn from the origin to the curve y = sin x, then their points of contact lie on the curve
(A) x – y = xy (B) x + y = xy (C) x2 – y2 = x2y2 (D) x2 + y2 = x2y2
x y
3. The line 1 touches the curve y = be–x/a at the point
a b
a b b
(A) (– a, 2b) (B) , (C) a, (D) (0, b)
2 2 e
x 2 , x 0
4. Let f(x) = 2 Equation of tangent line touching both branches of y = f(x) is
x 8 , x 0
(A) y = 4x + 1 (B) y = 4x + 4 (C) y = x + 4 (D) y = x + 1
5. The set of values of p for which the equation ln x px = 0 possess three distinct roots is
(A) p (0, 1/e) (B) p (0, e) (C) p (1/e, e) (D) p (1, e)
6. A curve is represented by the equations, x = sec 2 t and y = cot t where t is a parameter. If the tangent
at the point P on the curve where t = /4 meets the curve again at the point Q then PQ is equal to:
5 3 5 5 2 5 3 5
(A) (B) (C) (D)
2 2 3 2
7. The tangent to curve y = x – x 3 at point P meets the curve again at Q. The locus of one of the point of
trisection of PQ is
(A) y = x + 5x 3 (B) y = –x – 5x3 (C) y = x – 5x 3 (D) y = x – x 3
TN - 145
Tangent Normal & Its Applications in Conics
8. Length of the focal chord of the parabola y2 = 4ax at a distance p from the vertex is:
2a2 a3 4a3 p2
(A) (B) 2 (C) (D)
p p p2 a
9_. If P1Q1 and P2Q2 are two focal chords of the parabola y2 = 4ax. then the chords P1P2 and Q1Q2
intersect on
(A) tangent at the vertex of the parabola (B) the directrix of the parabola
(C) at x = –2a (D) y = 2a and x = –2a
10_. The vertex of the locus of a point that divides a chord of slope 2 of the parabola y2 = 4x. internally in the ratio
1 : 2 is
1 2 8 1 2 8 1 1
(A) , (B) , (C) , (D) ,
9 9 9 9 9 9 9 9
11. AB, AC are tangents to a parabola y2 = 4ax. p1, p2 & p3 are the lengths of the perpendiculars from A, B
& C respectively on any tangent to the curve, then p2, p1, p3 are in:
(A) A.P. (B) G.P. (C) H.P. (D) none of these
12. The mirror image of the parabola y2 = 4x in the tangent to the parabola at the point (1, 2) is
(A) (x – 1)2 = 4(y – 2) (B) (x + 3) 2 = 4(y + 2)
(C) (x + 1) = 4(y – 1)
2
(D) (x – 1)2 = 4 (y – 1)
x2 y2
13. The locus of point of intersection of tangents to an ellipse + = 1 at the points, the sum of
a2 b2
whose eccentric angles is constant, is :
(A) a hyperbola (B) an ellipse (C) a circle (D) a straight line
x2 y2
14. P is a point on the hyperbola = 1, N is the foot of the perpendicular from P on the transverse
a 2 b2
axis. The tangent to the hyperbola at P meets the transverse axis at T. If O is the centre of the
hyperbola, then OT. ON is equal to :
(A) e2 (B) a2 (C) b2 (D)b2/a2
x2 y2
15. Tangent at any point on the hyperbola –
= 1 cut the axes at A and B respectively. If the rectangle
a2 b2
OAPB (where O is origin) is completed then locus of point P is given by
a2 b2 a2 b2 a2 b2
(A) – 2 =1 (B) + 2 =1 (C) 2 – =1 (D) none of these
x2 y x2 y y x2
x2 y2
16. x 2y + 4 = 0 is a common tangent to y2 = 4x & = 1. Then the value of ‘b’ and the other
4 b2
common tangent are given by :
(A) b = 3 ; x + 2y + 4 = 0 (B) b = 3; x + 2y + 4 = 0
(C) b = 3 ; x + 2y 4 = 0 (D) b = 3 ; x 2y 4 = 0
17. The number of points from where a pair of perpendicular tangents can be drawn to the hyperbola,
x 2 sec 2 y2 cosec 2 = 1, (0, /4), is :
(A) 0 (B) 1 (C) 2 (D) infinite
TN - 146
Tangent Normal & Its Applications in Conics
18. A normal chord of the parabola subtending a right angle at the vertex makes an acute angle with the
x axis, then =
(A) arc tan 2 (B) arc tan 2 (C) arc cot 2 (D) arc cot2
19. If two normals to a parabola y2 = 4ax intersect at right angles then the chord joining their feet passes
through a fixed point whose co-ordinates are:
(A) ( 2a, 0) (B) (a, 0) (C) (2a, 0) (D) (–a, 0)
20^. An ellipse with major axis 4 and minor axis 2 touches both the coordinate axis, then Locus of its
centre is
x2
(A) x2 – y2 = 5 (B) x2.y2 = 5 (C) + y2 = 5 (D) x2 + y2 = 5
4
x2 y2
21. If the chord of contact of tangents from two points (x 1, y1) and (x 2, y2) to the hyperbola 2 – = 1 are
a b2
x1 x 2
at right angles, then is equal to
y1 y 2
a2 b2 b4 a4
(A) – (B) – (C) – (D) –
b2 a2 a4 b4
22^. If the normals from any point are drawn the parabola x 2 = 4y cuts the line y = 2 in points whose
abscissa are in A.P., then the slopes of the tangents at the 3 conormal points are in
(A) A.P. (B) G.P. (C) H.P. (D) None of these
23^. If a parabola whose length of latus rectum is 4a touches both the coordinate axes then the locus of its
focus is
(A) xy = a2 (x2 + y2) (B) x2y2 = a2 (x 2 + y2)
(C) x – y = a (x + y )
2 2 2 2 2
(D) x 2y2 = a2 (x 2 – y2)
24^. An ellipse with major axis 4 and minor axis 2 touches both the coordinate axes, then locus of its focus is
(A) (x2 – y2) (1 + x2y2) = 16 x2 y2 (B) (x2 – y2) (1 – x2y2) = 16 x2 y2.
(C) (x + y ) (1 + x y ) = 16 x y
2 2 2 2 2 2
(D) (x2 + y2) (1 – x2y2) = 16 x2 y2.
x2 y2
25. Tangents are drawn from any point on the hyperbola – = 1 to the circle x2 + y2 = 9, then the locus of
9 4
mid-point of the chord of contact is
2 2
x2 y2 x2 y2 x2 y2 x2 y2
(A) =
(B) =
9 4 9 9 4 9
2 2
x2 y2 x2 y2 x2 y2 x2 y2
(C) = (D) =
9 4 9 9 4 9
26_. In the parabola y2 = 4ax, the locus of middle points of all chords of constant length c is
(A) (4ax – y2)(y2 – 4a2) = a2c2 (B) (4ax + y2)(y2 + 4a2) = a2c2
(C) (4ax + y )(y – 4a ) = a c
2 2 2 2 2
(D) (4ax – y2)(y2 + 4a2) = a2c2
27. A tangent to the parabola x 2 + 4ay = 0 cuts the parabola x 2 = 4by at A and B the locus of the mid point
of AB is:
(A) (a + 2b) x2 = 4 b2y (B) (b + 2a) x2 = 4 b2y
(C) (a + 2b) y = 4 b x
2 2
(D) (b + 2x) x 2 = 4 a2y
TN - 147
Tangent Normal & Its Applications in Conics
28_. From the points on the circle x2 + y2 = a2, tangents are drawn to the hyperbola x2 – y2 = a2 ; then the locus
of the middle points of the chords of contact is
(A) (x2 – y2) = a2(x2 + y2) (B) (x2 – y2)2 = a2(x2 + y2)
(C) (x – y ) = a (x – y )
2 2 2 2 2
(D) (x2 – y2)2 = a2(x2 – y2)
x2 y2
30. If curves 1 and xy = c 2 intersect orthogonally, then
a2 b2
(A) a + b = 0 (B) a2 = b2 (C) a + b = c (D) none of these
31. The point(s) on the parabola y2 = 4x which are closest to the circle,
x 2 + y2 24y + 128 = 0 is/are:
(A) (0, 0)
(B) 2 , 2 2 (C) (4, 4) (D) none of these
3^. If at any point on a curve the length of subtangent and subnormal are equal, then the tangent is equal
to |ordinate| then the value of is
2 2
4_. A variable chord of the parabola y2 = 24x which subtend 90° angle at its vertex always passes through a fixed
point whose distance from the origin is :
6^. The distance between a tangent to the parabola y2 = 4 A x (A > 0) and the parallel normal with gradient
1 is 2 A then value of is
5
7_. If two curves y = 2sin x and y = x2 – 3x + 2 + 1 touch each other at some point then the value
6
2 3 18
of where 0 x is
5 5
8^. Through the vertex O of the parabola y² = 8x, a perpendicular is drawn to any tangent meeting it at
P & the parabola at Q, then the value of OP. OQ is
9^. The centre of the circle which passes through the focus of the parabola x 2 = 4 y & touches it at the point
(6, 9) is (, ) then | – | is
10^. The radius of the largest circle with centre (1, 0) that can be inscribed in the ellipse x2 + 4y2 =16 is where
and are prime number, then + is
TN - 148
Tangent Normal & Its Applications in Conics
11^. Common tangents are drawn to the parabola y2 = 4x & the ellipse 3x2 + 8y2 = 48 touching the parabola
at A & B and the ellipse at C & D, then the area of the quadrilateral ABCD is 2 the is equal to
12^. Points A, B & C lie on the parabola y² = 4ax. The tangents to the parabola at A, B & C, taken in pairs,
intersect at points P, Q & R. the ratio of the areas of the triangles ABC & PQR is where and are
co-prime number then + is
x2 y2
13^. A circle of radius r is concentric with the ellipse + = 1 and the common tangent is inclined to
42 32
r 2 2
the major axis at an angle of tan–1 2 2 ; r (b, a) then the value of + is
r
x2 y2
14^. If m1 and m2 are slopes of the tangents to the hyperbola – = 1 which passes through the point of
25 16
contact of 3x – 4y = 5 and x2 – 4y2 = 5 then 32(m1 + m2 – m1m2) = ....................
15^. Tangents are drawn from the point (, 2) to the hyperbola 3x2 2y2 = 6 and are inclined at angles &
to the x axis . If tan . tan = 2, then the value of 22 7 is
x2 y2
16. If the tangent on the point (3 sec , 4 tan ) (which is in first quadrant) of the hyperbola – = 1 is
9 16
perpendicular to 3x + 8y – 12 = 0, then the value of is (in degree)
x2 y2
17. C the centre of the hyperbola – = 1. The tangents at any point P on this hyperbola meets the
9 16
striaght lines 4x – 3y = 0 and 4x + 3y = 0 in the points Q and R respectively. Then CQ . CR =
x2 y2 x2 y2
18^. If radii of director circles of + = 1 and –
= 1 are 2r and r respectively and ee and eh be
a2 b2 a2 b2
the eccentricities of the ellipse and the hyperbola respectively then 4eh2 – ee2 is equal to
x2 y2
19^. If CF is perpendicular from the centre of the ellipse
= 1 to the tangent at P, and G is the
42 32
point where the normal at P meets the major axis, then the product CF PG is
20^. A normal is drawn to a parabola y2 = 4ax at any point other than the vertex and it cuts the parabola again at
a point whose distance from the vertex is not less than 6 a , then the value of is
21^. If three normal are drawn through (c, 0) to y2 = 4x and two of which of perpendicular then the value of c is
23^. The eccentricity of an ellipse whose focii are (2, 4) & (14, 9) and touches x-axis is then the
313
value of is
TN - 149
Tangent Normal & Its Applications in Conics
1
24^. The chord of the parabola y = - a2x 2 + 5ax - 4 touches the curve y = 1 x at the point x = 2 and is
26^. Minimum distance between the curves f(x) = ex & g(x) = n x is then the value of is
2
a b
2^. The x-intercept of tangent at arbitrary point of curve 2
= 1 is
x y2
(A) proportional to square of the abscissa of the point of tangency
(B) is independent of a
(C) proportional to cube of the abscissa of the point of tangency
(D) is independent of b
7
3. If tangent at point (1, 2) on the curve y = ax 2 + bx + be parallel to normal at (– 2, 2) on the curve
2
y = x 2 + 6x + 10, then
(A) a = 1 (B) a = – 1 (C) b = – 5/2 (D) b = 5/2
x 3 5x 2
4. The co-ordinates of the point(s) on the graph of the function, f(x) = + 7x – 4 where the tangent
3 2
drawn cut off intercepts from the co-ordinate axes which are equal in magnitude but opposite in sign,
is:
(A) (2, 8/3) (B) (3, 7/2) (C) (1, 5/6) (D) none of these
a
n n
x y
5. The equation of normal to the curve = 2 (n N) at the point with abscissa equal to 'a' can
a b
be:
(A) ax + by = a2 b2 (B) ax + by = a2 + b2
(C) ax by = a2 b2 (D) bx ay = a2 b2
6. For the curve represented parametrically by the equations, x = 2 n cot t + 1 & y = tan t + cot t
(A) tangent at t = /4 is parallel to x axis
(B) normal at t = /4 is parallel to y axis
(C) tangent at t = /4 is parallel to the line y = x
(D) tangent and normal intersect at the point (2, 1)
7^. The equation of line which is tangent at a point on curve 4x 3 = 27 y2 and normal at other point are :
(A) y = 2x–2 2 (B) y = – 2 x – 2 2
(C) y = 2x+2 2 (D) y = – 2 x + 2 2
TN - 150
Tangent Normal & Its Applications in Conics
8. For the curve x = t2 + 3t – 8, y = 2t2 – 2t – 5, at point (2, – 1)
(A) length of subtangent is 7/6. (B) slope of tangent = 6/7
(C) length of tangent = (85 ) / 6 (D) none of these
9. If one end of a focal chord of the parabola y2 = 4x is (1, 2), the other end lies on
(A) x2 y + 2 = 0 (B) xy + 2 = 0
(C) xy – 2 = 0 (D) x 2 + xy – y – 1 = 0
10. Equation of a tangent to the parabola y2 = 12x which make an angle of 45° with line y = 3x + 77 is
(A) 2x – 4y + 3 = 0 (B) x – 2y + 12 = 0 (C) 4x + 2y + 3 = 0 (D) 2x + y – 12 = 0
11^. Set of values of m for which a chord of slope m of the circle x 2 + y2 = 4 touches parabola y2 = 4x, may
lie in intervel
2 1
2 1
(A) , (B) (0 , 1/2) (C) , (D) (–1/2, 0)
2 2
12. The tangents at the extremities of a focal chord of a parabola
(A) are perpendicular (B) are parallel
(C) intersect on the directrix (D) intersect at the vertex
13. P is a point on the parabola y2 = 4x where abscissa and ordinate are equal. Equation of a circle passing
through the focus and touching the parabola at P is:
(A) x2 + y2 13x + 2y + 12 = 0 (B) x2 + y2 3x 18y + 2 = 0
(C) x + y + 13x 2y 14 = 0
2 2
(D) x 2 + y2 – x = 0
x2 y2
14._ If the chord joining the points whose eccentric angles are ‘’ and ‘’ on the hyperbola = 1 is a focal
a2 b2
chord then
(A) ± ecos = cos (B) ± ecos = cos
2 2 2 2
ke 1 ke 1
(C) tan(/2) tan(/2) + = 0 where k = ± 1(D) tan(/2) tan(/2) + = 0 where k = ± 1
ke 1 ke 1
x2 y2
15. The value of m for which y = mx + 6 is a tangent to the hyperbola – = 1 is
100 49
17 17 20 20
(A) (B) – (C) (D) –
20 20 17 17
17. The equation of the tangent lines to the hyperbola x 2 2y2 = 18 which are perpendicular to the line
y = x are :
(A) y = – x + 7 (B) y = x + 3 (C) y = – x – 4 (D) y = – x – 3
18. Let A() and B() be the extremeties of a chord of an ellipse . If the slope of AB is equal to the slope of
the tangent at a point C() on the ellipse, then the value of , is
(A) (B) (C) + (D) –
2 2 2 2
TN - 151
Tangent Normal & Its Applications in Conics
19. Let y2 = 4ax be a parabola and x 2 + y2 + 2 bx = 0 be a circle. If parabola and circle touch each other
externally then:
(A) a > 0, b > 0 (B) a > 0, b < 0 (C) a < 0, b > 0 (D) a < 0, b < 0
20. The tangent at any point ‘P’ on the standard ellipse with focii as S & S meets the tangents at the
vertices A & A in the points V & V, then :
(A) (AV) (A V) = b2 (B) (AV) (A V) = a2
(C) V SV = 90º (D) V S SV is a cyclic quadrilateral
21. Let F 1, F2 be two focii of the ellipse and PT and PN be the tangent and the normal respectively to the
ellipse at point P then
(A) PN bisects F 1 PF2 (B) PT bisects F1PF2
(C) PT bisects angle (180° – F 1PF2) (D) None of these
22^. If 1 be the equation of the common tangent in 1st quadrant to the circle x2 + y2 = 16 and ellipse
x2 y2
= 1 and 1 be the length of the intercept of the common tangent between the coordinate axes then
25 4
14
(A) 1 = (B) Equation of 1 is 2x 3 y 4 7
3
4
(C) 1 = (D) Equation of 1 is x 3y 4 7
3
23. Point/points, from which tangents to the ellipse 5x 2 + 4y2 = 20 are perpendicular, is/are :
(A) (1, 2 2 ) (B) (2 2 , 1) (C) (2, 5) (D) ( 5 , 2)
8 y2
24. The value of '', for which the line 2x – y = – 3 is a normal to the conic x 2 + = 1 is
3 4
3 1 3 3
(A) (B) (C) – (D)
2 2 2 8
25. If three normals can be drawn to the curve y2 = x from point (c, 0) then 'c' can be equal to
(A) 0 (B) 1/2 (C) 1 (D) 2
26. If the normal at P to the rectangular hyperbola x 2 y2 = 4 meets the axes in G and g and C is the centre
of the hyperbola, then
(A) PG = PC (B) Pg = PC (C) PG = Pg (D) Gg = PC
27. The normal to the curve x = a(cos + sin ), y = a(sin – cos ) at any point '' is such that
a
(A) it is at a constant distance from the origin (B) it passes through , – a
2
(C) it makes angle /2 + with the x-axis (D) it passes through the origin
28. Circles are drawn on chords of rectangular hyperbola xy = c 2 parallel to the line y = x as diameters. All
such circles pass through two fixed points whose co-ordinates are :
(A) (c, c) (B) (c, c) (C) ( c, c) (D) ( c, c)
2 y2
29^. If , , & be the eccentric angles of feet of four conormal points of a hyperbola x2 2 = 1 from
a b
any point in its plane then + + + can be
(A) 3 (B) 7 (C) 11 (D) 12
TN - 152
Tangent Normal & Its Applications in Conics
30_. If a circle and the rectangular hyperbola xy = c 2 meet in the four points t1, t2, t3 & t 4, then
(A) t1 t2 t3 t4 = 1
(B) The arthmetic mean of the four points bisects the distance between the centres of the two curves.
(C) The geometrical mean of the four points bisects the distance between the centres of the two
curves.
c 1 c1 1 1
(D) the centre of the circle through the points t1, t2 & t3 is : 2 t 1 t 2 t 3 t t t , 2 t t t t1 t 2 t 3
1 2 3 1 2 3
31. If P is a point on the curve 5x2 + 3xy + y2 = 2 and O is the origin, then OP has
1 2
(A) minimum value (B) minimum value (C) maximum value 11 (D) maximum value 2
2 11
32. The co-ordinates of a point on the parabola 2y = x 2 which is nearest to the point (0, 3) is
(A) (2, 2)
(B) 2 , 1 (C) 2, 1 (D) (– 2, 2)
PART - IV : COMPREHENSION
Comprehension # 1
Let PQ be a variable focal chord of the parabola y2 = 4ax where vertex is A. Locus of, centroid of
triangle APQ is a parabola ‘P1’
2. Vertex of parabola P1 is
2a 4a 8a a
(A) , 0 (B) , 0 (C) , 0 (D) , 0
3 3 3 3
3. Let 1 is the area of triangle formed by joining points T 1, T 2 and T 3 on parabola P1 and 2 be the area of
triangle T formed by tangents at T 1, T 2 and T 3, then
(A) 2 = 21
(B) 1 = 42
(C) orthocentre of triangle T lies on x = a/3.
(D) Both (A) and (C) are correct.
Comprehension # 2
x2 y2
Two tangents PA and PB are drawn from a point P(h, k) to the ellipse E :
1 (a > b). Angle of the
a2 b2
tangents with the positive x - axis are 1 and 2. Normals at A and B are intersecting at Q point.
On the basis of above information answer the following questions.
5. Circumcentre of QAB is
(A) mid point of AB (B) mid point of PQ (C) orthocentre of PAB (D) can't say
TN - 153
Tangent Normal & Its Applications in Conics
Comprehension # 3^
y
Asymptotes are lines whose distance from the curve at infinity y=
– b b x
a x = a
x2 y2 y
tends to zero Let y = mx + c is asymptote of H = 2
–1
a b2 x
= 0. Solving the two equations, we have (b2 – a2m2) x2 – 2a2mcx – a2 O
(b2 + c2) = 0. Both roots of this equation must be infinite so
b b
m=± and c = 0 which implies that y = ± x are asymptotes
a a (b2 + c2)
x2 y2
of = 1. Note that no real tangent can be drawn to the hyperbola from its centre and only one real
a2 b2
tangent can be drawn from a point lying on its asymptote other than centre. Further combined equation of
x2 y2 x2 y2
asymptotes is A = = 0 and conjugate hyperbola C = 2 +1=0.
a2 b2 a b2
Hence 2A = H + C, as we can see, equation of A, H and C vary only by a constant, for asymptotes which can
be evaluated by applying condition of pair of lines.
x2 y2
7. The points of contact of tangents drawn to the hyperbola = 1 from point (2, 1) are
3 2
9 2
(A) (3, 2), (1, 5) (B) (3, 2), , (C) (1, 2), (3, 4) (D) (3, 2), (3, 4)
5 5
8. The number of real distinct tangents drawn to hyperbola 4x 2 – y2 = 4 from point (1, 2) is
(A) 1 (B) 2 (C) 3 (D) 4
9. The number of real distinct tangents drawn from point (1, 2) to hyperbola x 2 – y2 – 2x + 4y – 4 = 0 is
(A) 1 (B) 2 (C) 3 (D) None of these
x2 y2
2. The minimum area of triangle formed by the tangent to the ellipse + = 1 and coordinate axes is
a2 b2
[IIT-JEE-2005, Scr.(3, –1) /84]
a2 b2 (a b ) 2 a 2 ab b 2
(A) ab sq. units (B) sq. unit (C) sq. units (D) sq. units
2 2 3
TN - 154
Tangent Normal & Its Applications in Conics
3*. The equation of common tangent(s) to the parabola y = x 2 and y = – x2 + 4x – 4 is /are
[IIT-JEE 2006, (5, –1), 184]
(A) y = 4(x – 1) (B) y = 0 (C) y = – 4(x – 1) (D) y = – 30 x – 50
Match the following [IIT-JEE 2006, (6, 0), 184]
4. Match the following. Normals are drawn at point P, Q and R lying on the parabola y2 = 4x which intersect at
(3, 0). Then
Column - Column -
(A) Area of PQR (p) 2
(B) Radius of circumcircle of PQR (q) 5/2
(C) Centroid of PQR (r) (5/2, 0)
(D) Circumcentre of PQR (s) (2/3, 0)
5. The tangent to the curve y = ex drawn at the point (c, ec) intersects the line joining the points (c – 1, ec–1) and
(c + 1, ec+1) [IIT-JEE 2007, Paper-1, (3, –1)/ 81]
(A) on the left of x = c (B) on the right of x = c
(C) at no point (D) at all points
Consider the circle x2 + y2 = 9 and the parabola y2 = 8x. They intersect at P and Q in the first and the
fourth quadrants, respectively. Tangents to the circle at P and Q intersect the x-axis at R and tangents to
the parabola at P and Q intersect the x-axis at S.
6. The ratio of the areas of the triangles PQS and PQR is
(A) 1 : 2 (B) 1 : 2 (C) 1 : 4 (D) 1 : 8
7. The radius of the circumcircle of the triangle PRS is
(A) 5 (B) 3 3 (C) 3 2 (D) 2 3
11*. The tangent PT and the normal PN to the parabola y2 = 4ax at a point P on it meet its axis at points T and N,
respectively. The locus of the centroid of the triangle PTN is a parabola whose
[IIT-JEE 2009, Paper-2, (4, –1), 80]
2a 2a
(A) vertex is , 0 (B) directrix is x = 0 (C) latus rectum is (D) focus is (a, 0)
3 3
TN - 155
Tangent Normal & Its Applications in Conics
12. The normal at a point P on the ellipse x2 + 4y2 = 16 meets the x-axis at Q. If M is the mid point of the line
segment PQ, then the locus of M intersects the latus rectum of the given ellipse at the points
[IIT-JEE 2009, Paper-2, (3, –1)/ 80]
3 5 2 3 5 19 1 4 3
(A) , (B) , (C) 2 3 , (D) 2 3 ,
2 7 2 4 7 7
13*. An ellipse intersects the hyperbola 2x2 – 2y2 = 1 orthogonally. The eccentricity of the ellipse is reciprocal of
that of the hyperbola. If the axes of the ellipse are along the coordinate axes, then
[IIT-JEE 2009, Paper-2, (4, –1), 80]
(A) Equation of ellipse is x2 + 2y2 = 2 (B) The foci of ellipse are (±1, 0)
(C) Equation of ellipse is x2 + 2y2 = 4 (D) The foci of ellipse are (± 2 , 0)
14*. Let A and B be two distinct points on the parabola y2 = 4x. If the axis of the parabola touches a circle of radius
r having AB as its diameter, then the slope of the line joining A and B can be
[IIT-JEE-2010, Paper-1(3, 0)/84]
1 1 2 2
(A) – (B) (C) (D) –
r r r r
x2 y2
Tangents are drawn from the point P(3, 4) to the ellipse = 1 touching the ellipse at point A and B.
9 4
15. The coordinates of A and B are [IIT-JEE 2010, Paper-2, (3, –1), 79]
8 2 161 9 8
(A) (3, 0) and (0, 2) (B) – 5 , 15 and – ,
5 5
8 2 161 9 8
(C) – 5 , 15 and (0, 2) (D) (3, 0) and – ,
5 5
16. The orthocentre of the triangle PAB is [IIT-JEE 2010, Paper-2, (3, –1), 79]
8 7 25 11 8 8 7
(A) 5 , (B) , (C) , (D) ,
7 5 8 5 5 25 5
17. The equation of the locus of the point whose distances from the point P and the line AB are equal, is
[IIT-JEE 2010, Paper-2, (3, –1), 79]
(A) 9x2 + y2 – 6xy – 54x – 62y + 241 = 0 (B) x2 + 9y2 + 6xy – 54x + 62y – 241 = 0
(C) 9x2 + 9y2 – 6xy – 54x – 62y – 241 = 0 (D) x2 + y2 – 2xy + 27x + 31y – 120 = 0
x2 y2
The circle x2 + y2 – 8x = 0 and hyperbola – = 1 intersect at the points A and B.
9 4
18. Equation of a common tangent with positive slope to the circle as well as to the hyperbola is
[IIT-JEE-2010, Paper-1(3, –1)/84]
(A) 2x – 5 y – 20 = 0 (B) 2x – 5 y + 4 = 0
(C) 3x – 4y + 8 = 0 (D) 4x – 3y + 4 = 0
TN - 156
Tangent Normal & Its Applications in Conics
19. Equation of the circle with AB as its diameter is [IIT-JEE-2010, Paper-1(3, –1)/84]
(A) x2 + y2 – 12x + 24 = 0 (B) x2 + y2 + 12x + 24 = 0
(C) x2 + y2 + 24x – 12 = 0 (D) x2 + y2 – 24x – 12 = 0
x2
y2
20. The line 2x + y = 1 is tangent to the hyperbola – = 1 . If this line passes through the point of
a2 b2
intersection of the nearest directrix and the x-axis, then find the eccentricity of the hyperbola.
[IIT-JEE-2010, Paper-1(3, 0)/84]
21. Consider the parabola y2 = 8x. Let 1 be the area of the triangle formed by the end points of its latus rectum
1
and the point P , 2 on the parabola, and 2 be the area of the triangle formed by drawing tangents at P
2
1
and at the end points of the latus rectum. Then is [IIT-JEE 2011, Paper-1, (4, 0), 80]
2
22*. Let L be a normal to the parabola y2 = 4x. If L passes through the point (9, 6), then L is given by
[IIT-JEE 2011, Paper-2, (4, 0), 80]
(A) y – x + 3 = 0 (B) y + 3x – 33 = 0 (C) y + x – 15 = 0 (D) y – 2x + 12 = 0
x2 y2
23. Let P(6, 3) be a point on the hyperbola 2
1 . If the normal at the point P intersects the x-axis at
a b2
(9, 0), then the eccentricity of the hyperbola is [IIT-JEE 2011, Paper-2, (3, –1), 80]
5 3
(A) (B) (C) 2 (D) 3
2 2
x2 y2
24*. Tangents are drawn to the hyperbola – 1 , parallel to the straight line 2x – y = 1. The points of
9 4
contacts of the tangents on the hyperbola are [IIT-JEE 2012, Paper-1, (4, 0), 70]
9 1 1
9
(A) , (B) – ,– (C) 3 3 , – 2 2 (D) – 3 3 , 2 2
2 2 2 2 2 2
Paragraph for Question Nos. 25 to 26
Let PQ be a focal chord of the parabola y2 = 4ax. The tangents to the parabola at P and Q meet at a
point lying on the line y = 2x + a, a > 0. [IIT-JEE - 2013]
26. If chord PQ subtends an angle at the vertex of y2 = 4ax, then tan =
2 –2 2 –2
(A) 7 (B) 7 (C) 5 (D) 5
3 3 3 3
27. The slope of the tangent to the curve (y – x 5)2 = x(1 + x 2)2 at the point (1, 3) is
[JEE (Advanced) 2014, Paper-1, (3, 0)/60]
28. The common tangents to the circle x 2 + y2 = 2 and the parabola y2 = 8x touch the circle at the points P,
Q and the parabola at the points R, S. Then the area of the quadrilateral PQRS is
[JEE (Advanced) 2014, Paper-2, (3, –1)/60]
(A) 3 (B) 6 (C) 9 (D) 15
TN - 157
Tangent Normal & Its Applications in Conics
Paragraph For Questions 29 and 30
Let a, r, s, t be nonzero real numbers. Let P(at2, 2at), R(ar2, 2ar) and S(as2, 2as) be distinct points on
the parabola y2 = 4ax. Suppose that PQ is the focal chord and lines QR and PK are parallel, where K is
the point (2a, 0) [JEE (Advanced) 2014, Paper-2, (3, –1)/60]
30. If st = 1, then the tangent at P and the normal at S to the parabola meet at a point whose ordinate is
(t 2 1)2 a(t 2 1)2 a( t 2 1)2 a( t 2 2)2
(A) (B) (C) (D)
2t 3 2t 3 t3 t3
31. Let the curve C be the mirror image of the parabola y2 = 4x with respect to the line x + y + 4 = 0. If A and
B are the points of intersection of C with the line y= – 5, then the distance between A and B is
[JEE (Advanced) 2015, P-1 (4, 0) /88]
32. If the normals of the parabola y2 = 4x drawn at the end points of its latus rectum are tangents to the
circle (x – 3)2 + (y + 2) 2 = r 2, then the value of r2 is [JEE (Advanced) 2015, P-1 (4, 0) /88]
33*. Let P and Q be distinct points on the parabola y2 = 2x such that a circle with PQ as diameter passes
through the vertex O of the parabola, If P lies in the first quadrant and the area of the triangle OPQ is
3 2 , then which of the following is (are) the coordinates of P ? [JEE (Advanced) 2015, P-1 (4, –2)/ 88]
1 1
(A) (4, 2 2 ) (B) (9, 3 2 ) (C) 4 , (D) (1, 2)
2
x2 y2
34. Suppose that the foci of the ellipse = 1 are (f 1, 0) and (f 2, 0) where f 1 > 0 and f 2 < 0. Let P1 and
9 5
P2 be two parabolas with a common vertex at (0, 0) and with foci at (f 1, 0) and (2f 2, 0), respectively. Let
T 1 be a tangent to P1 which passes through (2f 2, 0) and T 2 be a tangent to P2 which passes through
1
(f 1,0). If m 1 is the slope of T 1 and m 2 is the slope of T 2, then the value of m 2
is.
m2 2
1
[JEE (Advanced) 2015, P-2 (4, 0) / 80]
35*. Let E1 and E2 be two ellipses whose centers are at the origin. The major axes of E1 and E2 lie along the
x-axis and the y-axis, respectively. Let S be the circle x 2 + (y – 1)2 = 2. The straight line x + y = 3
2 2
touches the curves S, E1 and E2 at P,Q and R, respectively. Suppose that PQ = PR = . If e1 and e2
3
are the eccentricities of E1 and E2, respectively, then the correct expression(s) is (are)
[JEE (Advanced) 2015, P-2 (4, –2)/ 80]
43 7
(A) e12 + e22 = (B) e1e2 =
40 2 10
5 3
(C) | e12 – e 22 | = (D) e1e2 =
8 4
TN - 158
Tangent Normal & Its Applications in Conics
36*. Consider the hyperbola H : x 2 – y2 = 1 and a circle S with center N(x2, 0). Suppose that H and S touch
each other at a point P(x 1, y1) with x 1 > 1 and y1 > 0. The common tangent to H and S at P intersects
the x-axis at point M. If (l, m) is the centroid of the triangle PMN, then the correct expression(s)
is(are) [JEE (Advanced) 2015, P-2 (4, –2)/ 80]
dl 1 dm x1
(A) dx = 1 – for x1 > 1 (B) for x 1 > 1
1 3x12 dx1 3
x12 1
dl 1 dm 1
(C) dx = 1+ for x 1 > 1 (D) for y1 > 0
1 3x12 dy1 3
x2 y2
1 is : [AIEEE - 2005, (3, 0), 225]
a2 b2
(1) a hyperbola (2) a parabola (3) a circle (4) an ellipse
2. The equation of a tangent to the parabola y2 = 8x is y = x + 2. The point on this line from which the other
tangent to the parabola is perpendicular to the given tangent is [AIEEE-2007 (3, –1), 120]
(1) (–1, 1) (2) (0, 2) (3) (2, 4) (4) (–2, 0)
3. The shortest distance between the line y – x = 1 and the curve x = y2 is [AIEEE 2009(4, –1), 144]
3 2 2 3 3 2 3
(1) (2) (3) (4)
8 8 5 4
4
4. The equation of the tangent to the curve y = x + , that is parallel to the x-axis, is
x2
[AIEEE 2010 (4, –1), 144]
(1) y = 1 (2) y = 2 (3) y = 3 (4) y = 0
5. The shortest distance between line y – x = 1 and curve x = y2 is : [AIEEE 2011 (4, –1), 120]
3 3 2 8 4
(1) (2) (3) (4)
4 8 3 2 3
6. Statement-1 : An equation of a common tangent to the parabola y2 = 16 3 x and the ellipse 2x2 + y2 = 4
is y = 2x + 2 3 .
4 3
Statement-2 : If the line y = mx + , (m 0) is a common tangent to the parabola y2 = 16 3 x and the
m
ellipse 2x2 + y2 = 4, then m satisfies m4 + 2m2 = 24. [AIEEE 2012 (4, –1), 120]
(1) Statement-1 is false, Statement-2 is true.
(2) Statement-1 is true, statement-2 is true; statement-2 is a correct explanation for Statement-1.
(3) Statement-1 is true, statement-2 is true; statement-2 is not a correct explanation for Statement-1.
(4) Statement-1 is true, statement-2 is false.
7. The locus of the foot of perpendicular drawn from the centre of the ellipse x 2 + 3y2 = 6 on any tangent to it is:
[JEE(Main) 2014, (4, – ¼), 120]
(1) (x2 + y2)2 = 6x2 + 2y2 (2) (x2 + y2)2 = 6x2 – 2y2
(3) (x2 – y2)2 = 6x2 + 2y2 (4) (x2 – y2)2 = 6x2 – 2y2
TN - 159
Tangent Normal & Its Applications in Conics
8. The slope of the line touching both the parabolas y2 = 4x and x2 = – 32y is :
[JEE(Main) 2014, (4, – ¼), 120]
1 2 1 3
(1) (2) (3) (4)
8 3 2 2
9. The normal to the curve, x2 + 2xy – 3y2 = 0, at (1, 1) [JEE(Main) 2015, (4, – ¼), 120]
(1) does not meet the curve again
(2) meets the curve again in the second quadrant
(3) meets the curve again in the third quadrant.
(4) meets the curve again in the fourth quadrant.
10. The area (in sq.units) of the quadrilateral formed by the tangents at the end points of the latera recta to the
x2 y2
ellipse 1 , is [JEE(Main) 2015, (4, – ¼), 120]
9 5
27 27
(1) (2) 18 (3) (4) 27
4 2
11. Let O be the vertex and Q be any point on the parabola, x2 = 8y. If the point P divides the line segment OQ
internally in the ratio 1 : 3, then the locus of P is [JEE(Main) 2015, (4, – ¼), 120]
(1) x2 = y (2) y2 = x (3) y2 = 2x (4) x2 = 2y
2. Find the equation of tangent to the curve given by x = asin3t, y = b cos3t at a point where t = .
4
[CBSE 2009]
3. Find the slope of the tangent to the curve y = 3x2 – 4x at the point whose x-coordinate is 2.
[CBSE 2009, 2008]
x–7
4. Find the equation of tangent to the curve y = , at the point, where it cuts the x-axis.
( x – 2)( x – 3)
[CBSE 2010]
5. Find the point on the curve y2 = 2x which is at a minimum distance from the point (1, 4).
[CBSE 2011, 2009, 2007]
x2 y2
6. Find the equations of the tangent and normal to the curve = 1 at the point ( 2 a, b).
a2 b2
[CBSE 2014, 4]
TN - 160
Tangent Normal & Its Applications in Conics
EXERCISE - 1 3 1 1 1 1
E-3. , tan–1 E-4. – = –
2 5 a b a b
PART - I E-5. (2, 1) E-6. (– 6, 3)
E-7. 10
Section (A)
A-1. y=x A-2. a = 1, b = –2 PART - II
A-3. (1, –1) , (–1, –5) A-5. (9/4, 3/8) Section (A)
A-6. 2x + y = 4, y = 2x A-7_. x+y=3 A-1. (A) A-2. (C) A-3. (B)
A-4. (D) A-5. (B) A-6. (C)
4096 2048
A-8_. y = 0, y – (x – 8/3) A-7. (B) A-8. (B) A-9_. (A)
81 81 A-10_. (A) A-11. (B)
A-9. 4y = 9x + 4, 4y = x + 36 Section (B)
A-11. (9/2) B-1_. (D) B-2_. (B) B-3_. (A)
Section (B) B-4. (C) B-5. (A) B-6. (D)
B-1_. 8 2 B-2_. (4, 0), (–4, 0) B-7. (B) B-8. (A) B-9. (C)
B-10. (C) B-11. (B) B-12. (C)
4 4 B-13. (A) B-14. (A) B-15. (B)
B-3_. << B-4_. 35
5 17 Section (C)
C-1. (A) C-2. (D) C-3. (C)
8 C-4. (A) C-5. (B) C-6. (D)
B-5_. 10 B-7. (– , – 4] [4, )
3 C-7. (A) C-8. (B) C-9. (A)
B-8. x – 4y + 28 = 0 at (28, 14) C-10. (C) C-11. (A) C-12. (B)
B-9. (i) y = (x – a) tan , (ii) x = a, (iii) y = x
C-13. (C)
B-10. (yes, (5, 4)) B-11. (–2, 0)
B-12. 4x + 5y = 40, 4x – 35y = 200. Section (D)
B-14. =±6 B-15. n = ± 2 D-1. (C) D-2. (B) D-3. (B)
B-16. x 2 + 64 y2 = 80 & x2 + 4 y2 = 20 D-4. (C) D-5. (A) D-6. (B)
B-17. ex ± y = a, – ex ± y = a D-7. (A) D-8. (D) D-9. (C)
D-10. (A) D-11. (B)
B-19. x+y±3 3 =0 Section (E)
B-21. (i) –16/9 (ii) –20/9 E-1. (B) E-2. (A)
Section (C)
C-1. x + 3y = 33 C-5. (4a, 4a) PART - III
C-6. (3, 0) 1. (A p, q, r); (B r) ; (C p) ; (D q)
C-7. 12 x + 5 y = 48; 12 x 5 y = 48 2. (A) (s), (B) (r), (C) (q), (D) (p)
Section (D) 3. (A) (r), (B) (p), (C) (s), (D) (q)
4. (A) (q), (B) (s), (C) (s), (D) (r)
25 16
D-1. x + 2y – 3 = 0 D-2. ,
4 3 EXERCISE - 2
D-3. 48x + 25y – 169 = 0
D-4. y = x2 + 2
PART - I
5 3 1. (A) 2. (C) 3. (D) 4. (B)
D-5. y x ; x – 3 = 0 ; 8 sq. unit
12 4 5. (A) 6. (D) 7. (C) 8. (C)
9_. (B) 10_.(C) 11. (B) 12. (C)
x2 y2 y 13. (D) 14. (B) 15. (A) 16. (A)
D-6. y2 – 2ax + 8a2 = 0 D-7. 2 + 2 =
a b b 17. (D) 18. (B) 19. (B) 20^.(D)
D-8. (x + y ) = 16 x - 9 y
2 2 2 2 2 21. (D) 22^.(B) 23^.(B) 24^.(C)
Section (E) 25. (A) 26_.(D) 27. (A) 28_.(B)
29_.(A) 30. (B) 31. (C)
E-1. E-2. –1
3
TN - 161
Tangent Normal & Its Applications in Conics
PART - II PART - III
1^. 0 2^. 1 3^. 4 1. The curves x = y2 and x y = k intersect, where
4_. 24 5^. 19 6^. 2 y2 . y = k y = k1/3
7_. 1 8^. 16 9^. 23 x = k2/3 i.e., The intersect at P(k2/3, k1/3)
10^. 14 11^. 55 12^. 3
dy dy 1
13^. 7 14^. 22 15^. 4 Now, x = y2 1 = 2y =
16. 30 17. 25 18^. 6 dx dx 2 y
19^. 9 20^. 4 21^. 3
dy 1
22^. 1 23^. 13 24^. 1 m1 = = and xy = k
dx
P 2k1/ 3
25. 3 26^. 2
dy dy y
PART - III x +1·y=0 = –
dx dx x
EXERCISE - 3 dy b
m = dx =
t a
PART - I 4
3
1. (D) 2. (A) 3*. (AB) 1 a
Now x1 = asin = a
3 =
4. (A) (p), (B) (q), (C) (s), (D) (r) 4 2 2 2
5. (A) 6. (C) 7. (B)
3
8. (D) 1 b
9. (A) p, q, (B) p, q, (C) q, r, (D) q, r y1 = bcos = b
3 =
4 2 2 2
10. (B) 11*. (AD) 12. (C)
13*.(AB) 14*.(CD) 15. (D) a b
16. (C) 17. (A) 18. (B) Equation of tangent at , is
19. (A) 20. 2 21. 2 2 2 2 2
22*.(ABD) 23. (B) 24*.(AB)
25. (B) 26. (D) 27. 8 b b a
y– =– x –
28. (D) 29. (D) 30. (B) 2 2 a 2 2
31. 4 32. 2 33*.(A,D) [ y – y1 = m(x – x–1)]
34. 4 35*.(A,B) 36*.(A,B,D)
2 2y – b b 2 2x – a
PART - II = – a
2 2 2 2
1. (1) 2. (4) 3. (1) 4. (3)
5. (2) 6. (2) 7. (1) 8. (3) 2 2 bx + 2 2 ay = 2ab
9. (4) 10. (4) 11 (4)
TN - 162
Tangent Normal & Its Applications in Conics
dy ds y2
3. We have y = 3x2 – 4x
= 6x – 4
dx dy = 2 2 – 1 · y + 2(y – 4) = y3 – 8
The slope of the tangent at x = 2 is
dy d2 s ds
m= = 6(2) – 4 = 12 – 4 = 8 and 2 = 3y2 For max. or min. dy = 0
dx x 2 dy
y –8=0 3
y3 – 23 = 0
4. Given curve is (y – 2) (y + 2y + 4) = 0
2
( x 2 – 5 x 6) – (2x 2 – 5 x – 14 x 35 )
= x2 y2
( x – 2) 2 ( x – 3 ) 2 6. =1 differentiale. w.r.t.x
a2 b2
( x 2 – 5 x 6) – (2x 2 19 x 35) 2x 2y dy y dy x
= = 0 2
( x – 2 ) 2 ( x – 3 )2 a 2
b 2 dx b 2 dx
a
x 2 – 5 x 6 – 2x 2 19 x – 35 dy b2 x
= m= = 2 at point ( 2 a, b)
( x – 2) 2 ( x – 3 ) 2 dx a y
dy – x 2 14 x – 29 b 2 2a 2b
= slope of tangent = 2 b =
dx ( x – 2) 2 ( x – 3 ) 2 a a
Since tha curve cuts at x-axis
y=0 a
slope of normal = –
2b
x–7
From (i), we get 0 = x–7=0
x 2 – 5x 6 2b
x=7 Point of intersection is (7, 0) equation of tangent is y – b = (x – 2 a)
a
Slope of tangent at (7, 0) is
ay – ab = 2 bx – 2ab 2 bx – ay = ab
dy – (7)2 14(7) – 29
m = dx = a
( 7, 0 ) ( 7 – 2 ) 2 (7 – 3 ) 2 equation of normal is y – b = (x –
2b 2 a)
–49 98 – 29 20 1
= = = 2 y 2 by – 2 b = –ax +
2
2a
2
25 16 25 16 20
Equation of tangent at (7, 0) is ax + 2 by = 2 (a + b ).
2 2
1
y–0= (x – 7) [ y – y1 = n(x – x1)]
20
x – 7 = 20y x – 20y – 7 = 0
TN - 163
Tangent Normal & Its Applications in Conics
SUBJECTIVE QUESTIONS
2
1^. Find the slope of the normal at the point with abscissa x = – 2 of the graph of the function f(x) = x x
2^. Tangent at any point on the curve x2/3 + y2/3 = a2/3 meet the coordinate axes at P & Q. Find locus of mid
point of PQ.
3^. If a normal to a parabola y2 = 4ax make an angle with its axis, then find the angle where it again cuts
the curve.
4. If p be the length of perpendicular drawn from the origin upon the tangent to the curve x = a cos t + a t sin t
and y = a sin t – at cos t at the point t, then prove that :
dx dy
(i) p | t | (ii) p i ; where i 1
dt dt
5. Show that the equation of the tangent to the curve represented parametrically by the equations.
x y a
t t t t f t
x = a and y = a can be expressed in the form =0
f t f t ' t ' t f ' t
where f, g and h are the differentiable functions.
x2 y2
6. Show that the condition, for curves x 2/3
+y 2/3
=c 2/3
and + 2 = 1 to touch, is c = a + b.
a2 b
7_. Tangent at a point P1 (other than (0, 0)) on the curve y = x3 meets the curve again at P2 . The tangent at P2
meets the curve at P3, and so on, show that the abscissae of P1, P2, . . . ., Pn from a G.P. Also find the ratio
area P1P2P3
of area P P P .
2 3 4
8. If the relation between subnormal SN and subtangent ST at any point S on the curve by2 = (x + a) 3 is
p
p(SN) = q (ST)2, then find value of in terms of b and a.
q
9_. If r1, r2 be the length of the perpendicular chords of the parabola y2 = 4ax drawn through the vertex, then show
that (r1r2)4/3 = 16a2(r12/3 + r22/3).
10. Prove that the circle circumscribing the triangle formed by any three tangents to a parabola passes
through the focus.
11. A chord is a normal to a parabola and is inclined at an angle to the axis; prove that the area of the
triangle formed by it and the tangents at its extremities is 4a2 sec 3 cosec 3
12^. From an external point P, pair of tangent lines are drawn to the parabola, y2 = 4x. If 1 & 2 are the
inclinations of these tangents with the axis of x such that, 1 + 2 = , then find the locus of P .
4
13. TP and TQ are tangents to the parabola and the normals at P and Q meet at a point R on the curve ;
prove that the centre of the circle circumscribing the triangle TPQ lies on the parabola 2y2 = a(x – a).
14. From an external point P, tangents are drawn to the parabola; find the equation of the locus of P when
these tangents make angles 1 and 2 with the axis, such that cos 1 cos 2 = , which is constant.
x2 y2
15_. A straight line PQ touches the ellipse = 1 and the circle x2 + y2 = r2(b < r < a). RS is a focal chord
a2 b2
of the ellipse. If RS is parallel to PQ and meets the circle at points R and S. Find the length of RS.
TN - 164
Tangent Normal & Its Applications in Conics
16. A pair of tangents are drawn to the parabola which are equally inclined to a straight line whose inclination
to the axis is ; prove that the locus of their point of intersection is the straight line y = (x – a) tan 2.
x2 y2
17._ PQ is the chord joining the points whose eccentric angles are 1 and 2on the hyperbola = 1,
a2 b2
x2 y2
If 1 – 2 = 2, where is constant, prove that PQ touches the hyperbola cos –
2
= 1.
a2 b2
18. Prove that the normals at the points, where the straight line x + my = 1 meets the parabola y2 = 4ax,
4am 4am 2
meet on the normal at the point 2 ,
on the parabola.
19. Prove that the equation to the circle, which passes through the focus and touches the parabola y2 =4ax
at the point (at2, 2at), is x2 + y2 – ax(3t2 + 1) – ay (3t – t3) + 3a2t2 = 0.
Prove also that the locus of its centre is the curve 27ay2 = (2x – a) (x – 5a) 2.
20. Two tangents to the parabola y² = 8x meet the tangent at its vertex in the points P & Q. If
PQ = 4 units, prove that the locus of the point of the intersection of the two tangents is
y² = 8 (x + 2).
21^. Find locus of a point P if the three normals drawn from it to the parabola y2 = 4ax are such that two of
them make complementry angles with the axis of the parabola
x2 y2
22_. Find the locus of the mid-points of the chord of the hyperbola = 1 which subtend a right angle at the
a2 b2
origin
23. Prove that the orthocentre of any triangle formed by three tangents to a parabola lies on the directrix.
24. If tangent drawn at a point (t², 2t) on the parabola y² = 4x is same as the normal drawn at a point
( 5 cos 2 sin ) on the ellipse 4x² + 5y² = 20. Find the values of t &
x2 y2
25. Find the condition so that the line px + qy = r intersects the ellipse = 1 in points whose
a2 b2
eccentric angles differ by .
4
26. Prove that the minimum length of the intercept made by the axes on the tangent to the ellipse
x2 y2
2 + = 1 is equal to a + b.
a b2
27. Prove that the sum of the eccentric angles of the extremities of a chord of an ellipse, which is drawn in
a given direction is constant and is equal to twice the eccentric angle of the point at which the tangent
is parallel to the given direction.
28. If the normals at , , , on an ellipse are concurrent, prove that (cos)(sec ) = 4
29. If a chord joining the points P (a sec , a tan ) & Q (a sec , a tan ) on the hyperbola x 2 y2 = a2 is
a normal to it at P, then show that tan = tan (4 sec 2 1) .
x2 y2
30. Find the condition so that the line px + qy = r intersects the ellipse = 1 in points whose
a2 b2
eccentric angles differ by .
4
TN - 165
Tangent Normal & Its Applications in Conics
x2 y2
31. Show that the equation of the pair of tangents to the ellipse = 1 at the points of intersection
a 2 b2
x2 y2
with the line, p x + q y + 1 = 0 is, 2 2 1 (p2 a2 + q2 b2 1) = (p x + q y + 1)2.
a b
x2 y2 x2 y2
32. A tangent to the ellipse 1 meets the ellipse a b at the points P and Q; prove
a2 b2 a b
that the tangents at P and Q are at right angles.
33. Chords of the hyperbola x 2/a2 y2/b2 = 1 are tangents to the circle drawn on the line joining the foci as
diameter . Find the locus of the point of intersection of tangents at the extremities of the chords .
34. Find the locus of the point, the chord of contact of the tangents drawn from which to the
x2 y2
ellipse 2 2 1 touches the circle x2 + y2 = c 2, where c < b < a.
a b
35_. Find the locus of centre of a family of circles passing through the vertex of the parabola y2 = 4ax, and cutting
the parabola orthogonally at the other point of intersection.
x2 y2 x2 y2
36^. From any point on the hyperbola H1 : 2 = 1 tangents are drawn to the hyperbola H2 : 2 2 = 2.
a b2 a b
Then find the area cut-off by the chord of contact on the asymptotes of H2 .
37^. The chord PQ of the rectangular hyperbola xy = a2 meets the x-axis at A; C is the mid point of PQ & 'O'
is the origin. Then prove that the ACO is isosceles.
38._ If the normals at (xi, yi), i = 1, 2, 3, 4 on the rectangular hyperbola, xy = c2, meet at the point (, ) show that
(i) xi = (i) yi = (iii) xi = yi = –c4
(iv) xi =
2 2
(v) yi =
2 2
1 1 1
1^. 2^. 4 (x 2 + y2) = a2 3^. tan1 tan 7.
3 2 16
1 1
24. = – tan–1 2, t = ; = + tan12, t = ; = , t = 0
5 5 2
25. a2p2 + b2q2 = r 2sec 2 = (4 – 2 2 ) r2 30. a2p2 + b2q2 = r 2sec 2 = (4 – 2 2 ) r2
8 8
x2 y2 1 x2 y2 1
33. 34. 35. 2y2(2y2 + x2 – 12ax) = ax(3x – 4a)2
a 4 b 4 a 2 b2 4 4
a b c2
36^. 4 ab
TN - 166
Indefinite Integration
Indefinite Integration
But just as much as it is easy to find the differential of a given quantity, so it is difficult to find the integral of a given differential.
Moreover, sometimes we cannot say with certainty whether the integral of a given quantity can be found or not.
Bernoulli, Johann
If f & g are functions of x such that g(x) = f(x), then indefinite integration of f(x) with respect to x is
defined and denoted as f(x) dx = g(x) + C, where C is called the constant of integration.
Standard Formula:
ax bn 1
(i) (ax + b) n
dx =
a n 1
+ C, n 1
dx 1
(ii) = n |ax + b| + C
ax b a
1 ax+b
(iii) eax+b dx =
a
e +C
1 a pxq
(iv) apx+q dx =
p n a
+ C; a > 0
1
(v) sin (ax + b) dx =
a
cos (ax + b) + C
1
(vi) cos (ax + b) dx =
a
sin (ax + b) + C
1
(vii) tan(ax + b) dx =
a
n |sec (ax + b)| + C
1
(viii) cot(ax + b) dx =
a
n |sin(ax + b)| + C
1
(ix) sec² (ax + b) dx =
a
tan(ax + b) + C
1
(x) cosec²(ax + b) dx = –
a
cot(ax + b)+ C
1
(xi) sec (ax + b). tan (ax + b) dx =
a
sec (ax + b) + C
1
(xii) cosec (ax + b). cot (ax + b) dx = –
a
cosec (ax + b) + C
x
(xiii) secx dx = n |secx + tanx| + C OR n tan + C
4 2
IIM - 167
Indefinite Integration
dx x
(xv) a x2 2
= sin1
a
+C
dx 1 x
(xvi) a x
2 2
= tan1
a a
+C
dx 1 x
(xvii) x x2 a 2
=
a
sec 1
a
+C
dx x
(xviii) x2 a2
2 2
= n x x a + C OR sinh1
a
+C
dx x
(xix) x a2 2
2 2
= n x x a + C OR cosh1
a
+C
dx 1 ax
(xx) a x
2 2 =
2a
ln ax + C
dx 1 xa
(xxi) x a
2 2 =
2a
ln xa + C
x a2 x
(xxii) a 2 x 2 dx =
2
a 2 x2 +
2
sin1
a
+C
x a2 x x 2 a2
(xxiii) x a dx =
2
2
2
x a
2 2
+
2
n
a
+C
x a2 x x 2 a2
(xxiv) x a dx =
2
2
2
x a
2 2
2
n a +C
e ax
(xxv) eax. sin bx dx =
a 2 b2
(a sin bx b cos bx) + C
e ax
(xxvi) eax. cos bx dx =
a 2 b2
(a cos bx + b sin bx) + C
Theorems on integration
IIM - 168
Indefinite Integration
4x
5
Example # 1 Evaluate : dx
4 6 2 6
4x
5
Solution. dx = x +C= x + C.
6 3
7 2
x
3
Example # 2 Evaluate : 5x 2 4 dx
x x
7 2
x
3
Solution. 5x 2 4 dx
x x
7 2
= x
3
dx + 5x
2
dx – 4dx + x
dx + x
dx
1
x x x
3 2 1/ 2
= dx + 5 . dx – 4 . 1 . dx + 7 . dx + 2 . dx
x
x4 x3 x 1/ 2 x4 5 3
= +5. – 4x + 7 n | x | + 2 1/ 2 + C = + x – 4x + 7 n | x | + 4 x +C
4 3 4 3
e
xn a
Example # 3 Evaluate : e anx eana dx , a > 0
Solution. We have,
(e (e (a
xn a na x a a
e anx e ana ) dx = e nx e na ) dx = x
x a a a ) dx
ax x a 1
= a x dx + x a dx + a a dx =
n a
+
a 1
+ aa . x + C.
2x 3x
Example # 4 Evaluate : 5x
dx
2x 3x 2
x
3
x
(2 / 5) x
2x 3x (3 / 5 ) x
Solution. 5x
dx =
5x 5x
dx =
dx =
5 5 n
2 +
3
+C
n
5 5
sin
3
Example # 5 Evaluate : x cos3 x dx
1 1 1 3 sin 2x sin 6 x
sin (2 sin x cos x) sin
3 3
Solution. x cos3 x dx = dx = 3
2x dx = dx
8 8 8 4
1 1 3 1
=
32 (3 sin 2x sin 6x) dx = cos 2x 6 cos 6 x + C
32 2
x4
Example # 6 Evaluate : x 2
1
dx
x4 x4 1 1 x4 1 1
Solution. x 2
1
dx = x2 1
dx =
x 2 1 x 2 1
dx
1 x3
= ( x 2 1) dx + x 2
1
dx =
3
– x + tan–1 x + C
IIM - 169
Indefinite Integration
1
Example # 7 Evaluate : 4 9x 2 dx
Solution. We have
1 1 1 1
4 9x 2 dx =
9 4x
1
2
dx =
9 ( 2 / 3) 2
x2
dx
1 1 x 1 3x
= . tan–1 +C= tan–1 +C
9 ( 2 / 3) 2/3 6 2
1
tan 1 sin x
2
(1) Evaluate : x dx (2) Evaluate : dx
Integration by Substitution
If we substitution (x) = t in an integral then
(i) everywhere x will be replaced in terms of new variable t.
(ii) dx also gets converted in terms of dt.
x
3
Example # 9 Evaluate : sin x 4 dx
Solution. We have
x
3
= sin x 4 dx
1
Let x4 = t d(x 4) = dt 4x 3 dx = dt dx = dt
4x 3
1 1 1
=
4 sin t dt = – 4
cost + C = –
4
cos x 4 + C
(n x )2
Example # 10 Evaluate : x
dx
(n x )2 1
Solution. Let = x
dx Put nx = t
x
dx = dt
t3 ( n x )3
= t 2dt =
3
+c =
3
+C
(1 sin
2
Example # 11 Evaluate : x ) cos x dx
(1 sin
2
Solution. Let = x ) cos x dx Put sinx = t cosx dx = dt
t3 sin3 x
= (1 t 2 ) dt = t +
3
+ c = sin x +
3
+C
IIM - 170
Indefinite Integration
x
Example # 12 Evaluate : x 4
x2 1
dx
Solution. We have,
x x dt
= x 4
x 1 2 dx = (x 2 2
) x2 1
dx {Put x 2 = t x.dx =
2
}
1
=
1
2 2
1
t t 1
dt =
1
2
2
1 3
2 dt
t
2 2
t 1 1 2t 1 1 2x 2 1
1 1 2 +C = +C=
= . tan–1 tan–1 tan–1 + C.
2 3 3 3 3 3 3
2 2
( f ( x ))n1
Note: (i) [ f(x)]n f (x) dx =
n 1
+C
f ( x) ( f ( x ))1n
(ii) f (x)n
dx =
1 n
+C,n1
dx
(iii) x ( x 1)
n
; n N Take x n common & put 1 + x n = t.
dx
(iv) ( n1) ; n N, take xn common & put 1 + xn = tn
x x 1
2
n
n
dx
(v) ; take x n common as x and put 1 + x n = t.
n
x 1 x
n 1/ n
Integration by Parts : Product of two functions f(x) and g(x) can be integrate using formula :
d
f ( x) g(x) dx = f(x)
g(x) dx – dx f ( x) g( x) dx dx
(i) when you find integral g( x) dx then it will not contain arbitarary constant.
(ii) g( x) dx should be taken as same at both places.
(iii) The choice of f(x) and g(x) can be decided by ILATE guideline.
the function will come later is taken an integral function (g(x)).
Inverse function
L Logarithmic function
A Algebraic function
T Trigonometric function
E Exponential function
IIM - 171
Indefinite Integration
x tan
1
Example # 13 Evaluate : x dx
x tan
1
Solution. Let = x dx
x2 1 x2
= (tan–1 x)
2
– 1 x 2 .
2
dx
x2 1 x2 1 1 x2 1 1
=
2
tan–1 x –
2 x 12 dx =
2
tan–1 x –
2 1 x 2
dx
1
x2 1
= tan–1 x – [x – tan–1 x] + C.
2 2
x2 1 x2 x2 1 x 2 1 1
=
2
n (x + 1) –
2 x 1
dx =
2
n (x + 1) –
2 x 1
dx
x2 1 x2 1 1
=
2
n (x + 1) –
2
x 1 x 1 dx
x2 1 1
=
2
n (x + 1) –
2 ( x 1) x 1 dx
1 x x n | x 1 |
2
x2 +C
= n (x + 1) –
2 2 2
e
2x
Example # 15 Evaluate : sin 3x dx
e
2x
Solution. Let = sin 3x dx
cos 3 x cos 3 x
= e2x 2e
2x
– dx
3 3
1 2x 2
e
2x
=– e cos 3x + cos 3x dx
3 3
1 2x 2 2 x sin 3 x sin 3 x
=–
3
e cos 3x + e
3 3
2e 2 x
3
dx
1 2x 2 4
e
2x
=– e cos 3x + e2x sin 3x – sin 3x dx
3 9 9
1 2x 2 4
=– e cos 3x + e2x sin 3x –
3 9 9
4 e 2x
+ = (2 sin 3x – 3 cos 3x)
9 9
IIM - 172
Indefinite Integration
13 e 2x
= (2 sin 3x – 3 cos 3x)
9 9
e 2x
= (2 sin 3x – 3 cos 3x) + C
13
Note :
x
e
x
Example # 16 Evaluate : dx
( x 1)2
x 1 1 1 1 ex
Solution. Given integral = ex
( x 1)2
dx = ex dx =
( x 1) ( x 1)2
( x 1)
+C
1 sin x
e
x
Example # 17 Evaluate : dx
1 cos x
x x
1 2 sin cos
2 2
e
x
Solution. Given integral = 2 x dx
2 sin
2
1 x x x
e
x
= cos ec 2 cot dx = – ex cot +C
2 2 2 2
1
Example # 18 Evaluate : n (nx )
dx
(nx )2
1
Solution. Let = n (nx ) (nx ) 2 dx {put x = et et dt}
1 1 1 1
e e
t t
= nt 2 dt = nt 2 dt
t t t t
1 1
= et nt + C = x n ( nx ) +C
t nx
Self Practice Problems
IIM - 173
Indefinite Integration
Integration of Rational Algebraic Functions by using Partial Fractions:
PARTIAL FRACTIONS :
f (x)
If f(x) and g(x) are two polynomials, then g( x ) defines a rational algebraic function of x.
f (x)
If degree of f(x) < degree of g(x), then g( x ) is called a proper rational function.
f (x)
If degree of f(x) degree of g(x) then g( x ) is called an improper rational function.
f (x) f (x)
If g( x ) is an improper rational function, we divide f(x) by g(x) so that the rational function g( x ) is
( x )
expressed in the form (x) + g( x ) , where (x) and ( x ) are polynomials such that the degree of ( x )
f (x)
is less than that of g(x). Thus, g( x ) is expressible as the sum of a polynomial and a proper rational
function.
f (x)
Any proper rational function g( x ) can be expressed as the sum of rational functions, each having a
simple factor of g(x). Each such fraction is called a partial fraction and the process of obtained them is
f (x)
called the resolutions or decomposition of g( x ) into partial fractions.
f (x)
The resolution of g( x ) into partial fractions depends mainly upon the nature of the factors of g(x) as
discussed below :
f (x) A1 A2 An
g( x ) x a1 + x a 2 + ..... + x a n
where A1, A2, ...... An are constants and can be determined by equating the numerator on R.H.S. to the
numerator on L.H.S. and then substituting x = a1, a2, ........,an.
3x 2
Example # 19 Resolve into partial fractions.
x 6 x 2 11x 6
3
3x 2 3x 2
Solution. We have, =
3 2
x 6 x 11x 6 ( x 1)( x 2)( x 3)
3x 2 A B C
Let = + + . Then,
( x 1)( x 2)( x 3) x 1 x2 x 3
IIM - 174
Indefinite Integration
Putting x – 2 = 0 or, x = 2 in (i), we obtain
8 = B (2 – 1) (2 – 3) B = –8.
Putting x – 3 = 0 or, x = 3 in (i), we obtain
11
11 = C (3 – 1) (3 – 2) C = .
2
3x 2 3x 2 5 8 11
= = – +
3 2
x 6 x 11x 6 ( x 1)( x 2)( x 3) 2( x 1) x2 2( x 3)
Note : In order to determine the value of constants in the numerator of the partial fraction corresponding to the
non-repeated linear factor (px + q) in the denominator of a rational expression, we may proceed as
follows :
q
Replace x by – (obtained by putting px + q = 0) everywhere in the given rational expression except
p
in the factor px + q itself. For example, in the above illustration the value of A is obtained by replacing
3x 2
x by 1 in all factors of except (x – 1) i.e.
( x 1)( x 2)( x 3)
3 1 2 5
A= =
(1 2)(1 3) 2
Similarly, we have
3 2 1 33 2 11
B= = –8 and, C = =
(1 2)(2 3) (3 1)(3 2) 2
x 3 6 x 2 10 x 2
Example # 20 Resolve into partial fractions.
x 2 5x 6
Solution. Here the given function is an improper rational function. On dividing we get
x 3 6 x 2 10 x 2 ( x 4)
=x–1+ ...........(i)
2
x 5x 6 ( x 2 5 x 6)
x 4 x 4
we have, =
2
x 5x 6 ( x 2)( x 3)
x 4 A B
So, let = + , then
( x 2)( x 3) x2 x 3
– x + 4 = A(x – 3) + B(x – 2) ...........(ii)
Putting x – 3 = 0 or x = 3 in (ii), we get
1 = B(1) B = 1.
Putting x – 2 = 0 or x = 2 in (ii), we get
2 = A (2 – 3) A = – 2
x 4 2 1
= +
( x 2)( x 3) x2 x 3
x 3 6 x 2 10 x 2 2 1
Hence =x–1– +
2
x 5x 6 x2 x 3
IIM - 175
Indefinite Integration
CASE II When the denominator g(x) is expressible as the product of the linear factors such that some
of them are repeating.
1 1
Example g( x ) = this can be expressed as
( x a)k ( x a1 )( x a 2 ).......( x a r )
A1 A2 A3 Ak B1 B2 Br
+ 2 + 3 + ....+ k + ( x a ) + ( x a ) + ...... + ( x a )
x a ( x a) ( x a) ( x a) 1 2 r
Now to determine constants we equate numerators on both sides. Some of the constants are determined
by substitution as in case I and remaining are obtained by equating the coefficient of same power of x.
3x 2 (3 x 2)dx
Example # 21 Resolve 2
( x 1) ( x 1)( x 2)
into partial fractions, and evaluate ( x 1) (x 1)(x 2)
2
3x 2 A1 A2 A3 A4
Solution. Let 2 = + 2 + +
( x 1) ( x 1)( x 2) x 1 ( x 1) x 1 x2
3x – 2 = A1 (x – 1) (x + 1) (x + 2) + A2 (x + 1) (x + 2)
+ A3 (x – 1) 2 (x + 2) + A4 (x – 1) 2 (x + 1) .......(i)
1
Putting x – 1 = 0 or, x = 1 in (i) we get 1 = A2 (1 + 1) (1 + 2) A2 =
6
5
Putting x + 1 = 0 or, x = –1 in (i) we get –5 = A3 (–2) 2 (–1 + 2) A3 = –
4
8
Putting x + 2 = 0 or, x = –2 in (i) we get –8 = A4 (–3) 2 (–1) A4 =
9
Now equating coefficient of x 3 on both sides, we get 0 = A1 + A3 + A4
5 8 13
A1 = –A3 – A4 = – =
4 9 36
3x 2 13 1 5 8
2 = + 2 – +
( x 1) ( x 1)( x 2) 36( x 1) 6( x 1) 4( x 1) 9( x 2)
(3 x 2)dx
and hence ( x 1) (x 1)(x 2)
2
13 1 5 8
= n |x – 1| – – n |x + 1| + n |x + 2| + C
36 6( x 1) 4 9
CASE III When some of the factors of denominator g(x) are quadratic but non-repeating. Corresponding
Ax B
to each quadratic factor ax2 + bx + c, we assume partial fraction of the type 2 , where A and
ax bx c
B are constants to be determined by comparing coefficients of similar powers of x in the numerator of
A(2ax b) B
both sides. In practice it is advisable to assume partial fractions of the type 2 + 2
ax bx c ax bx c
The following example illustrates the procedure
IIM - 176
Indefinite Integration
2x 1 2x 1
Example # 22 Resolve
( x 1)( x 2)2 into partial fractions and evaluate (x 1)(x 2
2)
dx
2x 1 A Bx C 2x 1 A( x 2 2) (Bx C)( x 1)
Solution. Let 2 = + 2 . Then, 2 =
( x 1)( x 2) x 1 x 2 ( x 1)( x 2) ( x 1)( x 2 2)
2x – 1 = A (x2 + 2) + (Bx + C) (x + 1) ...(i)
Putting x + 1 = 0 or, x = –1 in (i), we get – 3 = A(3) A = –1.
Comparing coefficients of the like powers of x on both sides of (i), we get
A + B = 0, C + 2A = –1 and C + B = 2
–1 + B = 0, C – 2 = –1 (Putting A = –1) B = 1, C = 1
2x 1 1 x 1
=– + 2
( x 1)( x 2 2) x 1 x 2
2x 1 1 1 x
Hence (x 1)(x 2
2)
dx = – n |x + 1| +
2
n |x2 + 2| +
2
tan–1
2
+C
CASE IV When some of the factors of the denominator g(x) are quadratic and repeating fractions of the
A (2ax b) A2
A 0 (2ax b) A1 1
+ 2 2
form 2 2
ax bx c ax bx c ax bx c
2
2
ax bx c
A
2k 1( 2ax b ) A 2k
+ .......+ 2
ax bx c
k
2
ax bx c k
The following example illustrates the procedure.
2x 3
Example # 23 Resolve into partial fractions.
( x 1)( x 2 1)2
2x 3 A Bx C Dx E
Solution. Let 2 2 = + 2 + . Then,
( x 1)( x 1) x 1 x 1 ( x 2 1)2
2x – 3 = A(x 2 + 1)2 + (Bx + C) (x – 1) (x 2 + 1) + (Dx + E) (x – 1) ......(i)
1
Putting x = 1 in (i), we get – 1 = A (1 + 1)2 A = –
4
Comparing coefficients of like powers of x on both side of (i), we have
A + B = 0, C – B = 0, 2A + B – C + D = 0, C + E – B – D = 2 and A – C – E = –3.
1
Putting A = – and solving these equations, we get
4
1 1 5 2x 3 1 x 1 x5
B= = C, D = and E = = + +
4 2 2 ( x 1)( x 2 1)2 4( x 1) 4( x 2 1) 2( x 2 1)2
2x
Example # 24 Resolve into partial fractions.
3
x 1
2x 2x
Solution. We have, 3 =
x 1 ( x 1)( x 2 x 1)
2x A Bx C
So, let = + . Then, 2x = A (x2 + x + 1) + (Bx + C) (x – 1) ...(i)
( x 1)( x 2 x 1) x 1 x2 x 1
2
Putting x – 1 = 0 or, x = 1 in (i), we get 2 = 3 A A =
3
IIM - 177
Indefinite Integration
2
Putting x = 0 in (i), we get A – C = 0 C = A =
3
Putting x = – 1 in (i), we get –2 = A + 2B – 2 C.
2 4 2
–2= + 2B – B=–
3 3 3
2x 2 1 (– 2 / 3) x 2 / 3 2x 2 1 2 1 x
3 = . + or 3 = + 2
x 1 3 x 1 2
x x 1 x 1 3 x 1 3 x x 1
x2 1 1 1
Answers : (7) (i) n +C (ii) n |x + 1| – n (x 2 + 1) + tan–1 (x) + C
x3 2 4 2
Integration of type
,
,
dx
Express ax 2 + bx + c in the form of perfect square & then apply the standard results.
Example # 25 Evaluate : x 2 2x 5 dx
Solution. We have,
x 2 2x 5 = x 2 2x 1 4 dx = ( x 1)2 2 2
1 1
= (x + 1) ( x 1)2 2 2 + . (2) 2 n |(x + 1) + ( x 1)2 2 2 | + C
2 2
1
= (x + 1) x 2 2x 5 + 2 n |(x + 1) + x 2 2x 5 | + C
2
1
Example # 26 Evaluate : x 2
x 1
dx
1 1
Solution. x 2
x 1
dx = x 2
1
1 1
dx =
( x 1/ 2 )2 3 / 4
dx
x 1
4 4
1 x 1/ 2
= 2
( x 1/ 2) 3 / 2
2 dx =
1
3 /2
tan–1
3 /2
+C
2 2x 1
= tan–1 + C.
3 3
1
Example # 27 Evaluate : 9 8x x 2
dx
1 1 1
Solution. 9 8x x 2
dx = 2
{ x 8 x 9}
dx = 2
{ x 8 x 16 25}
dx
1 1 x 4
= {( x 4) 2
5 } 2 dx = 2
5 ( x 4) 2 dx = sin–1
5
+C
IIM - 178
Indefinite Integration
Self Practice Problems
1
(8) Evaluate : 2x 2
1
x 1
dx (9) Evaluate : 2
2x 3x 2
dx
1 2x 1 1 3 2 3
Answers : (8) n +C (9) log x 4 x 2 x 1 + C
3 2x 2 2
Integration of type
dx,
dx, (px q) ax 2 bx c dx
2x 3 ( 2x 4) 1 2x 4 1
Solution. x 2 4x 1
dx = x 2 4x 1
dx = x 2 4x 1
dx – x 2 4x 1
dx
dt 1
= – dx, where t = (x2 + 4x + 1) for Ist integral
t ( x 2) 2
3 2
=2 t – n | (x + 2) + x 2 4 x 1 | + C = 2 x 2 4 x 1 – n | x + 2 + x 2 4x 1 | + C
Example # 29 Evaluate : ( x 5) x 2 x dx
d
Solution. Let (x – 5) = . (x 2 + x) + . Then,
dx
x – 5 = (2x + 1) + .
Comparing coefficients of like powers of x, we get
1 11
1 = 2 and + = – 5 = and = –
2 2
Hence,
( x 5) x 2 x dx
1 11 1 11
= 2 (2x 1) 2 x 2 x dx = 2 (2x 1) x 2 x dx –
2 x 2 x dx
2 2
1 11 1 1
=
2 t dt –
2 x
2 2
dx (where t = x 2 + x for first integral)
2 2
1 t3 / 2 11 1 x 1 x 1 1
= . – 2 2 2 2
2 3/2 2
2 2
x 1 x 1 1
2
1 1
– . n 2 2 2 + C
2 2
1 3/2 11 2x 1 x 2 x 1 n x 1 x 2 x
= t – +C
3 2 4 8 2
1 2 11 2x 1 x 2 x 1 n x 1 x 2 x
= (x + x) 3/2 – +C
3 2 4 8 2
IIM - 179
Indefinite Integration
Self Practice Problems
x 1
(10) Evaluate : x 2
x3
dx
6x 5
(11) Evaluate : 3x 2 5x 1
dx
(12) Evaluate : ( x 1) 1 x x 2 dx
1 1 2x 1
Answers : (10) Ans. log |x 2 + x + 3| + tan–1 + C
2 11 11
(11) Ans. 2 3x 2 5x 1 + C
1 2 3 9
(12) Ans. (x + x + 1) 3/2 – (2x + 1) 2
3 8 1 x x 2 – 16 log (2x +1 + 2 x x 1 ) + C
(i)
OR
OR
(ii) OR
OR
x
Convert sines & cosines into their respective tangents of half the angles and then, put tan =t
2
(iii)
dx.
d
Express Nr A(Dr) + B (Dr) + C & proceed.
dx
1
Example # 30 Evaluate : 1 sin x cos x dx
1 1
Solution. = 1 sin x cos x dx = 2 tan x / 2 1 tan 2 x / 2
dx
1
1 tan x / 2 1 tan2 x / 2
2
1 tan 2 x / 2 sec 2 x / 2
= 1 tan 2
x / 2 2 tan x / 2 1 tan 2 x / 2
dx = 2 2 tan x / 2
dx
x 1 x
Putting tan = t and sec 2 dx = dt, we get
2 2 2
1 x
= t 1 dt = n | t + 1| + C = n tan
2
1 + C
IIM - 180
Indefinite Integration
3 sin x 2 cos x
Example # 31 Evaluate : 3 cos x 2 sin x dx
3 sin x 2 cos x
Solution. = 3 cos x 2 sin x dx
d
Let 3 sin x + 2 cos x = . (3 cos x + 2 sin x ) + (3 cos x + 2 sin x)
dx
3 sin x + 2 cos x = (3 cos x + 2 sin x ) (–3 sin x + 2 cos x)
Comparing the coefficients of sin x and cos x on both sides, we get
12 5
= and = –
13 13
3 sin x 2 cos x
=
1 . dx + 3 cos x 2 sin x dx
dt
=x+ t
, where t = 3 cos x + 2 sin x
12 5
= x + n | t | + C = x– n | 3 cos x + 2 sin x | + C
13 13
3 cos x 2
Example # 32 Evaluate : sin x 2 cos x 3 dx
Solution. We have,
3 cos x 2
= sin x 2 cos x 3 dx
6 3 8
– 2 = 0, 2 + = 3, 3 + = 2 = ,= and = –
5 5 5
cos x 2 sin x 1
= dx sin x 2 cos x 3 dx + sin x 2 cos x 3 dx
2 tan x / 2 1 tan 2 x / 2
Putting, sin x = , cos x = , we get
1 tan 2 x / 2 1 tan 2 x / 2
1
1 = 2 tan x / 2 2(1 tan 2 x / 2)
dx
3
1 tan2 x / 2 1 tan 2 x / 2
IIM - 181
Indefinite Integration
1 tan 2 x / 2
= 2 tan x / 2 2 2 tan 2
x / 2 3(1 tan 2 x / 2)
dx
sec 2 x / 2
= tan 2
x / 2 2 tan x / 2 5
dx
x 1 x x
Putting tan = t and sec 2 = dt or sec2 dx = 2 dt, we get
2 2 2 2
2dt
1 = t 2
2t 5
x
dt t 1 tan 1
2
=2 (t 1) 2
22
=
2
tan–1
2
= tan–1
2
2
x
tan 1
2 +C
Hence, = x + log | sin x + 2 cos x + 3 | + tan–1
2
6 3 8
where = ,= and = –
5 5 5
dx
Example # 33 Evaluate : 1 3 cos 2
x
Solution. Multiply Nr. & Dr. of given integral by sec 2x, we get
sec 2 x dx 1 tan x
= 2
tan x 4
=
2
tan–1
2
+C
40 9
Answer : (13) x+ log |5sinx + 4cosx| + C
41 41
sin
m
Integration of type x.cosn x dx
Case -
If m and n are even natural number then converts higher power into higher angles.
Case -
If at least one of m or n is odd natural number then if m is odd put cosx = t and vice-versa.
Case -
When m + n is a negative even integer then put tan x = t.
sin
5
Example # 34 Evaluate : x cos 4 x dx
sin
5
Solution. Let = x cos 4 x dx put cos x = t – sinx dx = dt
(1 t (t (t
2 2 4
=– ) . t4 . dt = – 2t 2 1) t4 dt = – 8
2t 6 t 4 ) dt
IIM - 182
Indefinite Integration
(sin x) (cos x ) 7 / 3 dx
1/ 3
Example # 35 Evaluate :
1 7
Solution. Here m + n = – = – 2 (a negative integer)
3 3
1
(sin x) (cos x ) 7 / 3 dx = (tan x)
1/ 3 1/ 3
dx {put tanx = t sec 2x dx = dt}
cos 2 x
3 4/3 3
t
1/ 3
= dt = t +C = (tanx) 4/3 + C
4 4
sin
2
Example # 36 Evaluate : x cos 4 x dx
1 1 1
sin sin sin
2 2 2
Solution. 2x(1 cos 2x )dx = 2x dx + 2x cos 2x dx
8 8 8
Integration of type
dx where K is any constant.
1
Divide Nr & Dr by x² & put x = t.
x
1 x2
Example # 37 Evaluate : 1 x2 x4
dx
1
1 2 dx 1
1 x2 1
Solution. Let = 1 x x 2 4 dx =–
2
x
1
{put x +
x
= t 1 2 dx = dt}
x
x 2 1
x
1
x 1
dt 1 t 1 1 x
= – t 12 =–
2
n
t 1
+C =–
2
n 1
x 1
+C
x
1
Example # 38 Evaluate : x 4
1
dx
Solution. We have,
1 2
1 x2 1 x2
= x 4
1
dx = x 2 1
dx =
2 x 2
1
dx
x2 x2
1 1 1 1
12
1 2 1 1
x2 x2
=
1
2
2
x
1 2
x
x 2 x 2
1
1 dx = 2 2
1
1 dx – 2 1 dx
x 2 x2 2
x x x x
1 1
1 1
1 x2 1 x2
=
2 1
2 dx –
2 1
2 dx
x 2 x 2
x x
IIM - 183
Indefinite Integration
1 1
Putting x – = u in 1st integral and x + = in 2nd integral, we get
x x
du d
=
1
2
u 2
2 2 –
1
2
2
2 2
1 u 1 2
= tan–1 – 1 n +C
2 2 2 2 2 2 2
1 x 1/ x 1 x 1/ x 2
= tan–1 – n +C
2 2 2 4 2 x 1/ x 2
1 x 2 1 1 x2 2 x 1
= tan –1
2 x – 4 2 n x 2 x 2 1 + C
2 2
Self Practice Problem :
x2 1
(14) Evaluate : x 4 7x 2 1
dx
1
x 3
1 x
Answers : (14) n 1 +C
6 x 3
x
1 y 1 y 2 1
(15) tan–1 + n + C where y = tan x –
2 2 2 2 y 2 tan x
Integration of type
OR
.
Put px + q = t 2.
1
Example # 39 Evaluate : ( x 3) x 1
dx
1
Solution. Let = ( x 3) x 1
dx {Put x + 1 = t2 dx = 2t dt}
1 2t
= (t 2
1 3) t 2
dt
dt 1 t2 1 x 1 2
=2 t 2
2 2 =2.
2( 2)
n
t2
+ C =
2
n
x 1 2
+ C.
IIM - 184
Indefinite Integration
x2
Example # 40 Evaluate : (x 2
3 x 3) x 1
dx
x2
Solution. Let = (x 2
3 x 3) x 1
dx
( t 2 1) 2t dt
Putting x + 1 = t , and dx = 2t dt, we get =
2
{(t 2
1)2 3( t 2 1) 3} t 2
1
1
( t 2 1) t2 1
2 t 4 2
t 1
dt = 2 1
t2 2 1
dt {put t –
t
= u}
t
1
du u t t
=2 u 3
2 2 =
2
3
tan–1
3
+C=
2
3
tan–1
3 +C
2 t 2 1 2 x
+C
= tan –1
t 3 + C = 3 tan
–1
3 3 ( x 1)
Integration of type
, put ax + b = ;
, put x =
Example # 41 Evaluate 1
1 1
Solution Let = 1
{put x + 1 =
t
dx = – 2 dt }
t
= 1 1 1
2 = 1 1 = 2
t t 1
t2 1 t 1
t t t t2 t
2
= 1 3
2 = – n t
1
2
1
t
2
3
4
+ C,
t
2 4
1
where t =
x 1
IIM - 185
Indefinite Integration
Example # 42 Evaluate (1 x 2
) 1 x 2
Solution. Put x =
1
t
dx = –
1
dt = (t 2
1) t 2 1
{put t2 – 1 = y2 tdt = ydy }
t2
1 y 1 x2
=– (y =– tan–1 + C = – 1 tan–1
2
2) y 2 2 2 2x +C
dx
(17) Evaluate : ( x 2 5 x 6) x 1
dx
(18) Evaluate : ( x 1) 1 x x 2
dx
(19) Evaluate : (2 x 1) 1 x 2
2
dx
(20) Evaluate : ( x 2 x 2) x 2 2 x 4
2
x 1
Answers : (16) 2 tan–1 x 1 + C (17) 2 tan–1
x 1 – –1
2 tan
2
+C
3
1 1 x2
2 x 1 1
(18) sin–1 +C (19) – tan–1 +C
5
3 3 x
2
x 2 2x 4 6 ( x 1)
1
(20) – n 2 +C
2 6 x 2x 4 6 ( x 1)
Integration of type
dx or dx; put x = cos2 + sin2
dx or dx; put x = sec2 tan2
; put x = t2 or x = t2.
IIM - 186
Indefinite Integration
tann1 x
n = sec 2 x tann2 x dx – n – 2 n =
n 1
– n – 2 , n 2
cot n1 x
cos ec x cot n2 x dx – n – 2
2
n = n = – – n – 2 , n 2
n 1
cos ec x dx = cos ec
n 2
4. n = x cosecn – 2 x dx
(cos ec
2
n = – cotx cosecn – 2x – (n – 2) x 1) cosec n – 2 x dx
(n – 1) n = – cotx cosec n – 2 x + (n – 2) n – 2
cot x cos ec n 2 x n2
n = +
– n 1 n 1 n – 2
Example # 43 Obtain reducation formula for n = sin x dx. Hence evaluate sin x dx
n 4
IIM - 187
Indefinite Integration
Self Practice Problems :
x 3
(21) Evaluate : x4
dx
dx
(22) Evaluate : [( x 1)(2 x )]3 / 2
dx
(23) Evaluate : [( x 2) ( x 1)6 ]1/ 7
8
dx dx
(24) Deduce the reduction formula for n = 4 n
(1 x )
and Hence evaluate 2 = (1 x 4 )2
.
x 4n 5
(24) n = +
4(n 1)(1 x 4 )n 1 4(n 1) n–1
1 1
x 3 1 x
x 1 x
x
2
2 = 4 + tan 1 n + C
4 (1 x ) 4 2 2 2 4 2 1
x 2
x
IIM - 188
Definite Integration & its Application
God does not care about our mathematical difficulties. He integrates empirically .........Einstein, Albert
Newton-Leibnitz formula.
b
d
Let dx (F(x)) = f(x) x (a, b). Then f ( x) dx =
a
lim F(x) – lim F(x).
x b x a
b
Note : 1. If a > b, then f ( x) dx =
a
lim F(x) – lim F(x).
x b x a
b
2. If F(x) is continuous at a and b, then f ( x) dx = F(b) – F(a)
a
2
dx
Example # 1 Evaluate
1
( x 1)( x 2)
1 1 1
Solution = – (by partial fractions)
( x 1)( x 2) x 1 x2
2
dx
1
( x 1)( x 2) = n( x 1) n( x 2)12
9
= n3 – n4 – n2 + n3 = n
8
Self practice problems :
3
x
(3) 1 sec x
0
dx
5 9 n 5 n 3 4
Answers : (1) 5– (2) + +2
2 4 2 1024 2
2 2
(3) – + 2 n
3 3
18 3
b b
Property (1)
a
f ( x ) dx = f (t) dt
a
i.e. definite integral is independent of variable of integration.
b a
Property (2) f ( x ) dx = – f ( x ) dx
b
a
DI - 189
Definite Integration & its Application
b c b
Property (3)
a
f ( x ) dx =
f ( x ) dx + f ( x ) dx, where c may lie inside or outside the interval [a, b].
a c
5
x3 : x 3
Example # 2 If f(x) = 2
3 x 1 : x 3
, then find f ( x ) dx.
2
3
5 3 5 3 x2 5
Solution
f ( x ) dx = f ( x ) dx + f ( x ) dx = ( x 3 ) dx + (3 x 1) dx =
2
3x + x 3 x
2
2
5
3
2 2 3 2 3
94 211
= + 3 (3 – 2) + 53 – 33 + 5 – 3 =
2 2
8
8 5 8
Solution | x 5 | dx =
2
( x 5) dx + ( x – 5) dx
5
=9
2
2 5 2
t dt.
2 3 9
1
1
e x ex e x e x ex e x
Solution 1 e x dx =
0
1 ex
1 ex
dx
1
1
e x e x e x (e x e x ) 1
(e 1 1) e2 1
=
0
1 ex
ex 1
dx =
(e
x
e x ) dx = e – 1 +
1
=
e
0
2
Example # 6 Evaluate cos x dx.
2
2 2
Solution cos x dx = 2 cos x
0
dx = 2 ( cos x is even function)
2
DI - 190
Definite Integration & its Application
1
2x
Example # 7 Evaluate log
1
e dx.
2 x
2x 2x 2x
Solution Let f(x) = loge f(–x) = loge = – log = – f(x)
2x 2x e
2x
1
2x
i.e. f(x) is odd function log
1
e dx = 0
2 x
b b a a
Property (5) f ( x ) dx =
a
f (a b x ) dx. Further
0
f ( x ) dx = f (a x) dx
0
a
2 2
g (sin x ) g (cos x )
Example # 8 Prove that
0
g (sin x ) g (cos x )
dx =
0
g (sin x ) g (cos x )
dx = .
4
2 2 g sin x
g (sin x ) 2
Solution Let = g (sin x ) g (cos x )
dx =
0 0 g sin x g cos x
2 2
2
g (cosx)
= g (cosx) g (sinx)
0
dx
on adding, we obtain
2 2
g (sin x ) g (cos x )
2 = g (sin x ) g (cos x ) g (cos x) g (sin x) dx
0
=
0
dx =
4
2 3
x sin x cos x dx
(12) sin 4
x cos 4 x
dx. (13)
1 tan x
0
6
Answers : (10) (11)
2 2
loge 1 2 (12)
2
16
(13)
12
DI - 191
Definite Integration & its Application
a
2 f x dx , if f (2a x) f (x)
2a a
Property (6) f ( x) dx
= ( f ( x ) f (2a x )) dx =
0
0 0
0, if f (2a x) f (x)
sin
3
Example # 9 Evaluate x cos 3 x dx.
0
sin
3
Solution Let f(x) = sin3x cos3x f( – x) = – f(x) x cos 3 x dx = 0
0
dx
Example # 10 Evaluate 1 2 sin
0
2
x
dx.
1 dx
Solution Let f(x) =
1 2 sin x 2 f( – x) = f(x) 1 2 sin
0
2
x
2 2 2 2
sec x dx sec 2 x dx 2
tan 3 tan x
dx
=2 1 2 sin
0
2
x
=2
0
1 tan2 x 2 tan2 x
=2 1 3 tan
0
2
x
=
3
1 2
0
tan is undefined, we take limit
2
=
2
Lt tan
1
3 tan x tan 1
3 tan 0
=
2
=
3 x 3 2 3
2
dx cos ec 2 x cos ec 2 x dx
Alternatively : 1 2 sin
0
2
x
= 0
cos ec 2 x 2
dx =
0
cot 2 x 3
Observe that we are not converting in terms of tan x as it is not continuous in (0, )
1 1 cot x 1 1 cot x cot x
tan = – Lt tan Lt tan 1
=–
3 3 0 3 x 3 x 0 3
1
=– 2 2 =
3 3
2 2 2
Example # 11 Prove that n sin x
0
dx = n cos x dx = n (sin 2x) dx = –
0 0
2
n 2 .
2
Solution Let = n sin x dx
0
..........(i)
2
=
n sin 2 x dx
0
(by property P – 5)
2
= n (cos x ) dx
0
..........(ii)
DI - 192
Definite Integration & its Application
Adding (i) and (ii)
2 2
sin 2x
2= n (sin x . cos x ) dx = n
0 0
2
dx
2 2
2= n (sin 2x ) dx – n 2 dx
0 0
2 = 1 – n 2 ..........(iii)
2
2
where = n (sin 2x ) dx
0
1
put 2x = t dx = dt
2
L.L:x=0 t=0
U.L:x= t=
2
2
1 = n (sin t ) ·
0
1
2
dt =
1
2
×2 n (sin t ) dt (by using property P – 6)
0
1 =
(iii) gives = – n 2
2
Self practice problems :
Evaluate the following
1
n x 1
x sin 1 x
2
Answers : (14) n 2 (15) n 2 (16) – n 2
2 2
(i) f ( x ) dx = n f ( x ) dx, n z
0
0
a nT T
(ii) a
f ( x ) dx = n f ( x ) dx, n z, a R
0
nT T
(iii) f ( x ) dx = (n – m) f (x ) dx, m, n z
0
mT
a nT a
(iv)
nT
f ( x ) dx = f ( x ) dx, n z, a R
0
b nT b
(v)
a nT
f ( x ) dx =
f ( x ) dx, n z, a, b R
a
DI - 193
Definite Integration & its Application
2
e
{x}
Example # 12 Evaluate dx.
1
2 1 3 1 1
e e e
{x} {x} {x} {x}
Solution dx = dx = 3 e dx = 3 dx = 3(e – 1)
1 1 0 0
n v
Example # 13 Evaluate | cos x | dx ,
0
2
< v < and n z.
n v v n v 2 v
2
= (1 – 0) – (sin v – 1) + 2n cos x
0
dx = 2 – sin v + 2n (1 – 0) = 2n + 2 – sin v
e sin 2x
{3 x }
(17)
1
dx. (18)
0
1 e sin x
dx. (19)
sin x cos 4 x
4
dx.
Answers : (17) 3 (e – 1) (18) 1000 (19)
4
Property (8) If (x) f(x) (x) for a x b, then
b b b
( x ) dx
a
f ( x ) dx
(x) dx
a a
b
Property (9) If m f(x) M for a x b, then m (b – a) f ( x ) dx M (b – a)
a
b
Further if f(x) is monotonically decreasing in (a, b), then f(b) (b – a) < f ( x ) dx < f(a) (b – a) and if f(x)
a
b
is monotonically increasing in (a, b), then f(a) (b – a) < f ( x ) dx < f(b) (b – a)
a
b b
Property (10) f ( x) dx
a
f ( x ) dx
a
DI - 194
Definite Integration & its Application
Solution 0 < x 3 < x2 x2 < x2 + x3 < 2x2
–2x2 < – x2 – x3 < –x2 4 – 2x2 < 4 –x2 – x3 < 4 – x2
4 2x 2 < 4 x2 x3 < 4 x2 f1(x) < f3(x) < f2(x) for x (0, 1)
1 1 1
0
f1( x ) dx <
0
f3 ( x ) dx < f ( x) dx
0
2
1 1 1 1
x dx 1 x dx
sin –1
2 0
<
0 4 x2 x3
<
2
sin –1
2 0 6
<
0 4 x2 x3
<
4 2
2
sin x
Example # 15 Estimate the value of 0
x
dx.
sin x
Solution Let f(x) =
x
x cos x sin x (cos x )( x tan x )
f(x) = 2 = <0
x x2
f(x) is monotonically decreasing function.
f(0) is not defined, so we evaluate
Lt Lt sin x = 1. Take f(0) = Lt f(x) = 1
f(x) = x 0
x 0 x x 0
2
f =
2
2 2
2 sin x sin x
. 0 <
2
0
x
dx < 1 . 0
2
1<
0
x
dx <
2
e
x2
Example # 16 Estimate the value of dx using (i) rectangle, (ii) triangle.
0
Solution (i) By using rectangle
1
e
x2
Area OAED < dx < Area OABC
0
e
x2
1< dx < 1 . e
0
e
x2
1< dx < e
0
(ii) By using triangle
1
e
x2
Area OAED < dx < Area OAED + Area of triangle DEB
0
1 1
x2
1
e x dx < e 1
2
DI - 195
Definite Integration & its Application
1 1
e dx .
2
Example # 17 Estimate the value of e x dx by using x
0 0
e e dx
x2 x
1×1< dx <
0 0
2
1< e x dx < e – 1
0
e e
x 2 x2
(20) Prove the following : cos x dx < cos 2 x dx
0 0
2 2
sin sin
n 1 2
(21) Prove the following : 0 < x dx < x dx , n > 1
0 0
1 1
2
x
(22) Prove the following : e 4 < ex dx < 1
0
1
x 3 cos x
(23) Prove the following : –
1
2
0
2 x2
dx <
1
2
2
(24) Prove the following : 1 <
0
sin x dx <
2
2
x dx 1
(25) Prove the following : 0 < 16 x
0
3 <
6
h( x )
Leibnitz Theorem : If F(x) =
f (t) dt , then
g( x )
dF( x )
= h(x) f(h(x)) – g(x) f(g(x))
dx
h( x )
dF( x )
= P(h(x)) h(x) – P(g(x)) g(x) = f(h(x)) h(x) – f(g(x)) g(x)
dx
x2
Example # 18 If F(x) =
x
sin t dt , then find F(x).
DI - 196
Definite Integration & its Application
e3 x
t
Example # 19 If F(x) = log e t
dt , then find first and second derivative of F(x) with respect to
e2 x
n x at x = n 2.
d 2F( x ) d d 1
2 = d (n x ) (e6x – e4x) = (e6x – e4x) × = (6 e6x – 4 e4x) x
d ( n x ) dx dnx / dx
First derivative of F(x) at x = n 2 (i.e. ex = 2) is 26 – 24 = 48
Second derivative of F(x) at x = n 2 (i.e. ex = 2) is (6 . 26 – 4 . 24) . n 2 = 5 . 26 . n 2.
2
x 2
e t dt
0
Example # 20 Evaluate xLt
x .
e
2t 2
dt
0
2
x 2
e t dt
Solution Lt 0 form
x x
e
2t 2
dt
0
2 2 2
2 . e t dt . e x 2 . e t dt 2
2 . ex
= Lt 0 = Lt 0 = xLt 2 =0
x
1 . e 2x
2 x
ex
2 2x . e x
x
dt
Example # 21 If f(x) = x t , then find f(x).
log ex
x x
1 1 1 1
Solution f(x) =
n x
x t 2
dt + 1 . 1 – 1
2x x
= +
1
x n x ( x t ) n x 2x x x n x
–
1 1 1 1 1 x 1 n x 1
= – x n x + – x x n x = – x x n x = x x n x
2x 2x x
x
x
dt
Alternatively : f(x) =
n x
n ( x t )
xt
(treating ‘t’ as constant)
n x
f(x) = n 2x – n (x + n x)
1 1 1 n x 1
f(x) = – x n x 1 = x x n x
x x
DI - 197
Definite Integration & its Application
1
xb 1
Example # 22 Evaluate
0
nx
, ‘b’ being parameter..
1 1
xb 1 d (b ) x b nx
Solution Let (b) =
0
nx dx db = nx
dx + 0 – 0 (using modified Leibnitz Theorem)
0
1
1
x b 1 1
=
0
x b dx =
b 1 = b 1 (b) =
0
n (b + 1) + c
b=0 (0) = 0
c=0 (b) = n (b+1)
1
tan 1(ax )
Example # 23 Evaluate
0 x 1 x2
dx , ‘a’ being parameter..
1
tan 1(ax )
Solution Let (a) =
0 x 1 x2
dx
1 1
d (a) x 1 dx
da
=
0
2 2
(1 a x ) x 1 x 2
dx = 0 (1 a 2 x 2 ) 1 x 2
Put x = sin t dx = cos t dt
L.L. : x = 0 t=0
U.L. : x = 1 t=
2
2 2
d (a) 1 1 dt
da
= 1 a
0
2
sin t2cos t
cos t dt = 1 a
0
2
sin 2 t
2
sec 2 t dt 1 1 a 2 tan t 2 = 1
= 1 (1 a
0
2
) tan t2
=
1 a 2
tan –1
0 1 a 2
.
2
2
(a) = n a 1 a + c
2
But (0) = 0 c=0
2
(a) = n a 1 a
2
x
t
(27) If f(x) = e g(x)
and g(x) = 1 t
2
4 dt, then find the value of f(2).
y
dt d2 y
(28) If x =
0 1 4t 2
and
dx 2
= Ry, then find R
DI - 198
Definite Integration & its Application
x2
x
2
(29) If f(x) = sin t dt , then find f(x).
x
x
(30) If (x) = cos x – ( x t ) ( t) dt, then find the value of (x) + (x).
0
(n t )
2
(31) Find the value of the function f(x) = 1 + x + 2n t dt, where f(x) vanishes.
1
x2
cos t
2
dt
Evaluate xLt n (1 b cos x) dx, ‘b’ being parameter..
0
(32) 0
. (33) Evaluate
x sin x
0
2
Answers : (26) 3x2 cos x 3 (27) (28) 4
17
(29) x2 (2x sin x2 – sin x) + (cos x – cos x2) 2x (30)– cos x
1 1 b2
2
(31) 1+ (32) 1 (33) n 2
e
Definite Integral as a limit of sum
Let f(x) be a continuous real valued function defined on the closed interval [a, b] which is divided into n parts
as shown in figure.
ba
The point of division on x-axis are a, a + h, a + 2h ..........a + (n – 1)h, a + nh, where = h.
n
Let Sn denotes the area of these n rectangles.
Then, Sn = hf(a) + hf(a + h) + hf(a + 2h) + ........+hf(a + (n – 1)h)
Clearly, Sn is area very close to the area of the region bounded by curve y = f(x), x–axis and the ordinates
x = a, x = b.
b
Hence
f ( x) dx =
a
n
Lt Sn
n 1 n 1
b ba (b a ) r
f ( x ) dx = Lt
n
h f (a rh) = Lt
n
r0
f a
n n
a r 0
Note :
1. We can also write
n
b
ba ba
Sn = hf(a + h) + hf (a + 2h) + .........+ hf(a + nh) and f ( x ) dx = Lt
n r 1
f a
n
r
n
a
n 1
1 1 r
2. If a = 0, b = 1, f ( x ) dx = n Lt
n f n
r 0
0
DI - 199
Definite Integration & its Application
Steps to express the limit of sum as definte integral :
r 1
Step 1. Replace by x, by dx and n Lt
by
n n
r
Step 2. Evaluate n Lt
n by putting least and greatest values of r as lower and upper limits respectively..
pn p
1 r r r
For example n Lt
r 1
f
n n = f (x) dx ( n Lt
n
r 1
= 0, n Lt
n
r np
= p)
0
Lt 1 1 1 1
Example # 24 : Evaluate n 1 n 2 n 3 n ......... 2n
n
1
Solution Lt
n
1 1 1 1
1 n 2 n 3 n ......... 2n = n Lt
r n
r 1
n 1
1 1 dx
= n Lt
n r = x 1 = n ( x 1)0 = n 2
1
r 1 1 0
n
n 1 n2 n3 3
Example # 25 : Evaluate n Lt
2 2 .........
5n
2 2 2 2 .
n 1 n 2 n 3
r
2n 2n 1
nr 1
Solution Lt
n
n
r 1
2
Lt
r2 = n
r 1
n
n
r
2
1
n
Lt r
= 0, when r = 1, lower limit = 0
n n
Lt r Lt 2n
and = = 2, when r = 2n, upper limit = 2
n n n n
2 2 2 2
1 x 1 1 2x 1 2 1
1 x
0
2 dx = 1 x
0
2 dx +
2 1 x
0
2 dx = [ tan–1x ]20 + 2 log e (1 x ) = tan–1 2 +
0 2
n 5
1
Lt n! n
Example # 26 : Evaluate n n .
n
1
Let y = n Lt
n! n Lt 1 n n! = Lt 1 n 1 . 2 . 3 ........n
Solution n n y = n
n n nn n
n nn
1
= – 1 – 0 = –1 y=
e
DI - 200
Definite Integration & its Application
Self Practice Problems :
Evaluate the following limits
1 1 1 1
(34) Lt ....
n n 2 2
n n 2
n 2n n n2
2
Lt 1 1 1 1
(35) n 1 n 2 n 3 n ....... 5n
1 3 2 3 n
Lt sin 2 sin 3 3 sin 3 ........ n sin3
(36) n 2 4n 4n 4n 4n
n
n 1
1
(37) Lt
n
r 0 n r2
2
3 n n n n
(38) Lt 1 ......
n
n n3 n6 n9 n 3(n 1)
(37) (38) 2
2
2
sin
n
Proof : n = x dx
0
2
n = sinn1 x cos x + (n 1) sin
2
0
n 2
x . cos 2 x dx
0
2
sin
n2
= (n – 1) x . (1 sin 2 x ) dx
0
2 2
sin sin
n2 n
= (n – 1) x dx (n 1) x dx
0 0
n + (n – 1) n = (n – 1) n–2
n 1
n =
n n–2
DI - 201
Definite Integration & its Application
2 2
sin cos
n n
Note : 1. x dx = x dx
0 0
n 1 n 3 n 5
2. n = ..... or according as n is even or odd. = , = 1
n n 2 n 4 0 1 0
2 1
n 1 n 3 n 5 1
........ . , if n is even
n n 2 n 4 2 2
Hence n =
n 1 n 3 n 5 2
........ . 1 , if n is odd
n n 2 n 4 3
4
1
tan
n
2. If n = x dx , then show that n + n–2 =
n 1
0
4 4
(tan x) (tan x)
n2 n2
Solution n = . tan2x dx = (sec2x – 1) dx
0 0
4 4 (tan x )n 1 4
(tan x) (tan x)
n2 n2
= sec2x dx – dx = – n–2
0 0 n 1 0
1 1
n = – n–2 n + n–2 =
n 1 n 1
2
m 1
sin
m
3. If m,n = x . cosn x dx , then show that m,n =
m n m–2, n
0
2 sinm 1 x . cosn 1 x 2 2
cos n 1 x
Solution m,n =
0
sinm 1 x (sin x cosn x ) dx =
n 1
+
0
0
n 1
(m – 1) sinm–2 x cos x dx
sin
2 2
m 1 m 1
=
sin x . cos n x . cos 2 x dx =
m2 m2
x . cos n x sinm x . cos n x dx
n 1 n 1
0 0
m 1 m 1 m 1 m 1
= – 1 =
n 1 m–2,n
n 1 m,n
n 1 m,n
n 1 m–2,n
m 1
m,n =
m n m–2,n
m 1 m 3 m 5
Note : 1. m,n = ........ or according as m is even or odd.
mn mn2 mn 4 0,n 1,n
2 2
1
cos sin x . cos
n n
0,n = x dx and 1,n = x dx =
n 1
0 0
DI - 202
Definite Integration & its Application
2. Walli’s Formula
(m 1) (m 3) (m 5 ) .........( n 1) (n 3) (n 5).......
when both m, n are even
(m n) (m n 2) (m n 4)........ 2
m,n =
(m 1) (m 3 ) (m 5) .........( n 1) (n 3) (n 5).......
otherwise
(m n) (m n 2) (m n 4)........
2
sin
2
Example # 27 : Evaluate x cos 2 x(sin x cos x ) dx .
2
2 2
sin sin
2
Solution Given integral =
3
x cos 2 x dx + x cos 3 x dx
2 2
2
sin
2
=0+2 x cos 3 x dx ( sin3x cos2x is odd and sin2x cos3x is even)
0
1. 2 4
= 2. 5 . 3 . 1 =
15
x sin
5
Example # 28 : Evaluate x cos 6 x dx .
0
x sin
5
Solution Let = x cos 6 x dx
0
sin
5
2 = . 2 x . cos 6 x dx
0
4 . 2 . 5 . 3 .1 8
= ; =
11 . 9 . 7 . 5 . 3 . 1 693
1
x
3
Example # 29 : Evaluate (1 x )5 dx .
0
Solution Put x = sin2 dx = 2 sin cos d
L.L :x=0 =0
U.L. :x=1 =
2
1 2 2
x sin sin
3
(1 x )5 dx = 6
(cos 2 )5 2 . sin . cos d = 2 . 7
cos11 d
0 0 0
6 . 4 . 2 . 10 . 8 . 6 . 4 . 2 1
= 2 . 18 . 16 . 14 . 12 . 10 . 8 . 6 . 4 . 2 =
504
DI - 203
Definite Integration & its Application
Self Practice Problems:
x sin 1 x dx .
6
sin sin
5 5 4
(39) x dx . (40) x cos x dx . (41)
0 0 0
2
a
x a
7
x
3/2
(42) 2
x2 2 dx . (43) 2 x dx.
0
0
8 8 16 a9
Answers : (39) (40) (41) – (42) (43)
15 315 14 245 9 2
Area included between the curve y = f(x), x-axis and the ordinates x = a, x = b
b
(a) If f(x) 0 for x [a, b], then area bounded by curve y = f(x), x-axis, x = a and x = b is f ( x) dx
a
Example # 1 : Find the area enclosed between the curve y = x 2 + 2, x-axis, x = 1 and x = 2.
Solution
Graph of y = x2 + 2
2 2
x3
Area =
2
x 2 dx 2x = 13
1
3 1 3
Example # 2 : Find area bounded by the curve y = n x + tan–1 x and x-axis between ordinates x = 1 and x = 2.
Solution y = n x + tan–1x
dy 1 1
Domain x > 0, = + >0
dx x 1 x2
y is increasing and x = 1, y = y is positive in [1, 2]
4
2
(n x tan
1
Required area = x ) dx
1
DI - 204
Definite Integration & its Application
2
1 1 2
= x n x x x tan x n (1 x )
2 1
1 1
= 2 n 2 – 2 + 2 tan–12 – n 5 – 0 + 1 – tan–1 1 + n 2
2 2
5 1
= n 2 – n 5 + 2 tan–12 – –1
2 2 4
Example # 3 The area cut off from a parabola by any double ordinate is k times the corresponding rectangle
contained by the double ordinate and its distance from the vertex. Find the value of k ?
Solution Consider y2 = 4ax, a > 0 and x=c
c
8
Area by double ordinate = 2 2 a x dx 0
3
a c3 / 2
2 2
Area = –
1
log 1 x dx = –
2
log e x . log 1 e dx
2
1
= – log 1 e . [ x loge x x ]12
2
Note : If y = f(x) does not change sign in [a, b], then area bounded by y = f(x), x-axis between
b
ordinates x = a, x = b is f ( x) dx
a
(c) If f(x) > 0 for x [a,c] and f(x) < 0 for x [c,b] (a < c < b) then area bounded by curve y = f(x) and x-axis
c b
between x = a and x = b is
a
f ( x ) dx f ( x ) dx
c
DI - 205
Definite Integration & its Application
Example # 5 : Find the area bounded by y = x3 and x- axis between ordinates x = –1 and x = 1
0 1
1 1 1
= 0 – + –0=
4 4 2
Note : Most general formula for area bounded by curve y = f(x) and x- axis between ordinates x = a and x = b is
b
| f ( x) | dx
a
Area included between the curve x = g(y), y-axis and the abscissas y = c, y = d
(a) If g (y) 0 for y [c,d] then area bounded by curve x = g(y) and y–axis between abscissa y = c and
d
y = d is
g(y)dy
y c
Example # 6 : Find area bounded between y = sin–1x and y–axis between y = 0 and y = .
2
Solution y = sin–1 x x = sin y
2
Required area = sin y dy
0
= cos y 2 = – (0 – 1) = 1
0
Note : The area in above example can also evaluated by integration with respect to x.
Area = (area of rectangle formed by x = 0, y = 0 , x = 1, y = ) – (area bounded by y = sin–1x,
2
x–axis between x = 0 and x = 1)
1
x sin 1 x 1 x 2
1
=
2
×1–
0
sin 1 x dx =
–
= – 0 0 1 = 1
0 2 2
Example # 7 : Find the area bounded by the parabola x2 = y, y-axis and the line y = 1.
Solution Graph of y = x2
1 1
2
Area OEBO = Area OAEO = | x | dy =
0 0
y dy =
3
DI - 206
Definite Integration & its Application
Example # 8 : Find the area bounded by the parabola x2 = y and line y = 1.
Solution Graph of y = x2
1 t 1 t
Example # 9 : For any real t, x = (e + e–t), y = (e – e–t) is point on the hyperbola x2 – y2 =1.Show that the area
2 2
bounded by the hyperbola and the lines joining its centre to the points corresponding to t1 and – t1 is t1.
Solution It is a point on hyperbola x2 – y2 = 1.
et1 e– t1 et1 e – t1
2 2
Area (PQRP) = 2
1
ydx = 2 1
x 2 – 1 dx
et1 e – t1
x 1 2 e2t1 – e –2t1
= 2 x 2 – 1 – n( x x 2 – 1) = – t1
2 2 1 4
(b) If g (y) 0 for y [c,d] then area bounded by curve x = g(y) and y–axis between abscissa y = c and
d
y = d is – g( y )dy
y c
Note : General formula for area bounded by curve x = g(y) and y–axis between abscissa y = c and
d
y = d is
y c
| g( y ) | dy
Curve-tracing :
To find approximate shape of a curve, the following phrases are suggested :
(a) Symmetry:
(i) Symmetry about x-axis :
If all the powers of 'y' in the equation are even then the curve (graph) is symmetrical about the x-axis.
E.g. : y2 = 4 a x.
DI - 207
Definite Integration & its Application
(ii) Symmetry about y-axis :
If all the powers of 'x' in the equation are even then the curve (graph) is symmetrical about the y-axis.
E.g. : x2 = 4 a y.
(iii) Symmetry about both axis :
If all the powers of 'x' and 'y' in the equation are even, then the curve (graph) is symmetrical about the
axis of 'x' as well as 'y' .
E.g. : x2 + y2 = a2.
(iv) Symmetry about the line y = x :
If the equation of the curve remain unchanged on interchanging 'x' and 'y', then the curve (graph) is
symmetrical about the line y = x.
E.g. : x3 + y3 = 3 a x y.
(v) Symmetry in opposite quadrants :
If the equation of the curve (graph) remain unaltered when 'x' and 'y' are replaced by '–x' and '–y'
respectively, then there is symmetry in opposite quadrants.
E.g. : xy = c2
(b) Find the points where the curve crosses the x-axis and the y-axis.
dy
(c) Find dx and equate it to zero to find the points on the curve where you have horizontal tangents.
(f) Asymptotes :
Asymptote(s) is (are) line (s) whose distance from the curve tends to zero as point on curve moves towards
infinity along branch of curve.
(i) If Lim f(x) = or Lt f(x) = – , then x = a is asymptote of y = f(x)
x a xa
DI - 208
Definite Integration & its Application
f (x)
(iii) If Lim
x
= m1, x Lt
(f(x) – m 1x) = c , then y = m 1x + c1 is an asymptote (inclined to right).
x
f (x)
(iv) If xLim
x
= m2, xLim
(f(x) – m2x) = c2, then y = m2x + c2 is an asymptote (inclined to left).
y = 0 is asymptote.
1
Example # 12 : Find asymptotes of y = x + and sketch the curve (graph).
x
1
Solution : Lim y = Lim x = + or –
x 0 x 0 x
x = 0 is asymptote.
1
Lim y = Lim x =
x 0 x 0 x
there is no asymptote of the type y = k
1
Lim y = Lim 1 =1
x x x x2
1
Lim (y – x) = Lim x x = Lim 1 = 0
x x x x x
y = x + 0 y = x is asymptote.
A rough sketch is as follows
x = b is f ( x ) g( x)dx .
a
DI - 209
Definite Integration & its Application
Example # 13 : Find the area enclosed by curve (graph) y = x2 + x + 1 and its tangent at (1,3) between ordinates
x = – 1 and x = 1.
dy
Solution = 2x + 1
dx
dy
= 3 at x = 1
dx
Equation of tangent is
y – 3 = 3 (x – 1)
y = 3x
1
(x
2
Required area = x 1 3 x ) dx
1
1 1
x3
2 2
= ( x 2x 1) dx 3 x x
1
1
1 1 2 8
= 1 1 – 1 1 = +2=
3 3 3 3
b
Note : Area bounded by curves y = f(x) and y = g (x) between ordinates x = a and x = b is | f ( x) g( x ) | dx .
a
Example # 14 : Find the area of the region bounded by y = sin x, y = cos x and ordinates x = 0, x = /2
/2
/4 /2
0
(cos x sin x ) dx +
/4
(sin x cos x ) dx = 2 ( 2 1)
6 x 36 x 2 20(2 x 2 1)
y=
10
3x 5 x 2
y=
5
y is real R.H.S. is also real.
– 5 <x< 5
If x=– 5 , y=3 5
If x= 5, y = –3 5
1
If x = 0, y=+
5
1
If y = 0, x=+
2
5 3x 5 x 2 3 x 5 x 2
dx
Required area = 5
5
5
DI - 210
Definite Integration & its Application
5 5
=
2
5
5 x 2 dx = 4
5 5 x 2 dx
5 0
Miscellaneous examples
Example # 16 : Find the area contained between the two arms of curves (y – x)2 = x3 between x = 0 and x = 1.
For arm
dy 3 1/2
y = x + x3/2 =1+ x >0 x > 0.
dx 2
y is increasing function.
For arm
dy 3 1/2
y = x – x3/2 =1– x
dx 2
1
dy 4 d2 y 3 2 4
=0 x= , x < 0 at x =
dx 9 dx 2 4 9
4
at x = , y = x – x3/2 has maxima.
9
1
(x x
3/2
Required area = x x 3 / 2 ) dx
0
1 1
2 x5 / 2 4
x
3/2
=2 dx =
5 / 2 5
0 0
Example # 17 : Let A (m) be area bounded by parabola y = x2 + 2x – 3 and the line y = mx + 1. Find the least area
A(m).
Solution Solving we obtain
x2 + (2 – m) x – 4 = 0
Let be roots = m – 2, = – 4
(mx 1 x (x
2 2
A (m) = 2x 3) dx = (m 2) x 4) dx
x3 x2 3 3 m 2 2
(m 2 ) 4 x ( 2 ) 4 ( )
= 3 2 = 3 2
1 2 (m 2)
= | – |. ( 2 ) ( ) 4
3 2
=
1 2
(m 2)
(m 2)2 16 3 (m 2) 4 2 (m 2) 4 = (m 2)2 16
1
6
(m 2 ) 2
8
3
DI - 211
Definite Integration & its Application
1 3/2
A(m) = 2
6 (m 2) 16
1 32
Least A(m) = (16)3/2 = .
6 3
Example # 18: A curve y = f(x) passes through the origin and lies entirely in the first quadrant. Through any point
P(x, y) on the curve, lines are drawn parallel to the coordinate axes. If the curve divides the area
formed by these lines and coordinate axes in m : n, then show that f(x) = cxm/n or f(x) = cxn/m (c-being
arbitrary).
Solution Area (OAPB) = xy
x
Area (OPBO) = xy – f ( t ) dt
0
Area (OAPO) m
Area (OPBO) n
x x
n f ( t ) dt m xy – f ( t ) dt
0 0
x x
n f ( t ) dt mx f ( x ) – m f ( t ) dt
0 0
Differentiating w.r.t. x
nf(x) = m f(x) + mx f(x) – m f(x)
f ( x ) n 1
f (x) m x
f(x) = cxn/m
similarly f(x) = cxm/n
Self practice problems :
(1) Find the area between curve y = x2 – 3x + 2 and x–axis
(i) bounded between x = 1 and x = 2.
(ii) bound between x = 0 and x = 2.
(2) Find the area included between curves y = 2x – x2 and y + 3 = 0.
(3) Find area between curves y = x2 and y = 3x – 2 from x = 0 to x = 2.
(4) What is geometrical significance of
3
2
(i) cos x dx
0
, (ii) cos x dx
0
(5) Find the area of the region bounded by the x-axis and the curves defined by y = tan x,
– 2
where x and y = cot x where x .
3 3 6 3
(6) Curves y = sinx and y = cosx intersect at infinite number of points forming regions of equal area
between them calculate area of one such region.
(7) Find the area of the region bounded by the parabola (y – 2)2 = (x – 1) and the tangent to it at ordinate
y = 3 and x–axis.
(8) Find the area included between y = tan–1x, y = cot–1x and y–axis.
(9) Find area common to circle x2 + y2 = 2 and the parabola y2 = x.
DI - 212
Definite Integration & its Application
4 x2
(10) Find the area included between curves y = and 5y = 3|x| – 6.
4 x2
1
(11) Find the area bounded by the curve |y| + = e–|x|.
2
(13) Find the area enclosed by |x| + |y| < 3 and xy > 2.
(14) Find are bounded by x2 + y2 < 2ax and y2 > ax, x > 0.
1 32
Answers : 1. (i) (ii) 1 2. 3. 1
6 3
3
5. n 6. 2 2
2
3 2 8
7. 9 8. n2 9. 10. 2 –
3 2 3 5
8 3 8 2
11. 2 (1–n2) 12. 13. 3–4n2 14. a
15 6
DI - 213
Differential Equation
Introduction :
An equation involving independent and dependent variables and the derivatives of the dependent vari-
ables is called a differential equation. There are two kinds of differential equation:
1.1 Ordinary Differential Equation : If the dependent variables depend on one independent vari-
able x, then the differential equation is said to be ordinary.
dy dz
for example + = y + z,
dx dx
dy d3 y dy
+ xy = sin x , 3
2 + y = ex ,
dx dx dx
3/2
dy 2 dy 2
d2 y dy
k = 1 , y = x + k 1 dx
dx 2 dx dx
1.2 Partial differential equation : If the dependent variables depend on two or more indepen-
dent variables, then it is known as partial differential equation
z 2z 2z 2z
for example y2 y = ax, 0
x y x2 y2
Order and Degree of a Differential Equation:
2.1 Order : Order is the highest differential appearing in a differential equation.
2.2 Degree :
It is determined by the highest degree of the highest order derivative present in it after the
differential equation is cleared of radicals and fractions so far as the derivatives are concerned.
Note : In the differential equation, all the derivatives should be expressed in the polynomial form
n n
dm y 1 dm1y 2 n
dy k
f 1 (x, y) m + f 2 (x, y) m1 + ........ f k(x, y) = 0
dx dx dx
The above differential equation has the order m and degree n1.
Example # 1 Find the order & degree of following differential equations.
1/ 4 dy d2 y
d2 y dy
6
dx dx 2
(i) = y (ii) y= e
dx 2 dx
dy d2 y
(iii) sin dx 2 = y (iv) ey – xy + y = 0
dx
4 6
d2 y
Solution : (i) = y dy order = 2, degree = 4
dx 2 dx
d2 y dy
(ii) 2 + = ny order = 2, degree = 1
dx dx
d2 y dy
(iii) + = sin–1 y order = 2, degree = 1
dx 2 dx
DE - 214
Differential Equation
d3 y d2 y
(iv) 3 –x +y=0
e dx dx 2
equation can not be expressed as a polynomial in differential coefficients, so degree is
not applicable but order is 3.
dy 1 dy d3 y d5 y
= n 5 1
(i) +y= (ii) dx dx 3
dx dy e dx
dx
2
dy 1 / 2 d2 y
(iii) y =
dx dx 2
Answer (1) (i) order = 1, degree = 2 (ii) order = 5, degree = not applicable.
(iii) order = 2, degree = 2
Example # 2 Form a differential equation of family of straight lines passing through origin.
Solution : Family of straight lines passing through origin is y = mx where’m’ is a parameter.
Differentiating w.r.t. x
dy
=m
dx
Eliminating ‘m’ from both equations, we obtain
dy y
= which is the required differential equation.
dx x
Example # 3 Form a differential equation of family of circles touching x-axis at the origin
Solution : Equation of family of circles touching x-axis at the origin is
x 2 + y2 + y = 0 ..........(i) where is a parameter
dy dy
2x + 2y + =0 .........(ii)
dx dx
Eliminating ‘’ from (i) and (ii)
dy 2xy
= 2
dx x y2
which is required differential equation.
DE - 215
Differential Equation
Self Practice Problems :
(2) Obtain a differential equation of the family of curves y = a sin (bx + c) where a and c being
arbitrary constant.
(3) Show that the differential equation of the system of parabolas y2 = 4a(x – b) is given by
2
d2 y dy
y 2 +
=0
dx dx
(4) Form a differential equation of family of parabolas with focus as origin and axis of symmetry
along the x-axis.
2
d2 y dy dy
Answer (2) + b 2y = 0 (4) y2 = y2 + 2xy
dx 2 dx dx
1 x y 1 1 1
x
dx =
y
dy x 1 dx = 1 y dy
n x + x = y – ny + c ny + nx = y – x + c
xy = cey–x
dy
Example # 5 Solve : = (ex + 1) (1 + y2)
dx
dy
Solution. The equation can be written as ( e x 1)dx
1 y2
Integrating both sides,
tan–1 y = ex + x + c.
DE - 216
Differential Equation
2 dy
dy
Example # 6 Solve : y – x y =a
dx dx
Solution. The equation can be written as
dy
y – ay2 = (x + a)
dx
dx dy
x a y ay 2
dx 1
x a y(1 ay ) dy
dx 1 a
x a y 1 ay dy
Integrating both sides,
n (x + a) = n y – n (1 – ay) + n c
cy
n (x + a) = n 1 ay
cy = (x + a) (1 – ay)
where 'c' is an arbitrary constant.
5.1.1 Polar coordinates transformations :
Sometimes transformation to the polar co-ordinates facilitates separation of variables. In this
connection it is convenient to remember the following differentials:
(a) If x = r cos ; y = r sin then,
(i) x dx + y dy = r dr (ii) dx 2 + dy2 = dr 2 + r 2d2 (iii) x dy – y dx = r2d
(b) If x = r sec & y = r tan then
(i) x dx – y dy = r dr (ii) x dy – y dx = r2 sec d.
Example # 7 Solve the differential equation xdx + ydy = x (xdy – ydx)
Solution. Taking x = r cos, y = r sin
x2 + y2 = r2
2x dx + 2ydy = 2rdr
xdx + ydy = rdr .........(i)
y
= tan
x
dy
x y
dx d
2 = sec2.
x dx
xdy – y dx = x sec . d
2 2
y 1
= c where – = c
x2 y2
(y + 1)2 = c(x2 + y2)
DE - 217
Differential Equation
5.1.2 Equations Reducible to the Variables Separable form : If a diff erential
equation can be reduced into a variables separable form by a proper substitution, then it is said
to be
dy
“Reducible to the variables separable type”. Its general form is = f(ax + by + c) a, b 0.
dx
To solve this, put ax + by + c = t.
dy
Example # 8 Solve = (4x + y + 1)2
dx
Solution. Putting 4x + y + 1 = t
dy dt
4+ =
dx dx
dy dt
= –4
dx dx
Given equation becomes
dt
– 4 = t2
dx
dt
2 = dx (Variables are separated)
t 4
Integrating both sides,
dt
4t 2 = dx
1 t 1 4 x y 1
tan–1 =x+c tan–1 =x+c
2 2 2 2
dy
Example # 9 Solve sin–1 =x+y
dx
dy
Solution. = sin (x + y)
dx
putting x + y = t
dy dt
= –1
dx dx
dt dt dt
– 1 = sin t = 1 + sin t 1 sin t = dx
dx dx
Integrating both sides,
dt
1 sin t = dx
1 sin t
cos 2
t
dt = x + c
(sec
2
t sec t tan t ) dt = x + c
tan t – sec t = x + c
1 sin t
– =x+c sin t – 1 = x cos t + c cos t
cos t
substituting the value of t
sin (x + y) = x cos (x + y) + c cos (x + y) + 1
DE - 218
Differential Equation
Self Practice Problems :
dy
(5) Solve the differential equation x2 y = (x + 1) (y + 1)
dx
xdx ydy ydx xdy
(6) Solve the differential equation =
2
x y 2 x2
dy
(7) Solve : = ex + y + x2ey
dx
dy
(8) Solve : xy = 1 + x + y + xy
dx
dy
(9) Solve = 1 + ex – y
dx
dy
(10) = sin(x + y) + cos (x + y)
dx
dy
(11) = x tan (y – x) + 1
dx
1 y
Answer(5) y – n (y + 1) = nx – +c (6) x2 y2 + =c
x x
1 x3
(7) – = ex + +c (8) y = x + n |x (1 + y)| + c
ey 3
xy x2
c
(9) e y–x
=x+c (10) log tan 1 = x + c (11) sin (y – x) = e2
2
dv
2v + (v2 – 1) v x =0
dx
dv 2v
v+x =– 2
dx v 1
dv v(1 v 2 )
x =
dx v2 1
v2 1 dx
v(1 v 2
)
dv = – x
2v 1
1 v 2
dv = – n x + c
v
DE - 219
Differential Equation
n (1 + v2) – n v = – n x + c
1 v 2
n .x =c
v
x2 y2
n y =c
x2 + y2 = yc' where c = ec
dy x2 y2
Solution. =–
dx 2xy
y = vx
dy dv
=v+
dx dx
dv 1 v 2
v+x =–
dx 2v
2v dx
1 v 2 dv = –
x
n (1 + v ) = – nx + c
2
at x = 1, y = 1 v=1
n 2 = c
2
1 y . x
n = n2
x 2
x2 + y2 = 2x
DE - 220
Differential Equation
dy dY
on dividing (i) by (ii) =
dx dX
dY X h 2( Y k ) 5
=
dX 2 X 2h Y k 4
X 2Y (h 2k 5)
=
2 X Y (2h k 4)
h & k are such that h + 2k – 5 = 0 & 2h + k – 4 = 0
h = 1, k = 2
dY X 2Y
= which is homogeneous differential equation.
dX 2X Y
Now, substituting Y = vX
dY dv
=v+X
dX dX
dv 1 2v
X = –v
dX 2v
2v dX
1 v 2 dv = X
1 3
2( v 1) 2(1 v ) dv = n X + c
1 3
n (v + 1) – n (1 – v) = n X + c
2 2
v 1
n = nX2 + 2c
(1 v )3
( Y X) X2
= e2c
( X Y )3 X 2
X + Y = c(X – Y)3 where e2c = c
x – 1 + y – 2 = c (x – 1– y + 2)3
x + y – 3 = c(x – y + 1)3
Special case :
a b
Case - 1 In equation (1) if , then the substitution ax + by = v will reduce it to the form in
A B
which variables are separable.
dy 2x 3 y 1
Example # 13 Solve = 4 x 6y 5
dx
Solution. Putting u = 2x + 3y
du dy
=2+3.
dx dx
1 du u 1
2 =
3 dx 2u 5
du 3u 3 4u 10
=
dx 2u 5
2u 5
7u 13 dx = dx
DE - 221
Differential Equation
2 9 1
7 1.du – 7 7u 13 . du = x + c
2 9 1
u– . n (7u – 13) = x + c
7 7 7
9
4x + 6y – n (14x + 21y – 13) = 7x + 7c
7
9
– 3x + 6y – n (14x + 21y – 13) = c
7
Case - 2 In equation (1), if b + A = 0, then by a simple cross multiplication equation (1) becomes
an exact differential equation.
dy x 2y 5
Example # 14 Solve = 2x y 1
dx
Solution. Cross multiplying,
2xdy + y dy – dy = xdx – 2ydx + 5dx
2 (xdy + y dx) + ydy – dy = xdx + 5 dx
2 d(xy) + y dy – dy = xdx + 5dx
On integrating,
y2 x2
2xy + –y= + 5x + c
2 2
x2 – 4xy – y2 + 10x + 2y = c where c = – 2c
(C) If the homogeneous equation is of the form :
yf(xy) dx + xg(xy)dy = 0, the variables can be separated by the substitution xy = v.
dy x 2y 3 dy x y 1
(14) = 2x y 3 (15) = 2x 2 y 3
dx dx
dy 3 x 2y 5
(16) = 3 y 2x 5
dx
y 1 y y
Answers (12) x 2 y 2 = e x tan x (13) x sin =C
x
DE - 222
Differential Equation
M N
NOTE : (i) The necessary condition for (1) to be exact is y x .
(ii) For finding the solution of exact differential equation, following results on exact differentials
should be remembered :
xdy ydx y
(a) xdy + y dx = d(xy) (b) = d (c) 2(x dx + y dy) = d (x 2 + y2)
x 2
x
xdy ydx 1
(g) = d xy
x2 y2
xdy ydx
Example # 15 Solve : y dx + x dy =
x2 y2
xdy ydx
Solution. ydx + xdy =
x2 y2
d (xy) = d (tan–1 y/x)
Integrating both sides
xy = tan–1 y/x + c
x2
2
Example # 16 Solve : (2x ny) dx + y 3y dy 0
Solution. The given equation can be written as
dy
ny (2x) dx + x2 y + 3y2 dy = 0 ny d (x2) + x2 d (ny) + d (y3) = 0
d (x2 ny) + d (y3) = 0
Now integrating each term, we get
x2ny + y3 = c
dm y dm1y dy
P0(x) + P1(x) m 1 + .................... + Pm–1 (x) dx + Pm (x) y = (x),
dx m dx
where P0(x), P1(x) ..................Pm(x) are called the coefficients of the differential equation.
dy
NOTE : + y2 sinx = lnx is not a Linear differential equation.
dx
DE - 223
Differential Equation
6.1 Linear differential equations of first order :
dy
The differential equation + Py = Q , is linear in y. (Where P and Q are functions of x only).
dx
Integrating Factor (I.F.) : It is an expression which when multiplied to a differential equation converts
it into an exact form.
Pdx
I.F for linear differential equation = e (constant of integration will not be considered)
after multiplying above equation by .F it becomes;
dy Pdx Pdx
. e Pdx + Py . e = Q. e
dx
d
dx
( y. e Pdx ) = Q. e Pdx y. e Pdx = Q. e
Pdx
C .
NOTE : Some times differential equation becomes linear, if x is taken as the dependent variable, and y as
dx
independent variable. The differential equation has then the following form : dy + P1 x = Q 1.
dy 3x 2 sin2 x
Example # 17 Solve + y =
dx 1 x3 1 x3
dy
Solution. + Py = Q
dx
3x 2
P=
1 x3
3x2
F = e P.dx = 1 x 3 dx = e n(1 x
3
) = 1 + x3
e
General solution is
y(F) = Q(IF).dx c
sin 2 x
y (1 + x3) = 1 x 3 (1 + x3) dx + c
1 cos 2 x
y(1 + x3) = 2
dx + c
1 sin 2x
y(1 + x3) = x– +c
2 4
dy
Example # 18 Solve : x nx + y = 2 n x
dx
dy 1 2
Solution. + y=
dx xnx x
1 2
P= ,Q=
xnx x
1
IF = e P.dx = xnx dx = e n( nx ) = n x
e
General solution is
2
y. (n x) = x .nx.dx c
y (n x) = (n x)2 + c
DE - 224
Differential Equation
Example # 19 Solve the differential equation
t (1 + t2) dx = (x + xt2 – t2) dt and it given that x = –/4 at t = 1
Solution. t (1 + t2) dx = [x (1 + t2) – t2] dt
dx x t
= t
dt (1 t 2 )
dx x t dx
2 which is linear in
dt t 1 t dt
1
Here, P = –
1
, Q=–
t
IF = e
t dt = e–nt = 1
t 1 t 2 t
General solution is -
1 1 t
x·
t
= t . 1 t 2
dt + c
x
= – tan–1 t + c
t
putting x = – /4, t = 1
– /4 = – /4 + c c=0
x = – t tan–1 t
6.2 Equations reducible to linear form
6.2.1 By change of variable .
Often differential equation can be reduced to linear form by appropriate substitution of the
non-linear term
dy
Example # 20 Solve : y sinx = cos x (sinx – y2)
dx
Solution. The given differential equation can be reduced to linear form by change of variable by a suitable
subtitution.
Substituting y2 = z
dy dz
2y =
dx dx
differential equation becomes
sin x dz
+ cos x.z = sin x cos x
2 dx
dz dz
+ 2 cot x . z = 2 cos x which is linear in
dx dx
IF = e
2 cot x dx
e 2n sin x = sin2 x
General solution is -
2 cos x. sin
2
z. sin2 x = x. dx c
2
y2 sin2x = sin3 x + c
3
6.2.2 Bernoulli’s equation :
dy
Equations of the form + Py = Q.yn, n 0 and n 1
dx
where P and Q are functions of x, is called Bernoulli’s equation and can be made linear in v by
dividing by yn and putting y –n+1 = v. Now its solution can be obtained as in (v).
dy
e.g. : 2 sin x – y cos x = xy3 ex .
dx
DE - 225
Differential Equation
dy y y 2
Example # 21 Solve : (Bernoulli's equation)
dx x x 2
Solution. Dividing both sides by y2
1 dy 1 1
..... (1)
y 2 dx xy x 2
1
Putting y = t
1 dy dt
–
y 2 dx dx
differential equation (1) becomes,
dt t 1
– 2
dx x x
dt t 1 dt
2 which is linear differential equation in
dx x x dx
1
IF = e x
dx
= enx = x General solution is -
1
t. x = x 2
. x dx + c
tx = – nx + c
x
y = – nx + c
Self Practice Problems :
dy
(19) Solve : x (x2 + 1) = y (1 – x2) + x2 nx
dx
dy
(20) Solve : (x + 2y3) =y
dx
dy
(21) Solve : x + y = y2 log x
dx
x 2 1
Answers (19) x y = x n x – x + c (20) x = y (c + y2)
Clairaut’s Equation :
The differential equation
dy
y = mx + f(m), ..............(1), where m =
dx
is known as Clairaut’s Equation.
To solve (10), differentiate it w.r.t. x, which gives
dy dm d f (m) dm
=m+x +
dx dx dm dx
DE - 226
Differential Equation
dm d f (m) dm
x + =0
dx dm dx
dm
either =0m=c ...........(2)
dx
or x + f(m) = 0 ............(3)
NOTE : (i) If m is eliminated between (1) and (2), the solution obtained is a general solution of (1)
(ii) If m is eliminated between (1) and (3), then solution obtained does not contain any arbitrary
constant and is not particular solution of (1). This solution is called singular solution of (1).
dy
Example # 22 Solve : y = mx + m – m 3 where, m =
dx
Solution. y = mx + m – m3 ..... (i)
The given equation is in clairaut's form.
Now, differentiating wrt. x -
dy dm dm dm
mx 3m 2
dx dx dx dx
dm dm dm
m=m+x 3m 2
dx dx dx
dm
(x + 1 – 3m 2) = 0
dx
dm
=0 m=c ..... (ii)
dx
x 1
or x + 1 – 3m 2 = 0 m2 = ..... (iii)
3
Eliminating 'm' between (i) & (ii) is called the general solution of the given equation.
y = cx + c – c3 where, 'c' is an arbitrary constant.
Again, eliminating 'm' between (i) & (iii) is called singular solution of the given equation.
y = m (x + 1 – m2)
1/ 2 1/ 2
x 1 x 1 x 1 2
y=± x 1 y=± (x + 1)
3 3 3 3
3/2
x 1 4
y=± 2 y2 = (x + 1)3
3 27
27y2 = 4 (x + 1)3
DE - 227
Differential Equation
Steps to find orthogonal trajectory :
(i) Let f (x, y, c) = 0 be the equation of the given family of curves, where 'c' is an arbitrary constant.
(ii) Differentiate the given equation w.r.t. x and then eliminate c.
dy dx
(iii) Replace by – in the equation obtained in (ii).
dx dy
(iv) Solve the differential equation obtained in (iii).
Hence solution obtained in (iv) is the required orthogonal trajectory.
Example # 23 Find the orthogonal trajectory of family of straight lines passing through the origin.
Solution. Family of straight lines passing through the origin is -
y = mx ..... (i)
where 'm' is an arbitrary constant. Differentiating wrt x
dy
m ..... (ii)
dx
Eliminate 'm' from (i) & (ii)
dy
y= x
dx
dy dx
Replacing by – dy ,
dx
dx
we get y = dy x
DE - 228
Differential Equation
Geometrical application of differential equation :
Form a differential equation from a given geometrical problem. Often following formulae are useful to
remember
y 1 m2 2
(i) Length of tangent (LT) = m (ii) Length of normal (LN) = y 1 m
y
(iii) Length of sub-tangent (LST) = (ii) Length of subnormal (LSN) = |my|
m
dy
where y is the ordinate of the point, m is the slope of the tangent
dx
Example # 25 Find the nature of the curve for which the length of the normal at a point 'P' is equal to the radius
vector of the point 'P'.
Solution. Let the equation of the curve be y = f(x). P(x, y) be any point on the curve.
dy
Slope of the tanget at P(x, y) is =m
dx
Slope of the normal at P is
P(x,y)
1
m = –
m
Equation of the normal at 'P'
1
Y–y=– (X – x)
m O G(x+my,0)
Co-ordinates of G (x + my, 0)
Now, OP2 = PG2
x2 + y2 = m2y2 + y2
x dy x
m=± y =± y
dx
Taking as the sign
dy x y2 x2
= y y . dy = x . dx = + x2 – y2 = – 2
dx 2 2
x2 – y2 = c (Rectangular hyperbola)
Again taking as –ve sign
dy x y2 x2
=– y y dy = – x dx =– +
dx 2 2
x2 + y2 = 2
x2 + y2 = c (circle)
Example # 26 Find the curves for which the portion of the tangent included between the co-ordinate axes is
bisected at the point of contact.
Solution. Let P (x, y) be any point on the curve. B
Equation of tangent at P (x, y) is -
dy P(
x,
Y – y = m (X – x) where m = y)
dx
is slope of the tangent at P (x, y).
mx y
Co-ordinates of A , 0 & B (0, y–mx)
A
m
P is the middle point of A & B
mx y
= 2x mx – y = 2mx
m
DE - 229
Differential Equation
dy
mx = – y x=–y
dx
dx dy
+ y =0 nx + ny = nc
x
xy = c
Example # 27 Show that (4x + 3y + 1) dx + (3x + 2y + 1) dy = 0 represents a hyperbola having the lines x + y =
0 and 2x + y + 1 = 0 as asymptotes
Solution. (4x + 3y + 1) dx + (3x + 2y + 1) dy = 0
4xdx + 3 (y dx + x dy) + dx + 2y dy + dy = 0
Integrating each term,
2x2 + 3 xy + x + y2 + y + c = 0
2x2 + 3xy + y2 + x + y + c = 0
which is the equation of hyperbola when h2 > ab & 0.
Now, combined equation of its asymptotes is -
2x2 + 3xy + y2 + x + y + = 0
which is pair of straight lines
1 1 3 1 1 9
=0 2.1 + 2 . . . –2. –1. – =0
2 2 2 4 4 4
=0 2x2 + 3xy + y2 + x + y = 0
(x + y) (2x + y) + (x + y) = 0 (x + y) (2x + y + 1) = 0
x+y=0 or 2x + y + 1 = 0
Example # 28 The perpendicular from the origin to the tangent at any point on a curve is equal to the abscissa of
the point of contact. Find the equation of the curve satisfying the above condition and which passes
through (1, 1)
Solution. Let P (x, y) be any point on the curve
Equation of tangent at 'P' is -
Y – y = m (X – x)
mX – Y + y – mx = 0
y mx
Now 2 =x
1 m
y2 + m2x2 – 2mxy = x2 (1 + m2)
y 2 x 2 dy
which is homogeneous equation
2xy dx
Putting y = vx
dy dv dv v2 1
=v+x v+x =
dx dx dx 2v
dv v 2 1 2v 2 2v dx
x
dx
2v
v 2
1
dv = –
x
n (v2 + 1) = –n x + n c
y2
x 2 1 = c
x
Curve is passing through (1, 1)
c=2
x2 + y2 – 2x = 0
DE - 230
Complex Number
Complex Numbers
The shortest path between two truths in the real domain passes through the complex domain. ......Hadamard, Jacques
To permit solutions of such polynomial equations, the set of complex numbers is introduced. We can
consider a complex number as having the form a + bi where a and b are real number. It is denoted by
z i.e. z = a + ib. ‘a’ is called as real part of z which is denoted by (Re z) and ‘b’ is called as imaginary
part of z, which is denoted by (Im z).
Any complex number is :
(i) Purely real, if b = 0 ;
(ii) Imaginary, if b 0.
(iii) Purely imaginary, if a = 0
Note : (a) The set R of real numbers is a proper subset of the Complex Numbers. Hence the complete
number system is N W I Q R C.
(b) Zero is purely real as well as purely imaginary but not imaginary.
Also i² = 1; i3 = i ; i4 = 1 etc.
(d) a b = a b only if atleast one of a or b is non - negative.
(e) If z = a + ib, then a – ib is called complex conjugate of z and written as z = a – ib
(f) Real numbers satisfy order relations where as imaginary numbers do not satisfy order relations
i.e. i > 0, 3 + i < 2 are meaningless.
CN - 231
Complex Number
Algebraic Operations:
Fundamental operations with complex numbers
In performing operations with complex numbers we can proceed as in the algebra of real numbers,
replacing i2 by – 1 when it occurs.
1. Addition (a + bi) + (c + di) = a + bi + c + di = (a + c) + (b + d) i
2. Subtraction (a + bi) – (c + di) = a + bi – c – di = (a – c) + (b – d) i
3. Multiplication (a + bi) (c + di) = ac + adi + bci + bdi2 = (ac – bd) + (ad+ bc)i
ac bd (bc ad)i ac bd bc ad
= = 2 2 + i
2
c d 2
c d c 2 d2
Inequalities in imaginary numbers are not defined. There is no validity if we say that imaginary number
is positive or negative.
e.g. z > 0, 4 + 2i < 2 + 4 i are meaningless.
In real numbers if a2 + b2 = 0 then a = 0 = b however in complex numbers,
z12 + z22 = 0 does not imply z1 = z2 = 0.
Example # 2 : Find the value of x and y for which (2 + 3i) x 2 – (3 – 2i) y = 2x – 3y + 5i where x, y R.
Solution (2 + 3i)x 2 – (3 – 2i)y = 2x – 3y + 5i 2x 2 – 3y = 2x – 3y x2 – x = 0
2
x = 0, 1 and 3x + 2y = 5
5
if x = 0,y = and if x = 1, y = 1
2
5
x = 0, y = and x = 1, y = 1
2
5
are two solutions of the given equation which can also be represented as 0, & (1, 1)
2
5
0, , (1, 1)
2
CN - 232
Complex Number
Example # 3 : Find the value of expression x 4 – 4x 3 + 3x 2 – 2x + 1 when x = 1 + i.
Solution : x=1+i x–1=i
(x – 1) 2 = –1 x 2 – 2x + 2 = 0
2 2
x (x – 4x + 3) – 2x + 1 x 2 (x 2 – 2x + 2) – 2x3 + x 2 – 2x + 1
–2x (2x – 2) + 2x – 2 – 2x + 1 –4x 2 + 4x – 1
–4(2x – 2) + 4x – 1 –8x + 8 + 4x – 1
–4x + 7 –4(1 + i) + 7
3 – 4i
Now x 4 – 4x3 + 3x 2 – 2x + 1
= (x2 – 2x + 2) (x2 – 3x – 3) – 4x + 7
when x = 1 + i i.e. x2 – 2x + 2 = 0
4 3 2
x – 4x + 3x – 2x + 1 = 0 – 4 (1 + i) + 7 = –4 + 7 – 4i = 3 – 4i
(x + iy)2 + x2 y2 = 0
x 2 – y2 + x 2 y 2 = 0 and 2xy = 0
x = 0 or y = 0
when x = 0 – y2 + | y | = 0
y = 0, 1, –1
z = 0, i, –i
when y = 0 x2 + | x | = 0
x=0 z=0
Ans. z = 0, z = i, z = – i
(5) Given that x, y R, solve : 4x² + 3xy + (2xy 3x²)i = 4y² (x 2/2) + (3xy 2y²)i
3 1 3K
Answers : (4) ± – i, 0, i (5) x = K, y = KR
2 2 2
CN - 233
Complex Number
Representation of a complex number :
Early in the 19 th century, Karl Friedrich Gauss (1777 – 1855) and W illiam Rowan Hamilton
(1805 – 1865) independently and almost simultaneously proposed idea of defining complex number as
ordered pair of real numbers. i.e. a + ib = (a, b)
To each complex number there corresponds one and only one point in plane, and conversely to each
point in the plane there corresponds one and only one complex number. Because of this we often refer
to the complex number z as the point z.
Length OP is called modulus of the complex number which is denoted by z & is called the
argument or amplitude.
y
z = x 2 y 2 and tan = (angle made by OP with positive xaxis)
x
Note : (i) Argument of a complex number is a many valued function. If is the argument of a complex
number then 2n + ; n I will also be the argument of that complex number. Any two
arguments of a complex number differ by 2n
(ii) The unique value of such that < is called the principal value of the argument.
Unless otherwise stated, amp z implies principal value of the argument.
(iii) By specifying the modulus & argument a complex number is defined completely. For the complex
number 0 + 0 i the argument is not defined and this is the only complex number which is only
given by its modulus.
(b) Trignometric/Polar Representation :
z = r (cos + i sin ) where z = r; arg z = ; z = r (cos i sin )
Note : cos + i sin is also written as CiS
(c) Euler's Formula :
z = rei, |z| = r, arg z =
z = re i
Proof of this formula is beyond scope of present discussion. A heuristic proof serving as motivation
for this formula is by considering expansion.
x x2 x3
ex = 1 + 1 ! + + + ..........
2! 3!
put x = i
2 4 3 5
ei 1 ....... + i
.......
2! 4! 3! 5!
= cos + i sin .
CN - 234
Complex Number
Agrument of a Complex Number :
Argument of a non-zero complex number P(z) is denoted and defined by arg(z) = angle which OP
makes with the positive direction of real axis.
If OP = |z| = r and arg(z) = , then obviously z = r(cos + isin), called the polar form of z. 'Argument of
z' would mean principal argument of z(i.e. argument lying in (–, ] unless the context requires otherwise.
Thus argument of a complex number z = a + ib = r(cos + isin) is the value of satisfying rcos = a
and rsin = b.
b
Let = tan–1
a
(ii) a = 0, b > 0 p.v. arg z =
2
(vi) a = 0, b < 0 p.v. arg z = –
2
CN - 235
Complex Number
|z|= ( 1)2 2 2
= 1 2 = 3
2
Arg z = – tan–1 1 = – tan–1 ( 2 ) = (say)
z= 3 (cos + i sin ) where = – tan–1 2
17 82
Answers : (6) – tan–1 , (7) 6, 50°
11 5
If two points P and Q represent complex numbers z1 and z2 respectively in the Argand plane, then the
sum z1 + z2 is represented by the extremity R of the diagonal OR of parallelogram OPRQ having
OP and OQ as two adjacent sides.
(ii) Geometric representation of substraction.
CN - 236
Complex Number
Theorem : For any two complex numbers z1, z2 we have |z1 z2| = |z1| |z2| and
arg (z1z2) = arg (z1) + arg (z2).
Proof : z1 = r1 e i1 , z2 = r2 ei2
z1z2 = r2r2 e i( 1 2 ) |z1z2| = |z1| |z2|
arg (z1z2) = arg (z1) + arg (z2)
i.e. to multiply two complex numbers, we multiply their absolute values and add their
arguments.
Note : (i) P.V. arg (z1z2) P.V. arg (z1) + P.V. arg (z2)
(ii) |z1 z2 .... zn| = |z1| |z2| ..... |zn|
(iii) arg (z1z2 .... zn) = arg z1 + arg z2 + ..... + arg zn
OR OP
= OR = OP.OQ i.e. OR = r1r2 and QÔR = 1
OQ OL
z1
arg z = arg (z1) – arg (z2)
2
z1
Note : P.V. arg z P.V. arg (z1) – P.V. arg (z2)
2
z1
complex number z , we take a point L on real axis such that OL = 1 and draw a triangle OPR
2
OP OR r1
similar to OQL. Therefore = OR = r
OQ OL 2
CN - 237
Complex Number
Conjugate of a complex Number :
P ro p ertie s
zz zz
(i) If z = x + iy, then x = ,y=
2 2i
(ii) z= z z is purely real
(iii) z+ z =0 z is purely imaginary
(iv) Relation between modulus and conjugate. |z|2 = z z
(v) zz
(vi) ( z1 z 2 ) = z1 ± z 2
(vii) ( z1 z 2 ) = z1 z 2
z1 ( z1 )
=
(viii) z
2 ( z 2 ) (z2 0)
Theorem : Imaginary roots of polynomial equations with real coefficients occur in conjugate
pairs
Proof : If z0 is a root of a0zn + a1zn–1 + ...... + an–1 z + an = 0,
a0, a1, ....... an R, then a 0 zn0 + a1z n01 + ....... + an–1 z0 + an = 0
By using property (vi) and (vii) we have a 0 z0n + a1z 0n1 + ....... + an 1z 0 + an = 0
z0 is also a root.
Note : If w = f(z), then w = f( z )
CN - 238
Complex Number
Self Practice Problem
z1 2 z 2
(8) If is unimodulus and z2 is not unimodulus then find |z1|.
2 z1z 2
(9) If z = x + iy and f(z) = x 2 – y2 – 2y + i(2x – 2xy), then show that f(z) = z 2 + 2iz
a ib a2 b2
(10) If x + iy = prove that (x 2 + y2)2 = 2
c id c d2
Answer : (8) |z1| = 2
|z1 – z2| = ( x1 x 2 )2 ( y1 y 2 )2
In triangle OAC
OC OA + AC
OA AC + OC
AC OA + OC
using these in equalities we have ||z1| – |z2|| |z1 + z2| |z1| + |z2|
Similarly from triangle OAB
we have ||z1| – |z2|| |z1 – z2| |z1| + |z2|
Note : (a) ||z1| – |z2|| = |z1 + z2|, |z1 – z2| = |z1| + |z2| iff origin, z1 and z2 are collinear and origin lies
between z1 and z2 .
(b) |z1 + z2| = |z1| + |z2|, ||z1| – |z2|| = |z1 – z2| iff origin, z1 and z2 are collinear and z1 and z2 lies
on the same side of origin.
Example # 8 : If |z – 5 – 7i| = 9, then find the greatest and least values of |z – 2 – 3i|.
Solution : We have 9 = |z – (5 + 7i)| = distance between z and 5 + 7i.
Thus locus of z is the circle of radius 9 and centre at 5 + 7i. For such a z (on the circle), we
have to find its greatest and least distance as from 2 + 3i, which obviously 14 and 4.
2 2 2 2
Solution : z =1 |z| z |z|+
z z z z
2 2
Let | z | = r r 1 r+
r r
2
r+ 1 r R+ ..............(i)
r
2 2
and r 1 –1 r – 1
r r
r [1, 2] ..............(ii)
from (i) and (ii) r [1, 2]
r [1, 2]
Ans. |z|max = 2, |z|min =1
CN - 239
Complex Number
Self Practice Problem
(11) |z – 3| < 1 and |z – 4i| > M then find the positive real value of M for which these exist at least
one complex number z satisfy both the equation.
1 1
(12) If z lies on circle |z| = 2, then show that
z 4z 2 3
4
3
Answer : (11) M (0, 6)
Important results :
(i) arg z = represnets points (non-zero) on ray eminating
from origin making an angle with positive direction of
real axis
2
Example # 11 : Solve for z, which satisfy Arg (z – 3 – 2i) = and Arg (z – 3 – 4i) = .
6 3
Solution : From the figure, it is clear that there is no z, which satisfy both ray
(i) (ii)
5 5
(14) (i) 25 (ii) 5 (iii) – tan–1 2 (iv) tan–1 2
CN - 240
Complex Number
Rotation theorem :
(i) If P(z1) and Q(zz) are two complex numbers such that |z1| = |z2|, then z2 = z1 eiwhere = POQ
(ii) If P(z1), Q(z2) and R(z3) are three complex numbers and
z3 z2 z3 z2
PQR = , then z z = z1 z 2 e
i
1 2
(iii) If P(z1), Q(z2), R(z3) and S(z4) are four complex numbers
z3 z 4 z3 z 4
i
and STQ = , then = z1 z 2 e
z1 z 2
z 1
Example # 13 : If arg = then interpret the locus.
z 1 3
z 1
Solution : arg =
z 1 3
1 z
arg =
1 z 3
1 z
Here arg represents the angle between lines joining –1 and z, and 1 and z. As this
1 z
angle is constant, the locus of z will be a larger segment of circle. (angle in a segment is
constant).
Example # 14 : If A(2 + 3i) and B(3 + 4i) are two vertices of a square ABCD (take in anticlock wise order) then
find C and D.
Solution : Let affix of C and D are z3 and z4 respectively.
Considering DAB = 90º and AD = AB
i
z 4 ( 2 3 i) AD 2
we get = e
(3 4i) – (2 3i) AB
z4 – (2 + 3i) = (1 + i) i
z4 = 2 + 3i+ i – 1 = 1 + 4i
i
z 3 (3 4 i) CB – 2
and = e
(2 3i) – (3 4i) AB
z3 = 3 + 4i – (1 + i) (–i) z3 = 3 + 4i + i – 1 = 2 + 5i
Self Practice Problems
(15) z1, z2, z3, z4 are the vertices of a square taken in anticlockwise order then prove that
2z2 = (1 + i) z1 + (1 – i) z3
(16) Check that z1z2 and z3z4 are parallel or, not
where, z1 = 1 + i z3 = 4 + 2i
z2 = 2 – i z4 = 1 – i
(17) P is a point on the argand diagram on the circle with OP as diameter, two point Q and R are
taken such that POQ = QOR = If O is the origin and P, Q, R are represented by complex
z1, z2, z3 respectively then show that z22 cos 2 = z1z3cos 2
(18) If a, b, c ; u, v, w are complex numbers representing the vertices of two triangles such that
c = (1 – r) a + rb, w = (1 – r) u + rv where r is a complex number show that the two triangles are
similiar.
Answers : (16) z1z2 and z3z4 are not parallel.
CN - 241
Complex Number
Demoivre’s Theorem:
Case
Statement :
If n is any integer then
(i) (cos + i sin )n = cos n + i sin n
(ii) (cos 1 + i sin 1) (cos 2 + i sin 2) (cos3 + i sin 2) (cos 3 + i sin 3) .....(cos n + i sin n)
= cos (1 + 2 + 3 + ......... n) + i sin (1 + 2 + 3 + ....... + n)
Case
Statement : If p, q Z and q 0 then
2k p 2k p
(cos + i sin )p/q = cos + i sin
q q
where k = 0, 1, 2, 3, ......, q – 1
Note : Continued product of the roots of a complex quantity should be determined using theory of equations.
CN - 242
Complex Number
Self Practice Problem
10
(21) Find (1 r 2r )
r 0
(22) It is given that n is an odd integer greater than three, but n is not a multiple of 3. Prove that
x 3 + x2 + x is a factor of (x + 1)n – x n – 1
(23) If x = a + b, y = a + b , z = a + b where , are imaginary cube roots of unity show that
xyz = a3 + b3
5 2
n 1
(24) 2
If x – x + 1 = 0, then find the value of
n 1
x n
x
Answers : (21) 12 (24) 8
n th Roots of Unity :
p
(ii) 1p + 1p + 2p +.... + n 1 = 0 if p is not an integral multiple of n
= n if p is an integral multiple of n
(iii) (1 1) (1 2)...... (1 n 1 ) = n &
(1 + 1) (1 + 2)....... (1 + n 1) = 0 if n is even and 1 if n is odd.
(iv) 1. 1. 2. 3......... n 1 = 1 or 1 according as n is odd or even.
Example # 17 : Find the roots of the equation z6 + 64 = 0 where real part is positive.
Solution : z6 = – 64
z6 = 26 . e i(2n + 1) n = 0, 1, 2, 3, 4, 5
i( 2n1)
z=2 e 6
5 7 3 11
i i i i i i
6 6
z=2 e 6 , 2e 2 , 2e , 2e , 2e 2 , 2e 6
i 11
i
roots with +ve real part are = 2 e 6 , 2 e 6
6
2k 2k
Example # 18 : Find the value sin 7
cos
7
k 1
6 6 6 6
2k 2k 2k 2k
Solution : sin
7
– cos
7
= sin 7 – cos 7 +1
k 1 k 1 k 0 k 0
=
k 0
(Sum of imaginary part of seven seventh roots of unity)
–
k 0
(Sum of real part of seven seventh roots of unity) + 1
=0–0+1=1
CN - 243
Complex Number
Self Practice Problems
(25) Resolve z7 – 1 into linear and quadratic factor with real coefficient.
2 4 6
(26) Find the value of cos + cos + cos .
7 7 7
(27) Find all values of ( 256)1/4. Interpret the result geometrically.
2 2 4 6
Answers : (25) (z – 1) z 2 cos z 1 . z 2 2 cos z 1 . z 2 2 cos z 1
7 7 7
1
(26) –
2
2r 1 2r 1
(27) 4 cos i sin , r = 0, 1, 2, 3; vertices of a square in a
4 4
circle of radius 4 & centre (0, 0)
sin n / 2 n 1
+ sin 2 + sin 3 +..... + sin n = sin / 2 sin
(ii) sin
2
Note : If = (2/n) then the sum of the above series vanishes.
1 1
Loge (+ i ) = Loge (² + ²) + i 2 n tan where n .
2
Example # 19 : Find the value of
(i) log (1 + 3 i) Ans. log2 + i(2n + )
3
(ii) log(–1) Ans. i(2n + 1)
(iii) 2i Ans. cos(ln2) + i sin(ln2) = ei(ln2)
( 4n 1).
(iv) ii Ans. e 2
( 8n 1).
(v) |(1 + i) i | Ans. e 4
1
(vi) arg ((1 + i)i) Ans. n(2).
2
i 2n
Solution : (i) log (1 + 3 i) = log 2 e 3 = log 2 + i
2n
3
(ii) log(–1) = loge1 + i(2n + ) = i (2n + 1)
(iii) 2i = ein 2 = [cos (n 2) + i sin (n 2)]
(iv) z = ii logz = i log i
log i = i (4n + 1)/2
log z = –(4n + 1)/2
z = e–(4n + 1)/2
CN - 244
Complex Number
(v) |(1 + i) i | z = (1 + i)i log z = i log (1 + i)
log(1 + i) = log 2 +i(2n + /4)
2 – (2n + /4)
log z = i log (1 + i) = i log 2 – (2n + /4) z = ei log
| z | = e–(8n + 1)/4
(vi) arg ((1 + i)i)
z = (1 + i)i
log z = i log (1 + i) log(1 + i) = log 2 +i(2n + /4)
2 – (2n + /4)
log z = i log (1 + i) = i log 2 – (2n + /4) z = ei log
= log 2
Self Practice Problem
(28) Find the real part of cos (1 + i)
1 e2
Answer : cos 1
2e
Geometrical Properties :
Section formula
If z1 and z2 are affixes of the two points P and Q respectively and point C divides the line segment
joining P and Q internally in the ratio m : n then affix z of C is given by
mz 2 nz1
z= where m, n > 0
mn
mz 2 nz1
If C divides PQ in the ratio m : n externally then z =
mn
Note : If a, b, c are three real numbers such that az1 + bz2 + cz3 = 0 ; where a + b + c = 0 and a,b,c are not
all simultaneously zero, then the complex numbers z1, z2 & z3 are collinear.
(1) If the vertices A, B, C of a are represented by complex numbers z1, z2, z3 respectively and
a, b, c are the length of sides then,
z1 z 2 z 3
(i) Centroid of the ABC = :
3
(ii) Orthocentre of the ABC =
asec A z1 b sec B z 2 csecCz 3 z1tan A z 2 tanB z 3 tan C
asec A bsec B c secC or tanA tan B tanC
(iii) Incentre of the ABC = (az1 + bz2 + cz3) (a + b + c).
z z 1
z1 z1 1 = 0. This is also the condition for three complex numbers z, z1, z2 to be collinear..
z 2 z2 1
The above equation on manipulating, takes the form z z r = 0 where r is real and is
a non zero complex constant.
Note : If we replace z by zei and z by ze – i then we get equation of a straight line which makes an angle
with the given straight line.
CN - 245
Complex Number
(7) The equation of circle having centre z0 & radius is :
z z0 = or z z z0 z z 0 z + z 0 z0 ² = 0 which is of the form
z z 2 z 3 z1 z z 2 z 3 z1 z z2 z3 z1
points z1, z2 & z3 can be taken as is real = .
z z1 z 3 z 2 z z 1 z 3 z 2 z z1 z 3 z 2
z z1
(10) Arg z z = represent (i) a line segment if =
2
line z + z + r = 0 (Provided z 0 z0 r 0 )
z z1
(16) If z z2 = k 1, 0, then locus of z is circle.
(17) If z – z1 – z – z2 = K < z1 – z2 then locus of z is a hyperbola, whose focii are
z 1 & z2 .
CN - 246
Complex Number
Match the following columns :
Column - Column -
(i) If | z – 3+2i | – | z + i | = 0, (i) circle
then locus of z represents ..........
z 1
(ii) If arg = , (ii) Straight line
z 1 4
then locus of z represents...
(iii) if | z – 8 – 2i | + | z – 5 – 6i | = 5 (iii) Ellipse
then locus of z represents .......
z 3 4i 5
(iv) If arg z 2 5i = , (iv) Hyperbola
6
then locus of z represents .......
(v) If | z – 1 | + | z + i | = 10 (v) Major Arc
then locus of z represents ........
(vi) |z–3+i|–|z+2–i|=1 (vi) Minor arc
then locus of z represents .....
(vii) | z – 3i | = 25 (vii) Perpendicular bisector of a line segment
z 3 5i
(viii) arg z i = (viii) Line segment
Ans. (i) (ii) (iii) (iv) (v) (vi) (vii) (viii)
(ii),(vii) (v) (viii) (vi) (iii) (iv) (i) (viii)
MISCELLANEOUS EXAMPLES
Example # 20 :If z1 and z2 are two complex numbers and c > 0, then prove that
|z1 + z2|2 (1 + c) |z1|2 + (1 + c–1) |z2|2
Solution : We have to prove : |z1 + z2|2 (1 + c) |z1|2 + (1 + c–1) |z2|2
i.e. |z1|2 + |z2|2 + z1 z 2 + z 1z2 (1 + c) |z1|2 + (1 +c–1) |z2|2
1
or z1 z 2 + z 1z2 c|z1|2 + c–1|z2|2 or c|z1|2 + |z |2 – z1 z 2 – z 1 z2 0
c 2
2
1
(using Re (z1 z 2) |z1 z 2|) or c z1 | z2 | 0 which is always true.
c
Example # 21 : If i [/6, /3], i = 1, 2, 3, 4, 5, and
3
z4 cos 1 + z3 cos 2 + z2 cos 3. + z cos 4 + cos5 = 2 3 , then show that |z| >
4
Solution : Given that cos1 . z4 + cos2 . z3 + cos3 . z2 + cos4 . z + cos5 = 23
or |cos1 . z4 + cos2 . z3 + cos3 . z2 + cos4 . z + cos5| = 23
23 |cos1 . z4 | + |cos2 . z3 | + |cos3 . z2 | + cos4 . z| + |cos5 |
i [/6, /3]
1 3
cosi
2 2
3 3 3 3 2 3 3
2 3 |z|4 + |z| + |z| + |z| +
2 2 2 2 2
3 |z|4 + |z|3 + |z|2 + |z|
CN - 247
Complex Number
Case I : If |z| > 1, then above result is automatically true
Case II : If |z| < 1, then
3 < |z| + |z|2 + |z|3 + |z|4 +|z|5 + .........
|z| 3
3 < 1 | z | 3 – 3 |z| < |z| |z| >
4
3
Hence by both cases, |z| >
4
Example # 22 : Two different non parallel lines cut the circle |z| = r in point a, b, c, d respectively. Prove that
a 1 b 1 c 1 d 1
these lines meet in the point z given by z =
a 1b 1 c 1d 1
Solution : Since point P, A, B are collinear
z z 1
a a 1
=0
z a b – z (a – b) + a b a b = 0 (i)
b b 1
Similarly, points P, C, D are collinear, so
z c d – z (c – d) + c d cd = 0 (ii)
On applying (i) × (c – d) – (ii) (a – b), we get
z a b (c – d) – z c d (a – b) = c d cd (a – b) – a b a b (c – d) (iii)
2
k k k
zz = r = k (say) a = a , b = b , c = c etc.
From equation (iii) we get
k k k k ck kd ak bk
z (c – d) – z (a – b) = (a – b) – (c – d)
a b c d d c b a
a 1 b 1 c 1 d 1
z=
a 1b 1 c 1d 1
Example # 23 : If z1 = a + ib and z2 = c + id are complex number such that |z1| = |z2| = 1 and Re ( z1z 2 ) = 0,
then show that the pair of complex numbers w1 = a + ic and w2 = b + id satisfies the following
(i) |w1| = 1 (ii) |w2| = 1 (iii) Re ( w 1w 2 ) = 0
Solution : a = cos , b = sin
c = cos , d = sin
n
Re ( z1z 2 ) = 0 –= n=±1 c = sin , d = – cos
2
w1 = cos + i sin
w2 = sin – i cos
|w1| = 1, |w2| = 1
w1 w 2 = cos sin – sincos + i(sin2 – cos2) = – i cos 2
Re ( w 1w 2 ) = 0
CN - 248
Complex Number
Example # 24 : Let z1 and z2 be complex numbers such that z1 z2 and |z1| = |z2|. If z1 has positive real part
z1 z 2
and z2 has negative imaginary part, then show that z z is purely imaginary..
1 2
Solution : z1 = r (cos + i sin ), – <<
2 2
z2 = r (cos + i sin ), –<<0
z1 z 2 3
= – i cot , – < <
z1 z 2 2 4 2 4
Hence purely imaginary.
Example # 25 : If z1, z2 & z3 are the affixes of three points A, B & C respectively and satisfy the condition
|z1 – z2| = |z1| + |z2| and |(2 - i) z1 + iz3 | = |z1| + |(1 – i) z1 + iz3| then prove that ABC in a
right angled.
Solution : |z1 – z2| = |z1| + |z2|
z1, z2 and origin will be collinear and z1, z2 will be opposite side of origin
Similarly |(2 - i) z1 + iz3 | = |z1| + |(1 – i) z1 + iz3|
z1 and (1 – i) z1 + iz3 = z4 say, are collinear with origin and lies on same side of
origin.
Let z4 = z1 , real
then (1 – i) z1 + iz3 = z1
i (z3 – z1) = ( – 1) z1
( z 3 z1 ) z 3 z1
i/2
z1 = ( – 1) i 0 z1 = me , m = – 1
z3 – z1 is perpendicular to the vector 0 – z1 .
i.e. also z2 is on line joining origin and z1
so we can say the triangle formed by z1 , z2 and z3 is right angled.
CN - 249
Probability
Probability
Lest men suspect your tale untrue Keep probability in view........Gay, John
There are various phenomena in nature, leading to an outcome, which cannot be predicted apriori
e.g. in tossing of a coin, a head or a tail may result. Probability theory aims at measuring the
uncertainties of such outcomes.
(I) Important terminology :
(i) Random experiment :
It is a process which results in an outcome which is one of the various possible outcomes that are
known to us before hand e.g. throwing of a die is a random experiment as it leads to fall of one
of the outcome from {1, 2, 3, 4, 5, 6}. Similarly taking a card from a pack of 52 cards is also a random
experiment.
(ii) Sample space :
It is the set of all possible outcomes of a random experiment e.g. {H, T} is the sample space associated
with tossing of a coin.
Example # 1 : Write the sample space of the experiment ‘A coin is tossed and a die is thrown’.
Solution : The sample space S = {H1, H2, H3, H4, H5, H6, T1, T2, T3, T4, T5, T6}.
Example # 2 : Write the sample space of the experiment ‘A coin is tossed, if it shows head a coin tossed
again else a die is thrown.
Solution : The sample space S = {HH, HT, T1, T2, T3, T4, T5, T6}
Example # 3 : Find the sample space associated with the experiment of rolling a pair of dice (plural of die) once.
Also find the number of elements of the sample space.
Solution : Let one die be blue and the other be green. Suppose ‘1’ appears on blue die and ‘2’ appears on green
die. We denote this outcome by an ordered pair (1, 2). Similarly, if ‘3’ appears on blue die and ‘5’
appears on green die, we denote this outcome by (3, 5) and so on. Thus, each outcome can be
denoted by an ordered pair (x, y), where x is the number appeared on the first die (blue die) and
y appeared on the second die (green die). Thus, the sample space is given by
S = {(x, y) x is the number on blue die and y is the number on grey die}
We now list all the possible outcomes (figure)
1 2 3 4 5 6
1 (1, 1) (1, 2) (1, 3) (1, 4) (1, 5) (1, 6)
2 (2, 1) (2, 2) (2, 3) (2, 4) (2, 5) (2, 6)
3 (3, 1) (3, 2) (3, 3) (3, 4) (3, 5) (3, 6)
4 (4, 1) (4, 2) (4, 3) (4, 4) (4, 5) (4, 6)
5 (5, 1) (5, 2) (5, 3) (5, 4) (5, 5) (5, 6)
6 (6, 1) (6, 2) (6, 3) (6, 4) (6, 5) (6, 6)
Figure
Number of elements (outcomes) of the above sample space is 6 × 6 i.e., 36
Self practice problems :
(1) A coin is tossed twice, if the second throw results in head, a die is thrown then write sample
space of the experiment.
PR - 250
Probability
(2) An urn contains 3 red balls and 2 blue balls. Write sample space of the experiment ‘Selection
of a ball from the urn at random’.
Answers : (1) {HT, TT, HH1, HH2, HH3, HH4, HH5, HH6, TH1, TH2, TH3, TH4, TH5, TH6}.
(2) {R1, R2, R3, B1, B2 }. (Here the balls are distinguished from one and other by
naming red balls as R 1, R2 and R3 and the blue balls as B1 and B2.)
(iii) Event :
It is subset of sample space. e.g. getting a head in tossing a coin or getting a prime number in
throwing a die. In general if a sample space consists ‘n’ elements, then a maximum of 2n events can
be associated with it.
Example # 4 : Write down all the events of the experiment ‘tossing of a coin’.
Solution : S = {H, T}
the events are , {H}, {T}, {H, T}
Example # 5 : A die is thrown. Let A be the event ‘ an odd number turns up’ and B be the event ‘a number
divisible by 3 turns up’. Write the events (a) A or B (b) A and B
Solution : A = {1, 3, 5}, B = {3, 6}
A or B = A B = {1, 3, 5, 6}
A and B = A B = {3}
(3) A coin is tossed and a die is thrown. Let A be the event ‘H turns up on the coin and odd number
turns up on the die’ and B be the event ‘ T turns up on the coin and an even number turns up
on the die’. Write the events (a) A or B (b) A and B.
(4) In tossing of two coins, let A = {HH, HT} and B = {HT, TT}. Then write the events
(a) A or B (b) A and B.
Answers : (3) (a) {H1, H3, H5, T2, T4, T6} (b)
(4) (a) {HH, HT, TT} (b) {HT}
(vii) Equally likely events :
If events have same chance of occurrence, then they are said to be equally likely.
e.g
(i) In a single toss of a fair coin, the events {H} and {T} are equally likely.
(ii) In a single throw of an unbiased die the events {1}, {2}, {3} and {4}, are equally likely.
(iii) In tossing a biased coin the events {H} and {T} are not equally likely.
PR - 251
Probability
(viii) Mutually exclusive / disjoint / incompatible events :
Two events are said to be mutually exclusive if occurrence of one of them rejects the possibility of
occurrence of the other i.e. both cannot occur simultaneously.
In the vein diagram the events A and B are mutually exclusive. Mathematically, we write
AB=
Events A1, A2, A3, ....... An are said to be mutually exclusive events iff
Ai Aj = i, j {1, 2, ..., n} where i j
Example # 6 : In a single toss of a coin find whether the events {H}, {T} are mutually exclusive or not.
Solution : Since {H} {T} = ,
the events are mutually exclusive.
Example # 7 : In a single throw of a die, find whether the events {1, 2}, {2, 3} are mutually exclusive or not.
Solution : Since {1, 2} {2, 3} = {2}
the events are not mutually exclusive.
Example # 8 : In throwing of a die, let A be the event ‘even number turns up’, B be the event ‘an odd prime
turns up’ and C be the event ‘a numbers less than 4 turns up’. Find whether the events
A, B and C form an exhaustive system or not.
Solution : A {2, 4, 6}, B {3, 5} and C {1, 2, 3}.
Clearly A B C = {1, 2, 3, 4, 5, 6} = S. Hence the system of events is exhaustive.
Example # 9 : Three coins are tossed. Describe
(i) two events A and B which are mutually exclusive
(ii) three events A, B and C which are mutually exclusive and exhaustive.
(iii) two events A and B which are not mutually exclusive.
(iv) two events A and B which are mutually exclusive but not exhaustive.
(v) three events A, B and C which are mutually exclusive but not exhaustive.
PR - 252
Probability
Ans. (i) A : “getting at least two heads” B : “getting at least two tails”
(ii) A : “getting at most one heads” B : “getting exactly two heads”
C : “getting exactly three heads”
(iii) A : “getting at most two tails” B : “getting exactly two heads”
(iv) A : “getting exactly one head” B : “getting exactly two heads”
(v) A : “ getting exactly one tail” B : “getting exactly two tails”
C : “getting exactly three tails”
[Note : There may be other cases also]
Self practice problems :
(7) In throwing of a die which of the following pair of events are mutually exclusive ?
(a) the events ‘coming up of an odd number’ and ‘coming up of an even number’
(b) the events ‘coming up of an odd number’ and ‘coming up of a number 4’
(8) In throwing of a die which of the following system of events are exhaustive ?
(a) the events ‘an odd number turns up’, ‘a number 4 turns up’ and ‘the number 5 turns
up’.
(b) the events ‘a number 4 turns up’, ‘a number > 4 turns up’.
(c) the events ‘an even number turns up’, ‘a number divisible by 3 turns up’, ‘number
1 or 2 turns up’ and ‘the number 6 turns up’.
Answers (7) (a) (8) (b)
If an experiment results in a total of (m + n) outcomes which are equally likely and if ‘m’ outcomes
are favorable to an event ‘A’ while ‘n’ are unfavorable, then the probability of occurrence of the event
m
i.e. P(A) = .
mn
We say that odds in favour of ‘A’ are m : n, while odds against ‘A’ are n : m.
n
Note that P( A ) or P(A) or P(AC), i.e. probability of non-occurrence of A = = 1 – P(A)
mn
In the above we shall denote the number of out comes favourable to the event A by n(A) and the total
number of out comes in the sample space S by n(S).
n( A )
P(A) = .
n(S)
Example # 10 : In throwing of a fair die find the probability of the event ‘ a number 4 turns up’.
Solution : Sample space S = {1, 2, 3, 4, 5, 6} ; event A = {1, 2, 3, 4}
n(A) = 4 and n(S) = 6
n( A ) 4 2
P(A) = = = .
n(S) 6 3
Example # 11 : In throwing of a fair die, find the probability of turning up of an odd number 4.
Solution : S = {1, 2, 3, 4, 5, 6}
Let E be the event ‘turning up of an odd number 4’
then E = {5}
n (E) 1
P(E) = n (S) = .
6
PR - 253
Probability
Example # 12 : In throwing a pair of fair dice, find the probability of getting a total of 8.
Solution : When a pair of dice is thrown the sample space consists
{(1, 1) (1, 2) .......... (1, 6)
(2, 1,) (2, 2,)......... (2, 6)
.... ..... .... ...
.... ... ... ...
(6, 1), (6, 2) ........ (6, 6)}
Note that (1, 2) and (2, 1) are considered as separate points to make each outcome as equally
likely.
To get a total of ‘8’, favourable outcomes are, (2, 6) (3, 5) (4, 4) (5, 3) and (6, 2).
5
Hence required probability =
36
Example # 13 : A four digit number is formed using the digits 0, 1, 2, 3, 4 without repetition. Find the probability that
it is divisible by 4
Each of these 96 numbers are equally likely & mutually exclusive of each other.
Now, A number is divisible by 4, if last two digits of the number is divisible by 4
6 ways
4 ways
4 ways
6 ways
__________
Total number of ways 30 ways
favorable outcomes 30 5
probability = Total outcomes = = Ans.
96 16
(9) A bag contains 4 white, 3 red and 2 blue balls. A ball is drawn at random. Find the probability
of the event (a) the ball drawn is white or red (b) the ball drawn is white as well as red.
(10) In throwing a pair of fair dice find the probability of the events ‘ a total of of less than or equal
to 9”’.
Answers (9) (a) 7/9 (b) 0 (10) 5/6.
PR - 254
Probability
Distributive laws : (a) A (B C) = (A B) (A C)
(b) A (B C) = (A B) (A C)
For any three events A, B and C we have the figure
(i) P(A or B or C) = P(A) + P(B) + P(C) – P(A B) – P(B C) – P(C A) + P(A B C)
(ii) P (at least two of A, B, C occur) = P(B C) + P(C A) + P(A B) – 2P(A B C)
(iii) P(exactly two of A, B, C occur) = P(B C) + P(C A) + P(A B) – 3P(A B C)
(iv) P(exactly one of A, B, C occur) =
P(A) + P(B) + P(C) – 2P(B C) – 2P(C A) – 2P(A B) + 3P(A B C)
Example # 14 : A bag contains 4 white, 3red and 4 green balls. A ball is drawn at random. Find the probability
of the event ‘the ball drawn is white or green’.
Solution : Let A be the event ‘the ball drawn is white’ and B be the event ‘the ball drawn is green’.
8
P(The ball drawn is white or green) = P (A B) = P(A) + P(B) – P(A B) =
11
Example # 15 : In throwing of a die, let A be the event ‘an odd number turns up’, B be the event ‘a number
divisible by 3 turns up’ and C be the event ‘a number 4 turns up’. Then find the probability
that exactly two of A, B and C occur.
Solution : Event A = {1, 3, 5}, event B = {3, 6} and event C = {1, 2, 3, 4}
A B = {3}, B C = {3}, A C = {1, 3} and A B C = {3}.
Thus P(exactly two of A, B and C occur)
= P(A B) + P(B C) + P(C A) – 3P(A B C)
1 1 2 1 1
= + + – 3 × =
6 6 6 6 6
(11) In throwing of a die, let A be the event ‘an odd number turns up’, B be the event ‘a number
divisible by 3 turns up’ and C be the event ‘a number 4 turns up’. Then find the probability
that atleast two of A, B and C occur.
(12) In the problem number 11, find the probability that exactly one of A, B and C occurs.
1 2
Answers (11) (12)
3 3
PR - 255
Probability
Example # 16 : If P(A/B) = 0.2 and P(B) = 0.5 and P(A) = 0.2. Find P(A B ).
Solution : P(A B ) = P(A) – P(A B)
P( A B)
Also P(A/B) = P(A B) = 0.1
P(B)
From given data,
P(A B ) = 0.1
Example # 17 : If P(A) = 0.25, P(B) = 0.5 and P(A B) = 0.14, find probability that neither ‘A’ nor ‘B’ occurs. Also
find P A B
Solution :
We have to find P A B = 1 – P(A B) (by De-Morgan’s law)
Also, P(A B) = P(A) + P(B) – P(A B)
putting data we get, P A B = 0.39
Hence P A B = P(A) – P(A B) = 0.11
Example # 18 : A pair of fair coins is tossed yielding the equiprobable space S = {HH, HT, TH, TT}. Consider
the events:
A = {head on first coin} = {HH, HT}, B = {head on second coin} = {HH, TH}
C = {head on exactly one coin} = {HT, TH}
Then check whether A, B, C are independent or not.
2 1
Solution : P(A) = P(B) = P(C) = = .
4 2
1 1 1
Also P(A B) = = P(A) P(B), P(A C) = = P(A) P(C), P(B C) = = P(B) P(C)
4 4 4
but P(A B C) = 0 P(A) P(B) P(C)
A, B & C are not independent
PR - 256
Probability
Example # 19 : In drawing two balls from a box containing 6 red and 4 white balls without replacement, which
of the following pairs is independent ?
(a) Red on first draw and red on second draw
(b) Red on first draw and white on second draw
Solution : Let E be the event ‘Red on first draw’, F be the event ‘Red on second draw’ and G be the event
‘white on second draw’.
6 6 4
P(E) = , P(F) = , P(G) =
10 10 10
6
P2 1
(a) P(E F) = 10 =
P2 3
3 3 9 1
P(E) . P(F) = × =
5 5 25 3
E and F are not independent
6 4 6
(b) P(E) . P(G) = × =
10 10 25
6
P1 4 P1 4
P(E G) = 10 =
P2 15
P(E) . P(G) P(E G)
E and G are not independent
Example # 20 : If two switches S1 and S2 have respectively 90% and 80% chances of working. Find the probabilities
that each of the following circuits will work.
9 8 1 2 49
= 1 – 1 1 =1 – × =
10 10 10 10 50
PR - 257
Probability
Example # 21 : A speaks truth in 60% of the cases and b in 90% of the cases. In what percentage of cases are they
likely to contradict each other in stating the same fact?
Solution : Let E be the event that A speaks truth and F be the event that B speaks truth. Then E and F are
independent events such that
60 3 90 9
P(E) = = and P(F) = =
100 5 100 10
A and B will contradict each other in narrating the same fact in the following mutually exclusive
ways:
(i) A speaks truth and B tells a lie i.e. E F
(ii) A tells a lie and B speaks truth lie i.e. E F
P(A and B contradict each other)
= P(I or II) = (I II) = P[(E F ) ( E F)]
= P(E F ) + P ( E F) [ E F and E F are mutually exclusive]
= P(E) P( F ) + P( E ) P(F) [ E and F are in dep.]
3 9 3 9 3 1 2 9 21
= × 1 + 1 × = × + × =
5 10 5 10 5 10 5 10 50
Example # 22 : A box contains 5 bulbs of which two are defective. Test is carried on bulbs one by one untill the two
defective bulbs are found out. Find the probability that the process stops after
(i) Second test (ii) Third test
Solution : (i) Process will stop after second test. Only if the first and second bulb are both found to be
defective
2 1 1
probability = × = (Obviously the bulbs drawn are not kept back.)
5 4 10
(ii) Process will stop after third test when either
2 3 1 1
(a) DND × × = Here ‘D’ stands for defective
5 4 3 10
3 2 1 1
or (b) NDD × × = and ‘N’ is for not defective.
5 4 3 10
3 2 1 1
or (c) NNN × × =
5 4 3 10
3
hence required probability =
10
1 1 E1
Example # 23 : If E1 and E2 are two events such that P(E1) = ; P(E2) = ; P = 1 , then choose the correct
4 2 E
2 4
options.
(i) E1 and E2 are independent (ii) E1 and E2 are exhaustive
(iii) E1 and E2 are mutually exclusive (iv) E1 & E2 are dependent
E1 E2
Also find P E and
2 E1
E1
Solution : Since P E = P(E1) E1 and E2 are independent of each other
2
Also since P(E1 E2) = P(E1) + P(E2) – P(E1) . P(E2) 1
Hence events are not exhaustive. Independent events can’t be mutually exclusive.
Hence only (i) is correct
Further since E1 & E2 are independent; E1 and E2 or E1 , E2 are E1 , E2 are also independent.
E
E
Hence P 1 = P E1 =
E
3
4
and P 2 = P (E2) =
E
1
2
2 1
PR - 258
Probability
Example # 24 : If cards are drawn one by one from a well shuffled pack of 52 cards without replacement, until an ace
appears, find the probability that the fourth card is the first ace to appear.
48
C 3 4 C1
Solution : Probability of selecting 3 non-Ace and 1 Ace out of 52 cards is equal to 52
C4
Since we want 4th card to be first ace, we will also have to consider the arrangement, Now 4 cards
in sample space can be arranged in 4! ways and, favorable they can be arranged in 3 ! ways as we
want 4th position to be occupied by ace
48
C 3 4 C1 3!
Hence required probability = 52 × 4!
C4
Aliter :
‘NNNA’ is the arrangement then we desire in taking out cards, one by one
48 47 46 4
Hence required chance is × × ×
52 51 50 49
(16) In throwing a pair of dies find the probability of getting an odd number on the first die and a
total of 7 on both the dies.
(17) In throwing of a pair of dies, find the probability of getting a doublet or a total of 4.
(18) A bag contains 8 marbles of which 3 are blue and 5 are red. One marble is drawn at random, its
colour is noted and the marble is replaced in the bag. A marble is again drawn from the bag and its
colour is noted. Find the probability that the marbles will be
(i) blue followed by red (ii) blue and red in any order (iii) of the same colour.
(19) A coin is tossed thrice. In which of the following cases are the events E and F independent ?
(i) E : “the first throw results in head”.
F : “the last throw result in tail”.
(ii) E : “the number of heads is two”.
F : “the last throw result in head”.
(iii) E : “the number of heads is odd ”.
F : “the number of tails is odd”.
49 16 56 2 1
Answers : (14) (i) (ii) (iii) (15) (i) (ii)
121 121 121 3 2
1 2 15 15 17
(16) (17) (18) (i) (ii) (iii)
12 9 64 32 32
(19) (i)
(VI) Binomial probability theorem :
If an experiment is such that the probability of success or failure does not change with trials, then
the probability of getting exactly r success in n trials of such an experiment is nCr pr qn – r, where ‘p’
is the probability of a success and q is the probability of a failure in one particular experiment. Note
that p + q = 1.
PR - 259
Probability
Example 25 : A pair of dice is thrown 5 times. Find the probability of getting a doublet twice.
1
Solution : In a single throw of a pair of dice probability of getting a doublet is
6
1
considering it to be a success, p =
6
1 5
q = 1 – =
6 6
number of success r = 2
2 3
1 5 625
P(r = 2) = 5C2 p2 q3 = 10 . . =
6 6 3888
Example # 26 : A pair of dice is thrown 4 times. If getting ‘a total of 9’ in a single throw is considered as a
success then find the probability of getting ‘a total of 9’ thrice.
4 1
Solution : p = probability of getting ‘a total of 9’ = =
36 9
1 8
q = 1 – =
9 9
r = 3, n = 4
3
1 8 32
P(r = 3) = 4C3 p3 q = 4 × . =
9 9 6561
Example # 27 : In an examination of 10 multiple choice questions (1 or more can be correct out of 4 options). A
student decides to mark the answers at random. Find the probability that he gets exactly two
questions correct.
Solution : A student can mark 15 different answers to a MCQ with 4 option i.e. 4C1 + 4C2 + 4C3 + 4C4 = 15
1
Hence if he marks the answer at random, chance that his answer is correct = and being
15
14 1 14
incorrecting . Thus p= ,q= .
15 15 15
2 8
1 14
P (2 success) = C2 × ×
10
15
15
Example # 28 : A family has three children. Event ‘A’ is that family has at most one boy, Event ‘B’ is that family has
at least one boy and one girl, Event ‘C’ is that the family has at most one girl. Find whether events
‘A’ and ‘B’ are independent. Also find whether A, B, C are independent or not.
Solution : A family of three children can have
(i) All 3 boys (ii) 2 boys + 1 girl (iii) 1 boy + 2 girls (iv) 3 girls
3
1 1
(i) P (3 boys) = C0 =
3
(Since each child is equally likely to be a boy or a girl)
2
8
2
1 1 3
(ii) P (2 boys +1girl) = 3C1 × × = (Note that there are three cases BBG, BGB, GBB)
2 2 8
1 2
1 1 3
(iii) P (1 boy + 2 girls) = C2 × × =
3
2
2
8
PR - 260
Probability
1
(iv) P (3 girls) =
8
1
Event ‘A’ is associated with (iii) & (iv). Hence P(A) =
2
3
Event ‘B’ is associated with (ii) & (iii). Hence P(B) =
4
1
Event ‘C’ is associated with (i) & (ii). Hence P(C) =
2
3
P(A B) = P(iii) = = P(A) . P(B) . Hence A and B are independent of each other
8
P(A C) = 0 P(A) . P(C) . Hence A, B, C are not independent
Self practice problems :
(20) A box contains 2 red and 3 blue balls. Two balls are drawn successively without replacement.
If getting ‘a red ball on first draw and a blue ball on second draw’ is considered a success,
then find the probability of 2 successes in 3 performances.
(21) Probability that a bulb produced by a factory will fuse after an year of use is 0.2. Find the
probability that out of 5 such bulbs not more than 1 bulb will fuse after an year of use.
2304
Answers (20) 189/1000 (21)
3125
(VI I ) Expectation :
If there are n possibilities A1, A2, .... An in an experiment having the probabilities p1, p2, .........pn
respectively. If value M1, M2, ....., Mn are associated with the respective possibility. Then the expected
n
Example # 29 : A fair die is tossed. If 2, 3 or 5 occurs, the player wins that number of rupees, but if 1, 4, or
6 occurs, the player loses that number of rupees. Then find the possible payoffs for the player.
Ai 2 3 5 1 4 6
Mi 2 3 5 –1 –4 –6
Solution :
Pi 1/6 1/6 1/6 1/6 1/6 1/6
Then expected value E of the game payoffs for the player
1 1 1 1 1 1 1
= 2 + 3 + 5 – 1 – 4 – 6 = –
6
6
6
6
6
6
6
Since E is negative therefore game is unfavorable to the player.
Example # 30 : There are 100 tickets in a raffle (Lottery). There is 1 prize each of Rs. 1000/-, Rs. 500/- and
Rs. 200/-. Remaining tickets are blank. Find the expected price of one such ticket.
Solution : Expectation = piMi outcome of a ticket can be
pi Mi piMi
1
(i) I prize 1000 10
100
1
(ii) II prize 500 5
100
1
(iii) III prize 200 2
100
97
(iv) Blank 0 0
100
________________
piMi = 17
________________
Hence expected price of one such ticket Rs. 17
PR - 261
Probability
Example # 31 : A purse contains four coins each of which is either a rupee or two rupees coin. Find the expected
value of a coin in that purse.
Solution : Various possibilities of coins in the purse can be
pi Mi piMi
1 4
(i) 4 1 rupee coins 4
16 16
4 20
(ii) 3 one Rs. + 1 two Rs. 5
16 16
6 36
(iii) 2 one Rs. + 2 two Rs. 6
16 16
4 28
(iv) 1 one Rs. + 3 two Rs. 7
16 16
1 8
(v) 4 two Rs. 8
16 16
________________
6/-
________________
Hence expected value is Rs. 6/-
Note : (that since each coin is equally likely to be one Rs. or two Rs. coin, the probability is determined
using Binomial probability; unlike the case when the purse contained the coins with all possibility
1
being equally likely, where we take pi = for each.)
5
Self practice problems :
(22) From a bag containing 2 one rupee and 3 two rupee coins a person is allowed to draw 2 coins
simultaneously ; find the value of his expectation.
Answer : Rs. 3.20
Proof :
The event A occurs with one of the n mutually exclusive and exhaustive events
B1, B 2, B 3,........,B n
A = (A B1) (A B2) (A B3) ........ (A Bn)
n
Now,
P(A Bi) = P(A) . P(Bi/A) = P(Bi) . P(A/Bi)
n
P(A) = P(B ) . P( A / B )
i1
i i
PR - 262
Probability
Example # 32 : Box - contains 5 red and 4 white balls while box - contains 4 red and 2 white balls. A fair
die is thrown. If it turns up a multiple of 3, a ball is drawn from box - else a ball is drawn from
box - . Find the probability that the ball drawn is white.
Solution : Let A be the event ‘a multiple of 3 turns up on the die’ and R be the event ‘the ball drawn is
white’
then P (ball drawn is white)
2 4 2 2 10
= P(A) . P(R / A) + P ( A ) P(R / A ) = × + 1 =
6 9 6 6 27
Example # 33 : Cards of an ordinary deck of playing cards are placed into two heaps. Heap - consists of
all the red cards and heap - consists of all the black cards. A heap is chosen at random and
a card is drawn, find the probability that the card drawn is a king.
Solution : Let and be the events that heap - and heap - are choosen respectively. Then
1
P() = P() =
2
Let K be the event ‘the card drawn is a king’
2 2
P (K / ) = and P(K /) =
26 26
1 2 1 2 1
P(K) = P () P(K / ) + P() P(K /) = × + × = .
2 26 2 26 13
Self practice problems :
(23) Box - contains 3 red and 2 blue balls while box - II contains 2 red and 3 blue balls. A fair
coin is tossed. If it turns up head, a ball is drawn from box - , else a ball is drawn from
box - . Find the probability that the ball drawn is red.
(24) There are 5 brilliant students in class XI and 8 brilliant students in class XII. Each class has
50 students. The odds in favour of choosing the class XI are 2 : 3. If the class XI is not chosen
then the class XII is chosen. Find the probability of selecting a brilliant student.
1 17
Answers : (23) (24) .
2 125
P(B i ) . P( A / Bi )
P(Bi / A) = n
Proof :
i 1
P(B i ) . P( A / Bi )
The event A occurs with one of the n mutually exclusive and exhaustive events
B1, B 2, B 3,........,B n
A = (A B1) (A B2) (A B3) ........ (A Bn)
n
Now,
P(A Bi) = P(A) . P(Bi/A) = P(Bi) . P(A/Bi)
P(Bi ) . P( A / Bi ) P(Bi ) . P( A / B i )
P (Bi/A) = =
P( A ) n
P( A Bi )
i 1
P(B i ) . P( A / Bi )
P(Bi/A) =
P(B ) . P(A / B )
i i
PR - 263
Probability
2
Example # 34 : Pal’s gardener is not dependable, the probability that he will forget to water the rose bush is . The
3
1
rose bush is in questionable condition any how, if watered the probability of its withering is , if not
2
3
watered, the probability of its withering is . Pal went out of station and upon returning, he finds
4
that the rose bush has withered, what is the probability that the gardener did not water the bush.
[Here result is known that the rose bush has withered, therefore. Bayes’s theorem should be used]
Solution : Let A = the event that the rose bush has withered
Let A1 = the event that the gardener did not water.
A2 = the event that the gardener watered.
By Bayes’s theorem required probability,
P( A 1 ) . P( A / A 1 )
P(A1/A) = P( A ) . P( A / A ) P( A ) . P( A / A ) .....(i)
1 1 2 2
2 1
Given, P(A1) = P(A2) =
3 3
3 1
P(A/A1) = , P(A/A2) =
4 2
2 3
.
3 4 6 3
From (1), P(A1/A) = 2 3 1 1 = =
. . 6 2 4
3 4 3 2
Example # 35 : There are 5 brilliant students in class XI and 8 brilliant students in class XII. Each class has
50 students. The odds in favour of choosing the class XI are 2 : 3. If the class XI is not chosen
then the class XII is chosen. A student is a chosen and is found to be brilliant, find the probability
that the chosen student is from class XI.
Solution : Let E and F be the events ‘Class XI is chosen’ and ‘Class XII is chosen’ respectively.
2 3
Then P(E) = , P(F) =
5 5
Let A be the event ‘Student chosen is brilliant’.
5 8
Then P(A / E) = and P(A / F) = .
50 50
2 5 3 8 34
P(A) = P(E) . P(A / E) + P(F) . P(A / F) = . + . = .
5 50 5 50 250
P(E) . P( A / E ) 5
P(E / A) = P(E ) . P( A / E) P(F) . P( A / F) = .
17
Example # 36 : A pack of cards is counted with face downwards. It is found that one card is missing. One card is
drawn and is found to be red. Find the probability that the missing card is red.
Solution : Let A be the event of drawing a red card when one card is drawn out of 51 cards (excluding missing
card.) Let A1 be the event that the missing card is red and A2 be the event that the missing card is
black.
Now by Bayes’s theorem, required probability,
P( A 1 ) . (P( A / A 1 )
P(A1/A) = P( A ) . P( A / A ) P( A ) . P( A / A ) ..........(i)
1 1 2 2
PR - 264
Probability
26
C1 26 1
Now P(A1) = probability that the missing card is red = 52 = =
C1 52 2
26 1
P(A2) = probability that the missing card is black = =
52 2
P(A/A1) = probability of drawing a red card when the missing card is red.
25
=
51
[Total number of cards left is 51 out of which 25 are red and 26 are black as the missing card is red]
26
Again P(A/A2) = Probability of drawing a red card when the missing card is black =
51
1 25
.
2 51 25
Now from (i), required probability, P(A1/A) = 1 25 1 26 =
. . 51
2 51 2 51
Example # 37 : A bag contains 6 white and an unknown number of black balls ( 3). Balls are drawn one by one with
replacement from this bag twice and is found to be white on both occassion. Find the probability that
the bag had exactly ‘3’ Black balls.
Solution : Apriori, we can think of the following possibilies
(i) E1 6W , 0B
(ii) E2 6W , 1B
(iii) E3 6W , 2B
(iv) E4 6W , 3B
1
Clearly P(E1) = P(E2) = P(E3) = P(E4) =
4
Let ‘A’ be the event that two balls drawn one by one with replacement are both white therefore we
E4
have to find P
A
A
P P(E4 )
E4 E4
By Baye’s theorem P =
A A A A A
P P(E1 ) P . P(E2 ) P . P(E3 ) P . P(E4 )
E1 E2 E3 E4
A 6 6 A 6 6 A 6 6 A 6 6
P E = × ; P E = × ; P E = × ; P E = × ;
4 9 9 3 8 8 2 7 7 1 6 6
1 1
E4 81 4
Putting values P =
A 1 1 1 1 1
4 81 64 49 36
Self practice problems :
(25) Box- contains 3 red and 2 blue balls while box- contains 2 red and 3 blue balls. A fair coin
is tossed. If it turns up head, a ball is drawn from box-, else a ball is drawn from box-. If the
ball drawn is red, then find the probability that the ball is drawn from box-.
(26) Cards of an ordinary deck of playing cards are placed into two heaps. Heap - consists of all
the red cards and heap - consists of all the black cards. A heap is chosen at random and
a card is drawn, if the card drawn is found to be a king, find the probability that the card drawn
is from the heap - .
2 1
Answers : (25) (26)
5 2
PR - 265
Probability
(X) Probability distribution :
(i) A probability distribution spells out how a total probability of 1 is distributed over several values
of a random variable (i.e. how ............................... possibilities)
(ii) Mean of any probability distribution of a random variable is given by :
pi x i
µ = = pi x i (Since pi = 1)
pi
(iii) Variance of a random variable is given by, 2 = (x i – µ)2 . pi
2 = pi x i2 – µ 2 (Note that SD = + 2 )
(iv) The probability distribution for a binomial variate ‘X’ is given by :
P(X = r) = nCr pr qn – r where P(X = r) is the probability of r successes.
P(r 1) nr p
The recurrence formula = . , is very helpful for quickly computing P(1) . P(2)
P(r ) r 1 q
. P(3) etc. if P(0) is known.
Mean of Binom ial Probability Distribution = np ; variance of Binomial Probability
Distribution = npq.
(v) If p represents a person’s chance of success in any venture and ‘M’ the sum of money which
he will receive in case of success, then his expectations or probable value = pM
Example # 38 : A random variable X has the following probability distribution :
X 0 1 2 3 4 5 6 7
P(X) 0 k 2k 2k 3k k2 2k2 7k2 + k
Determine
(i) k (ii) P(X < 3) (iii) P(X > 6) (iv) P(0 < X < 3)
[Hint : Use P(X) = 1 to determine k, P(X < 3) = P(0) + P(1) + P(2), P(X > 6) = P(7) etc.]
Example # 39 : A pair of dice is thrown 5 times. If getting a doublet is considered as a success, then find
the mean and variance of successes.
1
Solution : In a single throw of a pair of dice, probability of getting a doublet =
6
1
considering it to be a success, p =
6
1 5
q = 1 – =
6 6
1 5 1 5 25
mean = 5 × = , variance = 5 × . =
6 6 6 6 36
Example # 40 : A pair of dice is thrown 4 times. If getting a total of 9 in a single throw is considered as a
success then find the mean and variance of successes.
4 1
Solution : p = probability of getting a total of 9 = =
36 9
1 8
q = 1 – =
9 9
1 4
mean = np = 4 × =
9 9
1 8 32
variance = npq = 4 × × =
9 9 81
Example # 41 : Difference between mean and variance of a Binomial variate is ‘1’ and difference between their
squares is ‘11’. Find the probability of getting exactly three success
Solution : Mean = np & variance = npq
therefore, np – npq = 1 ..........(i)
n2p2 – n2p2q2 = 11 ..........(ii)
Also, we know that p + q = 1 ..........(iii)
PR - 266
Probability
5 1
Divide equation (ii) by square of (i) and solve, we get, q = ,p= & n = 36
6 6
3 33
1 5
Hence probability of ‘3’ success = 36C3 × ×
6 6
Self practice problems :
(27) A box contains 2 red and 3 blue balls. Two balls are drawn successively without replacement.
If getting ‘a red ball on first draw and a blue ball on second draw’ is considered a success,
then find the mean and variance of successes.
(28) Probability that a bulb produced by a factory will fuse after an year of use is 0.2. If fusing of
a bulb is considered an failure, find the mean and variance of successes for a sample of 10
bulbs.
Answers : (27) mean = 2.1, 2 = .63 (28) mean = 8 and variance = 1.6 (29) A
PR - 267
Matrices & Determinant
As for everything else, so for a mathematical theory, beauty can be perceived but not explained..... Cayley Arthur
Introduction :
Any rectangular arrangement of numbers (real or complex) (or of real valued or complex valued
expressions) is called a matrix. If a matrix has m rows and n columns then the order of matrix is
written as m × n and we call it as order m by n
The general m × n matrix is
Notes :
(i) The elements a11, a22, a33,........ are called as diagonal elements. Their sum is called as
trace of A denoted as tr(A)
(iii) Order of a matrix : If a matrix has m rows and n columns, then we say that its order is "m by n",
written as "m × n".
1
Example # 1 : Construct a 3 × 2 matrix whose elements are given by aij = | i –3j |.
2
a11 a12
Solution : In general a 3 × 2 matrix is given by A = a21 a22 .
a31 a32
1
aij = | i – 3j |, i = 1, 2, 3 and j = 1, 2
2
1 1 5
Therefore a11 = | 1 – 3 × 1 | = 1 a12 = | 1 – 3 × 2 | =
2 2 2
1 1 1
a21 = |2–3×1|= a22 = |2–3×2|=2
2 2 2
1 1 3
a31 = |3–3×1|=0 a32 = |3–3×2|=
2 2 2
5
1 2
1
Hence the required matrix is given hy A = 2 2
3
0 2
Types of Matrices :
Row matrix :
A matrix having only one row is called as row matrix (or row vector).General form of row matrix
is A = [a11, a12, a13, ...., a1n]
MT & DT - 268
Matrices & Determinant
Column matrix :
A matrix having only one column is called as column matrix (or column vector).
a11
a 21
Column matrix is in the form A =
...
a m1
This is a matrix of order "m × 1" (or a column matrix of order m)
Zero matrix :
A = [aij]m × n is called a zero matrix, if aij = 0 i & j.
0 0 0
0 0 0
e.g. : (i) (ii) 0 0 0
0 0 0
0 0 0
Square matrix :
A matrix in which number of rows & columns are equal is called a square matrix. The general
form of a square matrix is
Diagonal matrix :
A square matrix [aij]n is said to be a diagonal matrix if aij = 0 for i j. (i.e., all the elements of
the square matrix other than diagonal elements are zero)
Note : Diagonal matrix of order n is denoted as Diag (a11, a22, ......ann).
a 0 0 0
a 0 0
0 b 0 0
0 b 0
e.g. : (i) (ii) 0 0 0 0
0 0 c
0 0 0 c
Scalar matrix :
Scalar matrix is a diagonal matrix in which all the diagonal elements are same. A = [a ij]n is a
scalar matrix, if (i) aij = 0 for i j and (ii) aij = k for i = j.
a 0 0
a 0
0 a 0
e.g. : (i) (ii)
0 a 0 0 a
1 0 0
1 0
eg. 2 = , 3 = 0 1 0 .
0 1 0 0 1
MT & DT - 269
Matrices & Determinant
Upper triangular matrix :
A = [aij]m × n is said to be upper triangular, if aij = 0 for i > j (i.e., all the elements below the
diagonal elements are zero).
a b c d a b c
e.g. : (i) 0 x y z (ii) 0 x y
0 0 u v 0 0 z
a 0 0 a 0 0 0
b c 0 b c 0 0
e.g. : (i) (ii)
x y z x y z 0
Comparable matrices : Two matrices A & B are said to be comparable, if they have the same order
(i.e., number of rows of A & B are same and also the number of columns).
2 3 4 3 4 2
e.g. : (i) A = & B= are comparable
3 1 2 0 1 3
3 0
2 3 4
4 1
e.g. : (ii) C = & D= are not comparable
3 1 2 2 3
Equality of matrices :
Two matrices A and B are said to be equal if they are comparable and all the corresponding
elements are equal.
Let A = [aij] m × n & B = [bij]p × q
A = B iff (i) m = p, n = q
(ii) aij = bij i & j.
x 3 z 4 2y – 7 0 6 3 y – 2
– 6 a – 1 0 – 6 – 3 2c 2
Example # 3 : If = , then find the values of a, b, c, x, y and z.
b – 3 – 21 0 2b 4 – 21 0
Solution : As the given matrices are equal, therefore, their corresponding elements must be equal.
Comparing the corresponding elements, we get
x+3=0 z+4=6 2y – 7 = 3y – 2
a–1=–3 0 = 2c + 2 b – 3 = 2b + 4
a = – 2, b = – 7, c = – 1, x = – 3, y = – 5, z = 2
MT & DT - 270
Matrices & Determinant
Multiplication of matrix by scalar :
Let be a scalar (real or complex number) & A = [aij]m × n be a matrix. Thus the product A is
defined as A = [bij]m × n where bij = aij i & j.
2 1 3 5 6 3 9 15
e.g. : A = 0 2 1 3 & – 3A (–3) A = 0 6 3 9
0 0 1 2 0 0 3 6
Addition of matrices :
Let A and B be two matrices of same order (i.e. comparable matrices). Then A + B is defined to
be.
A + B = [aij]m × n + [bij]m × n.
= [cij]m × n where c ij = aij + bij i & j.
1 1 1 2 0 1
e.g. : A =
2 3 , B = 2 3 ,A + B = 0 0
1 0 5 7 6 7
Substraction of matrices :
Let A & B be two matrices of same order. Then A – B is defined as A + (– B) where – B is (– 1)
B.
Properties of addition & scalar multiplication :
Consider all matrices of order m × n, whose elements are from a set F (F denote Q, R or C).
Let Mm × n (F) denote the set of all such matrices.
Then
(a) A Mm × n (F) & B Mm × n (F) A + B Mm × n(F)
(b) A+B= B+A
(c) (A + B) + C = A + (B + C)
(d) O = [o]m × n is the additive identity.
(e) For every A Mm × n(F), – A is the additive inverse.
(f) (A + B) = A + B
(g) A = A
(h) ( 1 + 2) A = 1A + 2A
8 0 2 – 2
4 – 2 and B = 4 2
Example # 4 : IF A = , then find the matrix X, such that 2A + 3X = 5B
3 6 – 5 1
Solution : We have 2A + 3X = 5B.
3X = 5B – 2A
1
X= (5B – 2A)
3
2 – 2 8 0 10 – 10 – 16 0
1 1
X= 5 4 2 – 2 4 – 2 = 20 10 – 8 4
3 3
– 5 1 3 6 – 25 5 – 6 – 12
– 10
–6 –10 –2 3
10 – 16 – 10 0 1 14
1
X = 20 – 8 10 4 = 3 12 14 = 4
3 3
– 25 – 6 5 – 12 31 –7 – 31 – 7
3 3
MT & DT - 271
Matrices & Determinant
Multiplication of matrices :
Let A and B be two matrices such that the number of columns of A is same as number of rows
of B. i.e., A = [aij]m × p & B = [bij]p × n.
p
Then AB = [c ij]m × n where cij = a
k 1
ik b kj , which is the dot product of ith row vector of A and j th
column vector of B.
0 1 1 1
1 2 3 3 4 9 1
0 0 1 0
e.g. : A = , B= , AB =
2 3 1 1 1 2 0 1 3 7 2
Notes : (1) The product AB is defined iff the number of columns of A is equal to the number of rows
of B. A is called as premultiplier & B is called as post multiplier. AB is defined
/ BA is
defined.
(2) In general AB BA, even when both the products are defined.
(3) A (BC) = (AB) C, whenever it is defined.
Notes : (1) Let A = [aij]m × n. Then An = A & m A = A, where n & m are identity matrices of order
n & m respectively.
(2) For a square matrix A, A2 denotes AA, A3 denotes AAA etc.
1 2 3
3 – 2 1
Example # 5 : If A = , then show that A3 – 23A – 40 I = O
4 2 1
1 2 3 1 2 3 19 4 8
3 – 2 1 3 – 2 1 = 1 12 8
Solution : We have A2 = A.A =
4 2 1 4 2 1 14 6 15
63 46 69 1 2 3 1 0 0
69 – 6 23 3 – 2 1 0 1 0
Now A3 – 23A – 40I = – 23 – 40
92 46 63 4 2 1 0 0 1
MT & DT - 272
Matrices & Determinant
63 – 23 – 40 46 – 46 0 69 – 69 0
69 – 69 0 – 6 46 – 40 23 – 23 0
=
90 – 92 0 46 – 46 0 63 – 23 – 40
0 0 0
0 0 0
= =O
0 0 0
cos sin
(1) If A() = , verify that A() A() = A( + ).
sin cos
Hence show that in this case A(). A() = A() . A().
4 6 1 2 4
(2) Let A = 3 0 2 , B = 0 1 and C = [3 1 2].
1 2 5 1 2
Then which of the products ABC, ACB, BAC, BCA, CAB, CBA are defined. Calculate the product
whichever is defined.
Answer (2) Only CAB is defined. CAB = [25 100]
Transpose of a matrix :
Let A =[aij]m × n. Then the transpose of A is denoted by A( or AT) and is defined as
A = [bij]n × m where bij = aji i & j.
i.e. A is obtained by rewriting all the rows of A as columns (or by rewriting all the columns of A as
rows).
1 a x
1 2 3 4
a b c d
2 b y
e.g. : A = , A = 3 c z
x y z w
4 d w
Results : (i) For any matrix A = [aij]m × n, (A) = A
(ii) Let be a scalar & A be a matrix. Then (A) = A
(iii) (A + B) = A + B & (A – B) = A – B for two comparable matrices A and B.
(iv) (A1 ± A2 ± ..... ± An) = A1 ± A2 ± ..... ± An, where Ai are comparable.
(v) Let A = [aij]m × p & B = [bij]p × n , then (AB) = BA
(vi) (A1 A2 .......An)= An. An – 1 ...........A2 . A1, provided the product is defined.
MT & DT - 273
Matrices & Determinant
a h g
e.g. A = h b f is a symmetric matrix.
g f c
o x y
B = x o z is a skew-symmetric matrix.
y z 0
Notes : (1) In a skew-symmetric matrix all the diagonal elements are zero.
( aii = – aii aii = 0)
(2) For any square matrix A, A + A is symmetric & A – A is skew-symmetric.
(3) Every square matrix can be uniqualy expressed as a sum of two square matrices of
which one is symmetric and the other is skew-symmetric.
1 1
A = B + C, where B = (A + A) & C = (A – A).
2 2
– 2
4
Example # 6 : If A = , B = [1 3 – 6], verify that (AB)' = B'A'.
5
Solution : We have
– 2
4
A = , B = [1 3 –6]
5
– 2 – 2 – 6 12
4 4 12 – 24
Then AB = [1 3 –6] =
5 5 15 – 30
1
3
Now A' = [–2 4 5], B' =
– 6
1 – 2 4 5
3 –6 12 15
B'A' = [–2 4 5] = = (AB)'
– 6 12 – 24 – 30
Clearly (AB)' = B'A'
2 – 2 – 4
–1 3 4
Example # 7 : Express the matrix B = as the sum of a symmetric and a skew symmetric
1 – 2 – 3
matrix.
2 –1 1
–2 3 – 2
Solution : Here B' =
– 4 4 – 3
3 3
4 – 3 – 3 2 –
2
–
2
3
1 1 – 3 6 2
Let P= (B + B') = = – 2 3 1
2 2 – 3 2 – 6
3
– 2 1 – 3
MT & DT - 274
Matrices & Determinant
–3 – 3
2 2 2
– 3
Now P' = 3 1 =P
2
– 3
2 1 – 3
1
Thus P= (B + B') is a symmetric matrix.
2
–1 – 5
0 – 1 – 5 0 2 2
1
1 1 1 0
6 0 3
Also, Let Q = (B – B') = = 2
2 2 5 – 6 0
5
2 –3 0
1 5
0 2 3
1
Now Q' = – 0 – 3 = – Q
2
5
– 2 3 0
1
Thus Q= (B – B') is a skew symmetric matrix.
2
– 3 – 3 –1 – 5
2 0 2 2 2 – 2 – 4
2 2 1
– 3
Now P+Q= 3 1 + 0 3 = – 1 3 4
=B
2 2 1 – 2 – 3
– 3 5
2 1 – 3 2 – 3 0
Thus, B is reresented as the sum of a symmetric and a skew symmetric matrix.
Example # 8 : Show that BAB is symmetric or skew-symmetric according as A is symmetric or skew-
symmetric (where B is any square matrix whose order is same as that of A).
Solution : Case - A is symmetric A = A
(BAB) = (B)AB = BAB BAB is symmetric.
Case - A is skew-symmetric A = – A
(BAB) = (B)AB
= B ( – A) B
= – (BAB)
BAB is skew-symmetric
Self practice problems :
(3) For any square matrix A, show that AA & AA are symmetric matrices.
(4) If A & B are symmetric matrices of same order, then show that AB + BA is symmetric and
AB – BA is skew-symmetric.
Submatrix : Let A be a given matrix. The matrix obtained by deleting some rows or columns of A is called
as submatrix of A.
a b c d
eg. A = x y z w
p q r s
a c a b c
a b d
Then x z , p q s , x y z are all submatrices of A.
p r p q r
MT & DT - 275
Matrices & Determinant
Determinant of a square matrix :
To every square matrix A = [aij] of order n, we can associate a number (real or complex) called determinant
of the square matrix.
Let A = [a]1×1 be a 1×1 matrix. Determinant A is defined as |A| = a.
e.g. A = [– 3]1×1 |A| = – 3
a b
Let A = , then |A| is defined as ad – bc.
c d
5 3
e.g. A= , |A| = 23
1 4
Minors & Cofactors :
Let be a determinant. Then minor of element aij, denoted by Mij, is defined as the determinant
of the submatrix obtained by deleting ith row & j th column of . Cofactor of element aij, denoted
by C ij, is defined as Cij = (– 1)i + j Mij.
a b
e.g. 1 =
c d
M11 = d = C 11
M12 = c, C12 = – c
M21 = b, C21 = – b
M22 = a = C 22
a b c
e.g. 2 = p q r
x y z
q r
M11 = = qz – yr = C111.
y z
a b
M23 = = ay – bx, C23 = – (ay – bx) = bx – ay etc.
x y
Determinant of any order : Let A = [aij]n be a square matrix (n > 1). Determinant of A is defined as the
sum of products of elements of any one row (or any one column) with
corresponding cofactors.
MT & DT - 276
Matrices & Determinant
Properties of determinant :
(1) |A| = |A| for any square matrix A.
i.e. the value of a determinant remains unaltered, if the rows & columns are inter changed,
a1 b1 c1 a1 a 2 a3
i.e. D = a2 b2 c 2 b1 b 2 b 3 = D
a3 b3 c3 c1 c 2 c3
(2) If any two rows (or columns) of a determinant be interchanged, the value of determinant
is changed in sign only.
a1 b1 c1 a2 b2 c2
e.g. Let D1 = a 2 b 2 c2 & D2 = a1 b1 c 1 Then D2 = – D1
a 3 b3 c3 a 3 b3 c 3
(3) Let be a scalar. Than |A| is obtained by multiplying any one row (or any one column)
of |A| by
a1 b1 c1 Ka1 Kb1 Kc1
D = a2 b2 c2 and E = a 2 b2 c2 Then E= KD
a3 b3 c3 a3 b3 c3
(4) | AB | = | A | | B |.
(5) |A| = n |A|, when A = [aij]n.
(6) A skew-symmetric matrix of odd order has deteminant value zero.
(7) If a determinant has all the elements zero in any row or column, then its value is zero,
0 0 0
i.e. D = a 2 b2 c2 = 0.
a 3 b3 c3
(8) If a determinant has any two rows (or columns) identical (or proportional), then its value
is zero,
a1 b1 c1
i.e. D = a1 b1 c 1 = 0.
a 3 b3 c 3
(9) If each element of any row (or column) can be expressed as a sum of two terms then
the determinant can be expressed as the sum of two determinants, i.e.
a1x b1y c1z a1 b1 c1 x y z
a2 b2 c2 a2 b2 c 2 a2 b2 c2
a3 b3 c3 a3 b3 c3 a3 b3 c3
(10) The value of a determinant is not altered by adding to the elements of any row (or
column) a constant multiple of the corresponding elements of any other row (or column),
a1 b1 c1 a1 ma 2 b1 mb 2 c 1 mc 2
i.e. D1 = a 2 b2 c2 and D2 = a2 b2 c2 . Then D2= D1
a3 b3 c3 a 3 na1 b3 nb1 c 3 nc 1
(11) Let A = [aij]n. The sum of the products of elements of any row with corresponding
cofactors of any other row is zero. (Similarly the sum of the products of elements of
any column with corresponding cofactors of any other column is zero).
MT & DT - 277
Matrices & Determinant
a b c
Example # 9 Simplify b c a
c a b
Solution : Let R1 R1 + R2 + R3
0 0 1
= (a + b + c) b c c a a
c a a b b
= (a + b + c) ((b – c) (a – b) – (c – a)2)
= (a + b + c) (ab + bc – ca – b2 – c 2 + 2ca – a2)
= (a + b + c) (ab + bc + ca – a2 – b2 – c 2) 3abc – a3 – b3 – c3
a b c
Example # 10 Simplify a2 b2 c2
bc ca ab
Solution : Given detereminant is equal to
a2 b2 c2 a2 b2 c2
1 abc
= a3 b3 c3 = a3 b3 c3
abc abc
abc abc abc 1 1 1
Apply C1 C1 – C2, C2 C2 – C3
a2 b2 b2 c 2 c2
= a3 b 3 b3 c 3 c3
0 0 1
ab bc c2
2 2
= (a – b) (b – c) a ab b b 2 bc c 2 c3
0 0 1
= (a – b) (b – c) [ab + abc + ac + b + b C + bc 2 – a2b – a2c – ab2 – abc – b3 – b2c]
2 2 3 2
ab c 2a 2a
2b bc a 2b
(7) Prove that = (a + b + c) 3.
2c 2c c ab
MT & DT - 278
Matrices & Determinant
1 a bc
(8) Show that 1 b ca = (a – b) (b – c) (c – a) by using factor theorem .
1 c ab
Answers : (5) 0 (6) 0
Application of determinants : Following examples of short hand writing large expressions are:
(i) Area of a triangle whose vertices are (x r, yr); r = 1, 2, 3 is:
x1 y1 1
1 x2 y2 1
D= If D = 0 then the three points are collinear.
2 x y 1
3 3
x y 1
(ii) Equation of a straight line passing through (x 1, y1) & (x 2, y2) is x1 y1 1 = 0
x2 y2 1
(iii) The lines: a1x + b1y + c1 = 0........ (1)
a2x + b2y + c2 = 0........ (2)
a3x + b3y + c3 = 0........ (3)
a1 b1 c1
are concurrent if, a2 b2 c 2 = 0.
a3 b3 c3
Condition for the consistency of three simultaneous linear equations in 2 variables.
(iv) ax² + 2 hxy + by² + 2 gx + 2 fy + c = 0 represents a pair of straight lines if:
a h g
abc + 2 fgh af² bg² ch² = 0 = h b f
g f c
Singular & non singular matrix : A square matrix A is said to be singular or non-singular according
as |A| is zero or non-zero respectively.
Cofactor matrix & adjoint matrix : Let A = [aij] n be a square matrix. The matrix obtained by
replacing each element of A by corresponding cofactor is called as
cofactor matrix of A, denoted as cofactor A. The transpose of cofactor
matrix of A is called as adjoint of A, denoted as adj A.
i.e. if A = [aij]n
then cofactor A = [c ij]n when c ij is the cofactor of aij i & j.
Adj A = [dij]n where dij = c ji i & j.
Properties of cofactor A and adj A:
(a) A . adj A = |A| n = (adj A) A where A = [aij]n.
(b) |adj A| = |A|n – 1, where n is order of A.
In particular, for 3 × 3 matrix, |adj A| = |A|2
(c) If A is a symmetric matrix, then adj A are also symmetric
matrices.
(d) If A is singular, then adj A is also singular.
Example # 11 : For a 3×3 skew-symmetric matrix A, show that adj A is a symmetric matrix.
0 a b c2 bc ca
2
Solution : A = a 0 c cof A = bc b ab
b c 0 ca ab a 2
c2 bc ca
adj A = (cof A) = bc b 2 ab which is symmetric.
ca ab a 2
MT & DT - 279
Matrices & Determinant
Inverse of a matrix (reciprocal matrix) :
1
Let A be a non-singular matrix. Then the matrix adj A is the
|A|
multiplicative inverse of A (we call it inverse of A) and is denoted by A–1.
We have A (adj A) = |A| n = (adj A) A
1 1
A adj A = n = adj A A, for A is non-singular
| A | | A |
1
A–1 = adj A.
|A|
Remarks :
1. The necessary and sufficient condition for existence of inverse of A is that A is non-singular.
2. A–1 is always non-singular.
3. If A = dia (a11, a22, ....., ann) where aii 0 i, then A–1 = diag (a11– 1, a22–1, ...., ann–1).
4. (A–1) = (A)–1 for any non-singular matrix A. Also adj (A) = (adj A).
5. (A–1)–1 = A if A is non-singular.
1 –1
6. Let k be a non-zero scalar & A be a non-singular matrix. Then (kA)–1 = A .
k
1
7. |A–1| = | A | for |A| 0.
8. Let A be a non-singular matrix. Then AB = AC B = C & BA = CA B= C.
9. A is non-singular and symmetric A–1 is symmetric.
10. (AB)–1 = B–1 A–1 if A and B are non- singular.
11. In general AB = 0 does not imply A = 0 or B = 0. But if A is non-singular and AB = 0, then B = 0.
Similarly B is non-singular and AB = 0 A = 0. Therefore, AB = 0 either both are singular or one of
them is 0.
1 3 3
1 4 3
Example # 12 : If A = , then verify that A adj A = | A | . Also find A–1
1 3 4
Solution : We have | A | = 1 (16 – 9) – 3 (4 – 3) + 3 (3 – 4) = 1 0
Now C11 = 7, C12 = – 1, C13 = – 1, C21 = – 3, C22 = 1, C23 = 0,C 31 = – 3, C32 = 0, C33 = 1
7 – 3 – 3
–1 1 0
Therefore adj A =
– 1 0 1
1 3 3 7 – 3 – 3 7 – 3 – 3 – 3 3 0 – 3 0 3
1 4 3 – 1 1 0 7 – 4 – 3 – 3 4 0 – 3 0 3
Now A(adj A) = =
1 3 4 – 1 0 1 7 – 3 – 4 – 3 3 0 – 3 0 4
1 0 0 1 0 0
0 1 0 0 1 0
= = (1) = |A|. I
0 0 1 0 0 1
7 – 3 – 3 7 – 3 – 3
1 1
– 1 1 0 = – 1 1 0
Also A–1 = adj A =
|A| 1 – 1 0 1 – 1 0 1
MT & DT - 280
Matrices & Determinant
2 3
Example # 13 : Show that the matrix A = satisfies the equation A2 – 4A + I = O, where I is 2 × 2 identity
1 2
matrix and O is 2 × 2 zero matrix. Using the equation, find A–1 .
2 3 2 3 7 12
Solution : We have A2 = A.A = =
1 2 1 2 4 7
7 12 8 12 1 0 0 0
Hence A2 – 4A + I = – + = =0
4 7 4 8 0 1 0 0
Now A2 – 4A + I = 0
Therefore A A – 4A = – I
or AA(A–1) – 4 A A–1 = – I A–1 (Post multiplying by A–1 because |A| 0)
or A (A A–1) – 4I = – A–1 or AI – 4I = – A–1
4 0 2 3 2 – 3
or A–1 = 4I – A = – =
0 4 1 2 – 1 2
2 – 3
Hence A–1 =
– 1 2
Example # 14 : For two non-singular matrices A & B, show that adj (AB) = (adj B) (adj A)
Solution : We have (AB) (adj (AB)) = |AB| n
= |A| |B| n
A (AB)(adj (AB)) = |A| |B| A–1
–1
1
B adj (AB) = |B| adj A ( A–1 = | A | adj A)
MT & DT - 281
Matrices & Determinant
(ii) Using column transformations :
If A is a matrix such that A–1 exists, then to find A–1 using elementary column operations,
Step I : Write A = AI and
Step II : Apply a sequence of column operations on A = AI till we get, I = AB.
The matrix B will be inverse of A.
Note : In order to apply a sequence of elementary column operations on the matrix equation X = AB, we
will apply these row operatdions simultaneously on X and on the second matrix B of the product AB on
RHS.
0 1 2
1 2 3
Example # 15 : Obtain the inverse of the matrix A = using elementary operations.
3 1 1
0 1 2 1 0 0
1 2 3 = 0 1 0 A
Solution : Write A = IA, i.e.,
3 1 1 0 0 1
1 2 3 0 1 0
or 0 1 2 = 1 0 0 A (applying R1 R2)
3 1 1 0 0 1
1 2 3 0 1 0
or 0 1 2
=
1 0 0
A (applying R3 R3 – 3R1)
0 – 5 – 8 0 – 3 1
1 0 – 1 – 2 1 0
or 0 1 2
=
1 0 0
A (applying R1 R1 – 2R2)
0 – 5 – 8 0 – 3 1
1 0 – 1 – 2 1 0
0 1 2 = 1 0 0
or A (applying R 3 R3 + 5R2)
0 0 2 5 – 3 1
1 0 – 1 – 2 1 0
or 0 1 2 = 1 0 0 A (applying R 1 R )
0 0 1 3
2 3
5 –3 1
2 2 2
1 1 1
2 –
1 0 0 2 2
or 0 1 2 = 1 0 0
A (Applying R1 R1 + R3)
0 0 1 5 3 1
2 –
2 2
1 –1 1
1 0 0 2 2 2
or 0 1 0 = – 4 3 – 1 A (Applying R R – 2R )
0 0 1 2 2 3
5 –3 1
2 2 2
MT & DT - 282
Matrices & Determinant
1 1 1
2 –2 2
Hence A–1 = – 4 3 – 1
5 3 1
2 –2 2
b1
a11 a12 .......... a1n x1
a 21 a 22 .......... a 2n
x2
b 2
Let A = ,X= & B = ... .
..... ..... .......... ..... ....
...
a m1 a m2 .......... a mn x
n b
n
Then the above system can be expressed in the matrix form as AX = B.
The system is said to be consistent if it has atleast one solution.
Results : (1) If A is non-singular, the system has only the trivial solution (zero solution) X = 0
(2) If A is singular, then the system has infinitely many solutions (including the trivial
solution) and hence it has non-trivial solutions.
xyz6
Example # 16 : Solve the system x y z 2 using matrix inverse.
2x y z 1
1 1 1 x 6
Solution : Let A = 1 1 1 , X = y & B = 2 .
2 1 1 z 1
Then the system is AX = B.
|A| = 6. Hence A is non singular.
0 3 3
Cofactor A = 2 3 1
2 0 2
MT & DT - 283
Matrices & Determinant
0 2 2
adj A = 3 3 0
3 1 2
0 2 2 0 1/ 3 1/ 3
1 1
A–1 = | A | adj A = 3 3 0 = 1 / 2 1 / 2 0
6
3 1 2 1/ 2 1/ 6 1/ 3
0 1 2
(13) A = 1 2 3 . Find the inverse of A using |A| and adj A.
3 1 1
(14) Find real values of and µ so that the following systems has
(i) unique solution (ii) infinitely many solutions (iii) No solution.
x+y+z=6
x + 2y + 3z = 1
x + 2y + z = µ
(15) Find so that the following homogeneous system have a non zero solution
x + 2y + 3z = x
3x + y + 2z = y
2x + 3y + z = z
1 1 1
2 2 2
Answers : (13) 4 3 1 (14) (i) 3, µ R (ii) = 3, µ = 1 (iii) = 3, µ 1 (15) = 6
5 3 1
2 2 2
MT & DT - 284
Vector
What if angry vectors veer Round your sleeping head, and from. There's never need to fear Violence of the poor world's abstract storm.
........Warren,RobertPenn
Vector quantities are specified by definite magnitude and definite direction. A vector is generally
represented by a directed line segment, say AB . A is called the initial point and B is called the
terminal point. The magnitude of vector AB is expressed by AB .
Zero vector:
A vector of zero magnitude i.e. which has the same initial and terminal point, is called a zero vector.
It is denoted by O. The direction of zero vector is indeterminate.
Unit vector:
A vector of unit magnitude in the direction of a vector a is called unit vector along a and is denoted by
a
â , symbolically â .
|a|
2 2 2
then |a| = ax ay az
|a| = 14
a 1 2 3
â = | a | = – ĵ +
14 î 14 14
k̂
Equal vectors:
Two vectors are said to be equal if they have the same magnitude, direction and represent the same
physical quantity.
Collinear vectors:
Two vectors are said to be collinear if their directed line segments are parallel irrespective of their
directions. Collinear vectors are also called parallel vectors. If they have the same direction they are
named as like vectors otherwise unlike vectors.
Symbolically, two non-zero vectors a and b are collinear if and only if, a b , where R
a1 a a
= 2 = 3 ( =)
b1 b2 b3
a1 a2 a3
Vectors a = a1 î + a 2 ĵ + a 3k̂ and b = b1 î + b 2 ĵ + b 3k̂ are collinear if = =
b1 b2 b3
Coplanar vectors:
A given number of vectors are called coplanar if their line segments are all parallel to the same plane.
Note that “two vectors are always coplanar”.
(iii) AD + EB + PC = 4 AB
(2) The vector î ĵ k̂ bisects the angle between the vectors c and 3 î 4 ĵ . Determine the unit
vector along c .
(3) The sum of the two unit vectors is a unit vector. Show that the magnitude of the their difference
is 3 .
1 2 14
Answers : (2) î ĵ k̂
3 15 15
Addition of vectors :
(i) If two vectors a and b are represented by OA and OB , then their sum a b is a vector
Example # 3 : If a î 2 ĵ 3k̂ and b 2 î 4 ĵ 5k̂ represent two adjacent sides of a parallelogram, find unit
vectors parallel to the diagonals of the parallelogram.
Solution : Let ABCD be a parallelogram such that AB = a and BC = b .
Then, AB + BC = AC
AC = a b = 3 î 6 ĵ 2k̂
| AC | = 9 36 4 = 7
AB + BD = AD
BD = AD AB = b a = î 2 ĵ 8k̂
| BD | = 1 4 64 = 69
AC
Unit vector along AC =
| AC |
=
1
7
3 î 6 ĵ 2k̂
BD 1
and Unit vector along BD =
| BD |
=
69
î 2 ĵ 8k̂
Example # 4 : ABCDE is a pentagon. Prove that the resultant of the forces AB , AE , BC , DC , ED and AC
is 3 AC .
Solution : Let R be the resultant force
R = AB + AE + BC + DC + ED + AC
R = ( AB + BC ) + ( AE + ED + DC ) + AC
R = AC + AC + AC
R = 3 AC . Hence proved.
SECTION FORMULA
If a and b are the position vectors of two points A and B, then the p.v. of
RESONANCE VECTOR - 287
na m b
a point which divides AB in the ratio m: n is given by r .
mn
ab
Note : Position vector of mid point of AB = .
2
Example # 5 : ABCD is a parallelogram. If L, M be the middle point of BC and CD, express AL and AM in
3
terms of AB and AD . Also show that AL + AM = AC .
2
Solution : Let the position vectors of points B and D be respectively b and d referred to A as origin of
reference.
Then AC = AD + DC = AD + AB [ DC = AB ]
AC = d + b AB = b , AD = d
i.e. position vector of C referred to A is d + b
=
1
2
[p.v. of B + p.v. of C] =
2
1
b d b = AB +
1
2 AD
Similarly
1
AM = 2 d d b = AD + 21 AB
1 1
AL + AM = b + d + d+ b
2 2
3 3 3 3
= b + d = (b + d ) = AC .
2 2 2 2
Example # 6 : If ABCD is a parallelogram and E is the mid point of AB. Show by vector method that DE
trisect AC and is trisected by AC.
Solution : Let AB = a and AD = b
Then BC = AD = b and AC = AB + AD = a + b
Also let K be a point on AC, such that AK : AC = 1 : 3
1
AK = AC
3
1
AK = (a + b ) .........(i)
3
1
Again E being the mid point of AB, we have AE = a
2
Let M be the point on DE such that DM : ME = 2 : 1
AD 2AE ba
AM = = ..........(ii)
1 2 3
From (i) and (ii) we find that
1
AK = ( a + b ) = AM , and so we conclude that K and M coincide. i.e. DE trisect AC and is
3
trisected by AC. Hence proved.
(5) If a, b are position vectors of the points (1, –1), (–2, m), find the value of m for which a and b
are collinear.
(7) The vertices P, Q and S of a PQS have position vectors p, q and s respectively..
(i) If M is the mid point of PQ, then find position vector of M in terms of p and q
(ii) Find t , the position vector of T on SM such that ST : TM = 2 : 1, in terms of p, q
and s .
(iii) If the parallelogram PQRS is now completed. Express r , the position vector of the
point R in terms of p, q and s
(8) D, E, F are the mid-points of the sides BC, CA, AB respectively of a triangle.
1
Show FE = BC and that the sum of the vectors AD , BE , CF is zero.
2
(9) The median AD of a ABC is bisected at E and BE is produced to meet the side AC
1 1
in F. Show that AF = AC and EF = BF..
3 4
(10) Point L, M, N divide the sides BC, CA, AB of ABC in the ratios 1 : 4, 3 : 2, 3 : 7 respectively.
1 1 1
(6) 1:2 (7) m = (p q) , t = (p q s ) , r = (q p s)
2 2 2
Hence bisector of the angle between the two vectors a and b is a b , where R+. Bisector
of the exterior angle between a and b is a b , R+.
Note that the equations of the bisectors of the angles between the lines r = a + b and r = a + c
are :
r = a + t b c and r = a + p c b .
Scalar product (Dot Product) of two vectors :
a.b
(iv) The angle between a and b is given by cos , 0
|a| |b|
(v) a . b a b cos , (0 )
note that if is acute, then a . b > 0 and if is obtuse, then a . b < 0
(vi) a b = | a |2 | b |2 2 | a || b | cos , where is the angle between the vectors
2
(vii) a . a a a2 (viii) a . b b . a (commutative)
Note:
(a) Maximum value of a . b is a b
(b) Minimum value of a . b is – a b
Example # 7 : Find the value of p for which the vectors a 3 î 2 ĵ 9k̂ and b î p ĵ 3k̂ are
(i) perpendicular (ii) parallel
Solution : (i)
ab
a.b = 0 3 î 2 ĵ 9k̂ . î p ĵ 3k̂ = 0
3 + 2p + 27 = 0 p = – 15
(ii) vectors a = 3 î 2 ĵ 9k̂ and b = î p ĵ 3k̂ are parallel iff
3 2 9 2 2
= = 3= p=
1 p 3 p 3
Example # 8 : If a + b + c = 0 , | a | = 3, | b | = 5 and | c | = 7, find the angle between a and b .
Solution : We have, a b c 0
ab = –c
a b . a b = c . c
2 2
2 2
ab = | c |2 a + b + 2a . b = c
2
2 2
a + b +2 a b cos = c
1
9 + 25 + 2 (3) (5) cos = 49 cos = = .
2 3
Example # 9 : Find the values of x for which the angle between the vectors a = 2x 2 î + 4x ĵ + k̂ and b = 7 î
– 2 ĵ + x k̂ is obtuse.
a.b
Solution : The angle between vectors a and b is given by cos =
|a||b|
a.b
Now, is obtuse cos < 0 <0
|a||b|
a.b < 0 [ | a |, | b | 0 ]
14x2 – 8x + x < 0 7x (2x – 1) < 0
1
x(2x – 1) < 0 0<x<
2
Hence, the angle between the given vectors is obtuse if x (0, 1/2)
Solution : We have AB = AD + DB
AB2 = ( AD DB )2
Also we have AC = AD + DC
2
AC2 = ( AD DC)
Adding (i) and (ii), we get AB2 + AC2 = 2AD2 + 2BD2 + 2 AD . (DB DC )
AB2 + AC 2 = 2(AD 2 + BD2) DB + DC = 0
Example # 11 : If a = î + ĵ + k̂ and b = 2 î – ĵ + 3 k̂ , then find
(i) Component of b along a . (ii) Component of b in plane of a & b
but to a .
a.b
Solution : (i) Component of b along a is 2 a
| a |
Here a . b = 2 – 1 + 3 = 4
| a |2 = 3
a.b 4 4
Hence 2 a = a = ( î + ĵ + k̂ )
|
a | 3 3
a.b 1
(ii) Component of b in plane of a & b but to a is b – 2 a . = 2 î 7 ĵ 5k̂
3
| a |
(13) Let a = x 2 î 2 ĵ 2k̂ , b = î ĵ k̂ and c = x 2 î 5 ĵ 4k̂ be three vectors. Find the values of x
for which the angle between a and b is acute and the angle between b and c is obtuse.
(14) The points O, A, B, C, D are such that OA a , OB b , OC 2a 3b , OD a 2b .
Given that the length of OA is three times the length of OB . Show that BD and AC are
perpendicular.
(15) ABCD is a tetrahedron and G is the centroid of the base BCD. Prove that
AB2 + AC2 + AD2 = GB2 + GC2 + GD2 + 3GA2
(iii) î î ĵ ĵ k̂ k̂ 0 ; î ĵ k̂, ĵ k̂ î, k̂ î ĵ
î ĵ k̂
(iv) If a = a1 î +a2 ĵ + a3 k̂ and b = b1 î + b2 ĵ + b3 k̂ , then a b a1 a 2 a3
b1 b 2
3 b
(v) a x b b x a (not commutative)
(vi) ( )
(m a ) b = a m b = m a b (associative), where m is a scalar..
( )
(vii) a x ( b c ) (a x b ) (a x c ) (distributive)
(viii) a b 0 a and b are parallel (collinear) (a 0 , b 0 ) i.e. a K b , where K is a scalar..
ab
(ix) Unit vector perpendicular to the plane of a and b is n̂ =
| ab |
r (a b )
(x) A vector of magnitude ‘r’ and perpendicular to the plane of a and b is
| ab |
axb
(xi) If is the angle between a and b, then sin
a b
(xii) If a, b and c are the position vectors of 3 points A, B and C respectively, then the vector area of
1
ABC = a x b bx c cx a . The points A, B and C are collinear if a x b b x c c x a 0
2
1
(xiii) Area of any quadrilateral whose diagonal vectors are d1 and d2 is given by d1 x d2
2
2
2 2 a.a a.b
(xiv) Lagrange's Identity : For any two vectors a and b ; (a x b) a b (a . b ) 2
a.b b.b
Example # 12 : Find a vector of magnitude 9, which is perpendicular to both the vectors 4 î – ĵ 3k̂
and 2 î ĵ 2k̂ .
RESONANCE VECTOR - 293
Solution : Let a = 4 î ĵ 3k̂ and b = 2 î ĵ 2k̂ . Then
î ĵ k̂
4 1 3
ab = = (2 – 3) î – (–8 + 6) ĵ + (4 – 2) k̂ = î 2 ĵ 2k̂
2 1 2
| ab | = ( 1)2 2 2 2 2 = 3
ab 9
Required vector = 9 =
( î 2 ĵ 2k̂ ) = ± ( 3 î 6 ĵ 6k̂ )
| a b | 3
Example # 13 : For any three vectors a, b, c , show that a (b c ) b ( c a ) c ( a b ) 0 .
Solution : We have, a × (b c ) + b × (c a ) + c × (a b )
= ab ac bc ba c a c b [Using distributive law]
= ab ac bc ab ac bc = 0 [ b a – a b etc]
Example # 14 : For any vector a , prove that | a î |2 + | a ĵ |2 + | a k̂ |2 = 2 | a |2
Solution : Let a = a1î a 2 ĵ a 3 k̂ . Then
a î = (a1î a 2 ĵ a 3k̂ ) × î = a1 ( î î ) + a2 ( ĵ î ) + a3 (k̂ î ) = –a2 k̂ a 3 ĵ
| a î |2 = a22 + a32
a ĵ = (a1î a 2 ĵ a 3k̂ ) × ĵ = a1k̂ a 3 î
| a ĵ |2 = a21 + a32
a k̂ (a i î a 2 ĵ a 3 k̂ ) × k = – a i ĵ a 2 î
| a k̂ |2 = a12 + a22
| a î |2 + | a ĵ |2 + | a k̂ |2 = a22 + a33 + a12 + a32 + a12 + a22
= 2 (a12 + a22 + a32) = 2 | a |2
Example # 15 : Let OA = a , OB = 10 a + 2b and OC = b where O is origin. Let p denote the area of the
quadrilateral OABC and q denote the area of the parallelogram with OA and OC as adjacent
sides. Prove that p = 6q.
Solution : We have,
p = Area of the quadrilateral OABC
1 1
p= | OB AC | = | OB (OC OA ) |
2 2
1 1
p= | (10a 2b) (b a) | = | 10(a b) 10(a a) 2(b b) 2(b a) |
2 2
1
p= | 10(a b) 0 0 2(a b) | = 6 | a b | .....(i)
2
and q = Area of the parallelogram with OA and OC as adjacent sides
q = | OA OC | = | a b | ........(ii)
From (i) and (ii), we get p = 6q
RESONANCE VECTOR - 294
Self Practice Problems :
(16) If p and q are unit vectors forming an angle of 30º. Find the area of the parallelogram having
a p 2q and b 2p q as its diagonals.
(17) Prove that the normal to the plane containing the three points whose position vectors are
a, b, c lies in the direction b c c a a b
(18) ABC is a triangle and EF is any straight line parallel to BC meeting AC, AB in E, F respectively.
If BR and CQ be drawn parallel to AC, AB respectively to meet EF in R and Q respectively,
prove that ARB = ACQ.
Let LM be the shortest distance vector between the lines L1 and L2 . Then LM is perpendicular to both
p and q i.e. LM is parallel to p × q . Therefore the magnitude of the shortest distance vector (i.e.
| LM |) would be equal to that of the projection of AB along the direction of the line of shortest distance.
| LM | Pr ojection of AB on LM = Pr ojection of AB on p x q
AB . (p x q) (b a) . (p x q)
=
pxq pxq
(i) The two lines directed along p and q will intersect only if shortest distance = 0
i.e. (b a) . (p x q) 0 i.e. b a lies in the plane containing p and q . b a · p q 0 .
(ii) If two parallel lines are given by r1 a1 Kb and r2 a 2 Kb , then distance (d) between them is
b x (a 2 a1 )
given by d
b
where is the angle between a , b (i.e. ) and is the angle between a x b and c
(i.e. = ) . It is (i.e. a b . c ) also written as a b c and spelled as box product.
a1 a 2 a3
(v) If a = a1 î + a2 ĵ + a3 k̂ ; b = b1 î +b2 ĵ +b3 k̂ and c = c 1 î + c 2 ĵ + c 3 k̂ , then [ a b c ] b1 b 2 b 3 .
c1 c 2 c 3
In general, if a a1 a 2 m a 3n ; b b 1 b 2 m b 3 n and c c 1 c 2 m c 3n
a1 a 2 a3
then a b c b1 b 2
b 3 m n , where , m and n are non-coplanar vectors.
c1 c 2 c3
(vi) If a, b and c are coplanar [ a b c ] 0 .
(vii) Scalar product of three vectors, two of which are equal or parallel is 0 [ a b c]0 ,
(viii) If a , b , c are non-coplanar, then [a b c] 0 for right handed system and [a b c] 0 for left handed
system.
(ix) [ î ĵ k̂ ] = 1 (x) [Ka b c ] K [ a b c ] (xi) [(a b) c d ] [ a c d ] [ b c d ]
a b
(xii) b c c a = 0 and a b b c ca = 2 a b c .
a.a a.b a.c
2
(xiii) a b c = b.a b .b b.c
c .a c .b c .c
(a) The volume of the tetrahedron OABC with O as origin and the position vectors of A, B and C being
a, b and c respectively is given by V 1 a b c
6
(b) If the position vectors of the vertices of tetrahedron are a, b, c and d , then the position vector of its
1
centroid is given by (a b c d) .
4
note that this is also the point of concurrency of the lines joining the vertices to the centroids of the
opposite faces and is also called the centre of the tetrahedron. In case the tetrahedron is regular it is
equidistant from the vertices and the four faces of the tetrahedron.
Example # 16 : Find the volume of a parallelopiped whose sides are given by 3 î 7 ĵ 5k̂ , 5 î 7 ĵ 3k̂
and 7 î 5 ĵ 3k̂
Solution : Let a 3 î 7 ĵ 5k̂ , b 5 î 7 ĵ – 3k̂ and c 7 î 5 ĵ 3k̂ .
We know that the volume of a parallelopiped whose three adjacent edges are a, b, c is
[a b c ] .
Example # 17 : Simplify [ a b bc c a]
Solution : We have :
[ a b b c c a ] = {(a b) (b c )} . (c a ) [By definition]
= (a b a c b b b c ) . ( c a ) [By distribution law]
= (a b c a b c ) . ( c a ) [ bb 0 ]
= (a b) . c – (a b) . a + (c a) . c – (c a) . a + (b c ) . c – (b c ) . a [By distribution law]
= [a b c ] – [a b a] + [ c a c ] – [ c a a ] + [ b c c ] – [ b c a ]
= [a b c ] – [ b c a ] [ When any two vectors are equal, scalar triple product is zero ]
= [a b c ] – [a b c ] = 0 [ [ b c a ] = [a b c ] ]
Example # 18 : Find the volume of the tetrahedron whose four vertices have position vectors a , b , c and d .
Solution : Let four vertices be A, B, C, D with position vectors a , b , c and d respectively..
DA = (a – d)
DB = (b – d)
DC = (c – d)
1
Hence volume V = [a – d b – d c – d]
6
1
= ( a – d ) . [( b – d ) × ( c – d )]
6
1
= (a – d) . [b × c – b × d + c × d]
6
1
= {[ a b c ] – [ a b d ] + [ a c d ] – [ d b c ]}
6
1
= {[ a b c ] – [ a b d ] + [ a c d ] – [ b c d ]}
6
Example # 19 : Show that the vectors a 2 î 4 ĵ 2 k̂ , b 4 î 2 ĵ 2 k̂ and c 2 î 2 ĵ 4 k̂ are coplanar..
2 4 2
Solution : [a b c ] = 4 2 2 = – 2(–8 – 4) – 4(16 – 4) – 2(–8 – 4)
2 2 4
= 24 – 48 + 24 = 0
So vectors a , b , c are coplanar
(19) Show that {( a + b + c ) × ( c – b )} . a = 2 a b c .
(20) Show that a . (b c ) (a b c ) 0
(21) One vertex of a parallelopiped is at the point A (1, –1, –2) in the rectangular cartesian co-ordinate. If
three adjacent vertices are at B(–1, 0, 2), C(2, –2, 3) and D(4, 2, 1), then find the volume of the
parallelopiped.
(22) Find the value of m such that the vectors 2 î ĵ k̂ , î 2 ĵ 3k̂ and 3 î m ĵ 5k̂ are coplanar..
(23) Show that the vector a, b, c are coplanar if and only if b c , c a , a b are coplanar..
Answers : (21) 72 (22) –4
Example # 20 : For any vector a , prove that î (a î ) ĵ (a ĵ ) k̂ (a k ) = 2a
Solution : Let a a1î a 2 ĵ a 3 k̂ .
Then î (a î ) ĵ (â ĵ ) k̂ (a k̂ )
= { ( î . î ) a ( î . a) î } + {( ĵ . ĵ ) a ( ĵ . a) ĵ} + {(k̂ . k̂ ) a (k̂ . a) k̂ }
= {(a ( î . a) î } {a ( ĵ . a) ĵ} + {a (k̂ . a) k̂ }
= 3a {( î . a) î ( ĵ . a) ĵ (k̂ . a) k̂ = 3a (a1î a 2 ĵ a 3k̂ ) = 3a a 2a
Example # 21 :
Prove that a {b (c d)} = (b . d)(a c ) – (b. c ) (a d)
Solution : We have, a {b (c d)} = a {(b . d) c (b . c ) d}
= a {(b . d) c } a {(b . c ) d} [by dist. law]
Example # 22 : Let a = î + 2 ĵ – 3k̂ , b = î + 2 ĵ – 2k̂ and c = 2 î – ĵ + k̂ . Find the value(s) of , if any,,
such that
ab bc
× ca = 0 .
Solution :
ab bc
× ca = a b c b × ca
= a b c
a.b c b.c a
which vanishes if (i) a . b c = b . c a (ii) a b c = 0
(i)
a . b c = b . c a leads to the equation 2 3 + 10 + 12 = 0, 2 + 6 = 0 and 6 – 6 = 0,
which do not have a common solution.
(ii)
a b c =0
2 3
1 2 2 2
=0 3 = 2 =
2 1 3
ab a
Example # 23 : If A B = a , A . a = 1 and A B = b , then prove that A = and
| a |2
b a a | a |2 1
B = .
| a |2
Solution : Given A B a .....(i)
a. A B = a.a
a . A a .B = a . a
2
1 + a.B = |a|
a . B = | a |2 – 1 ...........(ii)
Given A B b
a A B = a × b
a .B A – a . A B ab
| a |2 1 A B = a b ...........(iii) [Using equation (ii)]
solving equation (i) and (iii) simultaneously, we get
A =
ab a
and B =
b a a | a |2 1
| a |2 | a |2
Example # 24 : Solve for r satisfying the simultaneous equations r b c b , r . a 0 provided a is not
perpendicular to b .
Solution : (r c) × b = 0 r c and b are collinear
r c kb
Example # 25 : If x a k x b , where k is a scalar and a, b are any two vectors, then determine x in terms
of a, b and k.
Solution : x a k x b ..........(i)
Premultiply the given equation vectorially by a
a ( x a ) + k (a x ) = a b
(a . a) x (a . x ) a k(a x ) a b ..........(ii)
Premultiply (i) scalarly by a
[a x a] + k ( a . x ) = a . b
k(a . x ) a . b .......(iii)
Substituting x a from (i) and a . x from (iii) in (ii) we get
1 (a . b)
x = a2 k 2 kb (a b) a
k
(25) Find the unit vector coplanar with î + ĵ + 2 k̂ and î + 2 ĵ + k̂ and perpendicular to î + ĵ + k̂ .
(26) Prove that a {a (a b)} (a . a) (b a) .
1 1
(27) Given that x (p . x ) p q , show that p . x p . q and find x in terms of p and q .
2 2
p
(28) If x . a = 0, x . b = 0 and x . c = 0 for some non-zero vector x , then show that [a b c ] = 0
( r . a ) ( b c ) ( r . b ) ( c a ) ( r . c ) (a b )
(29) Prove that r = + +
[a b c ] [a b c ] [a b c ]
where a, b, c are three non-coplanar vectors
1 p.q
Answers : (25) ± ( – ĵ + k̂ ) and x = q – 2 p
2 2|p|
Linear combinations :
Given a finite set of vectors a , b , c ,...... , then the vector r xa yb zc ........ is called a linear
(b) Fundamental Theorem in plane : Let a , b be non zero, non collinear vectors, then any vector r
coplanar
with a , b can be expressed uniquely as a linear combination of a and b
i.e. there exist some unique x, y R such that xa yb r .
(c) If a , b , c are nonzero, noncoplanar vectors, then
xa yb zc x' a y' b z' c x x' , y y' , z z'
(d) Fundamental theorem in space: Let a , b , c be nonzero, noncoplanar vectors in space.
Then any vector r can be uniquely expressed as a linear combination of a , b , c i.e. there exist
some unique x,y, z R such that xa yb zc r .
(e) If x 1,x 2 ,......, x n are n non zero vectors and k 1 ,k 2 ,.....,k n are n scalars and if the linear
com bination k 1x1 k 2 x 2 ....... k n x n 0 k 1 0, k 2 0 ,....., k n 0 , then we say that vectors
x1, x 2, ......, x n are linearly independent vectors.
(f) If x 1,x 2 ,......, x n are not linearly independent then they are said to be linearly dependent vectors.
i.e. if k 1x1 k 2 x 2 k 3 x 3 ...... k r x r ...... k n x n 0 and if there exists at least one k r 0, then
x1, x 2, ......, x n are said to be linearly dependent vectors.
Note 1: If k r 0; k 1x1 k 2 x 2 k 3 x 3 ...... k r x r ...... k n x n 0
k r x r k 1x1 k 2 x 2 ....... k r 1 . x r 1 k r 1 . x r 1 ...... k n x n
1 1 1 1 1
k r x r k1 x1 k 2 x 2 ..... k r 1 . x r 1 ..... k n xn
kr kr kr kr kr
x r c 1x 1 c 2 x 2 ...... c r –1x r –1 + c r 1x r 1 ....... c n x n
i.e. x r is expressed as a linear combination of vectors x1, x 2 ,........, x r 1, x r 1, .......... , x n
Hence x r with x1, x 2 , ........, x r 1, x r 1, ......., x n forms a linearly dependent set of vectors.
Note 2:
If a = 3 î + 2 ĵ + 5 k̂ then a is expressed as a Linear Combination of vectors î , ĵ , k̂ . Also a , î , ĵ ,
k̂ form a linearly dependent set of vectors. In general, in 3 dimensional space every set of four vectors
is a linearly dependent system.
Example # 27 : Prove that the vectors 5 a + 6 b + 7 c , 7 a – 8 b + 9 c and 3 a + 20 b + 5 c are linearly
dependent, where a , b , c being linearly independent vectors.
Solution : We know that if these vectors are linearly dependent , then we can express one of them as a
linear combination of the other two.
Now let us assume that the given vector are coplanar, then we can write
5 a + 6 b + 7 c = ( 7 a – 8 b + 9 c ) + m (3 a + 20 b + 5 c )
where , m are scalars
Comparing the coefficients of a , b and c on both sides of the equation
5 = 7 + 3m ..........(i)
6 = – 8 + 20 m ..........(ii)
7 = 9 + 5m ..........(iii)
From (i) and (iii) we get
1
4 = 8 = = m which evidently satisfies (ii) equation too.
2
Hence the given vectors are linearly dependent .
(30) Does there exist scalars u, v, w such that ue1 ve 2 we 3 î where e1 k̂ , e 2 ĵ k̂ ,
e 3 ĵ 2k̂ ?
(31) Consider a base a, b, c and a vector 2a 3b c . Compute the co-ordinates of this vector
relatively to the base p , q, r where p 2a 3b , q a 2b c , r 3a b 2c .
(32) If a and b are non-collinear vectors and A =(x + 4y) a + (2x + y + 1) b and B = (y – 2x + 2)
a + (2x – 3y – 1) b , find x and y such that 3 A 2B .
(33) If vectors a, b,c be linearly independent, then show that
(34) Given that î ĵ , î 2 ĵ are two vectors. Find a unit vector coplanar with these vectors and
perpendicular to the first vector î ĵ . Find also the unit vector which is perpendicular to the
plane of the two given vectors.
(35) If with reference to a right handed system of mutually perpendicular unit vectors î, ĵ, k̂ ,
3 î ĵ , 2 î ĵ 3k̂ . Express in the form 1 2 where 1 is parallel to and
2 is perpendicular to .
(36) Prove that a vector r in space can be expressed linearly in terms of three non-coplanar,,
[ r b c] a [ r c a ] b [ r a b ] c
non-zero vectors a, b, c in the form r
[ a b c]
Answers : (30) No (31) (0, – 7/5, 1/5) (32) x = 2, y = –1
1 3 1 1 3
(34) ± ( î ĵ ) ; k̂ (35) 1 î ĵ , 2 î ĵ 3k̂
2 2 2 2 2
Example # 28 : If a , b , c and a, b, c be the reciprocal system of vectors, prove that
(i) a . a b . b c . c 3 (ii) a a b b c c 0
Solution : (i) We have : a . a = b . b = c . c = 1
a . a + b . b + c . c = 1 + 1 + 1 = 3
1
(ii) We have : a = (b c ) , b = (c a ) and c = (a b) , where =
[a b c ]
a a a (b c ) {a (b c )} {(a . c ) b (a . b) c }
b b b (c a) {b (c a )} {(b . a) c (b . c ) a}
and c c c (a b) {c (a b )} {(c . b) a (c . a ) b}
a a b b c c
= {(a . c ) b (a . b) c } {(b . a) c (b . c ) a} {(c . b) a (c . a) b}
= [(a . c ) b (a . b) c (b . a ) c (b . c ) a (c . b ) a (c . a) b]
= [(a . c ) b (a . b) c (a . b ) c (b . c ) a (b . c ) a (a . c ) b] = 0 0
Equation of a plane :
(i) The equation ( r r0 ) . n 0 represents a plane containing the point with position vector
r0 , where n is a vector normal to the plane.
The above equation can also be written as r . n d , where d = r0 . n
(ii) Angle between two planes is the angle between two normals drawn to the planes and the angle
between a line and a plane is the compliment of the angle between the line and the normal to
the plane.
(iii) The length of perpendicular (p) from a point having position vector a to the plane r . n d is
|a.nd|
given by p =
|n|
(iv) If ( r a ) . n1 0 and ( r a ) . n2 0 are the equations of two planes, then the equation of line
of intersection of these planes is given by r a (n1 n 2 ) .
Test of collinearity :
Three points A,B,C with position vectors a, b, c respectively are collinear, if & only if there exist
scalars x, y, z not all zero simultaneously such that xa yb zc = 0 , where x + y + z = 0.
Example # 29 : Show that the vectors 2a b 3c , a b 2c and a b 3c are non-coplanar vectors.
Example # 30 : Prove that four points 2a 3b c , a 2b 3c , 3a 4b 2c and a 6b 6c are coplanar..
Solution : Let the given four points be P, Q, R and S respectively. These points are coplanar if the vectors
PQ , PR and PS are coplanar. These vectors are coplanar iff one of them can be expressed
as a linear combination of other two. So let PQ = x PR + y PS
a 5b 4c = x a b c + y a 9b 7c
a 5b 4c = (x – y) a + (x – 9y) b + (–x + 7y) c
x – y = –1, x – 9y = –5, –x + 7y = 4 [Equating coeff. of a, b, c on both sides]
1 1
Solving the first two equations of these three equations, we get x = – ,y= .
2 2
These values also satisfy the third equation. Hence the given four points are coplanar.
(37) If a, b, c, d are any four vectors in 3-dimensional space with the same initial point and such
that 3a 2b c 2d 0 , show that the terminal A, B, C, D of these vectors are coplanar. Find
the point (P) at which AC and BD meet. Also find the ratio in which P divides AC and BD.
(38) Show that the vector a b c , b c a and 2a 3b 4c are non-coplanar, where a, b, c are
any non-coplanar vectors.
(39) Find the value of for which the four points with position vectors ĵ k̂ , 4 î 5 ĵ k̂ , 3 î 9 ĵ 4k̂
(c) The moment of F about ’O’ is defined as M r x F , where r is the position vector of P w.r.t. ’O’. The
direction of M is along the normal to the plane OPN such that r , F & M form a right handed system.
(d) Moment of the couple = ( r1 r2 ) x F , where r1 and r2 are position vectors of the point of the application
of the forces F and F.
Example # 31 : Forces of magnitudes 5 and 3 units acting in the directions 6 î 2 ĵ 3k̂ and 3 î – 2 ĵ 6k̂
respectively act on a particle which is displaced from the point (2, 2, –1) to (4, 3, 1). Find the
work done by the forces.
Solution : Let F be the resultant force and d be the displacement vector. Then,
(6 î 2 ĵ 3k̂ ) (3 î – 2 ĵ 6k̂ ) 1
F = 5 +3 =
7
(39 î 4 ĵ 33k̂ )
36 4 9 9 4 36
and d = ( 4 î 3 ĵ k̂ ) – (2 î 2 ĵ k̂ ) = 2 î ĵ 2k̂
1
Total work done = F . d = (39 î 4 ĵ 33k̂ ) . (2 î ĵ 2k̂ )
7
1 148
= (78 + 4 + 66) = units.
7 7
(40) A point describes a circle uniformly in the î , ĵ plane taking 12 seconds to complete one
revolution. If its initial position vector relative to the centre is î and the rotation is from î to ĵ ,
find the position vector at the end of 7 seconds. Also find the velocity vector.
(41) The force represented by 3 î 2k̂ is acting through the point 5 î 4 ĵ 3k̂ . Find its moment
(42) Find the moment of the couple formed by the forces 5 î k̂ and 5 î k̂ acting at the points
(9, –1, 2) and (3, –2, 1) respectively
Answers : (40) –
1
2
3 î ĵ , 12 î 3 ĵ (41) 2 î 20 ĵ 3k̂ (42) î ĵ 5k̂
RESONANCE VECTOR - 306
Miscellaneous solved examples
Example # 32 : Show that the points A, B, C with position vectors 2 î ĵ k̂ , î 3 ĵ 5k̂ and 3 î 4 ĵ 4k̂
respectively are the vertices of a right angled triangle. Also find the remaining angles of the
triangle.
Solution : We have,
= (2 î ĵ k̂ ) – (3 î 4 ĵ 4k̂ ) = î 3 ĵ 5k̂
Since AB + BC + CA = ( î 2 ĵ 6k̂ ) + (2 î ĵ k̂ ) + ( î 3 ĵ 5k̂ ) = 0
So A, B and C are the vertices of a triangle.
Now, BC . CA = (2 î ĵ k̂ ) . ( î 3 ĵ 5k̂ ) = –2 – 3 + 5 = 0
BC CA BCA =
2
Hence ABC is a right angled triangle.
Since A is the angle between the vectors AB and AC . Therefore
AB . AC ( î 2 ĵ 6k̂ ) . ( î 3 ĵ 5k̂ )
cos A = =
| AB | | AC | ( 1)2 ( 2)2 ( 6)2 12 ( 3)2 ( 5)2
1 6 30 35 35
= = =
1 4 36 1 9 25 41 35 41
35
A = cos –1
41
BA .BC ( î 2 ĵ 6k̂ ) . (2 î ĵ k̂ )
cos B = =
| BA | | BC | 1 2 2 6 2 2 2 ( 1)2 (1)2
2
226 6 6
cos B = = B = cos–1
41 6 41 41
Example # 33 : If a, b, c are three mutually perpendicular vectors of equal magnitude, prove that a b c is
equally inclined with vectors a, b and c .
Solution : Let | a | = | b | = | c | = (say). Since a, b, c are mutually
perpendicular vectors, therefore a . b = b . c = c . a = 0 ..............(i)
2
Now, abc = a . a + b . b + c . c + 2a . b + 2b . c + 2c . a
2
= | a | | + | b | 2 + | c |2 [Using (i) ]
= 3 2 [ | a | = | b | = | c | = ]
RESONANCE
| a b c | = 3 ..............(ii)
VECTOR - 307
Suppose a b c makes angles 1, 2, 3 with a, b and c respectively. Then,
a . (a b c ) a.aa.ba.c
cos1 = =
|a||a bc | | a || a b c |
| a |2 |a| 1
= = = = [Using (ii)]
|a||a bc | |abc | 3 3
1
1 = cos –1
3
1 1
Similarly, 2 = cos –1 and = cos –1
3 3
3
1 = 2 = 3.
Hence, a b c is equally inclineded with a, b and c
Example # 34 : Prove using vectors : If two medians of a triangle are equal, then it is isosceles.
Solution : Let ABC be a triangle and let BE and CF be two equal medians. Taking A as the origin, let the
position vectors of B and C be b and c respectively. Then,
1 1
P.V. of E = c and P.V. of F =
2 2 b
1
BE = 2 (c 2b)
1
CF = 2 (b 2c )
Now, BE = CF | BE | = | CF |
2 2
1 1
| BE | = | CF |2
2 (c 2b) = (b 2c )
2 2
1 1
| c 2b |2 = | b 2c |2 | c 2b |2 = | b 2c |2
4 4
(c 2b) . (c 2b) = (b 2c ) . (b 2c )
c . c – 4b . c + 4b . b = b . b – 4b . c + 4c . c
| c |2 – 4b . c + 4 | b |2 = | b |2 – 4b . c + 4 | c |2
2
3 | b | 2 = 3 | c |2 | b |2 = | c |
AB = AC Hence triangle ABC is an isosceles triangle.
Example # 35 : Using vectors : Prove that cos (A + B) = cos A cos B – sin A sin B
Solution : Let OX and OY be the coordinate axes and let î and ĵ be unit vectors along OX and OY
respectively. Let XOP = A and XOQ = B. Drawn PL OX and QM OX.
LP = (OP sin A) ĵ
RESONANCE VECTOR - 308
Now, OL + LP = OP
Now, OM + MQ = OQ
BC + CA = – AB ......(i)
Squaring both sides
( BC )2 + ( CA )2 + 2 ( BC ). CA = ( AB )2
a2 + b2 + 2 ( BC . CA ) = c2 c 2 = a2 + b2 + 2 ab cos ( – C)
c 2 = a2 + b2 – 2ab cosC
(ii) ( BC + CA ). AB = – AB . AB
BC . AB + CA . AB = – c2
– ac cosB – bc cos A = – c 2
acosB + bcosA = c.
Example # 37 : If D, E, F are the mid-points of the sides of a triangle ABC, prove by vector method that area of
1
DEF = (area of ABC)
4
Solution : Taking A as the origin, let the position vectors of B and C be b and c respectively. Then the
1 1 1
position vectors of D, E and F are (b c ) , c and respectively..
2 2 2 b
1 1 b
Now, DE = c – (b c ) =
2 2 2
1 1 c
and DF = b – ( (b c ) =
2 2 2
1 1 b c
Vector area of DEF = (DE DF ) =
2 2 2 2
1 1 1 ( AB AC ) 1
= (b c ) = = (vector area of ABC)
8 4 2 4
1
Hence area of DEF = area of ABC.
4
RESONANCE VECTOR - 309
Example # 38 : P, Q are the mid-points of the non-parallel sides BC and AD of a trapezium ABCD. Show that
APD = CQB.
Solution : Let AB = b and AD = d
Now DC is parallel to AB there exists a scalar t such that DC = t AB = t b
AC = AD + DC = d t b
1 1
The position vectors of P and Q are (b d t b) and respectively..
2 2 d
Now 2 APD = AP × AD
1 1
= (b d t b ) × d = (1 + t) (b d)
2 2
1
Also 2 CQB = CQ × CB = d (d tb) × [b (d t b)]
2
1
= d t b d (1 t ) b
2
=
1
2
(1 t ) (d b) t (b d)
1 1
= (1 t 2t ) (b d) = (1 t ) (b d) = 2 APD
2 2
Hence Prove.
Example # 39 : Let u and v are unit vectors and w is a vector such that (u v ) u = w and w u v then find
the value of u v w .
Solution : Given (u v ) u = w and w u v
(u v ) u × u w u
u v × u u u = v (as w u v )
u . u v v . u u u u v (using u . u = 1 and u u = 0, since unit vector)
u . v u 0
v ( v . u) u v
u.v = 0 (as u 0) .............(i)
Now u . (v w )
u . (v ((u v) u)) (given w (u v ) + u)
u . ( v (u v ) v u) u . (( v . v ) u ( v . u) v v u)
u . (| v |2 u 0 v u) (as u . v 0 from (i))
| v |2 (u . u) – u . ( v u)
| v |2 | u |2 – 0 (as u v u = 0)
1 (as | u | = | v | = 1)
u v w = 1
Example # 40 : In any triangle, show that the perpendicular bisectors of the sides are concurrent.
Solution : Let ABC be the triangle and D, E and F are respectively middle points of sides BC, CA and
AB. Let the perpendicular bisectors of BC and CA meet at O. Join OF. We are required to
prove that OF is to AB. Let the position vectors of A, B, C with O as origin of reference
be a , b and c respectively..
Example # 41 : A, B, C, D are four points in space. using vector methods, prove that
AC2 + BD2 + AD2 + BC2 AB2 + CD2 what is the implication of the sign of equality.
Solution : Let the position vector of A, B, C, D be a , b , c and d respectively then
AC2 + BD2 + AD2 + BC2 = c a . c a + d b . d b + d a . d a + c b . c b
2 2 2
= | c |2 + | a | – 2 a . c + | d |2 + | b |2 – 2 d . b + | d |2 + | a | – 2 a . d + | c | + | b |2 – 2b . c
2 2 2
2
= | a | + | b |2 – 2 a . b + | c | + | d |2 – 2 c . d + | a | + | b |2 + | c | + | d |2
+ 2 a . b + 2c . d – 2a . c – 2b . d – 2a . d – 2b . c
= ab . ab + c d . c d + abc d
2
= AB2 + CD2 + a b c d . a b c d
AB + CD
2 2
A PLANE
If line joining any two points on a surface lies completely on it then the surface is a plane.
OR
If line joining any two points on a surface is perpendicular to some fixed straight line. Then this surface
is called a plane. This fixed line is called the normal to the plane.
Equation of a plane :
(i) The equation ( r r0 ) . n 0 represents a plane containing the point with position vector
r0 , where n is a vector normal to the plane.
The above equation can also be written as r . n d , where d = r0 . n
(ii) Angle between two planes is the angle between two normals drawn to the planes and the angle
between a line and a plane is the compliment of the angle between the line and the normal to
the plane.
(iii) The length of perpendicular (p) from a point having position vector a to the plane r . n d is
|a.nd|
given by p =
|n|
(iv) If ( r a ) . n1 0 and ( r a ) . n2 0 are the equations of two planes, then the equation of line
of intersection of these planes is given by r a (n1 n 2 ) .
(v) Normal form of the equation of a plane is x + my + nz = p, where, ,m, n are the direction
cosines of the normal to the plane and p is the distance of the plane from the origin.
(vii) The equation of a plane passing through the point (x1, y1, z1) is given by
a (x x 1) + b( y y1) + c (z z1) = 0 where a, b, c are the direction ratios of the normal
to the plane.
(viii) Plane through three points: The equation of the plane through three noncollinear points
x y z 1
x 1 y 1 z1 1
(x 1, y1, z1), (x 2, y2, z2), (x 3, y3, z3) is =0
x 2 y2 z2 1
x 3 y3 z3 1
x y z
(ix) 1
Intercept Form: The equation of a plane cutting intercept a, b, c on the axes is
a b c
(x) Vector form: The equation of a plane passing through a point having position vector a &
normal to vector n is ( r a ). n = 0 or r . n = a . n
Note: (a) Vector equation of a plane normal to unit vector n̂ and at a distance d from the origin is
. =d
r n
(b) Coordinate planes
(i) Equation of yzplane is x = 0
(ii) Equation of xzplane is y = 0
(iii) Equation of xyplane is z = 0
VECTOR - 312
Vectors & Three Dimensional Geometry
(c) Planes parallel to the axes:
If a = 0, the plane is parallel to x axis i.e. equation of the plane parallel to the x axis is
by + cz + d = 0.
Similarly, equation of planes parallel to yaxis and parallel to zaxis are ax + cz +d = 0
and ax + by + d = 0 respectively.
(e) Transformation of the equation of a plane to the normal form: To reduce any equation
ax + by + cz d = 0 to the normal form, first write the constant term on the right hand
side and make it positive, then divide each term by a 2 b 2 c2 , where a, b, c are
coefficients of x, y and z respectively e.g.
ax by cz d
+ + =
a 2 b2 c2 a 2 b2 c2 a 2 b2 c2 a 2 b2 c2
Where (+) sign is to be taken if d > 0 and ( ) sign is to be taken if d < 0.
(g) Equation of a plane passing through a given point & parallel to the given vectors:
The equation of a plane passing through a point having position vector a and parallel to
b & c is r = a + b + c (parametric form) where & are scalars.
or r . (b c ) = a . (b c ) (non parametric form)
(h) A plane ax + by + cz + d = 0 divides the line segment joining (x 1, y1, z1) and (x2, y2, z2). in the
(i) The xyplane divides the line segment joining the points (x1, y1, z1) and (x2, y2, z2) in the ratio
z1 x y
. Similarly yzplane in 1 and zx plane in 1
z2 x2 y2
(j) Coplanarity of four points
The points A(x 1 y1 z1), B(x 2 y2 z2) C(x3 y3 z3) and D(x 4 y4 z4) are coplaner then
x 2 x1 y 2 y1 z 2 z1
x 3 x1 y 3 y1 z3 z1
=0
x 4 x1 y 4 y1 z 4 z1
very similar in vector method the points A ( r1 ), B( r2 ), C( r3 ) and D( r4 ) are coplanar if
[ r4 – r1 , r4 – r2 , r4 – r3 ] = 0
Example # 62 : Find the equation of the plane upon which the length of normal from origin is 10 and direction
ratios of this normal are 3, 2, 6.
Solution : If p be the length of perpendicular from origin to the plane and , m, n be the direction cosines
of this normal, then its equation is
VECTOR - 313
Vectors & Three Dimensional Geometry
x + my + nz = p ..... (1)
Here p = 10
Direction ratios of normal to the plane are 3, 2, 6
32 22 62 = 7
Direction cosines of normal to the required plane are
3 2 6
= ,m= ,n=
7 7 7
3 2 6
Putting the values of , m, n, p in (1), equation of required plane is x + y + z = 10
7 7 7
or, 3x + 2y + 6z = 70
Example # 63 : Show that the points (0, – 1, 0), (2, 1, – 1), (1, 1, 1), (3, 3, 0) are coplanar.
x 1 my1 nz 1 1
On dividing (2) by (3), we get
x 12 y 12 z12 p
VECTOR - 314
Vectors & Three Dimensional Geometry
x y z
Example # 65 : A point P moves on a plane = 1. A plane through P and perpendicular to OP meets
s
a b c
the co-ordinate axes in A, B and C. If the planes through A, B and C parallel to the planes
x = 0, y = 0, z = 0 intersect in Q, find the locus of Q.
x y z
Solution : Given plane is 1 ..... (1)
a b c
Let P (h, k, )
h k
Then =1 ..... (2)
a b c
OP = h2 k 2 2
h k
Direction cosines of OP are , ,
h2 k 2 2 h2 k 2 2 h2 k 2 2
Equation of the plane through P and normal to OP is
h k
x y z h2 k 2 2
2 2 2 2 2 2 2 2 2
h k h k h k
or, hx + ky + z = (h + k + ) 2 2 2
h2 k 2 2 h2 k 2 2 h2 k 2 2
A 0, , 0 0, 0,
, B , C
, 0, 0
h k
h2 k 2 2 h2 k 2 2 h2 k 2 2
Let Q (, , ), then = ,= ,= ..... (3)
h k
1 1 1 h2 k 2 2 1
Now 2
2
2
2 2 2 2
..... (4)
(h k ) (h k 2 2 )
2
h2 k 2 2
From (3), h =
h h2 k 2 2 k h2 k 2 2 h2 k 2 2
. Similarly and
a a b b c c
h2 k 2 2 h2 k 2 2 h2 k 2 2 h k
= 1 [from (2)]
a b c a b c
1 1 1 1 1 1 1
or, 2 2 2 2 [from (4)]
a b c h k
2 2
1 1 1 1 1 1
Required locus of Q (, , ) is 2 2 2 .
ax by cz x y z
Self practice problems :
(71) Check whether given points are coplanar if yes find the equation of plane containing them
A (1, 1, 1), B (0, – 1, 0), C (2, 1, –1), D (3, 3, 0)
(72) Find the plane passing through point (– 3, – 3, 1) and perpendicular to the line joining the
points (2, 6, 1) and (1, 3, 0).
(73) Find the equation of plane parallel to plane x + 5y – 4z + 5 = 0 and cutting intercepts on the
axes whose sum is 150.
VECTOR - 315
Vectors & Three Dimensional Geometry
(74) Find the equation of plane passing through (2, 2, 1) and (9, 3, 6) and perpendicular to the
plane x + 3y + 3z = 8.
(75) Find the equation of the plane parallel to î ĵ k̂ and î ĵ and passing through (1, 1, 2).
(76) Find the equation of the plane passing through the point (1, 1, – 1) and perpendicular to the
planes x + 2y + 3z – 7 = 0 and 2x – 3y + 4z = 0.
Answers : (71) yes, 4x – 3y + 2z = 3 (72) x + 3y + z + 11 = 0
3000
(73) x + 5y – 4z = (74) 3x + 4y – 5z = 9
19
(75) x + y – 2z + 2 = 0 (76) 17x + 2y – 7z = 26
Sides of a plane:
A plane divides the three dimensional space in two equal parts. Two points A (x 1 y 1 z 1 )
and B (x2 y2 z2) are on the same side of the plane ax + by + cz + d = 0 if ax 1 + by1 + cz1 + d and
ax 2 + by2 + cz2 + d are both positive or both negative and are opposite side of plane if both of these
values are in opposite sign.
Example # 66 : Show that the points (1, 2, 3) and (2, – 1, 4) lie on opposite sides of the plane
x + 4y + z – 3 = 0.
Solution : Since the numbers 1+ 4 × 2 + 3 – 3 = 9 and 2 – 4 + 4 – 3 = – 1 are of opposite sign, then points
are on opposite sides of the plane.
A plane & a point
ax 'by 'cz' d
(i) Distance of the point (x , y, z) from the plane ax + by + cz+ d = 0 is given by .
a2 b2 c 2
(ii) The length of the perpendicular from a point having position vector a to plane r . n = d
| a.n d |
is p = .
|n|
(iii) The coordinates of the foot of perpendicular from the point (x1, y1, z1) to the plane
x x1 y y1 z z1 (ax1 by1 cz1 d)
ax + by + cz + d = 0 are =–
a b c a 2 b2 c 2
(iv) To find image of a point w.r.t. a plane.
Let P (x1, y1, z1) is a given point and ax + by + cz + d = 0 is given plane. Let (x, y, z) is the
image of the point, then
(a) x – x 1 = a, y – y1 = b, z – z1 = c
x = a + x 1, y = b + y1, z = c + z1 ...... (i)
x x 1 y y 1 z z1
(b) a + b + c =0 ...... (ii)
2 2 2
from (i) put the values of x, y, z in (ii) and get the values of and subtitute in (i) to get
(x y z).
The coordinate of the image of point (x1 , y1 , z1) w.r.t the plane ax + by + cz + d = 0 are given
x ' x 1 y ' y 1 z' z1 (ax1 by1 cz1 d)
by =–2
a b c a 2 b2 c 2
(v) The distance between two parallel planes ax + by + cx + d = 0 and ax + by + cx + d’ = 0
| d d' |
is
a b2 c 2
2
VECTOR - 316
Vectors & Three Dimensional Geometry
Example # 67 : Find the image of the point P (3, 5, 7) in the plane 2x + y + z = 0.
Example # 69 : A plane passes through a fixed point (a, b, c). Show that the locus of the foot of perpendicular
to it from the origin is the sphere x2 + y2 + z2 – ax – by – cz = 0 O(0, 0, 0)
Solution : Let the equation of the variable plane be
x + my + nz + d = 0 ..... (1)
Plane passes through the fixed point (a, b, c)
a + mb + nc + d = 0 ..... (2) P(, , )
Let P (, , ) be the foot of perpendicular from origin to plane (1).
Direction ratios of OP are
– 0, – 0, – 0 i.e. , ,
From equation (1), it is clear that the direction ratios of normal to the plane i.e. OP are , m, n;
, , and , m, n are the direction ratios of the same line OP
1
= = = (say)
m n k
= k, m = k, n = k ..... (3)
Putting the values of , m, n in equation (2), we get
ka + kb + kc + d = 0 ..... (4)
Since , , lies in plane (1)
+ m + n + d = 0 ..... (5)
VECTOR - 317
Vectors & Three Dimensional Geometry
Putting the values of , m, n from (3) in (5), we get
k2 + k 2 + k2 + d = 0 ..... (6)
or k + k + k – ka – kb – kc = 0
2 2 2
(77) Find the intercepts of the plane 3x + 4y – 7z = 84 on the axes. Also find the length of
perpendicular from origin to this plane and direction cosines of this normal.
1 4 1 13 5 1 7
(78) (i) (ii) , , (iii) , ,
6 3 6 6 3 3 3
(ii) Equation of bisector of the angle containing origin: First make both the constant terms positive.
VECTOR - 318
Vectors & Three Dimensional Geometry
Family of planes
(i) Any plane passing through the line of intersection of nonparallel planes or equation of
the plane through the given line in non symmetric form.
a 1x + b 1y + c 1z + d 1 = 0 & a2x + b2y + c 2z + d2 = 0 is
a1x + b1y + c 1z + d1 + (a2x + b2y + c2z + d2) = 0, where R
(ii) The equation of plane passing through the intersection of the planes r . n1 = d 1
& r . n 2 = d2 is r . (n1 + n 2 ) = d1 d2 where is arbitrary scalar
Example # 70 : The plane x – y – z = 4 is rotated through 90° about its line of intersection with the plane
x + y + 2z = 4. Find its equation in the new position.
Solution : Given planes are x – y – z = 4 ..... (1)
and x + y + 2z = 4 ..... (2)
Since the required plane passes through the line of intersection of planes (1) and (2)
its equation may be taken as
x + y + 2z – 4 + k (x – y – z – 4) = 0
or (1 + k)x + (1 – k)y + (2 – k)z – 4 – 4k = 0 ..... (3)
Since planes (1) and (3) are mutually perpendicular,
(1 + k) – (1 – k) – (2 – k) = 0
2
or, 1+k–1+k–2+k=0 or k=
3
2
Putting k = in equation (3), we get 5x + y + 4z = 20
3
This is the equation of the required plane.
Example # 71 : Find the equation of the plane through the point (1, 1, 1) which passes through the line of
intersection of the planes x + y + z = 6 and 2x + 3y + 4z + 5 = 0.
Solution : Given planes are x + y + z – 6 = 0 ..... (1)
and 2x + 3y + 4z + 5 = 0 ..... (2)
Given point is P (1, 1, 1).
Equation of any plane through the line of intersection of planes (1) and (2) is
x + y + z – 6 + k (2x + 3y + 4z + 5) = 0 ..... (3)
If plane (3) passes through point P, then
3
1 + 1 + 1 – 6 + k (2 + 3 + 4 + 5) = 0 or, k=
14
From (3) required plane is 20x + 23y + 26z – 69 = 0
Example # 72 : Find the planes bisecting the angles between planes 2x + y + 2z = 9 and
3x – 4y + 12z + 13 = 0. Which of these bisector planes bisects the acute angle between the
given planes. Does origin lie in the acute angle or obtuse angle between the given planes ?
Solution : Given planes are – 2x – y – 2z + 9 = 0 ..... (1)
and 3x – 4y + 12z + 13 = 0 ..... (2)
2 x y 2 z 9 3 x 4 y 12z 13
Equations of bisecting planes are
( 2)2 ( 1)2 ( 2)2 3 2 ( 4)2 (12)2
or, 13 [– 2x – y – 2z + 9] = ± 3 (3x – 4y + 12z + 13)
or, 35x + y + 62z = 78, ..... (3) [Taking +ve sign]
and 17x + 25y – 10z = 156 ..... (4) [Taking – ve sign]
Now a1a2 + b1b2 + c 1c 2 = (– 2) (3) + (– 1) (– 4) + (– 2) (12)
= – 6 + 4 – 24 = – 26 < 0
Bisector of acute angle is given by 35x + y + 62z = 78
a1a2 + b1b2 + c 1c 2 < 0, origin lies in the acute angle between the planes.
VECTOR - 319
Vectors & Three Dimensional Geometry
Example # 73 : If the planes x – cy – bz = 0, cx – y + az = 0 and bx + ay – z = 0 pass through a straight line,
then find the value of a2 + b2 + c 2 + 2abc.
Solution : Given planes are x – cy – bz = 0 ..... (1)
cx – y + az = 0 ..... (2)
bx + ay – z = 0 ..... (3)
Equation of any plane passing through the line of intersection of planes (1) and (2) may be
taken as x – cy – bz + (cx – y + az) = 0
or, x (1 + c) – y (c + ) + z (– b + a) = 0 ..... (4)
If planes (3) and (4) are the same, then equations (3) and (4) will be identical.
1 c (c ) b a
b a 1
(i) (ii) (iii)
From (i) and (ii), a + ac = – bc – b
(a bc )
or, =– ..... (5)
(ac b)
From (ii) and (iii),
–(ab c)
c + = – ab + a2 or = ..... (6)
1 a2
(a bc ) (ab c )
From (5) and (6), we have .
ac b (1 a 2 )
or, a – a3 + bc – a2bc = a2bc + ac 2 + ab2 + bc
or, a2bc + ac2 + ab2 + a3 + a2bc – a = 0
or, a2 + b2 + c 2 + 2abc = 1.
(81) Find the equations of the planes bisecting the angles between the planes x + 2y + 2z – 3 = 0,
3x + 4y + 12z + 1 = 0 and sepecify the plane which bisects the acute angle between them.
(82) Show that the origin lies in the acute angle between the planes
x + 2y + 2z – 9 = 0 and 4x – 3y + 12z + 13 = 0
Let A (x1, y1, z1), B (x 2, y2, z2), C (x 3, y3, z3) be the vertices of a triangle, then = (2x 2y 2z )
y1 z1 1 z1 x1 1 x1 y1 1
1 1
where x = y2 z2 1 , y = z x2 1 and z = x 2 y2 1
2 2 2
y3 z3 1 z3 x3 1 x3 y3 1
VECTOR - 320
Vectors & Three Dimensional Geometry
Vector Method From two vector AB and AC . Then area is given by
i j k
1 1
| AB x AC | = x 2 x1 y 2 y1 z2 z1
2 2
x 3 x1 y 3 y1 z3 z1
Example # 74 : Through a point P (h, k, ) a plane is drawn at right angles to OP to meet the co-ordinate axes
p5
in A, B and C. If OP = p, show that the area of ABC is , where O is the origin.
2hk
Solution : OP = h2 k 2 2 = p
h k
Direction cosines of OP are , ,
2 2 2 2 2 2
h k h k h k 2 2
2
Since OP is normal to the plane, therefore, equation of the plane will be,
h k
x y z h2 k 2 2
2 2 2 2 2 2 2 2 2
h k h k h k
or, hx + ky + z = h2 + k 2 + 2 = p2 ..... (1)
p2 p2 p2
A h , 0, 0 , B 0, k , 0 , C 0, 0,
p2
0 1
h
1 p2 1 p4
= Mod of 0 1 =
2 k 2 | hk |
0 0 1
4
1 p 1 p4
Similarly, Ayz = and Azx =
2 | k | 2 | h |
1 p8 1 p8 1 p8
2 =
4 h 2k 2 4 k 2 2 4 2h 2
p8 p10 p5
= 2 2 2 (2 + k 2 + h2) = 2 2 2 or = .
4h k 4h k 2hk
Volume of a tetrahedron :
Volume of a tetrahedron with vertices A (x1, y1, z1), B( x2, y2, z2), C (x 3, y3, z3) and D (x 4, y4, z4) is
x1 y1 z1 1
1 x2 y2 z2 1
V = modulus of
6 x3 y3 z3 1
x4 y4 z4 1
VECTOR - 321
Vectors & Three Dimensional Geometry
Angle between a plane and a line:
x x1 y y1 z z1
(i) If is the angle between line = = and the plane ax + by + cz + d = 0, then
m n
a bm cn
sin = .
(a 2 b2 c2 ) 2 2 2
m n
b.n
(ii) Vector form: If is the angle between a line r = ( a + b ) and r . n = d then sin = .
| b | | n |
m n
(iii) Condition for perpendicularity = = b xn = 0
a b c
(iv) Condition for parallel a + bm + cn = 0 b.n = 0
Condition for a line to lie in a plane
x x1 y y1 z z1
(i) Cartesian form: Line = = would lie in a plane
m n
ax + by + cz + d = 0, if ax1 + by1 + cz1 + d = 0 & a + bm + cn = 0.
(ii) Vector form: Line r = a + b would lie in the plane r . n = d if b . n = 0 & a . n = d
Coplanar lines :
x y z x ' y ' z '
(i) If the given lines are = = and = = , then condition
m n ' m' n'
x y z x y z
the above two lines is m n =0 or m n =0
' m' n' ' m' n'
(ii) Condition of coplanarity if both the lines are in general form Let the lines be
ax + by + cz + d = 0 = ax + by + cz + d &
x + y + z + = 0 = x + y + z +
a b c d
a' b' c' d'
They are coplanar if =0
Alternative method
' ' ' '
get vector along the line of shortest distance as
î ĵ k̂
u = m n
m n
VECTOR - 322
Vectors & Three Dimensional Geometry
Now get unit vector along u , let it be û
Let v = ( – ) î + ( – ) ĵ + ( – ) k̂
S. D. = û . v
Example # 75 : Find the distance of the point (1, 0, – 3) from the plane x – y – z = 9 measured parallel to the
x2 y2 z6
line .
2 3 6
Solution : Given plane is x – y – z = 9 ..... (1)
x2 y2 z6
Given line AB is ..... (2)
2 3 6
Equation of a line passing through the point Q(1, 0, – 3) and parallel to line (2) is
x 1 y z 3
= r.. ..... (3)
2 3 6
Co-ordinates of any point on line (3) may be taken as
P (2r + 1, 3r, – 6r – 3)
If P is the point of intersection of line (3) and plane (1), then P lies on plane (1),
B
(2r + 1) – (3r) – (– 6r – 3) = 9
r=1 Q (1, 0, – 3)
or, P (3, 3, – 9)
Distance between points Q (1, 0, – 3) and P (3, 3, – 9) A
Example # 76 : Find the equation of the plane passing through (1, 2, 0) which contains the line
x 3 y 1 z 2
.
3 4 2
Solution : Equation of any plane passing through (1, 2, 0) may be taken as
a (x – 1) + b (y – 2) + c (z – 0) = 0 ..... (1)
where a, b, c are the direction ratios of the normal to the plane. Given line is
x 3 y 1 z 2
..... (2)
3 4 2
If plane (1) contains the given line, then
3a + 4b – 2c = 0 ..... (3)
Also point (– 3, 1, 2) on line (2) lies in plane (1)
a (– 3 – 1) + b (1 – 2) + c (2 – 0) = 0
or, – 4a – b + 2c = 0 ..... (4)
a b c
Solving equations (3) and (4), we get
8 2 8 6 3 16
a b c
or, = k (say). ..... (5)
6 2 13
Substituting the values of a, b and c in equation (1), we get
6 (x – 1) + 2 (y – 2) + 13 (z – 0) = 0.
or, 6x + 2y + 13z – 10 = 0. This is the required equation.
VECTOR - 323
Vectors & Three Dimensional Geometry
x 1 y 1 z 3
Example # 77 : Find the equation of the projection of the line on the plane x + 2y + z = 9.
2 1 4
x 1 y 1 z 3
Solution : Let the given line AB be ..... (1)
2 1 4
Given plane is x + 2y + z = 9 ..... (2)
Let DC be the projection of AB on plane (2)
Clearly plane ABCD is perpendicular to plane (2).
Equation of any plane through AB may be taken as (this plane passes through the point
(1, – 1, 3) on line AB)
a (x – 1) + b (y + 1) + c (z – 3) = 0 ..... (3) B
where 2a – b + 4c = 0 ..... (4) A
[normal to plane (3) is perpendicular to line (1)]
Since plane (3) is perpendicular to plane (2),
a + 2b + c = 0 ..... (5)
a b c
Solving equations (4) & (5), we get . D C
9 2 5
Substituting these values of a, b and c in equation (3), we get
9 (x – 1) – 2 (y + 1) – 5 (z – 3) = 0
or, 9x – 2y – 5z + 4 = 0 ...... (6)
Since projection DC of AB on plane (2) is the line of intersection of plane ABCD and plane
(2), therefore equation of DC will be
9x 2y 5z 4 0 .....( i)
and ..... (7)
x 2y z 9 0 .....(ii)
Let , m, n be the direction ratios of the line of intersection of planes (i) and (ii)
9– 2m – 5n = 0 ..... (8)
and + 2m + n = 0 ..... (9)
m n m n
2 10 5 9 18 2 4 – 7 10
Let any point on line (7) is (, , 0) 9 – 2 + 4 = 0
1 17
+ 2 – 9 = 0 = , =
2 4
1 17
x– y–
So equation of line is 2 4 z–0
4 –7 10
x 3 y 1 z 2 x–7 y z7
Example # 78 : Show that the lines and are coplanar. Also find the
2 3 1 –3 1 2
equation of the plane containing them.
x 3 y 1 z 2
Solution : Given lines are = r (say) ..... (1)
2 3 1
x7 y z7
and = R (say) ..... (2)
3 1 2
If possible, let lines (1) and (2) intersect at P.
Any point on line (1) may be taken as (2r + 3, – 3r – 1, r – 2) = P (let).
Any point on line (2) may be taken as (– 3R + 7, R, 2R – 7) = P (let).
2r + 3 = – 3R + 7
or, 2r + 3R = 4 ..... (3)
VECTOR - 324
Vectors & Three Dimensional Geometry
Also – 3r – 1 = R
or, – 3r – R = 1 ..... (4)
and r – 2 = 2R – 7
or, r – 2R = – 5. ..... (5)
Solving equations (3) and (4), we get,
r = – 1, R = 2
Clearly r = – 1, R = 2 satisfies equation (5).
Hence lines (1) and (2) intersect.
lines (1) and (2) are coplanar.
x 3 y 1 z 2
Equation of the plane containing lines (1) and (2) is 2 3 1 =0
3 1 2
or, (x – 3) (– 6 – 1) – (y + 1) (4 + 3) + (z + 2) (2 – 9) = 0
or, – 7 (x – 3) – 7 (y + 1) – 7 (z + 2) = 0
or, x–3+y+1+z+2=0 or, x + y + z = 0.
Example # 79: Find the equation of the sphere having centre at (1, 2, 3) and touching the plane
x + 2y + 3z = 0.
Solution : Given plane is x + 2y + 3z = 0 ..... (1) H
Let H be the centre of the required sphere.
Given H (1, 2, 3) P
Radius of the sphere,
VECTOR - 325
Vectors & Three Dimensional Geometry
HP = length of perpendicular from H to plane (1)
| 1 2 2 3 3 |
= = 14
14
Equation of the required sphere is (x – 1) 2 + (y – 2) 2 + (z – 3)2 = 14
or x 2 + y2 + z2 – 2x – 4y – 6z = 0
Example # 80: Find the equation of the sphere if it touches the plane r .( 2 î 2 ĵ k̂ ) = 0 and the position
Equation of the required sphere is | r c | = a
2
or | x î yĵ zk̂ (3 î 6 ĵ 4k̂ ) |
3
4
or | (x – 3) i + (y – 6) j + (z + 4) k |2 =
9
4
or (x – 3)2 + (y – 6)2 + (z + 4) 2 =
9
or 9 (x2 + y2 + z2 – 6x – 12y + 8z + 61) = 4
or 9x 2 + 9y2 + 9z2 – 54x – 108y + 72z + 545 = 0
Example # 81 : Find the equation of the sphere passing through the points (3, 0, 0), (0, – 1, 0), (0, 0, – 2) and
whose centre lies on the plane 3x + 2y + 4z = 1
Solution : Let the equation of the sphere be
x 2 + y2 + z2 + 2ux + 2vy + 2wz + d = 0 ..... (1)
Let A (3, 0, 0), B (0, – 1, 0), C (0, 0, – 2)
Since sphere (1) passes through A, B and C,
9 + 6u + d = 0 ..... (2)
1 – 2v + d = 0 ..... (3)
4 – 4w + d = 0 ..... (4)
Since centre (– u, – v, – w) of the sphere lies on plane
3x + 2y + 4z = 1
– 3u – 2v – 4w = 1 ..... (5)
(2) – (3) 6u + 2v = – 8 ..... (6)
(3) – (4) – 2v + 4w = 3 ..... (7)
2v 8
From (6), u = ..... (8)
6
From (7), 4w = 3 + 2v ..... (9)
2v 8
Putting the values of u, v and w in (5), we get 2v – 3 – 2v 1
2
2v + 8 – 4v – 6 – 4v = 2 v=0
08 4
From (8), u=
6 3
VECTOR - 326
Vectors & Three Dimensional Geometry
3
From (9), 4w = 3 w=
4
From (3), d = 2v – 1 = 0 – 1 = – 1
8 3
From (1), equation of required sphere is x2 + y2 + z2 – x+ z–1=0
3 2
or 6x 2 + 6y2 + 6z2 – 16x + 9z – 6 = 0.
Example # 82 : Find the equation of the sphere with the points (1, 2, 2) and (2, 3, 4) as the extremities of a
diameter. Find the co-ordinates of its centre.
Solution : Let A (1, 2, 2), B (2, 3, 4)
Equation of the sphere having (x1, y1, z1) and (x 2, y2, z2) as the extremities of a diameter is
(x – x1) (x – x2) + (y – y1) (y – y2) + (z – z1) (z – z2) = 0
Here x 1 = 1, x 2 = 2, y1 = 2, y2 = 3, z1 = 2, z2 = 4
required equation of the sphere is
(x – 1) (x – 2) + (y – 2) (y – 3) + (z – 2) (z – 4) = 0
or x 2 + y2 + z2 – 3x – 5y – 6z + 16 = 0
Centre of the sphere is middle point of AB
3 5
Centre is , , 3
2 2
(89) Find the value of k for which the plane x + y + z = 3 k touches the sphere
x + y + z – 2x – 2y – 2z – 6 = 0.
2 2 2
(90) Find the equation to the sphere passing through (1, – 3, 4), (1, – 5, 2) and (1, – 3, 0) which
has its centre in the plane x + y + z = 0
(91) Find the equation of the sphere having centre on the line 2x – 3y = 0, 5y + 2z = 0 and passing
through the points (0, – 2, – 4) and (2, – 1, – 1).
(92) Find the centre and radius of the circle in which the plane 3x + 2y – z – 7 14 = 0 intersects
the sphere x 2 + y2 + z2 = 81.
(93) A plane passes through a fixed point (a, b, c) and cuts the axes in A, B, C. Show that the
a b c
locus of the centre of the sphere OABC is = 2.
x y z
VECTOR - 327